Sunteți pe pagina 1din 190

RESUMENES PROEDUMED   

ENARM XL 
 
 
“El cansancio es temporal, el orgullo es para siempre” 
 
TEMAS 
Patología esofágica 
  Acalasia   
  Enfermedad por reflujo gastroesofágico 
  Esófago de Barret 
Enfermedad ácido péptica y sus complicaciones 
  Úlcera péptica 
Colecistitis, colelitiasis y sus complicaciones 
  Colecistitis 
  Colelitiasis 
Sangrado de tubo digestivo alto y bajo 
  Hemorragia digestiva 
  Várices esofágicas 
Patología abdominal (resolución quirúrgica de urgencia) 
  Abdomen agudo* 
  Apendicitis* 
  Colecisitits* 
  Coledocolitiasis y colangitis* 
  Enfermedad diverticular* 
  Hemorragia de tubo digestivo alto y bajo* 
  Hernia inguinal y femoral* 
  Hernia hiatal* 
  Hernia umbilical y de pared* 
  Oclusión intestinal en adultos* 
  Volvulus 
Alteraciones electrolíticas y ácido básicas 
Pancreatitis 
  Pancreatitis aguda 
Infecciones óseas y articulares 
  Ostemielitis 
  Artritis séptica 
Trauma de miembros 
  Trauma de codo 
  Esguince de tobillo 
  Fractura intracapsula del exgtremo proximal del femur 
  Sx de hombro doloroso 
Cervicalgias, lumbalgias y tumores 
  Cervicalgia 
  Lumbalgia 
  Osteocarcoma 
  Sarcoma de Ewing 
Patología articulares 
  Luxación glenohumeral 
  Luxación de cadera 
  Halux Valgus 
  Lesiones comunes en rodilla 
  Pie plano 
  Displasia del desarrollo de la cadera 
Patología de oído externo 
  Otitis externa 
Osteomielitis de la base del cráneo 
Otitis media aguda 
  Otitis media crónica 
  Hematoma auricular 
Patología del oido interno 
  Hipoacusia 
Presbiacusia 
Otosclerosis 
Patología del sistema vestibular 
  Generalidades de vértigo* 
Vértigo postural paroxístico* 
Neuronitis vestibular* 
  Enfermedad de Meniere* 
Patología de la nariz y senos paranasales 
  Epistaxis 
  Sinusitis aguda y crónica 
Patología de faringe y laringe 
  Faringoamigalitis aguda 
  Celulitis y abscesos periamigdalinos 
  Procesos inflamatorios de laringe 
  Tumores laringeos 
  Quiste de conducto tirogloso 
  Quistes faringeos 
Enfermedades corneales y de la órbita 
  Celulitis y abscesos periorbitarios 
Catarata 
  Catarata 
Enfermedades de la retina 
Retinopatía diabética 
  Retinopatías vasculares e hipertensivas 
  Glaucoma 
  Glaucoma de ángulo abierto 
  Glaucoma de ángulo cerrado 
Enfermedades de los párpados, aparato lagrimal y conjuntiva 
  Conjuntivitis generalidades 
Conjuntivitis bacteriana del adulto* 
  Conjuntivitis neonatal* 
  Toxoplasmosis* 
  Tracoma* 
  Orzuelo y chalazión* 
  Celulitis preseptal y orbitaria* 
  Perlas de oftalmología* 
Patología de colon, ano y recto 
  Físura anal* 
Absceso anorrectal* 
  Fístula anal* 
  Incontinencia fecal* 
  Hemorroides* 
  Cáncer de colon y recto 
  Diverticulosis 
  Trombosis mesentérica* 
Cálculo uretrales y uropatía obstructiva 
  Cálculos urinarios 
  Urolitiasis add 
Incontinencia urinaria 
  Incontinencia urinaria 
Patología de próstata 
  Cáncer de próstata 
Hiperplasia benigna de próstata  * 
Prostatitis* 
  Epididimitis* 
  Disfunción eréctil 
  Torsión testicular* 
  Criptorquidia* 
Trauma craneoencefálico 
  Trauma cráneoencefálico 
Trauma de cuello 
  Esguine cervical 
Trauma de tórax 
  Trauma de tórax 
  Taponamiento cardiaco 
  Neumotórax 
  Hemotórax 
Trauma abdominal 
  Trauma abdominal cerrado y abierto 
Choque 
  Choque hipovolémico traumático 
   
Angiología 
Insuficiencia arterial aguda* 
Enfermedad arterial periférica* 
Síndrome de reperfusión* 
Síndrome compartimental* 
Criterios de amputación* 
Enfermedad carotídea* 
Insuficiencia venosa* 
Enfermedad tromboembólica venosa* 
Trombosis venosa profunda* 
Pie diabético* 
Secuela postrombótica* 
Várices* 
 
   
     
 
 

 
 
PATOLOGÍA ESOFÁGICA 
ACALASIA 
DEFINICIÓN 
Aperistalsis del cuerpo esofágico, Ausencia de relajación del EEI 
 
SALUD PÚBLICA 
20‐50 años de edad. 
 
DIAGNÓSTICO 
Disfagia temprana, progresiva, a sólidos y líquidos, 2 años antes de buscar atención.   
Pérdida de peso variable. 
Regurgitación y vómito son utilizados para vaciar el esófago, puede haber broncoaspiración (complicación broncopulmonar más 
frecuente [10%]. 
Prueba inicial: Esofagograma de bario [imagen en punta de lápiz] 
Estándar de oro: manometria [ausencia de relajación del EEI] 
Endoscopia: descara pseudoacalasia, acalasia secundaria.   
 
TRATAMIENTO 
Incurable y el objetivo es el manejo de los síntomas.   
Médico nitratos, bloqueadores de canales de calcio vía SL. 
Dilataciones endoscópica 
Miotomía de Heller con funduplicatura (de elección) 
 
ACALASIA 
[r]HOMBRE DE 30 AÑOS DE EDAD, ACUDE POR PRESENTAR DISFAGIA Y REGURGITACIÓN PROGRESIVA. USTED SOSPECHA LA POSIBILIDAD 
DE ACALASIA. 
EL ESTUDIO QUE DEBERÁ SOLICITAR PARA CONFIRMAR SU SOSPECHA DIAGNÓSTICA ES: 
A) UN ESOFAGOGRAMA CON BARIO. 
B) UNA ENDOSCOPÍA. 
C) UNA PH METRÍA DE 24 HRS. 
D) UNA MANOMETRÍA. 
 
EN CASO DE CONFIRMAR SU DIAGNÓSTICO EL PACIENTE DEBERÁ SER TRATADO CON 
A) ANTAGONISTAS DE CALCIO. 
B) TOXINA BOTULINICA. 
D) DILATACIONES NEUMÁTICAS. 
E) MIOTOMÍA. 
 
RESPUESTAS D, E 
Para el ENARM es muy importante comprender qué es lo que te están preguntando.   
El estudio inicial en paciente con acalasia es el esofagograma  con bario, el cual demustra la imagen clásica en "punta de lápiz". Este 
estudio orienta al diagnóstico pero no lo confirma. La manometría es el estudio ideal para confirmarlo y demostrar ausencia de relajación 
del esfinter esofágico inferior y aperistalsis del cuerpo esofágico. Hay que recordar que en estadios iniciales aun no es posible encontrar 
la clásica imagen en el esofagograma. 
La endoscopia y la TAC son estudios útiles para diferenciar de acalasia de pseudoacalasia. 
La pHmetría en 24 hrs mide el reflujo ácido y es gold standar de ERGE, sin utilidad diagnóstica en acalasia. 
La miotomía y las dialtaciones son los únicos tratamiento duraderos en acalasia, y son más efectivos en pacientes vírgenes a tratamientos 
médicos. La miotomía de Heller con funduplicatura parcial tiene menor riesgo y es más eficaz que las dilatacines, por lo que en este 
paciente es gold standar. 
 
 
 
UN HOMBRE DE 65 AÑOS DE EDAD SE PRESENTA A SU MÉDICO QUEJÁNDOSE DE DIFICULTAD PARA DEGLUTIR, DOLOR EN EL PECHO DE 
VEZ EN CUANDO, Y REGURGITACIÓN DE ALIMENTOS. EN LOS ÚLTIMOS 2 MESES HA PERDIDO ALREDEDOR DE 7 KG. LOS RESULTADOS DE 
UN ESTUDIO DE DEGLUCIÓN DE BARIO SE MUESTRAN EN LA IMAGEN. ¿QUÉ PRUEBA SE DEBE REALIZAR PARA BUSCAR POSIBLES CAUSAS 
DE SU CONDICIÓN? 
 
 
A.  PHMETRÍA DE 24 HORAS 
B. MANOMETRÍA ESOFÁGICA 
C. MEDICIÓN DEL NIVEL DE GASTRINA SÉRICA 
D. ENDOSCOPIA SUPERIOR 
E. PRUEBA DEL ALIENTO CON UREA 
RESPUESTA D 
(YO) 
 
 
 
 
 
ENFERMEDAD POR REFLUJO GASTROESOFÁGICO 
DEFINICIÓN 
ERGE solo cuando produce síntomas molestos y/o complicaciones (ej. esofagitis, pérdida de peso, estancamiento ponderoestatural]) 
 
SALUD PÚBLICA 
15% la padecen 1 vez por semana, 7% diariamente. 
 
PATOGENIA 
Menanismos antireflujo 
‐Esfinter esofágico interior 
  Debilidad sin causa evidente 
  Esclerodermia 
  Embarazo 
  Tabaquismo 
  Anticolinérgicos 
  Relajantes del m. liso (beta adrenérgicos,aminofilina, nitratos, antagonistas de calcio, inh. de fosfodiesterasa) 
‐Músculo crural diafragmático. 
  Incompentencia 
‐Localización anatómia del EEI. 
 
DIAGNÓSTICO 
Típicos: pirosis y regurgitación 
Atípicos tos crónica, disfonía, dolor torácico.   
Otros:    eructos, disfagia, hipersalivación, náuseas, hipo.   
 
Endoscopia: si sospecha de estenosis, Barret, esofagitis, adenicarcinoma, datos de complicación (anemia ferropénica, pérdida de peso, 
tumoración gástrica, disfagia, vómito persistente) 
pHmetría: quienes    no respondan a manejo empírico o si no hubo lesiones endoscópicas. 
Impedancia esofágica: nos permite diferencia reflujo ácido de no ácido y es más S y E que pHmetría y manometría para detectar 
episodios.   
 
TRATAMIENTO 
Gold stándar: fundupliatura vía laparoscópica (Nissen) 
  Fracaso a menejo médico, joven de 25‐35 años, deseo del paciente.   
La funduplicatura no previene adenocarcinoma. 
 
PRONÓSTICO 
Escamoso==> columnar o cilíndroco (metaplasia o Barret) 
 
REFLUJOGASTROESOFÁGICO 
[r]HOMBRE DE 51 AÑOS DE EDAD QUIEN PADECE DESDE HACE 10 AÑOS ENFERMEDAD POR REFLUJO GASTROESOFÁGICO 
TRATADA SÓLO DE MANERA PARCIAL, ES SOMETIDO A ENDOSCOPÍA, REPORTÁNDOSE EPITELIO ROSA SALMÓN ARRIBA 
DE LA UNIÓN GASTROESOFÁGICA. 
 
CON BASE EN EL HALLAZGO ENDOSCÓPICO LO MÁS PROBABLE ES QUE EL PACIENTE PRESENTA EN ESTE MOMENTO: 
A) ESOFAGITIS. 
B) CARCINOMA ESCAMOCELULAR. 
C) METAPLASIA. 
D) ACALASIA. 
 
CORRESPONDE AL TRATAMIENTO DE ELECCIÓN EN ESTE CASO: 
A) INHIBIDORES DE LA BOMBA DE PROTONES. 
B) ANTIHISTAMÍNICOS H2. 
C) ESOFAGECTOMÍA. 
D) FUNDUPLICATURA 
 
RESPUESTA C, A   
Se trata de paciente con ERGE es somentido a endoscopia y se muestras el hallazgo de epitelio color salmón.   
Hay que recordar que una metaplasia es el cambio de una células por otra similar, se dice que que hay displasia cuando 
hay pérdida de la arquitectura habitual de la célula. El esófago de Barret es una metaplasia que endoscópicamente luce 
como  una  sona  de  epitelio  color  salmón,  razón  por  la  cual  el  apciente  presenta  una  metaplasia.  Y  el  tratamiento 
conducente es con IBP.   
 
Por medio de endoscopia la esofagitis se puede clasificar en grados: 
  Grado I‐  Erosiones pequeñas, circulares, no confluyen. 
  Grado II‐  Erosiones lineales cubiertas por tejido de granulación que sangran fácil si se tocan.   
  Grado III‐ Erosiones lineales o circulares que confluyen para formar área circular sin epitelio 
  Grado IV‐ Estenosis (obligada la toma de biopsias) 
  Grado V‐Esófago de Barret 
 
Respecto al tratamiento hay que recordar que de primera elección están los IBP. 
  Displasia leve: IBP x 1‐2meses y repetir endoscopia. 
  Displasia de alto grado: vigilancia estricta endoscópica c/ 3 meses. 
Los inhibidores H2 solo están indicados junto con funduplicatura en pacientes con displasia leve y después del tratamiento 
inicial con IBP. 
Esofagectomía inciada en jovenes que no están dispuestos a control endoscópico trimestral.   
La funduplicatura está inciada en quienes no teleran los IBP o no responden a ellos.   
 
MUJER DE 38 AÑOS DE EDAD, QUE ACUDE POR PRESENTAR DESDE HACE TRES AÑOS PIROSIS, REGURGITACIÓN Y DISFAGIA 
PROGRESIVA QUE ACTUALMENTE ES A SÓLIDOS. SE REALIZA ENDOSCOPÍA, LA CUAL REPORTA ESOFAGITIS CON ESTENOSIS 
A NIVEL DEL TERCIO MEDIO DEL ESÓFAGO. 
  
CON BASE EN EL REPORTE ENDOSCÓPICO LA ESOFAGITIS REPORTADA DEBERÁ CONSIDERARSE: 
A) GRADO I. 
B) GRADO II. 
C) GRADO III. 
D) GRADO IV. 
 
EL DIAGNÓSTICO NOSOLÓGICO DE BASE MÁS PROBABLE ES: 
A) ACALASIA. 
B) PSEUDOACALASIA. 
C) ENFERMEDAD POR REFLUJO GASTROESOFÁGICO. 
D) ENFERMEDAD DE BARRET. 
 
Los  hallazgos  endoscópicos  se  clasifican  según  Savary‐Miller.  Importante  recordar  que  la  estenosis  esofágica  es 
característica de un grado IV, aunque podríamos encontrarlo en grado III también.   
RESPUESTA    D, C 
 
MUJER DE 58 AÑOS DE EDAD, CON ANTECEDENTE FAMILIAR DE ESÓFAGO DE BARRET Y ALCOHOLISMO DESDE HACE 25 
AÑOS,  ACUDE  A  LA  CONSULTA  GENERAL.  DURANTE  EL  INTERROGATORIO  LA  PACIENTE  REFIERE  SINTOMATOLOGÍA 
PERSISTENTE  DE  ENFERMEDAD  POR  REFLUJO  GASTROESOFÁGICO  QUE  REMITE  POR  ALGUNOS  DÍAS  DESPUÉS  DE 
TRATAMIENTO  ALTERNATIVO.  EN  SU  ÚLTIMA  CONSULTA  REFIERE  DISFAGIA  DE  1  MES  DE  EVOLUCIÓN.  SE  SOLICITA 
INTERCONSULTA PARA ENDOSCOPÍA. 
 
EL SIGUIENTE FACTOR JUSTIFICA LA REALIZACIÓN DE UNA ENDOSCOPÍA: 
   
A) LA PRESENCIA DE DISFAGIA. 
B) LA EDAD MAYOR A 50 AÑOS. 
C) EL ALCOHOLISMO DE 25 AÑOS DE EVOLUCIÓN. 
D) EL ANTECEDENTE FAMILIAR DE ESÓFAGO DE BARRET. 
 
La  realización  de  endoscopia  en  paciente  con  ERGE  su  justifica  si:  1)  sospecha de  estenosis,  2)  esófago  de  Barret  o  3) 
adenocarcinoma de esófago. O en aquellos que presenten disfagia,    vómito persitente,    hemorragia gastrointestinal, 
anemia ferropénica, pérdida de peso sin razón aparente o tumoración epigástrica.   
RESPUESTA A 
 
 
 
 
 
ESÓFAGO DE BARRETT 
DEFINICIÓN 
Metaplasia de escamosos ==> cilíndrico==> displasia ==> adenocarcinoma 
 
PATOGENIA 
Reflujo ácido==> esofagitis erosiva==> cicatrización 
 
DIAGNÓSTICO 
Endoscopia: epitelio rosa salmon (Barrett) 
Esofagograma: hernias hiatales, estenosis, ulceraciones.   
 
TRATAMIENTO 
Esófago de Barret metaplásico: IBP o funduplicatura* 
  *IBP eliminan síntomas pero no garantizan el control del refliujo ácido. 
  *IBP no eliminan relujo biliar (causa importa de Barret) 
  *Funduplicatura promueve regresión de epilio cilíndrico 
Esófago de Barret con displasia de bajo grado IBP x 3 meses a triple dosis. ==> endoscopia y biopsia. 
  si se confirma continuar con IBP o funduplicatura* 
  *revisión endoscópica cada 6m‐12m 
Esófago de Barret con displasia de alto grado. 
  1) Revisión endoscópica cada 3 meses, biopsia de 4 cuadrantes de cada centímetro de Barret.   
  Después de displasia de alto grado tarda aprox 5 años a progresar a Ca el 50% de pacientes. 
  2)Esofagectomía en jóvenes 
    ‐30% de las displasia tienen Ca   
    ‐50% desarrollar Ca durante la vigilancia endoscópica 
    ‐En centros especializados hay muy buenos resultados.   
  
Antihistamínicos H2? 
Buen día PROEDUMED, en el subtema esófago de Barret, apartado de Manejo Terapéutico dice: "el enfermo podrá seguir con 
antihistamínicos H2". ¿Con anthistiamínicos H2 se refiere a inhibidores H2 como ranitidina? 
¿Está inciada para el manejo de esófago de Barret? 
 
ENFERMEDAD ÁCIDO PÉPTICA Y SUS COMPLICACIONES 
ÚLCERA PÉPTICA 
 
 
DEFINICIÓN 
Daño necrótico de la mucosa que supera la muscular de la mucosa, secundaria a pepsia y ác. clorhídrico. 
  Úlcera péptica complicada aguda: comprometa la vida (sangrado o perforación) 
  Úlcera péptia complicada crónica: obstrucción? 
 
SALUD PÚBLICA 
Perforación 2‐10 de úlcera pépticas. 
Una de las 3 causas comunes de hemorragia del tubo digestivo. 
FR para sangrado: AINES, edad avanzada, H. pylori, alcohol y tabaco. 
 
PATOGENIA 
H pylori y AINES son causa de la mayoría de úlcera duodenales, ác gástrico lesiona mucosa pero no tiene efecto primordial. 
Úlceras duodenales: secrecion de HCO reducida, secreción ác basal y nocturna aumentada. H pylori podría estar asociada. 
Úlceras gástrias: Puede estar productica por H pylori o AINES, 
 
DIAGNÓSTICO 
Sospecha de sangrado de úlcera:==>endoscopia las    1eras 24 hrs o colocacion de SNG con lavado gástrico con solución a temp 
ambiente 
  Antecednte de enf ác. péptica 
  Hematemesis y melena 50% 
  Hematemesis 30% 
  Melena 20% 
  Hematoquecia 5% 
  Sx anémico 
Sospecha de perforación==>1 era TAC, sino placas simples y contrastadas con material HIDROSOLUBLE 
  Antec de enf ác. péptica 
  Dolor transflictivo, súbico abdomen alto, aumenta con movimientos, se irradia a abdomen bajo, hombros.   
  Datos de iritación peritoneal 
  Choque, sepsis 
Obstrucción secundaria de úlcera péptica (x edema y estenosis) ==>1eropanendoscopia con toma de biopsia, 2do serie EGD 
  Antec de enf ác péptica 
  Infección por H pylori no tratada 
  Desnutrición y pérdida de peso 
  Vómits postprandiales 
  Alcalosis metabólica hipoclorémica e hipokalémica 
  Plenitud postprandial y saciedad temprana 
  Distensión abdominal 
  Masa epigástrica timpánica 
 
 
TRATAMIENTO 
Sangrado: líquidos, paquete globular si pérdida >30% 
  IBP 80mg IV en bolo, 8mg7gr x 72hrs, 40mg cada 24 x 3 semans (una vez dada erradicación) 
  Endoscopia temprana con método hemostático (sin urgencia vital) 
  Cirugía de emergencia si hay compromiso vigtal o si >2cm. 
Perforación: Cristaloides, AB de amplio espetro, SNG a succión. 
  Cirugía abierta si urgencia o perforación >6mm 
  Irrigación caviad peritoneal con 30lts de sol salina 
  SNG 48hrs 
Obstrucción pilórica: Corregir anormalidaes HE, desequilibrio A/B, nutrición parenteral total, erradicación,    drenaje. 
  Erradicación como primer aopci´n, pensar dilatación con balón. 
 
 
 
 
 
 
 
 
 
 
 
 
 
 
 
 
 
 
Erradicación 
1era elección: Claritromicina 500m c12, amoxoicilina 1gr cada 12, Omepraz 40mg cada12 x 14 días.   
2da elección: tinidazol 1gr c 12, tetraciclina 500mg c6, bismuto 525mg cada 6, omepra 40mg    cada 12 x 14 días.   
3era elección: azitromicina 500mg x 3 dias, omeprazol 40mg c 24 con furazolidona 200mg c8    x 10 días. 
PRONÓSTICO 
Mortalidad por perforación 5‐30%. 
 
RESPUSTAS 
18D, 19C, 20B 
COLECISTITIS, COLELITIASIS Y SUS COMPLICACIONES 
COLECISTITIS 
 
DEFINICIÓN 
Colecistitis es inflamación de la vesícula generalmente por litos, lodo biliar y en raras ocasiones ninguna de estas.   
Colelitiasis es la presencia de litos en vesículas. 
 
SALUD PÚBLICA 
M2:1H 
FR: mujer, >40 años, obesidad, embarazo, fármacos (fibratos, ceftriaxona), enf del íleon, dislipidemias, enf hepáticas, enf metabólicas, 
AO, terapia hormonales sustitutiva con estrógenos. 
 
PATOGENIA 
80% son mixtos y de colesterol (>50% de colesterol, sales cálcicas, pigmentos) 
20% son pigmentarios (bilirrubinato cálcico y <20% colesterol) 
 
Mecanismos   
Sobresaturación de bilis por colesterol 
Aumento de la actividad de la hidroximetilglutaril CoA reductasa. 
Incremento de la captación hepática de colesterol en sangre.. 
Menor conversión hepática de colesterol en ác biliares 
Def CYP7A1==> def de 7 hidroxilasa de colesterol. 
Mutación gen MDR3 codifica para bomba de exportación de fosfolípidos    a la bilis (estos disminuyen y aumentan colesterol) 
 
Nucleación    de colesterol 
Nucleación rápida del colesterol el la bilis (exceso de factores pronuecleadores o déficil de factores antinucleadores. 
 
Retraso de vaciamiento y estasis biliar 
Hipomotilidad vesicular 
 
 
 
 
Cálculos de pigmento: hemólisis crónica, cirrosis alcohólica, anemia perniciosa, fibrosis quística, infección crónica de vías biliares, 
envejecimiento, enfermedad de íleon, extirpación o derivación de tal segmento. 
 
DIAGNÓSTICO 
Colecistitis/colelitiasis: Murphy, dolor CSD, masa CSD, resistencia muscular CSD, náusea, vómito. 
Colecistitis/colelitiasis agudas: vesícula palpable, >39°, escalocríos, inestabilidad hemodinámica.   
Colasco y peritonitis: irritación abdominal, distensión abdominal, taquicardia, taquipnea, acidosis metabólica, hipotensión, choque 
 
Si agudo precisa un síntoma local y un síntoma sistémico, USG que lo sugiera. BHC reportará leucocitosis, PCR ayudará a corroborar que 
hay inflamación.   
Amilasa sérica útil para identificar una coledocolitiasis como complicación. 
 
USG reportará colelitiasis aguda: 
  Pared vesicular >5mm 
  Líquido perivesicular 
  Murphy USGnográfico 
  Alargamiento vesicular 8cm axial, 4cm diametral. 
  Lito encarcelado 
  Imagen de doble riel 
  Sombra acústica 
  Ecos intamurales 
Gammagrafía biliar solo si no es concluyente el USG y la clínica 
TAC 
  Engrosamiento y alargamiento vesicular 
  Alargamiento vesicular 
  Áreas de alta densidad en tejido graso perivesicular 
RMN 
  Alargamiento vesicular 
  Engrosamiento de la pared. 
Gammagrafía con Tc HIDA 
  Exclusión vesicular 
  Signo de RIM 
 
5‐10% alitiásica (quemados, traumatisados, puerperas, postoperados, NPT prolongada, ==> imagen vesícula tensa, estática, agrandada 
sin cálculos. 
 
TRATAMIENTO 
 

 
 
Ác. ursodesoxicólico, quennodeoxicolico x 1‐2 años. 
 
Diclofenaco 75mg, meperidina, antibiótico.   
 
Litotripsia si: lito único, no calcificado, diámetro 20‐30mm, sin contraindicaciones (panreatitis, alt de la coagulación, quistes o 
aneurismas en el trayecto de las ondas de choque). 
 
Grado I  Cole temprana laparoscópica 
Grado II  Cole temprana laparoscópica, si inflamación grave drenaje percutáneo 
Grado III  Manejo urgente de falla orgánica, drenaje, desinflamar, mejorar condiuciones, colelaparoscópica. 
 
Complicaciones de cole laparoscópica: lesión del conducto biliar, intestino e hígado.   
Complicaciones de cole abierta: infecciones, íleo, hemorragia intraperitoneal, atelectasia, trombosis venosas profundas, IVU. 
 
Si coledocolitiasis: colelap+CPRE en el mismo ingreso pero días después. 
 

 
 
 

 
 
 
SANGRADO DE TUBO DIGESTIVO ALTO Y BAJO 
HEMORRAGIA DIGESTIVA INFERIOR 
DEFINICIÓN 
Aquella que se produce distal al ligamento de Treitz (entre el intestino delgado y el ano). Generalmente x malf vasculares, divertículos y 
neoplasias. 
 
SALUD PÚBLICA 
Aprox 65 años. 
 
PATOGENIA 
Etiología: Diverticulosis. Angiodisplasia, Neoplasia, Enf perianales, Diverículo de Meckel, Colitis    (Infecciosa, no infecciosa, otras). 
 
Diverticulosis: causa entre 20‐55%. Aunque slo del 3‐15 sangran (muchas diverticulosis no sangran). El sangrado de los divertículos es 
como hematoquezia. 70% en colon izquierdo, pero el sangrado es más común en el derecho 50‐90%. 
Angiodisplasia: causa de 3‐12% de STDB, principal causa de sangrado gradual e intermitente, el 66% son >70 años, 45% da sangrado 
constante pero imperceptible, afecta más colon derecho. 
Neoplasias: raramente por si solos sangran, con frec asocian divertículos. 
Enf perianales: Hemorroides (en general no producen dolor cuando sangran, prurito, si se trombosa hace coágulo , ulcera y sangra),   
fístulas perianales (fred x traumatism por defecación, dolor, comezón, rectorragia). 
Divertículo de Meckel: con frec en intestino delgado, sangra más en niños que en adultos, puede confundirse con peritonitis. 
Colitis: infecciosa generalmente cuasada por amibiasis, disentería, tuberculosos, campylobacter, E.coli, clostridium difficile. 
 
DIAGNÓSTICO 
Se sugiere realizar una colonoscopia a todos los pacientes con sangrado de tubo digestivo bajo. Es diagnóstica y terapéutica. 
Glóbulos rojos marcados con TC99: determina hemorragias >0.1ml/min, limitado prque no prmite identifiar la causa del sangrado. 
Angiografía: detecta hemorragias de 0.5ml/min. S 30‐50%, E 100%. 
Tomografía: detecta causa y extensión de algunas alteraciones, uso limitado porque no siempre hay tomógrafo.   
Estudio de video‐cápsulas: identificación del sitio 55‐65%, contraindicaci´n es el uso de marcapasos. 
 
Gold standar en el diagnóstico de angiodisplasias: colonoscopia. 
 
TRATAMIENTO 
El de cada uno en particular.   
 
PRONÓSTICO 
Tasa de mortalidad 2‐4% 
 
WEBINAR 
Pérdidas de tubo digestivo 
10%  SV normales 
10‐20%  15mmHg por debajo, piel fria, piloereccion 
20‐25%  Sistolicas <80, frialdad de piel, tegumentos choque 
 
Antes de cualquier cosa: primero lo reestableces. 
Choque III= IC a UCI 
Choque II‐III Venas permables en ambas extremidades + catéter venoso central 
 
Definir si variceal o no.  PFHy antecedentes 
Tx estabilizar 
Endoscopia + ligadura de várices 
Si es no variceal SNG 200ml de sol fisiológica 5‐10%.  endoscópica 
PATOLOGÍA ABDOMINAL Y RESOLUCIÓN QUIRÚRGICA DE URGENCIA 
ABDOMEN AGUDO 
[Resumen la Salle] 
 
GENERALIDADES 
Abdomen agudo: dolor abdominal de instauración reciente (generalmente menos de 48 hrs) que no sobrepasa los 6 días. Con 
repercusión al estado general. Requiere diagnótico rápido y preciso ante la posibilidad de que sea susceptible a tratamiento quirúrgico 
urgente.   
Tipos de dolor 
1) Somático (o visceral) 
Las fibras C lo transmite. Al ser fibra amielínicas el dolor es difuso y poco localizado. 
 
2) Parietal 
El dolor pArietal surge cuando se irritan las fibras A del peritoneao, es entonces cuando el dolor se localiza. 
 
3) Referido 
 
La ubicación del dolor parietal en el áre abdominal puede orientar a la patología de origen de abdomen agudo:   
Cuadrante superior derecho: pancreatitis aguda, hepatitis aguda, hepatomegalia congestiva aguda, neumonia con irritación 
pleural, pielonefritis aguda, angor, absceso hepático. 
Cuadrante superior izquierdo: pancreatitis aguda, aneurisma aórtico, colitis aguda, pielonefritis, neumonía e IAM. 
Periumbilical: obstrucción intestinal, pancreatitis aguda, diverticulitis. 
CID: EPI, salpingitis, quiste ovárico complciado, embarazo ectópico, apendicitis aguda.   
CII: diverticulitis, EPI, salingitis, quiste de ovario complicado, embarazo ectípico roto, tumor o cuerpo extaño en colo 
descendente, litiasis reno‐ureteral.   
 
30% de los abdómenes agudos corresponden a situaciones que se resolverán quirúrgicamente. La principal causa es apendicitis. 
 
DIAGNÓSTICO 
 
¿Ciuáles son las pruebas iniciales en abdomen agudo? QS, BHC, EGO, ES, TP/TTP, lipasa‐amilasa y PFH si duele tercio superior del 
abdomen, PIE si mujer en edad fertil y duele tercio inferior del abdomen, tele de tórax, rx simple de abdomen de pie y decúbito.   
 
Estudios no invasivos correspondientes a la sospecha: USG, TC con doble contraste, RMI. 
Estudios invasivos: laparoscopia, laparotomía exploradora. 
 
El dolor comienza en epigastrio si hay afección a esófago, estómago y vesícula.   
El dolor comienza en el mesogastrio y se irradia postriormente a FID si hay afección a apéndice.   
 
Contraindicaciones absoltas para laparoscopia diagnóstica:   
  Choque      Obstrucción 
  Sepsis      Trastornos graves de coagulación 
  Peritonitis      Tumores gigantes 
  Hernia externa irreductible  Enf cardiorrespiratorias severas 
 
Contraindicaciones relativas de laparoscopia diagnóstica: 
  Cirugía previa    Hernia hital 
  Infección de pared abdominal  Cardiopatía isquémica 
  Hipertension portal    Trastornos de coagulación corregida 
  Obesidad extrema    Paciente no cooperador 
 
Complicaciones de laparoscopia son: 
  Hiperextensión de plexo braquial       Enfisema 
  Insuflación extraperitoneal inadecuada      Embolia grasa 
  Trauma de contenido abdominal, intestinos, vasos.    Neumotórax o enfisema mediastínicos 
  Fascitis necrosante 
Paresia o parestesia de miembros inferiores por compresion de region poplítea 
 
ESCOLAR DE 8 AÑOS QUE ES VALORADO EN URGENCIAS CON DOLOR DE 6 HRS DE EVOLUCIÓN, INICIADO EN OMBLIGO Y POSTERIOR 
APARICIÓN EN CUADRANTE INFERIOR DERECHO, LOS PADRES LE REFIEREN QUE NO HA PRESENTADO FIEBRE, CON ATAQUE LEVE AL 
ESTADO GENERAL, ASTENIA Y ANOREXIA. 
 
¿CON QUÉ ESTUDIO APOYA SU DIAGNÓSTICO? 
A) TELE DE TÓRAX 
B) PLACA DE ABDOMEN 
C) USG ABDOMINAL 
D) TC ABDOMINAL 
 
ES EN ESTE ESTUDIO QUE SE PUEDE ENCONTRAR UN DATO QUE SE REALCIONA DE MANERA MÁS SENSIBLE Y ESPECÍFICA CON SU 
DIAGNÓSTICO. ¿CUÁL ES? 
A) SIGNO DE MCBURNEY 
B) IMAGNE DE TIRO AL BLANCO (DIANA) 
C) IMAGEN RADIO‐OPACA EN FID 
D) SIGNO DE DOBLE BURBUJA 
 
¿ES EL SÍNTOMA MÁS CONSTANTE HALLADO EN ESTA PATOLOGÍA DESPUÉS DEL DOLOR? 
A) ASTENIA 
B) FIEBRE 
C) ANOREXIA 
D) NÁUSEA 
 
¿Cómo debemos interpretar las preguntas en los casos clínicos? En el caso clínico correspondiente a abdomen agudo, nos preguntan 
“con qué estudio apoya su diagnóstico”, ¿nos están pidiendo estudio inicial, de primera elección o el gold standard? 
 
La imagen radio‐opaca en FID es patognomónica pero solo está presenta en 8% de los casos.   
La imagen de tiro al blanco (diana) es la imagen patognomónica por USG, esta imagen esta dada por el edema de pared y no debe 
exceder los 7mm, de ser así es diagnóstica. Como el caso clínico está haciendo alusión a la placa de abdomen no podrías resonder de 
esta forma.   
 
De todos los síntomas de apendicitis aguda, el 85% son generales de irritación peritoneal (fiebre, astenia, dolor, náuseas), 15% son 
propios de apendicitis aguda y el más freucente es anorexia‐hiporexia (aparece en el 80‐90%).   
RESPUESTAS B, C, C 
APENDICITIS 
[Resumen la Salle] 
DEFINICIÓN 
Abdomen agudo: dolor que no sobrepasa los 6 días. 
Apendicitis aguda: inflamación, obstrucción, moco, pérdida de elasticidad.   
Apendicitis simple: apéndice inflamado, sin gangrena ni perforación o absceso periapendicular.   
 
SALUD PÚBLICA 
Frecuente entre 10‐30 años. Rara en extremos de la vida. 
 
Grupo etáreo:   
Escolares (por tener más tejido linfático), las IVRS activan el tejido linfático. En el apéndice la luz es de 1mm y la hiperplasia linfática es la 
principal causa. 
Adultos jóvenes: fecalitos, otros (semillas, ascaris) 
En la mayoría el fenómeno inicial es la ulceración de la mucosa, se propone etiología vírica, infección por Yersinia (39% hay evidencia). 
Aumento de folículos linfoides causados por infección vírica (sarampión), bario concentrado, gusanos (oxiuros, Ascaris, Taenia) o 
tumores (carcinoides    carcinoma), pueden obstruir la luz.   
 
 
50% de los abdómenes agudos operables son apedicitis. 
20% de las laparotomías son blancas (apendicectomías incidentales) 
30% se perfora. 
Para minimizar perforación el cirujano puede favorecer laparotomía temprana pese a que el 20% de los laparotomizados son normales 
 
PATOGENIA 
Anatomía del apéndice 
La confluencia de las tenias no permite encontrar el apéndice. 
  Mesoapéndice 
  Arteria apendicular==>    ileocólica==> mesentérica superior 
  No tiene vena propia==> Vena ileocólica ==> mesentérica inferior==> porta==> absceso hepático piógeno 
  Ganglios linfáticos ileocólicos 
  Inervación simpática y parasimpática 
 
2/3 son retrocecal (indica donde está la punta) 
1/3 es pélvico 
Ectópico: subhepática==> pasa porque hay alguna alteración en la rotación intestinal en la embriogénesis, estos pacientes no tienen 
colon derecho. 
 
Isquemia==> edema, moco==> pus por bacterias==> destrucción de pared apendicular. 
1) linfáticos se detienen, 2) drenaje venoso se detiene, 3) flujo arterial se detiene.   
Si empeora==> peritonitis 
El epiplón fija y trata de bloquear el proceso y forma un plastrón. En pediátricos el epiplón no está bien desarrollado razón por la cual se 
tiende a complicar más fácilmente. 
 
Apendicitis aguda puede ser la primera manifestación de enfermedad de Crohn.   
La frecuencia de apendicitis aguda recidivante aumentará conforme se disponga de AB con más libertad y queden muñones 
apendiculares largos como consecuencia del mayor uso de apendicetomía laparoscópica.   
 
DIAGNÓSTICO 
Cuadro atípico: edades extremas, en pediátricos el epiplón no está bien formado, cuando se modifica por medicamentos.   
 
SÍNTOMAS 
Abdomen agudo (cólico, localizado periumbilical, incremento rápido, antes de 24 hrs migra a CID, incrementa al toser o caminar.   
Náuseas y vómitos no muy numerosos. Puede haber fiebre. 
Si fuera muy largo el apéndice puede dar dolor en el otro lado (FII) aunque dolor FII se explica mejor por diverticulitis. 
   
Los tres datos pivote son:   
1 Dolor epigástrico con migración a CID 
  2 Datos de irritación peritoneal (hipersensibilidad FID, rebote, defensa, rigidez) 
  3 Datos de respuesta inflamatoria (leucocitosis con predominio de neotrófilos) 
 
Sospecho apéndice si la clínica aparece en el siguiente orden: Dolor ==> náusea ==> vómito   
Otros datos anorexia, fascies dolorosa, taquicardia, posición en gatillo. 
 
Descarto apéndice:    Dolor que no migra 
    Dolor en FII     
    FID, dolor que no sigue secuencia (ej. náusea==> vómito==> dolor) 
EF:   
Hipersensibilidad FID 
Defensa muscular involuntaria,   
McBurney     
Rovsing    Dolor en FID al presionar FII 
Cope [psoas]    Aumento del dolor en FID al flexionar activamente la cadera derecha.   
Cope [obturador]  Dolor en hipogastrio al flexionar el muslo derecho y rotar la cadera hacia adentro.   
Summer    Aumento de tensión de los músculos abdominales a la palpación superficial de FID. 
Von Blumber  Rebote   
Talopercusión  Percusión en talón derecho y dolor en FID. 
Dunphy:      incremento del dolor FID con tos. 
Para apendicitis retrocecal: 1) Capurro (pellizcar espina iliaca anterosuperior para adentro) 2) As Schultze?: es para afuera 
 
PARACLÍNICOS:   
BHC: Leucos 10,000‐18,000 con neutrofilia (ocasionalmente bandemia). Si solo presentan 1 o 2, pedir EGO, BHC y PIE. 
Prueba triple (PCR >8mcg/ml, leucocitos >11,000, neutrofilia >76%) sugiere fuertemente apendicitis.   
PCR >55mg/l genera sospecha de apendicitis perforada.   
EGO patológico es para ver concomitancias, puede haber leucocitosis en pacientes con apendicitis. 
Rx: liquido libre (vidrio despulido). Es el estudio de imagen de inicio y para fines de examen, sigue USG y TAC.   
USG: hacer después de rx (sens 86, esp 96), imagen en diana y líquido periapendicular 
TAC: gold standar si sospecho de apendicitis pero queda la duda pese a USG. En embarazadas se usa siempre y cuando >20sdg. 
 
Rx es el estudio de imagen de inicio y para fines de examen sigue USG y TAC.   
 
I    Congestiva   Hiperemia + edema 
II  Supurativa   Hiperemia + edema + natas fibrinopurulentas 
III  Necrótica    Hiperemia + edema + natas fibrinopurulentas + necrosis segmentaria 
IV  Perforada    Perforada, peritonitis focal.   
 
Bacteroides fragilis principal asilado.   
 
En pediatría: 6‐10años, M1:F2, dolor abdominal náusea  vómitofiebre. En lactantes hay diarrea. Náusea y vómito antes del 
dolor nos da la sospecha de apéndice retrocecal ascendente. En múltiples ocasiones el paciente puede cursar afebril. Leucocitosis 
>15,000 con neutrofilia. (Si hay sospecha de IVU tendrá >20 leucos + nitritos positivos,) Niños con 1, 2, 3==> USG ==> TAC. 
 
En embarazadas: Primera causa quirúrgica en el embarazo es apendicitis, es más frecuente en el tercer trimestre, signo pivote es dolor 
en flanco o hipoconrio derechos, la Qx se hace donde le duele, si es en el1er trimestre el porcentaje de pérdida del producto es del 30‐
50% (esta última cifra si se perfora, y en promedio sin importad edad 30%). En segundo trimestre asocia pretérmino en 11%. La fiebre y 
taquicardia están presentes en cuadros apendiculares complicados con perforación o absceso apendicular. 
Leucocitosis >16mil, con neutrofilia y bandas.     
 
En adulto mayor: atípica, insidiosa, dolor poco intenso, distermia, más de 3 días, distensión, disminución de ruidos intestinales, 
meteorismo, parálisis intestinal, poca defensa abdominal, cuadro confusional agudo, generalmente no hay leuocitosis, alt 
hidroelectrliticas, aumento de creatinina. Adultos mayores con 1,2,3  TAC 
 
Anciano con ileo paralítico y alt hidroelectrolíticas, pensar que puede ser secundario a apendicitis.   
 
En los viejitos con frecuencia se complican porque no sienten tanto, sin embargo, el síntoma pivote es la diarrea (menos frec, náusea y 
vómito) 
  Ej.  Diarrea==>  medico  gral==>  GEPI==>  BHC  no  elevaciones==>ES  alterados==>  Tx  para  reponer  ES  ==>  lo  complica  más==>  Cr  está 
elevada. 
 
No es el dolor el síntoma pivote en ancianos.   
 
"Joven que sangra en la adolescencia, con dolor en fosa iliaca derecha y mesogastrio (pienso en Meckel)" 
Gold standar diagnóstico de Meckel: gammagrafía con TC‐99 
 
DIAGNÓSTICO DIFERENCIAL 
Embarazo ectópico (mujer en edad fértil), quimioterapia?, si opero y descubro ectópico (aunque esté perforado), se mitiga el sangrado, 
se realiza curetaje y se conserva la tuba. 
Solo si está en cuerno de la trompa se realiza histerectomía.   
 
 
TRATAMIENTO 
Tratamiento de elección apendicectomía laparoscópica o abierta dependiente del paciente y el cirujano. 
 
Si hay duda: observación corta y laparoscopia.   
Apendicectomía laparoscópica contraindicada en <5 años y adulto mayor. 
Laparoscópica reduce complicaciones postoperatorias en infección y de íleo paralítico en niños.   
Laparoscópica ideal en fértiles. 
 
En neonato y lactante: lo invaginan, dan un punto, se esfacela y lo evacúan C: 
 
Apéndice blanco o normalapendicectomía incidental 15% 
Si rebasa el 15% quiere decir que no se está diagnosticando bien. Se meten muchos sanos. Si el promedio baja quiere decir que estamos 
dejando ir apenditis y que se están complicando.   
 
Si quito apéndice sano, reviso si hay Meckel (remanente de onfalomsentéricos, tx qx con divertículectomía en cuña para no dejar 
mucosa gástrica o pancreática y que pueda sangrar). aunque esté sano lo quito, porque puede sangrar al tener mucosa gástrica y 
pancreática o puede hacer hernia de Litré.     
 
Complicación de cirugía complicada: fiebre y diarrea continua TAC  absceso interasa. (absceso residual) 
Indicadores de drenaje cerrado a presión negativa (VAG, drenovag] : cirugía pancreática, urológica, pélvico    y absceso localizado. 
Penrose está proscrito. 
 
Peritonitis fecal tiene un 40% de infección de la herida por ser contaminada. 
Al 5to día la infección da clínica, si no la hay puede suturar o poner vendoletes (permabond‐resistol), dejo abiertas las perforadas.   
 
[meterzmer. Herida limpia 10%, limpia contaminada 20%, contaminada 30%, sucia 40%] 
 
Abordaje si inicial y no complicada: laparoscípica 
Abordaje si complicación o peritonitis: Rocky Davids, McBurney. 
Vía oral 6‐12 hrs si no complicada 
Ayuno hasta que el íleo se quite si fue complicada.   
 
Antibióticos:   
  I  Cefalosporina (cefalotina, cefalexina) preoperatoria + 2 dosis postqx 
  II  Cefalosporina (cefalotina) preoperateoria + 3 dosis postqx 
  III  Cefalosporina (cefalotina) preoperatoria + 7 dosis postqx 
  IV  Cefalosporina (cefalotina) preoperatoria + 10 dosis postqx + metronidazol [Acorde a la Salle] 
 
Antibióticos: cefoxitina 2gr IV al momento de la inducción antestésica (pero puede ser antes, durante y/o después) 
Alt: cefazolina 1‐2grs IV + metronidazl 500mg IV DU.    o  cefotaxima si hipersensisbilidad amikacina.    [Acorde a proedumed] 
 
CONTENIDO ADICIONAL CLAVE 
Desviación a la izquierda: neutrófilos, agudo 
Desviación a la derecha: crónico 
 
Inicial 
abordan estudios simple complejo   
menos ivasivo mas invasivo 
BHC, qS, ES; tele de torax, palca de abdomen. 
 
Estudio siguiente 
usg 
 
Ideal, gold estandard, corrobora dx,   
TAC 
 
FEMENINA DE 25 AÑOS DE EDAD QUE HACE 24 HRS PRESENTO DOLOR PERIUMBILICAL, QUE POSTERIORMENTE SE IRRADIA A FOSA ILIACA 
DERECHA.  FIEBRE  CUANTIFICADA  DE  38.5°C.  EL  DÍA  HOY  SE  INTENSIFICA  EL  DOLOR  VOLVIÉNDOSE  INCAPACITANTE.  NIEGA 
SINTOMATOLOGÍA URINARIA. AL LLEGAR AL HOSPITAL PRESENTA 2 VÓMITOS DE CONTENIDO GASTROALIMENTARIO. A LA EXPLORACIÓN 
SE  ENCUENTRA  FEBRIL  DE  38.7°C,  FASCIES  DOLOROSA,  TAQUICARDIA,  DOLOR  EN  FOSA  ILIACA  DERECHA  A  LA  PALPACIÓN  Y 
DESCOMPRESIÓN. SE SOLICITAN LABORATORIOS QUE REPORTAN HEMOGLOBINA DE 12, LEUCOCITOS DE 15000. LA RADIOGRAFÍA DE 
ABDOMEN MUESTRA BORRAMIENTO DEL PSOAS DERECHO. 
 
EL DIAGNÓSTICO CLÍNICO MÁS PROBABLE ES:   
A) APENDICITIS AGUDA. 
B) QUISTE DE OVARIO TORCIDO. 
C) EMBARAZO ECTÓPICO. 
D) TIFLITIS. 
 
EL ESTUDIO QUE SERÍA MAS ÚTIL PARA CONFIRMAR EL DIAGNÓSTICO ES: 
A) PRUEBA INMUNOLÓGICA DE EMBARAZO. 
B) TOMOGRAFÍA AXIAL COMPUTADA. 
C) ULTRASONOGRAMA. 
D)RESONANCIA MAGNÉTICA. 
 
EL TRATAMIENTO QUE DEBE INDICARSE EN ESTA PACIENTE ES: 
A) LAPAROSCOPÍA EXPLORADORA. 
B) CISTECTOMÍA Y/O SALPINGOOFORECTOMÍA. 
C) APENDICECTOMÍA. 
D) ANTIBIOTICOTERAPIA ENDOVENOSA Y REANIMACIÓN CON LÍQUIDOS. 
 
RESPUESTAS A, C, C 
Datos semiológicos clave, el dolor apendicular inicia en epigatrio e irradia a FID, dolor que no migra habrá que dudar de apendicitis.   
Genermente incia con anorexia, dolor y vómito, cuando este orden se altera también habrá que dudar de apendicitis.   
La Rx aunque no es el estudio de gabitene confirmatorio si es el inicial y podremos encontrar escoliosis antiálgica, borramiento del psoas 
derecho, nivel hidroaéreo en cuadrante superior derecho, borramiento de la grasa preperitoneal, borramiento de la grasa prevesical, 
imagen de fecalito, asa centinela, etc. 
 
La  tiflitis  es  cuadro  que  se  caracteriza  por  inflamación  del  ciego,  colon  ascendente.  FR  neutropenia,  quimioterapia,  leucemia  aguda. 
Fisiopatológicamente puediera intervenir CMV, isquemia, hemorragia, infiltración paraneoplásica.    Dx: estudio de elección es la TAC 
(riesgo  de  perforación  x  colonoscopia  o  colon  x  enema)  Tx  reposición  de  líquidos,  electrolitos,  antibióticos,  NPT,  reposo  gástrico, 
analgésicos. Qx en caso de complicacines: hemorragia, perforación, obstrucción, absceso, necrosis, sepsis no controlada. 
 
La USG es el estudio de elección para corroborar apendicitis (S55‐96, E85‐98), se inditifica como un saco ciego a nivel ciego, aperistáltico. 
Con la compresión máxima se mide al diámetro anteroposterior del apéndice.   
Confirmatorios:    apéndice no compresible de 6mm o más y/o presencia de apendicolito establece el diagnóstico.     
Sugestivos: engrosamiento de pared apendicular y líquido periapendicular.     
USG reduce apéndice blancas del 37 al 13%. 
La resonancia es ideal si USG no concluyente en embarazadas ya que no podemos emplear TAC. ¿En embarazadas <20sdg? 
La TAC es el gold standar en caso de duda (luego de USG), es más cara y el paciente se expone a radiaciones, razón por la cual es de 
primera línea?.   
 
En embarazo ectópico se usa la salpingectomía lineal si es menor de 4cm y se localiza en la ampolla.   
La salpingectomía parcial o total se indica cuando el producto está en el istmo. 
 
Tiflitis==> TAC==> AB endovenosa y líquidos.   
Quiste ovárico torcido ===> Cistectomía y/o salpingooforectomía 
Embarazo ectópico==> prueba inmunológica==>Laparoscopia exploradora 
Apendicitis==> USG ==> apendicectomía 
 
300 mRADS es lo que genera el tomógrafo, lo permitido en el embarazo son 5 rads. 
 
[r]HOMBRE DE 34 AÑOS ACUDE AL SERVICIO DE URGENCIAS PORQUE DESDE HACE 24 HORAS PRESENTA DOLOR QUE COMIENZA EN 
MESOGASTRIO Y QUE POSTERIORMENTE SE LOCALIZA EN CUADRANTE INFERIOR DERECHO CON IRRADIACIÓN A MUSLO Y ESCROTO. SE 
ACOMPAÑA DE ASTENIA, ADINAMIA, HIPOREXIA, NÁUSEAS INTENSAS, VÓMITOS EN 3 OCASIONES Y TRES EVACUACIONES DISMINUIDAS 
DE CONSISTENCIA. A LA EXPLORACIÓN PRESENTA TEMPERATURA DE 37.9°C, TA 120/90 MMHG, FC 100/MIN, FR 20/MIN, ABDOMEN CON 
RESISTENCIA MUSCULAR VOLUNTARIA, DOLOR A LA DESCOMPRESIÓN SÚBITA EN EL CUADRANTE INFERIOR DERECHO DEL ABDOMEN. SE 
REPORTA  BIOMETRÍA  CON  HEMOGLOBINA  DE  13MG/DL,  LEUCOS  DE  13000,  70%  DE  NEUTRÓFILOS.  EL  EXAMEN  GENERAL  DE  ORINA 
REPORTA 15 A 20 LEUCOCITOS POR CAMPO, BACTERIAS +++. 
EL DIAGNÓSTICO MAS PROBABLE ES: 
   
A) LITIASIS URETERAL DERECHA. 
B) APENDICITIS AGUDA. 
C) COLITIS INFECCIOSA. 
D) INFECCIÓN DE VÍAS URINARIA 
 
EL TRATAMIENTO INDICADO EN ESTE CASO ES: 
A) ANALGÉSICOS, ANTIESPASMÓDICOS Y REPOSO. 
B) APENDICECTOMÍA. 
C) ANTIESPASMÓDICOS, ANTIBIÓTICOS. 
D) LITOTRIPSIA EXTRACORPÓREA POR ONDAS DE CHOQUE. 
 
RESPUESTA B, B 
Los diagnósticos diferenciales factibles por la clínica son litiasis ureteral derecha y apendicitis aguda. Apendicitis aguda tiene dolor que 
migra a otro punto y en la litiasis el dolor es irradiado más no migratorio. La apendictis puede hacer leucocitosis, en cambio la litiasis 
debería tener EGO con cristales y hematuria. 
Pensaríamos en IVU si tuviera en este contexto, >30 eritrocitos x campo y >20 leucocitos por campo   
 
HOMBRE DE 87 AÑOS DE EDAD, PRESENTA CUADRO DE DOLOR ABDOMINAL DE 24 HRS DE EVOLUCIÓN, INICIA EN OMBLIGO, SE RECORRIÓ 
A CID, LE SOLICITA LABS QUE DEMUESTRAN LEUCOCITOS DE 12 MIL, HB DE 9.6 CON NEUTROFILIA DE 77% Y SIGNO DE REBOTE POSITIVO.   
¿QUÉ ESTUDIO DE IMAGEN DECIDIRÍA TOMAR? 
A) USG 
B) RX AP Y LATERAL   
C) TAC 
D) RMN 
 
LA PRUEBA TRIPLE PARA LA POBLACIÓN GENERAL PARA ESTA PATOLGÍA INCLUYE: 
A) CLINICA, LABORATORIOS Y USG 
B) PCR, VSG, LEUCOCITOS 
C) FECALITO, BORRAMIENTO DE PSOAS Y ASA CENTINELA 
D) LEUCOCITOSIS, PCR ELEVADA Y NEUTROFILIA 
 
SI SE TRATARA DE UNA PACIENTE EMBARAZADA, ¿HASTA CUANTO PUEDE DECIR QUE ES NEUTROFILIA FISIOLÓGICA POR LA GRAVIDEZ? 
A) 11MIL 
B) 15MIL 
C) 16 MIL 
D) 18 MIL 
 
RESPUESTAS CDC 
En el adulto mayor las causas de dolor abdominal por orden de frecuencia son: enf biliares, obstrucción intestinal, tumores y  causas 
vasculares.  La  frecuentes  de  apendicitis  en  el  anciano  es  de  3‐10%  del  total  de  las  apendicitis.    En  anciados  pedir  EGO,  BHC,  ES, 
creatinina, ES, rx simple de abdomen, de pie y decúbito, rx de tórax.   
 
La prueba triple sugiere la presencia de apendicitis ante un cuadro clínico susgestivo: PCR >8mcg/ml, leucocitosis >11,000 y neutrofilia 
>75%. Es especialmente útil en niños.   
La leucocitosis puede estar levada de manera normal en el embarazo en niveles hasta por arriba de 16mil cel/ml. 
 
Niños  con  cualquiera  de  las  3  manifestaciones  cardinales  (dolor  caracterísitco,  datos  de  irritación  peritneal,  datos  de  respuesta 
inflamatoria)  solicitar USG, si USG indeterminado TAC 
Adultos mayores con cualquiera de las 3 manifestaciones cardinales ==> TAC 
 
[r]HOMBRE    DE  21  AÑOS  DE  EDAD.  ACUDE  A  URGENCIAS  POR  CUADRO  DE  1  DÍA  DE  EVOLUCIÓN,  CARACTRIZADO  POR  DOLOR 
ABDOMINAL INTENSO EPIGÁSTRICO Y PERIUMBILICAL, QUE POSTERIOMENTE MIGRÓ HACIA FOSA ILAICA DERECHA, NÁUSEAS, VÓMITO. 
A LA EF SE ENCUENTRA FC90 FR 14 T 38 TA 112/70, HIPERSENSIBILDIAD EN CUADRANTE INFERIOR DERECHO, MC BURNEY Y ROVSING 
POSITIVO. SE SOLICITAN ESTUDIOS DE LABORTORIO DEMOSTRANDO LEUCOCITOSIS. 
¿CUÁL ES EL ESTUDIO DE ELECCIÓN PARA ESTE PACIENTE? 
A) USG 
B) TAC 
C) RX ABDOMINAL DE PIE 
D) RX ABDOMINAL DECÚBITO 
 
RESPUESTA B 
El  paciente  tiene  apendicitis  aguda.  Tema  comunmente  preguntand  en  el  ENARM.  La  migración  del  dolor  con  aumento  del  mismo, 
presencia  de  náuseas,  vómito  y  anorexia  seugieren  el  diangóstico  de  apendicitis  aguda.  Así  que  la  cracterística  del  dolor,  las 
manifestaciones de irritación peritoneal y los datos re respeusta inflamatoria confluyen en el diagnóstico de apendicitis. El patrón de oro, 
como estudio de elección para apendictisi es la TAC. La rx abdominal es anormal en menos del 10% de los casos. 
 
EN EL SERVICIO DE URGENCIAS, USTED RECIBE A UN PACIENTE FEMENINO DE 30 AÑOS DE EDAD, LA CUAL INICIA PADECIMIENTOA CTUAL 
DE 12 HRS DE EVOLUCIÓN CON PRESENCIA DE DOLOR EN REGIÓN DE MESOGASTRIO QUE MIGRÓ A FOSA ILAICA DERECHA, EL DOLOR HA 
IDO EN AUMENTO A TAL GRADO QUE HA PROVOCADO IMPOSIBILDIAD PARA DEAMBULACION, SUMÁNDOSE NÁUSEA QUE PROGRESÓ 
AL VÓMITO EN DOS OCASIONES, FIEBRE NO CUANTIFICADA A LA EXPLORACIÓN FÍSICA ENCUENTRA HIPERESTESIA E HIPERBARALGESIA, 
CON DATOS DE DOLOR A LA DESCOMPRESIÓN EN FOSA ILIACA DERECHA. 
  
SE REALIZA AP DE ABDOMEN DE PIE Y DECÚBITO, ¿CON QUE FRECUENCIA SE OBSERVA ESTE HALLAZGO? 
A) 5% 
B) 8% 
C) 12% 
D) 16% 
 
EN EL CASO PARTICUAR DE ESTE PACIENTE ¿QUÉ ESTUDIO DEBERÁ PRACTICARSE DE MANERA OBLIGADA? 
A) TC ABDOMINAL 
B) USG 
C) LAPAROSCOPIA DIAGNÓSTICA 
D) PRUEBA INMUNOLÓGICA DE EMBARAZO 
 
RESPUESTA B, D 
 
 
__ 
Meckel 60 cms valvula IC 100 cms atras 
diverticulo chipote perforaciones , mucosa ectpopica gastrica pancreatica   
hcl enzimas 
ulcera 
samgtado 
infantes 
obstruccion perforacion   
diverticulectomia en cuña cirre intestino priaira,   
gols 
 
gold st gamma Tc 99 
__ 
 
 
COLECISTITIS 
[Resumen la Salle] 
DEFINICIÓN 
Hidrocolecisto: llena de moco 
Piocolecisto: con pus, más frec en DM2 y inmunocomprometidos.   
Gangrenada: antes de la perforación 
Colasco: perforación de la vesícula,    produce peritonitis química, cinluso infectada si ya tenía dos? 
 
 
SALUD PÚBLICA 
Colecistitis: 90% secundaria a litiasis biliar==> ruta fisiopatológica igual al apéndice pero la obstrucción es en el cístico, también puede 
haber traslocación bacteriana.   
Puede ser alitiásica: ej sepsis, sarcoidosis, LES; SIDA, NOT, quemados.   
M3:H1. 
 
F emale 
F at 
F ourty 
F ertility (AO) 
F ea :C 
 
PATOGENIA 
De arriba a abajo la vesícula tiene: fondo, cuerpo y bolsa de Hartman. 
 
Función de la vesícula: almacernamiento y concentración de bilis, absorción, secreción de moco por cels caliciformes)==> de estas 
puede surgir cáncer ==> adenocarcinoma de tipo escirroso==> colecistectomía + disección en cuña donde está adosado al hígado 
(segmento IV y V) 
 
Triángulo de admiral:    Lecitina 
    Ac biliar   
    Colesterol (más amarillos) ==> generlamente multilitiásica (como esquite) 
 
80% de los litos son de colesterol. 
Solo se disuvleven:    10% (pigmento biliar) y otro 10% que son mixtos.   
 
Pólipos vesiculares <1cm vigilo 
Pólipos vesiculares >1cm opero    Lesión hiperplásica opero. ¿que quiere decir? 
 
Gorrofrigio: es la forma de la vesícula cuando está doblada, se opera porque altera la motilidad. 
Vesícula en procelana: calcificada, es premaligna y se opera (cáncer en 7%) 
Pared >3mm está patológica. Se mide a la altura del segmento IV y V. 
 
Enfisema: gas producido por bacteriaws 
Discinesia: no nacía bien, prueba de Boyden.   
 
Colecistoquinético==> comida al duodeno 1era porción==> secretina + colecistoquinina que promueven el vaciamiento. ==> litos se 
desplazan==> dolor, náusea, vómito==> desaparece el dolor porque deja de estimularse la secretina y colecistoquinina.   
 
DIAGNÓSTICO 
BHC    Leucocitosis 
PFH    aumentadas (TGO TGP, FA, GTT) 
 
Colecistitis==> USG gold standar 
Coledocolitiasis==>CPRE (Gold standar), colangiografía.   
 
 
PRONÓSTICO 
Complicaciones: Sx de Mirizzi,    Boberet (ileo biliar) y coledocolitiasis 
[Poner imagen] 
 
CONTENIDO ADICIONAL CLAVE [WEBINARS] 
Principales bacterias Klebsiella y E. coli 
Gold estándar para qx: cole lap 
14% presenta ictericia sin que haya obstrucción de colédoco. Por que vesícula está adherida a IV y V segmente hepático.   
La pared de la vesícula normal 3mm. 
 
Ayuno, betalactámico, analgésico (AINES)  si no se enfría el cuadro en 24‐48 hrs colecistectomía abierta con incisión de mini lap. 
No laparoscópica porque aumenta la pérdida del producto.   
COLEDOCOLITIASIS 
[Resumen la Salle] 
DEFINICIÓN 
2‐15 % es una complicación de CCL. 
Si es antes del años==> litiasis residual 
Si después deñ años ==> litiasis recidivante. 
 
Sintomática 25‐50%. 
Dolor tipo cólico 
Si microlitiasis==> pancreatitis. 
 
Triada de Charcot:    Dolor +    Fiebre + Ictericia 
 
Tríada de Reynolds: Choque + Confusión + Charcot 
 
Coledocolitiasis==>CPRE (Gold standar) 
 
 
CONTENIDO ADICIONAL CLAVE 
Los litos son de pigmentos biliares, no de colesterol como en la colecistitis. Generalmente son único pero grandes. 
ICTERICIA es el síntoma pivote.   
Se infecta la bilir  Colangitis: ictericia + dolor + fiebre choque séptico , problemas mentales pentada de raynolds 
Gold estándar Dx es COLANGIO RETROGRADA ENDOSCOPIA (si se tata de hígado y bilis) si se refiere a hígado es: COLANGIOGRAFÍA 
PNACREATICA RETROGRADA ENDOSCÓPICA.   
 
Primerp se hace la CPRE y luego la colecistectomía laparoscópica (se hace en las primeras 72hrs) 
 
 
ENFERMEDAD DIVERTICULAR DEL COLON 
[Resumen la Salle] 
DEFINICIÓN 
La enfermedad diverticular engloba tres condiciones: a) estado prediverticular, b) diverticulosis y c) diverticulitis.   
La Salle Onlina propone: divertulosis como término genérico para definir presencia de diverticulos. 
  Enfermedad diverticular cuando hay síntomas, estando dentro de esta dos presentaciones, a) hemorragia y b) diverticulitis. 
 
Pólipos protruyen hacia la luz. 
Enf diverticular  "chipotes" por debilidad de la pared.   
  Adquirida: son falsos divertículos, están en colon izquierdo, más en sigmoides, están cerca de la vejiga.   
  Congénita: son verdaderos, solo en colon derecho. 
 
SALUD PÚBLICA 
M3:H2 
50% en 80 años 
30% en 60 
5% en <40 
 
FR: bajo consumo de fibra. (Hay que consumir 32g/día) 
Prevención: Tx: psyllium plántago a dosis altas a dosis progresivas, Efectos secundarios: cólicos, distensión, malestar abdominal.   
  Consumir 2 lts de agua al día.   
 
 
Pueden ser de boca pequeña (peores porque se ocluyen) o de boca amplia, 
¿Que sitio es donde protruyen los divertículos? Vasos rectos (en el borde lateral) 
 
DIAGNÓSITCO 
Clínica: asintomáticos 70%, síntomas 30% (Diarrea/estreñimiento), moco, dolor en CII e hipogastrio.   
    
Diverticulitis: taquicardia y fiebre, dolor en CII, datos de irritación peritoneal. Prestar atención si hay pérdida de matidez hepática, si 
masa abdominal pensar en plastrón. 
 
1)Placa simple de abdomen (inicial) 
2)Colon por enema con doble contraste (Gold standar)?? ¿qué papel juega? 
3)Colonoscopia para saber si son de boca grande o pequeña y su localización. También en caso de hemorragia y tratamiento del 
sangrado.    En este caso entiendo que es diagnóstia y terapéutica. Está contraindicada si hay diverticulitis. ¿Por qué? ¿Riesgo de 
perforación? 
Un paciente que sangra por diverticulos NO tiene diverticulitis (porque en esta los vasos se obliteran).   
4) TAC con doble contraste (IV y VO) es el estudio más útil si hay divertículitis, es decir complicaciones. Se traduce con dolor intenso. 
1/4 hará signo de la flecha, colon >4mm diverticulitis.      Y la USG si la TAC no está disponible. 
 
Complicaciones:   
  Diverticulitis (es la más frec,10‐25%) 
  Peforación peritonitis, 
  Estenosis 
  Fístulas  zona de fistulización más frecuente es colovesical. 
Si tengo un paciente con diverticulitis (dolor), me salto el colon por enema porque el bario lo puede perforar y hago una TAC 
directamente. 
En resumen si diverticulosis  colon x enema, si diverticulitis TAC   
El dato pivote que nos orienta a una diverticulitis es el dolor, si no hay dolor puedo hacer colon por enama. 
La colonoscopia la podría hacer en caso de choque. 
 
Clasificación de Hinchey   
0  Enf diverticular leve      Enviar a casa no AB 
Ia  Inflamación pericólica      Enviar a casa no AB 
Ib  Absceso <5cm      AB 
II  Absceso >5cm      AB + drenaje precutáneo [hospitalizar] 
III  Peritonitis    purulenta      Qx urgente [colectomía] + Metronidazol + levofloxacino.   
IV  Peritonitis fecal      Qx urgente [colectomía] + Metronidazol + levofloxacino 
 
*Ib y II sin mejoría luego de 48 hrs se opera de urgencia. Hospitalario. Investigo si está complicado. 
 
Tx: ambulatorio si tolera la VO, doy líquidos.   
 
Dolor  Mesalazina + rifaximina , paracetamol y metamizol? 
AB  Ciproflox + metronidazol 
 
Fiebre, leucocitosis, dolor hospitalario 
Hospitalarios III y IV:    nunca morfina mejor meperidina.    Antibiótico gold standar Metronidazol + levofloxacino 
 
Tx Qx electivo si:    a)Inmunodeprimidos, <50 años, tras 2do episodio 
    b)Pacientes con fístula, estenosis o sospecha de cáncer. 
    Resección del tramo que se complicó y anastomosis aunque haya más divertículos. Vía laparoscópica. 
    No está justificada la Qx luego de solo un episodio en menores de 50 años. 
    Sigmoidectomía en todo inmunodeprimido. 
Tx de primera elección: resección de sigmoides con anastomosis primaria e ileostomía protectora. [Siempre que las condiciones la 
permitan] 
Prodecimiento de 2da elección: resección de sigmoides, colostomía y colocación de bolsa de Hartmann 
 
 
Tx Qx urgente si: sin respuesta al tx (48hr), sepsis, perforado, diverticulitis complicada con fístula. 
    Se realiza en tres tiempos: resección, colostomía, ileostomía derivativa? 
 
 
 
PRONÓSTICO 
STDB grave: 24‐42% será enf diverticular 
5% de los que sangran y la colonoscopia no fue útil==> angiografía y embolización, si fracasa se hace resección.   
Debuta cmo hematoquesia, está mezclada con las heces, cesa 80‐90% de forma espontánea.   
  
CONTENIDO ADICIONAL CLAVE 
Dolor en: 
  Epigastrio + FII      = enf diverticular 
  Epigastrio + FID = apéndice 
 
WEBINAR 
 
Estudios inciales: placa de abdomen, bhc, es. 
Si pienso que es diverticulitis: TAC con doble contraste. 
 
Principal complicación de enf diverticular: diverticulitis 
Otras complicaciones: estenosis, sangrado, perforación, fistulización.   
Sitio más frecuente de fistulizacion vejiga 
 
 
Paciente joven de 2‐3 eventos en un año o con comorbildiades (DM) Hinchey I se puede operar. 
 
 
FUENTES ADICIONALES 
1) ALLER DE LA FUENTE, R.. Enfermedad diverticular del colon. Rev. esp. enferm. dig. [online]. 2005, vol.97, n.6 [citado   
2016‐04‐21], pp.458‐458. Disponible en: <http://scielo.isciii.es/scielo.php?script=sci_arttext&pid=S1130‐
01082005000600009&lng=es&nrm=iso>. ISSN 1130‐0108. 
 
RECIBE EN SU CONSULTORIO A PACIENTE FEMENINO DE 65 AÑOS DE EDAD, CON ANTECEDENTE DE PADECER DOLOR TIPO CÓLICO EN 
REGIÓN DE FOSA ILIACA IZQUIERDA, DE DOS AÑOS DE EVOLUCIÓN QUE INICLAMENTE SE PRESENTÓ COMO DOLOR CÓLICO LEVE, 
DESPARECIENDO DE MANERA ESPONTÁNEA, SIN EMBARGO DESDE HACE 3 MESES, PRESENTE DOS EVENTOS DE DOLOR INTENS, EL 
ÚLTIMO LO OBLIGÓ A ADOPAR POSICIÓN ANTIÁLGICA, AGREGÁNDOSE EN ESTE EVENTO FIEBRE NO CUANTIFICADA.   
AL MOMENTO DE LA EXPLORACIÓN ENCUENTRA DOLOR DE PREDOMINIO EN FLANCO Y FOSA ILÍACA IZQUIERDA, EL CUAL SE PRESENTA 
A LA PRESIÓN MEDIA Y PROFUNDA, SIN DATOS DE IRRITACIÓN PERITONEAL, SIN PLASTRONES NI BORRAMIENTO DE ÁREA HEPÁTICA. 
 
¿CUÁL ES EL ESTUDIO DE ELECCIÓN? 
A) PLACA DE ABDOMEN 
B) ANOSCOPIA 
C) RECTOSIGMOIDOSCOPIA 
D) TC DOBLE CONTRASTE. 
 
Recuerda que el estudio endoscópico si está indicado, pero a modo de hemorragia ya que sería diagnostico y terapéutico. 
 
EN EL ESTUDIO QUE SOLICITA, SE DESCRIBE LA PRESENCIA DE ZONAS DE INFLAMACIÓN SEVERA A NIVEL DE COLON SIGMOIDES CON 
PRESENCIA DE UN ABSCESO DE 4CM A NIVEL DE LA ZONA INFLAMATORIA MÁS INTENSA Y OTRO DE MÁS DE 5 CM EN LA ZONA DE 
UNIÓN DE SIGMOIDES CON RECTO. ¿Cuál ES LA CONDUCTA A SEGUIR? 
A) ANTIBIOTICOTERAPIA CON DOBLE ESQUEMA 
B) ANTIBIOTICOTERAPIA CON TRIPLE ESQUEMA 
C) ANTIBIOTICOTERAPIA MÁS DRENAJE PERCUTÁNEO 
D) CIRUGÍA URGENTE 
 
LA FORMA DE USO DE ANTIBIÓTICO EN LOS PACIENTES CON TAL PATOLOGÍA ES: 
A) QUINOLONA Y NITROIMIDAZOL 
B) FLUROQUINOLONAS Y METRONIDAZOL 
C) AMINUGLUCÓSIDOS Y METRONIDAZOL 
D) MONOTERAPIA CON IMIPENEM Y MEROPENEM 
 
RESPUESTA D, C, B 
 
HOMBRE DE 45 AÑOS DE EDAD EL CUAL SERA SOMETIDO A CIRUGÍA DE COLÓN POR DIVERTÍCULOS. 
CONSISTE EN EL TRATAMIENTO DE ELECCIÓN PARA LA PREPARACIÓN INTESTINAL PREOPERATORIA EN ESTE PACIENTE: 
   
A) NEOMICINA + ERITROMICINA. 
B) METRONIDAZOL + CIPROFLOXACINO. 
C) CEFALOSPORINA DE SEGUNDA GENERACIÓN. 
D) NEOMICINA + CLINDAMICINA. 
 
RESPUESTA 
Nenomicna y eritromicina son de primera elección para la preparación intesitnal. 
La opción B podría ser correcta si adicionara neomicina.   
La opción C es incorrecta para la preparación.   
La opción D es incorrecta porque clindamicina asocia enterocolitis necrotizante. 
 
 
HOMBRE  DE  65  AÑOS  DE  EDAD.  ACUDE  A  URGENCIAS  POR  UN  CUADRO  DE  4  DÍAS  DE  EVOLUCIÓN,  CARACTRIZADO  POR  DOLOR 
ABDOMINAL, ESCALOFRÍOS Y FIEBRE; EL DOLOR INICIÓ EN FOSA ILIACA IZQUIERDA PERO SE HA DISEMINADO. A LA EXPLORACIÓN FÍSICA: 
FC90  FR16,  T  38,  TA  110/65,  DOLOR  A  LA  PALPACIÓN  EN  FOSA  ILIACA  IZQUIERDA.  SE  SOLICITAN  ESTUDIOS  DE  LABORATORIO 
DEMOSTRANDO LEUCOCITOS DE 14,000.  LA TAC DE ABDOMEN DEMUESTRA MÚLTIPLES DIVERTÍCULOS EN COLON SIGMOIDES, YA LA 
PRESENCIA DE UNA COLECCIÓN SIMPLE BIEN DELIMITADA DE APROXIMADAMENTE 4X6X2CM EN EL ESPACIO SUBFRÉNICO DERECHO.   
¿CUÁL ES EL MANEJO MÁS ADECUADA DE LA COLECCIÓN ENCONTRADA EN LOS ESTUDIOS DE IMAGEN? 
A) ANTIBIOTICOTERAPIA 
B) RESECCIÓN INTESTINAL 
C) DRENAJE PERCUTÁNEO GUIADO POR IMAGEN 
D) ANTIBIÓTICOS Y PEDIR CONTROL DE TAC EN 72 HRS 
 
RESPUESTA C 
El paciente tiene diverticulitis aguda, con dicho antecedente deducimos que la colección encontrada en la tac es una absceso abdominal. 
El drenaje percutáneo guiado por USG o TAC está indocado en el tratamiento.    [Reto enarm] 
 
 
 
 
ALTERACIONES ELECTROLÍTICAS Y ÁCIDO BÁSICAS EN EL PACIENTE QUIRÚRGICO 
DEFINICIÓN 
Pueden ser alteracines de la concentración, composición, volumen o mixtas. 
 
[supertabla de alteraciones eletrolíticas] 
Concentración 
Hiponatremiua   
  <130meq/L 
  Causas: Causas importantes: SIADH, diarre importante, fístula entero cutánea, Diab insípida (cada 100mg de glucermia arriba 
  de lo normal, disminuye 1.6 a 3mEq/l 
  Cl: signos en SNC, HIC,   
   
Hipernatremia   
  >150mEq/L 
  Cl:Signos SNC y tisulares, EKg onda T en espiga.Resequedad y viscosisdad mucosa.     
  Tx: reponer agua. 
 
Hipokalemia*   
  <3.5mEq/L 
  Causas: líquidos parenterales prolngados sin potasio, NPT prolnagada sin potasios, pérdida digestiva, paso de K al interior de 
las    céls 
  Cl: falta de contractilidad de músculo esquelético, liso y cariaco. Sensibilidada aumenta a digitálicos, cambios EKG. 
  Tx: 40 mEq/L de cloruro o fosfato de potasios. 
  NPT con glucosa al 25% usando cateter central. 
Hiperkalemia 
  >5mEq/L 
  Causas: acidosis, trauma quiurgico, IR 
  Cl: náuseas, vómito, cólico intestinal, diarrea, bloqueo    cardiaco, paro diastólico, EKG T altas en espiga, QRS ancho, ST 
  deprimido.   
  Tx: gluconato cálcico I1g IV al 10%, 45mEq HCO3 1000 ml de dextrosa al 10% + 20 UI de insulina 
  Resinas de intermbio catiónico 
  Diálisis 
  [medidas antihiperkalémicas] 
   
Composición 
    Fisiopato        Causas 
Acidosis respiratoria  Retención de CO2 x hipoventilación  Depresión del centro respiratorio, afección pulmonar, EPOC, NAC 
Alcalosis respiratoria  Pérdida de CO2x hiperventilación   Emocional, dolor intenso, VMI, encefalitis 
 
Acidosis metabólica  Retención de H+ o pérdida de HCO  Diabetes, hiperazoemia, acum de ác láctico, inanición, diarrea,   
            fístulasi de intestino delgado 
Alcalosis metabólica* Pérdida de H+ o retención de HCO  Vómito, aspiración gástrica, ingesta de HCO, diuréticos.   
  
*Comunes en pacientes quirúrgicos. 
 
 
 
HERNIA UMBILICAL 
DEFINICIÓN 
Reparación quirúrgica hernia umbilical en niños: >1.5cm    o persistente después de los 2 años.   
Se recominda ténica de Mayo, material absorbible. 
 
Reparación en adultos: técnica de mayo, material absorbible, las mallas si hernias >3cm de diámetro. 
AB solo si la tasa hospitalaria de infecciones es superior a 5%. 
  
HERNIA INGUINAL 
DEFINICIÓN 

 
 
 
 
[HERNIA INGUINAL] 
MUJER DE 48 AÑOS DE EDAD, CON ANTECEDENTE DE HERNIA INGUINAL BILATERAL. ES VALORADA EN EL SERVICIO DE URGENCIAS POR 
PRESENTAR DOLOR INTENSO EN LA REGIÓN INGUINAL DERECHA. 
LA POSIBILIDAD DE QUE SE TRATE DE UNA HERNIA ESTRANGULADA SE CONFIRMARA EN CASO DE: 
 
A) QUE HAYAN DATOS DE OCLUSIÓN INTESTINAL. 
B) QUE HAYAN DATOS DE PERFORACIÓN INTESTINAL. 
C) QUE NO SEA REDUCTIBLE. 
D) QUE EXISTA COMPROMISO VASCULAR. 
 
RESPUESTA D 
Tanto en hernias incarderadas como estanguladas hay cierto grado de oclusión intestinal.   
La perforación intestinal puede ser producto de una isquemia prolongada.   
En una hernia estrangulada hay compromiso vascular,    está tensa y muy snesible, la piel que lo cubre tiene color rojizo o azulado. N 
existen ruidos intetinales dentro de la hernia. El paciente suele tener leucocitosis con desviación a la izquierda, se encuentra tóxico, 
deshidratacio, febril, GASA con acidosis metabólica.    El Tx es Qx de urgencia por riesgo de perforación, el tx de esta última complicación 
es la resección del segmento afectado.   
 
 
OCLUSIÓN INTESTINAL 
DEFINICIÓN 
Detención completa y persistnte del contenido intestinal en algún punto del tubo. Si no es completa y persiste se llama suboclusión 
intestinal. El 90% se explica por adherencias, hernias y tumores. 
 
ETIOLOGÍA 
Hay que distinguir si es oclusión alta o baja (en función a válvula ileocecal), si es mecánica o no mecánica, parcial o total, complicada o 
no complicada.   
 
Diferencia 2 entidades clínicas bien distintas: 
  Íleo paralítico: alt de la motilidad intestinal x parálisis del m. liso, raramente requiere manejo qx. 
  Íleo mecánico: auténtico obstáculo mecánico del contenido. Puede se intraluminal, extraluminal o parietal.   
 
Obstrucción mecánica estrangulante: obstrucción genera compromiso vascular intestinal. 
Oclusión en asa cerrada: onclusión en 2 puntos. Este tipo tiende más a la isquemia y perforación.   
 
Causa más frec de obstrucción intestinal baja: neoplasias de colon y recto.   
 
Ileo mecánico 
a) Extraluminal: Adherencias postQx (35‐40%), hernias externas 20‐25% (inguinales, crurales, umbiliales, etc), hernias internas, 
torsiones, vólvulos, invaginaciones, efecto masa extraluminal.   
b) Parietal: Neoplasias, alt congénitas (atresia, estenosis, duplicaciones), procesos inflamatorios (Crohn, postradiación, etc) 
c) Intraluminal: íleo biliar, bezoar, parasitosis, cuerpo extraño, impactación fecal, tumoraciones.   
 
Íleo paralítico   
a) Adinámico: postquirúrgic, peritonitis, reflejo (Sx retroperitoneal), compromiso medular, hiperuricemia, hipokalemia, coma diabético, 
mixedema, bloqueantes ganglionares, isquemia.   
b) Espástico: intoxicación por metales pesados, porfirias.   
c) Vascular: embolia arterial, trombo venoso.   
 
PATOGENIA 
El gas que se acumula proximal a la obstrucción se debe a 1) aire deglutido (es el principal componente, tiene mucho N+ razón por la 
cual no se absorbe en intestino, por eso se pone SNG), 2) CO2 por neutralización del bicarbonato (se absorbe bien en intestino), 3) 
gases orgánicos de la fermentación bacteriana.   
Hay hipocloremia, hipokalemia, hiponatremia, alcalosis metabólica, oliguria, hiperazoemia, hemoconcentración y deshidratación.    ==> 
si persiste==> taquiardia, disminución de PVC y GC==> hipotensión y shock hipovolémico.   
 
Distensión abdominal==>aumento de presión intrabdominal==> disminución de retorno venoso de las piernas==> hipoventilación. 
Translocación bacteriana y sepsis. 
 
En obstrucción de colon los cambios electrolíticos son más lentos, si la válvula ileocecal es competente se comporta como asa cerrada, 
esto aumenta el riesgo de perforación.   
 
DIAGNÓSTICO 
1 ¿Tiene una obstrucción intestinal? 
2 ¿Dónde está la obstrucción? 
3 ¿Cuál es la naturaleza anatómica y patológica de las lesiones que la provocan? 
4 ¿Existe estrangulación? 
5 ¿Cuál es el estado general del paciente? 
 
Síntomas cardinales: dolor, distensión, vómito, ausencia de emisión de gases y heces.   
Dolor: comienzo gradual, mal localizado, tipo cólico si es mecánica, continuo si es paralítico o isquémico. Intervalo de dolor de 4‐5 
minutos. Mientras más baja sea la obstrucción la frecuencia baja.   
Distensión abdminal: distendido y timpánico,     
Vómitos: primero son de origen reflejo, luego por regurgitación de las asas. Primero alimenticios, luego biliosos (intestinales), más 
tarde fecaloides.   
Ausencia de emisión de gases y heces: no es un síntomas constante, si la obstrucción es alta puede haber emisión.   
Recordar que en obstrucción alta los principales síntoams son náuseas y vómito, en obstrucción baja distensión abdominal. 
 
Exploración física:   
Inspección: distendido, ondas perisálticas en delgados, 
Palpación: dolor difuso, búsqueda de hernias, 
Auscultación: peristaltismo aumentado==> ausencia. 
 
Paraclínicos:     
Solicitar: BHC, QSC con CPK, PCR, ES, TP/TTP, GASA (acidosis metabólica?), tele de tórax, ap de abdomen de pie y decúbito.. 
Deshidratación hemoconcetración alt, electrolticas, leucocitosis. 
Rx: Gas intestinal y niveles hidroaéreos, si intesitno delgado u obstrucción alta (imagen en pila de monedas). Si se observa dilatación 
intestinal generalizada en tubo digestivo y heces en ampolla o gas, sospecha íleo paralítico. 
Aerobilia si íleo biliar, signo de grano de café si vólvulo 
Colon x enema: últil si obstrucción de intestino grueso. 
TAC con doble contraste: el más sensible si obstrucción sin complicación con material hidrosoluble. 
 
TRATAMIENTO 
Si es completa y mecánica cirugía. 
Si es parcial mecánica o no mecánica, SNG 48‐72 hs genera remisión en 90% de los casos.   
Datos predictivos: no resolución luego de 72 hrs, SNC con gasto >500ml, >40 años, adherencias complejas, nula presencia de aire en 
recto, CPK >130. 
 
El manejo conservador con monitoreo cada 8 hrs de perímetro abdominal, búsqueda de fiebre, taquiardia, irritación peritoneal y gasto 
aumentado de SNG. 
 
Gralmente quirúrgico, EXCEPTO adherencias y las incompletas o suboclusiones.   
Pseudoobstrucción intestinal e íleos funcionales==> recuperar equilibrio hidroelectlitico. 
Adherencias==> SNG, ayuno, control de iones, electrolitos.   
Si obstrucción incompleta de intestino grueso==>conservador y qx electiva posteior 
Hernia incarcerada==> observación y reducción . 
 
Indicación de Qs: TC con líquido intraperitoneal, edema de mesenterio, signod e las heces en intesitno, isquemia en apred intestinal o 
fuga del medio.   
Cirugía abdominal 6 semanas previas.   
Dolor intenso mayor a 4 en escala análoga + leucos >10mil, PCR >7.5mg/l. 
Peritonitis o hernia complicada. 
No solución luego de 72 hrs, aun luego de 50‐150ml de medio de contraste. 
Laparotomía exploradora es la técnica más recomendada. 
 
EN TU CONSULTORIO REVISAS A UN PACIENTE MASCULINO DE 68 AÑOS DE EDAD, EL CUAL INICIA PADECIMIENTO ACTUAL DE 24 HRS 
DE EVOLUCIÓN CON LA PRESENCIA DE DISTENCIÓN ABDOMINAL CON LA IMPOSIBILDIAD PARA CANALIZAR GASES Y DE EVACUAR, 
ADEMÁS DE SUMARSE ESTADO NAUSEOSO QUE PROGRESA AL VÓMITO EN VARIAS OCASIONES, AL INICIO FUE ESPORÁDICO PERO 
PAULATINAMENTE HA IDO EN AUMENTO, CON ESCALA DE 5 DE 10. APARECIENDO DOLOR DEL TIPO CÓLICO, EL CUAL ES INTENSO, QUE 
SE MODIFICABA CON EL VÓMITO AL INCIO, PERO DESPUÉS SE VOLVIÓ CONTINUO E INTENSO, EN LOS ANTECEDENTE DE 
INTERROGADOS, REFIERE HABER SIDO OPERADO DE COLECISTECTOMÍA, APENDICECTOMÍA POR APENDICITIS COMPLICADA, 
LAPARTOMÍA EXPLORADORA POR LESIÓN DE ARMA DE FUEGO HACE 5 AÑOS, CON RESECCIÓN DE INTESTINO DELGADO. 
 
TU SOSPECHA ES: 
A) APENDICITIS AGUDA 
B) ISQUEMIA MESENTÉRICA 
C) PANCREATITIS AGUDA 
D) OCLUSIÓN INTESTINAL 
 
EL GOLD ESTÁNDAR PARA ESTA PATOLOGÍA ES: 
A) PLACA SIMPLE DE ABDOMEN 
B) TRÁNSITO INTESTINAL 
C) TELE DE TÓRAX 
D) TAC ABDOMINAL 
 
RESPUESTA D, D 
 
HOMBRE DE 38 AÑOS DE EDAD, CON ANTECEDENTE DE APENDICECTOMÍA HACE 6 MESES Y EN ESTA OCASIÓN INGRESA CON DATOS DE 
OBSTRUCCIÓN INTESTINAL. 
LA CAUSA MÁS PROBABLE POR LA CUAL EL PACIENTE PRESENTA ESTE CUADRO CLÍNICO ES: 
   
A) ADHERENCIAS INTESTINALES. 
B) ABSCESO RESIDUAL. 
C) CÁNCER DE COLÓN. 
D) ADENITIS MESENTÉRICA. 
 
RESPUESTAS A 
El  bloqueo  de  la  luz  intestinal  se  produce  en  su  mayoría  por  factores  extrínsecos  (adherencias,  hernia  inguinal,  hernia  de  pared 
estrangulada) y en pocas ocasiones por factores intrínsecos.    Las adherencias intraabdominales por intervenciones quirúrgicas previas 
representan el 75% de los casos de obstrucción intestinal.   
 
PANCREATITIS 
PANCREATITIS AGUDA 
DEFINICIÓN 
15% es considerada grave por presentar necrosis pancreática, 85% leve por presentar daño intersticial. 
 
SALUD PÚBLICA 
50% secundaria a colelitiasis, 25% por alcohol y el resto otras causas e idiopática. 
FR colelitiasis, alcoholismo, hiperlipidemia, medicamentos, hipercalcemia, trauma, disfunción de Oddi, CPRE, pancreas divisu, Ca 
periampular, Ca pánreas, diverticulo periampular, vesculitis, infecciosas, autoinmune (LES, Sjögren) 
 
PATOGENIA 
A) activación de enzimas digestivas 
B) secuestro de neutrófilos en páncreas 
C) necrosis, edema,    SRIS, IRA y fallo multiorgánico. 
 
DIAGNÓSTICO 
2/3 criterios: 
  Clínica (dolor en epigastrio, irradiación a espalda, intensidad progresiva y súbito, náuseas, vómito, distensión abdominal. 
  Elevación de amilasa/lipasa al menos al triple. 
  Alteraciones estructurales en estudios de imagen. 
   
Se debe hacer en las primeras 48 hrs.   
Grey Turner's (equimosis en la pared lateral del abdomen)* 
Cullen's (equimosis al rededor de la pared abdominal)* 
Fox's (cambio de coloración a nivel de lig inguinal)* 
*Solo en el 3% de los casos e indican gravedad 
 
APACHE en los primeros 3 días nos define si es leve o grave. 
Hematócrito en admisión, 12, 24 hrs también nos permite distinguir. 
 
>55 años, IMC >30, falla orgánica al ingreso, derrame pleural son indicadores de severidad y se deben diagnostiar.   
 
Lipasa es más S y E para diagnóstico. Si la amilasa no llega al triple de la basal decimos que hay hiperamilasemia (causas. Mordidas, 
úlceras esofágicas, etc). 
Gold standar para valorar pronóstico: proteína C reactiva a las 48 hrs de evolución. 
Primer estudio de imagen a pedir: El primer estudio de imagen a pedir es rx de abdmen y torax de pie y decúbito no para diagnóstico, 
sino para diagnóstico diferencial, y abocado a pancreatitis==> USG 
TAC en las primeras 24 hrs es muy útil ya que nos da pronóstico, se repite a las 72‐120 hrs. El Dr. Sanchez medina dice que hasta las 
72hrs! No antes. Nos permite determinar criterios de Baltazar e índice de severidad tomográfica. 
  Si la TAC demustra algo de necrosis antibiótico para evitar muerte por sepsis (mepropenem, fluoroquinolonas, cefalos 
3era o 4ta asociada a metronidazol).    Si >30% de necrosis agregar fluconazol. 
  
Si pancreatitis + sospecha de litiasis biliar==> CPRE dentro de las primeras 72 hrs. 
 
Panceratitis leve:    Falla orgánica mínima, edema intersticial. Falta de respuesta luego de 48‐72 hrs pensar en complicacines. 
Pancreatitis grave: falla orgánica importante, necrosis, asbceso y pseudoquiste. 
  TAS <90 
  IRA PaO2 <60mmHg 
  Creatinina >2mg 
  Sangrad GI >500cc /24h 
 
  CID <100,000 plaq 
  Fibrinógeno <100mg/dl 
  Prod deg fibrina >80ug/ml 
  Hipocalcemia <7.5 
 
TRATAMIENTO 
Analgesia, coloides IV, ayuno, ASNG (evita que la liberación de gastrina en estómago y su ulterior llegada a duodeno) AB si necrosante, 
ectreótido reduce mortalidad pero no cambia tasa de complicaciones, gabexato reduce lesión pancreática pero no afecta mortalidad. 
 
Necrosis infectada==>extirpación de tejido necrótico   
Necrosis >30%, o áreas pequeñas con sepsis==>aspiración con aguja fina, tinción, cultivo. 
  Gram negativo==> imipenem, cirpfloxacino, ofloxacino, pefloxacino. 
  Gram positico==>vancomicina. 
 
No inicia vía oral según amilasa. Mejor amilasa en orina mínimo de 6hrs, si está disminuida  dieta líquida dieta blanda 
 
 
Tratamiento quirúrgico 
Necrosis pancreática estéril >50% + datos de deterioro.   
Ncrosis pnacreática esteril que luego de 2 semana no puede ser alimentado, dolor abdominal aumenta presenta íleo. 
Necrosis infectada: necrosectomía y drenaje. 
Abscesp pancreático==> drenaje percutáneo o endoscópico. 
Pancreatitis aguda con complicaciones, pseudoquiste pancreático==> drenaje. 
 
PRONÓSTICO 
Mortalidad 5% general, 3% si es leve, 17% si es grave.   
24‐48 hrs con Ranson 
>48 hrs APACHE No aumentar más de 8. 
Criterios para UCI:   
  Inestabilidad hemodinámica, disfunción respiratoria y signos de gravedad (ranson >3, apache II >8, insuf orgánica.   
PCR también da pronóstico, incluso procalcitonina. 
 
   
30‐40% hace pseudoquiste. Esperar 6 semanas, el 60% se reabsorve automáticamente. Si tiene una parede de 6mm y 60cc puede 
reabsorverse. El 40% no reabsorve luego de 6 semanas  endoscopia cistogastroanastomosis 

 
 

 
 
 
Criterios de Ranson 
Al ingreso: 
Edad    >55a 
Glucosa    >200mg/dl 
AST    >250UxL 
DHL    >350UxL 
Leuc    >16,000 
 
A las 48hrs 
Déficit de base      >4mEq 
Incremento de nit ureico    >5mg/dl 
Calcio sérico      <8mg/dl 
Disminución de Htc     >10% 
PaO2        <60mmHg 
Secuestro de líquidos    >6,000 ml 
 
 
 
INFECCIONES ÓSEAS Y ARTICULARES 
OSTEOMIELITIS 
 
DEFINICIÓN 
Proceso infeccioso que afecta al tejido óseo y su cavidad medular.   
 
ETIOLOGÍA 
Bacterias y parásitos (más comúnes) Etiología más frecuente en caso de bacterias S. aureus. 
Hongos (inmudeprimidos) 
Virus (no son causa) 
 
Niños 1‐4 años: S. aureus, después SBH. SARM en infecciones de tejidos blandos.   
Niños > 4años: ¿? 
 
FR: infecciones, cirugía previa, desnutrición, DM descompensada, inmunodeficiencia, adicciones, alt vascualres, traumatismos. 
 
PATOGENIA 
Hematógena (más común), inoculación externa (fx espuesta, Qx), extensión de estructura adyacente infectada. 
 
Localización en adultos: cualquier sitio anatómico óseo, 90% miembros pélvicos y solo un hueso.   
Localización en niños: nunca la fisis, siempre es metafisaria (invade a médula y cortical). 
 
Aguda  <2 sem 
Subaguda  2‐4sem 
Crónica  >4sem 
Residual secuelas 
 
 
DIAGNÓSTICO 
Fase aguda temprana (24‐48hrs): febrícula mal gral, marcha claudicante, rechazo al caminar, sensibilidad doloroosas.   
Fase aguda tardía: (4‐5días): absceso subperióstico. Fiebre, dolor a la movilización tumefacción edema. 
 
Síntomas: [Lactantes] pseudoparálisis de extr afectada, ausencia de fiebre, contractura muscular, irritabilidad, pérdida 
de apetito. 
  [Niño] Limitación de movilidad art, dolor articular, fiebre. 
  [Adulto] dolor localizado que aumenta tras la mov activa y pasiva de art adyacentes, aumento de vol, 
hiperemia, fiebre, escalofríos, celulitis, pérdida de la función. 
 
Paraclínicos: rx simple con técnica para partes blandas (al principio aumento de vol de partes blandas periférico al 
hueso), a partir del 7mo día podré pedir técnica para partes óseas donde se verá elevación del periostio, lisis, esclerosis,   
reducción de densidad ósea. USG cambio en tejidos blandos, fluidos subperiósticos o yuxtacorticales.   
BHC==> leucocitosis [máximo 10,000, neutrófilos totales >6600, segmentados >6%.] 
VSG==> elevada entre 1‐5 día[>20 mm/hr] 
PCR==> elevada desde 1eras 8 hrs con pico máximo al día 2 [>10mg/l] Recordar que infecciones bacteriana elevan de 4‐
20 si son leves, graves o quemaduras >20. Virales y emarazo 1‐4. 
 
Hemocultivo Dx en 50%, aspiración de la lesión en 70%. 
Luego de 15 días la rx podrá mostrar involucro o absceso subperióstico. 
RM: estudio de mayor S y E. 
 
TRATAMIENTO 

 
 
IV por 7‐14 dias==> VO 6‐8 sem (hasta que esté asintomático, se normalice VSG) 
 
Si lesión en piel, cubrir con toalla seca y cubrir con plástico. Si fiebre control x medios físicos. 
Crioterapia 10‐15min cada 8‐12hrs x 2 días directo en lesión afectada. 
 
MASCULINO DE 4 AÑOS SIN ANTECEDENTES DE IMPORTANCIA, QUE ACUDE A CONSULTA PORQUE LA MAMÁ REFIERE 
QUE LLEVA DOS DÍAS COJEANDO DE LA PIERNA IZQUIERDA, ACOMPÑADO DE MALESTAR GENERAL AL ESTAR JUGANDO Y 
HABERSE TROPEZADO. EL DÍA DEL INCIDENTE LO LLEVAN A UN HOSPITAL DONDE LO DAN DE ALTA CON DIAGNÓSTICO 
DE ESGUINCE DE RODILLA Y LO EGRESAN CON IBUPROFENO. 
A SU LLEGADA TA 138/88, FC 95, FR 26 T 37.6, PESO 28, INCAPACIDAD PARA LA MARCHA Y CLAUDICACIÓN A EXPENSAS 
DE MPI, DOLOR Y LIMITACIÓN A LOS ARCOS DE MOVIMIENTO DE CADERA Y RODILLA IZQUIERDAS, RESTO DE LA EF SIN 
DATOS PATOLÓGICO. RX AP DE PELVIS Y RX DE RODILLAS NORMALES. 
 
CON BASE EN LOS DATOS MENCIONADSO EL SIGUIENTE PASO EN EL ABORDJE DEL PACIENTE SERÍA: 
A) RB DE CADERA Y RODILLA 
B) BH, PCR Y VSG 
C) ASPIRADO DE LÍQUIDO ARTICULAR DE RODILLA Y CADERA IZQUIERDAS 
D) AP DE PELVIS EN POSICIÓN DE RANA 
 
¿CÓMO SE LE LLAMA AL ABSCESO SUBPERIÓSTICO, QUE SE PRODUCE ALREDEDOR DE LA CORTICAL SI SE DEJA LA 
EVOLUCIÓN NATURAL DE LA ENFERMEDAD? 
A) ABSCESO CORTICAL 
B) OSTEOMIELITIS 
C) SECUESTRO 
D) INVOLUCRO 
 
CON LOS RESULTADO DE LOS LABORATOROS POSITIVOS ¿CUÁL ES EL SIGUIENTE PASO QUE USTED SEGUIRÍA? 
A) TX EMPÍRICO VS S. AUREUS 
B) TX EMPÍRICO VS S.HEMOLÍTICO 
C) TX EMPÍRICO VS H. INFLUENZAE 
D) CULTIVO DE LÍQUIDO ARTICULAR E INICIAR TX ESPECÍFICO. 
 
Hueso alrededor de sitio infectado es un involucro, si hay varios segmentos afectados es secuestro. 
RESPUESTA B, D, A 
 
ESCOLAR DE 11 AÑOS, CON ANTECEDENTE DE TRAUMATISMO DIRECTO EN MUSLO DERECHO, HACE 12 HORAS PRESENTA DOLOR 
AGUDO EN LA ZONA DISTAL DEL MUSLO MOTIVO POR LO QUE ACUDE A LA CONSULTA. A LA EXPLORACIÓN CON TEMPERATURA DE 
38°C, INFLAMACIÓN EN EL TERCIO INFERIOR DEL MUSLO DERECHO Y DOLOR A LA MOVILIZACIÓN, LIMITACIÓN FUNCIONAL DE CADERA 
Y RODILLA. BIOMETRÍA HEMÁTICA CON LEUCOCITOSIS Y LA VELOCIDAD DE SEDIMENTACIÓN GLOBULAR AUMENTADA. RADIOGRAFÍAS 
DE FÉMUR NORMALES.   
    
1 ‐ EL GERMEN PATOGENO MAS PROBABLE DE ESTA PATOLOGIA ES: 
STAPHILOCOCUS AUREUS. 
ENTEROBACTER. 
ESCHERICHIA COLI. 
PSEUDOMONAS AERUGINOSA.   
        
2 ‐ EN CASO DE DEJARSE A LIBRE EVOLUCIÓN SE ESPERA ENCONTRAR CAMBIOS RADIOLÓGICOS EN EL SIGUIENTE TIEMPO. 
2 DÍAS. 
3 DÍAS. 
7 DÍAS. 
10 DÍAS.   
     
3 ‐ DESPUÉS DE 36 HRS DE INICIADOS LOS ANTIBIÓTICOS EL PACIENTE PERSISTE CON MANIFESTACIONES LOCALES Y SISTÉMICAS, POR 
LO QUE SE LE DEBE REALIZAR: 
COLOCACIÓN DE SISTEMAS DE IRRIGACIÓN. 
TRACCIÓN DE LA PIERNA. 
INMOVILIZACIÓN DE LA EXTREMIDAD. 
DESBRIDACIÓN DEL FOCO ÓSEO.   
 
Dolor a la movilización activa y pasiva en ARTICULACIONES ADYACENTES al sitio de osteomielitis es un dato pivote, en artritis séptica el 
dolor es agudo MONOARTICULAR (solo 10%‐20% lesiona múltiples articulaciones, generalmente se asocian a Neisseria, gram negativo y 
estrepto). 
RESPUESTA 
1A, 2C, 3D 
 
 
ARTRITIS SÉPTICA 
DEFINICIÓN 
Generalmente causada por S. aureus. Su tratamiento es una urgencia médico‐quirúrgica 
Presentación crónica se debe más a micobacterias u hongos filamentosos.   
Generalmente se debe a contaminación hematógena a partir de un foco primario,   
 
SALUD PÚBLICA 
75% son cocos gram+ (mas fre S. aureus) 
20% son bacilos gram‐ 
FR: catéteres intavasculares, cirugía de trauma, inmunodeficiencia (DM, LES, alt hepáticas, UDVP), hepatitis C, hipotiroidismo, cirrosis, 
EPOC. 
 
DIAGNÓSTICO 
Clínico: fiebre, disminución de la función, dolor progresivo, calor, tumefacción. 
Paraclínico: pedir BHC, VSG, PCR.   
Punción articular 3 tubos 
1) 5‐10ml en tubo estéril ==> cultivos y gram 
2)5ml con heparina o EDTA ==> citológico y bioquímico (glucosa, proteína y lactato) 
3)1ml en tubo seco para observar coagulación ==> sinovial normal no coagula.   
 
Si cultivos normales, sospecha de infección==> biopsia.   
Si sospecha de artritis séptica===> 2 hemocultivos 
 
Estudio de imagen inicial ==> rx (normal si reciente inicio) 
Estudio de imagen más específico===>USG (pedir si rx normal) 
 
TRATAMIENTO 
Si artritis séptica o sospecha: drenaje + AB 
Sin FR:     
  Dicloxacilina    >40kg: 1‐2hr IV c 6h 
      <40kg: 200mgkgd div 4 dosis 

  Amikacina *  >40kg: 1gr c 24h 
      <40kg: 20mgkgd c 24 
De forma IV por 14 días y prolongarse 1‐2 semanas VO con: 
  Dicloxacilina  >40kg: 500mg VO c6h 
      <40kg: 25mgkgd c6h 
 
 
Si alergia a penicilina: [se sustituye dicloxa]   
  Clindamicina  >40kg: 150‐300mg c6‐8h 
      <40kg: 20‐40mgkgd div 3 dosis 
 
Si alto riesgo a gram negativos:   
  Cefuroxima  >40kg: 750‐1500mg IV c 8‐12h 
+      <40kg: 100‐140mgkgd IV div 2‐3 dosis. 
  Amikacina*  >40kg: 1gr c 24h 
      <40kg: 20mgkgd c 24 
De forma IV por 14 días y prolongarse 1‐2 semanas VO con: 
  Cefuroxima  >40kg: 250‐500mg c 12 
      <40kg: 20‐30mgkgd div en 2 dosis 
 
Si alergia a penicilina sustiturir cefuroxima por clindamicina. 
 
Con FR: [hospitalización previa, úlceras, cateteres, hemodialisis, DM, UDVP] 
  Vancomicina  >40kg: 500mg IV c6h 
+      <40kg: 40mgkgd div 4 dosis. 
  Cefotaxima  >40kg: 1gr IV c 8 hr 
      <40kg: 150mgkgd div 3 dosis. 
10‐14 días posterior a drenaje quirúrgico y continuar con: 
  Rifampicina  >40kg:300mg VO c 12h 
  TMP/SMX    >40kg: 160/800 c 12 h 
 
  Rifampicina  <40kg:15mgkg dia VO div    12 hrs 
  TMP/SMX    <40kg: 7mgkgd div cada 12 hrs 
 
Si sospecha de Neisseria gonorrea o Neisseria meningitidis: 
  Ceftriaxona  >40kg: 1gr IM x 7 d 
      <40kg 50mgkgd c 24 hrs x 7 d. 
Médico: 
Sin FR      dicloxa* + amika==> dicloxa 
Si gram ‐    cefuroxima* + amika ==> cefuroxima 
Con Fr    vanco + cefota ==> rifam+TMP/SMX 
Si Neisseria  ceftriaxona 
 
*Si alergia a dicloxa agregar clinda.   
 
Quirúrgico: En todos los casos artrotomía, artroscopia, artroscopia + debridación, aspiración y lavado por punción. 
Artritis séptica del hombro, codo, cadera, rodilla==> lavado artroscópico 
Artritis séptica multifocal==> drenaje abierto 
 
 
TRAUMA DE MIEMBROS 
TRAUMA DE CODO 
PUNTOS BÁSICOS 

 
 
 
 
 
 
DEFINICIÓN 
Consiste en 3 articulaciones: humerocubital, húmero radial y radiocubital. 
 
SALUD PÚBLICA 
Trauma de extermidad superior más frec 1) luxación de hombro, 2) luxación de codo.   

 
 
 
DIAGNÓSTICO 
Lesión      Mecanismo          Síntomas 
Supracondilea humeral      carga axial con codo en flexión <90° 
Epitroclear o epicondilea    tracción de la extremidad            inestabilidad del codo. 
Olécranon     lesión directa sobre codo en flexicón          incapacidad para extensión activa. 
Apófisis coronoides      caída en flexión y luxación posterior del codo          dolor cara anteior de codo. 
Cabeza y cuello del radio    aída sobre la mano con codo en extensión o semiflexión  dolor anteroexterno del codo   
                  (prosupinación lo aumenta) 
Luxación del codo      caída sobre la mano en hiperextensión        dolor en art e incapacidad funcional 
 
Corroborar PA con codo en extensión 
  Lateral con codo a 90° de flexión 
  Sin férula 
 

 
 
 
 
 
 
TRATAMIENTO 
De primer contacto: 
Cerrada: inmovilización braquipalmar   
Abierta: cubrir con gasa estéril seca 
2do nivel de atención: 
Abierta estable tipo I: férula braquipalmar a 90° de flexión 
Cerrada inestable tipo I: ostesíntesis con fijación interna ? 
Abierta inestable tipo I: osteosíntesis con fijación externa. 
Luxación abierta de codo, reducir. 
 
Cierre de la herida. 
Luxaciones o fracturas del codo abiertas tipo I y II: cierre primario y drenaje 
Luxaciones o fractuas de codo abiertas tipo III: diferir 
 
Farmacológico 
Fractura abierta de codo, inciar AB en las primeras 6 hrs.   
  Tipo I  1) cefaloptina, 2) ciprofloxacino 
  Tipo II, III  1) cefalotina + amilkacina 
Si riesgo de anaerobios (granjas, zona rural) 
  Agregar penicilina sódica crisalina o metronidazol 
 
Aseo quirúrgico y debridación inmediata.   
 
Tratamiento cerradas: yeso por 4 semansa, si parcialmente desplazadas sedar y reducir, si desplazmiento importante quirúrgica.   
Las luxaciones solo yeso x 2 semanas.   
 
CONTENIDO ADICIONAL CLAVE 
Tendinitis 
Epicondilitis: dolor en epicóndilo (parte externa del codo) que aumenta con extender codo o muñeca (spiderman) 
  Evitar extensión y supinación del antebrazo 
Epitrocleitis: dolor en epitróclea que aumentacon flexicón de la muñeca. 
  Evitar flexión palmar de la muñeca y pronación del antebrazo.   
Si no mejoría en 2 semanas refernecia a 2do nivel, ==> esteroides inyectados localmente, rehabilitación, si 6 meses y no mejoría Qx 
 
Bursitis 
Aumento de    volumen en cara posteior de codo.   
Tx aines, inmovilización a 45° sin aplicar presión. si no mejora==> aspiración de bolsa preolecraneana, estudio de tinción, búsqueda de 
cristales.   
Si infecciosa dicloxacilina 500mg cada    6hrs x 10 días.   
2da TMP/SMX 
 
ESGUINCE DE TOBILLO 
Hueso Ligamento Hueso 
Hueso Tendón Músculo 
DEFINICIÓN 
Ruptura total o parcial de ligamentos en la articulación del tobillo.   
Rx AP, lateral de tobillo. 
        AP en rotación medial de 15‐30 grado en el tobillo lesionado.   
Solicitar siempre y cuando cumpla con reglas de Ottawa 
 
 
SALUD PÚBLICA 
Mecanismo común (supinación, aducción, inversión en flexión plantar) 
 
DIAGNÓSTICO 
 
 
TRATAMIENTO 
Piroxicam tabletas e 40mg. Tomar 1 tableta cada 12 hrs por 3 días, posteriormente cada 24 hrs 4 días. 
Paraccetamol.   
Si edema bimaleolar y dolor importante combinar AINE + paracetamol (analgésico) 
 
En las primeras 72hrs. 
  Hielo local cada 20 min x 8 hrs 
  Ventaje elástico no compresivo inicial 
  Movilización activa moderada 
Despúes    delas 72 hrs 
  Vendaje compesivor. 
  Ejercicios de propiocepión (arrugar toalla con los dedos, rodar botella) 
Inmovilización c férul d yeso esguinces grado I y II.   
No calor local ni masaje directo. 
 
REHABILITACIÓN 
Ameritan envío: 
Luxación y fractura asociada. 
Esguince grado III y IV 
 
 
 
FRACTURA INTRACAPSULAR DEL EXTREMO PROXIMAL DEL FÉMUR 
DIAGNÓSTICO 
Dolor inguinocrural a la movilización, imposibilidad para elevar el talón, rotación extera y acortamiento. 
RX Anteroposterior de cadera con rotación medial de 15 grado de la cadera afectada. 
Axial de cadera afectada. 
 
TRATAMIENTO 
Cefalosporina 1 o 2 da generación, 1 dosis antes, 2 después de cirugía. 
Antitrombosis: 12 hrs previas y 12 hrs posterior x 10 días con: enoxaparina o nadrparina   
Anaglesia:Metmaizol, ketorolaco, clorhidato de buprenorfina, diclofenaco.   
 
Reducción abierta con fijación interna en adultos mayors con actividad física, si falla==> sección artroplastica o hemiartroplastía. 
Fractura no desplazada, impactada que sea ambulatorio=> fijación interna anes de desplazmaiento, 
Fractura desplazada: hemiartroplastia primaria si adulto mayor a considerar reemplazo total. 
 
 
  
SÍNDROME DE HOMBRO DOLOROSO 
DIAGNÓSTICO 
Manguito rotador: dolor en región deltoidea con limitación de abducción, rotaciones. 
  Test de Neer, Hawkins‐Kennedy, Yocum 
Tendinitis bicipítal: dolor en cara anterior que irradia a tendón bicipital hasta antebrazo.   
  Test de speed, Yergason. 
Bursisits: dolor en tercio superior de hombro, limitación de abducción pasiva y activa. 
 
Si sospecha de rotura de manguito rotador==> USG o RMN 
 
 
TRATAMIENTO 
1era Anaglésico simple, 2da AINE, 3e 
 
[TENDINITIS DEL MANGUITO ROTADOR][r] 
MASCULINO DE 33 AÑOS DE EDAD, TENISTA PROFESIONAL DESDE LOS 15 AÑOS, PRESENTA DOLOR INTENSO EN AMBOS HOMBROS 
PREDOMINANDO LADO DERECHO, ASI COMO INCAPACIDAD PARA LA MOVILIZACION. ACUDE A LA CONSULTA Y LE DIAGNOSTICAN 
TENDINITIS DEL MANGUITO ROTADOR. 
 
LA CAPSULITIS ADHESIVA QUE ES SECUELA DE LA TENDINITIS DEL MANGUITO ROTADOR SE PUEDE PREVENIR: 
A) EVALUANDO LA MOVILIDAD CONTRARESISTIDA. 
B) EVITANDO LOS SOBREESFUERZOS. 
C) INFILTRANDO ESTEROIDES. 
D) INMOVILIZANDO BREVEMENTE EL HOMBRO. 
 
RESPUESTAS D 
Manguito de los rotadores: conjunto de músculos y tendones conectados a la cabeza del húmero y dan estabilidad al hombro.   
Supraespinoso  Fosa supraespinosa de la escápula ‐ tubérculo mayor del húmero.  Permite primeros 15‐20° de abducción 
Inftraespinoso    Fosa infraespinosa de la escápula    ‐    tubérculo mayor del húmero.  Rota brazo lateralmente 
Teres minor  Borde lateral de la escápula  ‐ tubéculo mayor del húmero    Rota brazo lateralmente 
Subescapular  Fosa subescapular ‐ tubérculo menor del húmero      Rota brazo medialmente 
 
Estos tendones pueden lesionarse con traumatismo o movimientos viciosis.   
 
La capsulitis adhesiva es la adhesión real de la cápsula del hombro a la cabeza del húmero, ocasionando una restrición del movimiento 
del hombro.   
Fase 1‐Dolorosa: dolor nocturno incidioso. 2‐9 meses 
Fase 2‐ Congelada o adhesiva: se agrega incapacidad para moverse a gran amplitud. 3‐9m 
Fase 3‐ Descongelación o regresiva: mejora dolor y limitación   
 
Evitar la inmovilización prolongada en pacientes que pueden estar predispuestos a desarrollar capsulitis adhesiva es importante. La 
infiltración de esteroides puede ayudar en el tratamiento pero no es de primera elección.   
Deberá inmovilizarse por un periodo no mayor a 15 días y se deberá anexar tratamiento analgésico y antiinflamatorio. Posteriormente 
rehabilitación. De hecho más que inmovilización debe preferirse reposo.   
 
CERVICALGIAS, LUMBALGIAS Y TUMORES 
CERVICALGIAS 
DEFINICIÓN 
Cervicobraquialgia cuando se irradia a ext superiores a través de raiz nerviosa cervical baja (C5‐C8). 
 
DIAGNÓSITCO 
AP y lateral de columna cervical   
TAC si origen traumático y no se visualiza C7   
 
LUMBALGIAS 
DIAGNÓSTCO 
Clasificar en: 1) lumbalgia inespecífica, 2) lumbalgia potencialmente asociada con radiculopatía, 3) lumalgia pontencialmente asocaida 
con enfermedad sistémica. 
  Pensar en neo si >50 años, ant de Ca, pérdida de peso, no mejora en4‐6semanas, dolor continuo o progresivo, dolor en reposo y 
nocturno.   
 
95% de casos de lumbalgia aguda es inespecífica, >50 tendrá alteraciones pr Rx.   
Pedir Rx AP de columna lumbar y lateral si fiebre x más de 48 hrs, osteoporosis, enf sistémica, déf sensitivio o motor, uso crónico de 
esteroides, >50 años, inmunosupresión, sospecha de espondilitis anquilosante.   
 
Prueba de Lassegue si sospecha de radiculopatía 91% sensible, 26% específica. Si duele entre los 30‐70° ==> hernia de disco. (si levanta 
la pierna opuesta y produce dolor 29 sensibilidad y 88 especificidad.   
Paresia aporta E 91% para compresión 

 
 
Retención urinaria 90% S, 95% E, y anestesia en silla de montar==> cauda equina 
<45 años, dolor que mejora con el movimiento, rigidez mayor de 3min?==> inflamatorio 
 
TRATAMIENTO 
1era línea paracetamol 
2 AINE periodos cortos menor tiempo posible (ibuprofeno tiene menos efectos gastrointestinales) 
No se recomienda AINE >4 semanas. 
3era opiaceos menores 
Relajante muscular no benzodiacepínico, no más de 2 semanas especialmente si espasmo muscular. 
Lumbalgia crónica con fracaso a tx convencionales antidepresivos tricíclicos.   
NO hay evidencia de que la vitamina B sea eficaz para el tx de dolor lumbar inespecífico.   
Masaje disminuye dolor y aumenta capacidad funcional. 
Aplicación de calor local. 
 
 
[LUMBALGIA] [r] 
MUJER DE 32 AÑOS DE EDAD, INSTRUCTORA DE UN GIMNASIO, CON ALCOHOLISMO Y TABAQUISMO POSITIVOS DESDE HACE 10 AÑOS. 
ACTUALMENTE SIENDO TRATADA POR EL SERVICIO DE PSIQUIATRÍA POR DEPRESIÓN. ES ENVIADA CON USTED PARA DESCARTAR 
PATOLOGÍA ORGÁNICA. REFIERE DOLOR EN REGIÓN LUMBAR DE 5 SEMANAS DE EVOLUCIÓN, EL CUAL ES INTENSO, INCAPACITANTE, 
AUMENTA CON CAMBIOS DE POSICIÓN Y SE IRRADIA A REGIÓN GLÚTEA DERECHA. 
 
EL SIGUIENTE ES EL PRINCIPAL FACTOR DE RIESGO PARA LUMBALGIA QUE DEBEMOS CONSIDERAR EN LA PACIENTE: 
   
A) ALCOHOLISMO 
B) EJERCICIO EN GIMNASIO 
C) DEPRESIÓN 
D) TABAQUISMO. 
  
CON BASE AL TIEMPO DE EVOLUCIÓN CLASIFICAREMOS LA LUMBALGIA DE LA PACIENTE COMO: 
A) AGUDA 
B) SUBAGUDA 
C) CRÓNICA 
D)RECIDIVANTE 
 
RESPUESTAS A, B 
Existe asociación entre malos hábitos alimenticios, sedentarismo,    sobrepeso, obesidad, alcoholismo y    posiciones visciosas con 
lumbalgia.    No hay evidencia de que el tabaco sea factor de riesgo para el desarrollo de dolor lumbar.   
El ejercicio benéfico y parte del tratamiento. 
La depresión favorece la cronicidad, es de decir la persistencia más no la causa.   
 
Respecto al tiempo de evolución: 
  Aguda si <6semanas.   
  Subaguda 6‐12 semanas 
  Crónica >12 semanas 
 
 
[r] 
MUJER DE 28 AÑOS DE EDAD, DIRECTORA DE UN JARDÍN DE NIÑOS. ACUDE SOLICITANDO ATENCIÓN MÉDICA YA QUE, TRAS CARGAR A 
UN NIÑO EL DÍA DE AYER, PRESENTA DOLOR INTENSO EN REGIÓN LUMBAR QUE LE INCAPACITA PARA REALIZAR SUS LABORES. LLAMA 
LA ATENCIÓN SU ACTITUD IMPACIENTE Y NERVIOSA, MUESTRA MOVIMIENTOS INVOLUNTARIOS EN PÁRPADO DE OJO DERECHO. 
COMO PARTE DE SU TRATAMIENTO ESTARÁ INDICADA EN ESTE MOMENTO LA REHABILITACIÓN MEDIANTE: 
   
A) LA APLICACIÓN DE CRIOTERAPIA 
B) EL USO DE SOPORTES LUMBARES 
C) LA REALIZACIÓN DE LOS EJERCICIOS DE WILLIAMS 
D) LA INDICACIÓN DE TERAPIAS COGNITIVO CONDUCTUALES 
 
RESPUESTA C 
La aplicación de calor local en lumbalgia aguda y subaguda,    solo se contraindica en caso de alt sensitivas, escaras, alt de coagulacióon 
y procesos neoplásicois. La crioterapia no va al caso.   
El uso de soportes lumbares no tienen efectividad comprobada. 
Los ejercicios de flexión o ejercicios de Williams están indicados para el tx del sx doloroso lumbar. 
La etapa cognitivo conductual no se recomienda para el tratamiento del dolor en etapa aguda, si en subagudo, previene cronicidad y 
recurrencias.   
[La pregunta claramente dice rehabilitación, eso hace pensar que es tx a largo plazo y posterior ¬¬] 
 
 
 
OSTEOSARCOMA 
 
DEFINICIÓN 
Tumor óseo más frec en la niñez y adolescencia. 
 
SALUD PÚBLICA 
10‐20 años. Si población negra o 0‐10 años descartar primero sarcoma de Ewing. 
 
PATOGENIA 
45% de todos los sarcomas óseos (neoplasia de cels fusiformes que produce matriz osteoide [hueso nimineralizado] o hueso. 
 
DIAGNÓSTICO 
Generalmente afecta metáfisis de huesos largos. 
Frec en porción distal del fémur, zona proximal de tibia y húmero.   
Síntomas: aumento de volumen en región afectada, puede no haber afección de tej blandos, hay disminución de movimiento y dolor. 
(En ostemielitis primero se ve la afección a tejidos blandos y después al hueso, también tiende a haber más fiebre o focos infecciosos a 
distancia) 
Paciente generalmente acude por dolor e inflamación.     
Paraclínicos: rx simple   
  lesión lítica de aspecto apolillado,   
  reacción perióstica con espículas (imagen en sol naciente),   
  triangulo de Codman 
TC útil para definir destrucción ósea y calcificación, RMN útil para precisar etrensión intramedular y afección a tejidos blandos. 
Rx y TC para descubrir mets pulmonares.   
Gammagrafía para descubrir mets óseas.   
Confirmación anatomopatológica: punción biopsia con aguja, y de ser posible con biopsia a cielo abierto.   
 

 
 
TRATAMIENTO 
QT + intervención para conservar de miembro    (>80% de los pacientes). 
QT==> doxorrubicina, ifosfamida, cisplatino, metotrexato + ac folínico.   
 
Osteosarcoma es radiorresistnete.   
 
PRONÓSTICO 
Principal factor pronóstico es la respuesta a QT 
Sobrevida a 5 años, 60‐70%. 
 
CONTENIDO ADICIONAL 

 
 
Diáfisis    Médula ósea 
Metáfisis    Cartílago de crecimiento 
Epífisis    Articulación, médula ósea. 
 
 
OSTEOSARCOMA 
MASCULINO DE 19 AÑOS, REFIERE DOLOR EN MUSLO IZQUIERDO INCREMENTÁNDOSE POR LAS NOCHES. NO HAY RELACIÓN CON LA 
ACTIVIDAD FÍSICA Y SE NIEGA FIEBRE O PÉRDIDA DE PESO. A LA EXPLORACIÓN SE PALPA MASA DURA Y FIJA A PLANOS PROFUNDOS EN 
TERCIO DISTAL DE FÉMUR. LA RADIOGRAFÍA PRESENTA LESIÓN PENETRANTE EN FÉMUR CON DESTRUCCIÓN CORTICAL CON ASPECTO DE 
"RAYOS SOLARES". 
 
EL DIAGNÓSTICO PROBABLE DE ESTE PACIENTE ES EL DE: 
A) OSTEOSARCOMA. 
B) QUISTE ÓSEO. 
C) OSTEOMIELITIS. 
D) OSTEOCONDROMA 
 
LOS HALLAZGOS RADIOLÓGICOS PRESENTES EN EL PACIENTE SON SECUNDARIOS A: 
A) INFECCIÓN EN UN SITIO CERCANO AL HUESO. 
B) DISPLASIA ÓSEA Y PLEOMORFISMO CELULAR METAFISIARIO. 
C) REMANENTE DE LA PLACA DE CRECIMIENTO DEL CARTÍLAGO 
D) INFECCIÓN QUE VÍA HEMATOGENA LLEGA A LA ARTICULACIÓN. 
 
RESPUESTAS 
A, B, 
 
 
MUJER  DE  23  AÑOS  DE  EDAD  PRESENTA  RADIOGRAFÍA  CON  DATOS  DE  TUMORACIÓN  MUTILOCULADA  EN  LA  ZONA  EPIFISARIA  DEL 
EXTREMO DISTAL DE FÉMUR IZQUIERDO. ¿CUÁL ES EL DIAGNÓSTICO MÁS PROBABLE? 
A) SARCOMA OSTEOGÉNICO 
B) TUMOR DE CÉLULAS GIGANTES 
C)SARCOMA DE EWING 
D)MIELOMA MÚLTIPLE 
  
RESPUESTA B 
EL ASPECTO MULTILOCULADO O EN POMPAS DE JABÓN ES CARACTERÍSTICO DEL TUMOR DE CÉLULAS GIGANTES, MUY FRECUENTE EN 
MUJRES DE 20‐40 AÑOS] CTO TRAUMATLOGÍA PAG 56 
El tumor de células gigantes es un tumor raro, constituye el 5% de los tumores óseos primarios. Ocurre en pacientes entre los 20 y 40 
años, siendo raro en los menores de 10 y en mayores de 50 años. Tiene un ligero predominio del sexo femenino y más frecuente en países 
orientales que occidentales. Habitualmente se loalizan en huesos largos, con afectación de la epífisis y de localización excéntrica, pueden 
propagarse a metáfisis, provocar destrucción cortical y extensión eventual a tejidos blando y al especio articular.   
Los tres sitios más habituales de localización en orden de frecuencia son: 1 extremo distal del fémur, 2 extremo proximal de tibia, 3 
extremo distal del radio.   
Clínicamente, el dolor es el signo más freucente de presentación, tanto si se asocia o no a fractura patológica. Localmente la piel puede 
estar hiperémica, eritematosa y si el tumoer crece puede aparecer ciruclación colateral con presencia de masa palpable. La clínica es 
infrecuente y si el TCG tiene localización raquídea o sacra presenta signos y síntomas neurológicos.   
El  diagnóstico  de  TCG  suele  realizarse  por  laws  manifestaciones  clínicas  y  radiográficas,  teniendo  en  cuenta  lo  spisible  sdiagnóstico 
diferenciales  Cmo  pruebas  complementarias,  la  RMN  es  el  método  más  aprovechoso  para  determinar  la  extensión  y  l  estadiaje,  la 
gammagrafía es uilizada para detectar TCG multicéntricos que son raros (<1%) m y los parámetros de laboratorios suelen ser normales.   
 
 
OSTEOSARCOMA PARTE2 
Epidemiología:Tumor maligno más frecuente en pacientes de 10‐25 años de edad. 
Localización: metáfisis de huesos largos (distal del fémur, proximal de la tibia) 2do lugar metáfisis proximal del húmero.   
Clasificación: hay 2 tipos el primario y secundario.   
El primario se origina ahí, el secundario es metástasis (1. Ca de pulmón, 2. Paget, 3. retinoblastoma) 
 
Clínica: Dolor progresivo, nocturno no cede con AINES , cede con paracetamol y actividad. Tumefacción (aumento de vol y temp). 
Fracturas en terreno patológicos. 
 
Paraclínicos: 
Rx: Estudio de primera elección siempre. 1) Destrucción trabecular de metáfisis, 2) patrón moteado, 3)Triángulo de Codman (invasión a 
la cortical pero sin romperala), 4) imagen en sol naciente (rompió hueso, cortical, invadio musculo nervios o vasos).   
RM es el siguiente estudio, valora extensión.   
DHL y FA elevados (tienen valor pronóstico). A mas elevados peor pronóstico. 
Gold estándar: histopatológico. Células fusiformes y trabéculas óseas.   
 
TX:    Solo resección y curetaje, QT neoadyuvante?. Nunca RT. 
 
PACIENTE FEMENINO DE 14 AÑOS DE EDAD QUE ACUDE A LA CONSULTA POR REFERIR UNA CAÍDA SOBRE RODILLA DERECHA HACE 3 
MESES. DESDE ENTONCES PADECE DOLOR INTERMITENTE SORDO QUE DISMINUYE CON LA ACTIVIDAD FÍSICA Y AUMENTA CON EL 
REPOSO Y POR LAS NOCHES.    LAS ÚLTIMAS DOS SEMANAS REFIERE AUMENTO DE VOLUMEN, TEMPERATURA Y RUBOR. LA INGESTA 
DE AINES O ANALGÉSICOS NO MEJORAN SUS SÍNTOMAS. A LA EF SE OBSERVA AUMENTO DE VOLUMEN EN 1/3 DISTAL DE RODILLA 
DERECHA, FIJO A PLANOS PROFUNDOS, NO MOVIL, DE CONTORNO IRREGULAR, ASÍ COMO AUMENTO DE VOLUMEN Y DOLOR A LA 
PALPACIÓN.   
 
CON LOS DATOS CLÍNICOS ARRIBA MENCIONADOS EL ESTUDIO DE PRIMERA ELECCIÓN QUE USTED SOLICITARÍA ES: 
A) RM CON MEDIO DE CONTRASTE 
B) RESONANCIA MAGNÉTICA SIMPLE 
C) TAC 
D) RX SIMPLE AP Y LATERAL 
E) USG 
 
La rx y clínica confirman el 90% de los daignósticos. Excelente opción costo‐beneficio. 
 
TENIENDO YA UN DIAGNÓSTICO PRESUNCIONAL LA IMAGEN QUE USTED ESPERA OBSERVA EN EL ESTUDIO SOLICITADO ES:   
A) IMAGEN HIPERINTENSA CON CAPTACIÓN EN CÓNDILO LATERAL DE FÉMUR 
B) PATRÓN DIFUSO INTRAMEDULAR E IMAGEN EN SOL NACIENTE.   
C) IMAGEN RADIOPACA EN AROS DE CEBOLLA 
D) IMAGEN HIPOECOICA SOBRE EL CÓNDILO LATERAL DEL FÉMUR 
E) IMAGEN HIPODENSA EN METÁSTASIS DISTAL DEL FEMUR. 
 
Si tenemos la certeza de que el estudio de primera elección es una radiografía y dentro de las respuestas se usan términos como 
"captación en cóndilo", "imagen hipoecóica", "hipodensa" podemos eliminar varias respuestas, ya que hacen alusión a contrastes, USG 
y TAC.   
 
CON EL DIAGNÓSTICO YA CONFIRMADO EL TRATAMIENT DE ELECCIÓN PARA ESTE PACIENTE ES: 
A) RT ADYUVANTE, QUIMIOTERAPIA Y CURETAJE.   
B) ARTROSCOPIA DIAGNÓSTICA    Y TOMA DE BIOPSIA 
C) RESECCIÓN TOTAL DE LA TUMORACIÓN 
D) RESECCIÓN CON APLICACIÓN DE INJERTO Y QT 
E) PRIMERO QT, RESECCIÓN TOTAL DEL TUMOR Y RT.   
 
Primero QT y luego QX 
RESPUESTA D, D 
 
SARCOMA DE EWING 
Tumor más frec en niños 5‐15 años. Para el ENARM 5‐10. 
Localizacion: metáfisis de huesos largos (diáfisis de fémur, metáfisis distal de fémur) 
Etiología: traslocación 11‐22 
Aumento de volumen dolor, eritema, aumento de temp y fiebre.    (se puede confundir con osteomilitis) 
DHL, PCR, VSG elvados. 
 
Imagen EN AROS DE CEBOLLA ES CARACTERÍSTICA ES EN SARCOMA DE EWING. Hay error en CENETEC. Hay destrucción difusa de la 
diáfisis y metáfisis, evidente.   
De primera elección rx 
Gold estándar: patológico (cels pequeñas, redonda y de color azul).   
Tx ÚNICO TUMOR ÓSEO RADIOSENSIBLE. Resección quir´rugica y curetaje del tumor. Qx y RT neoadyuvante.   
 
 
 
PATOLOGÍAS ARTICULARES GENERALIDADES 
Una luxación es la pérdida de congruencia entre dos o más superficies óseas que se articulan, no existe el término subluxación, está 
luxada o no lo está. Requieren reducción urgente a la brevedad.   
 
LUXACIÓN GLENOHUMERAL 
DEFINICIÓN 
Es la más frecuentes de todas las luxaciones. 
Localización más frecuente es anterior. 
Común en jóvenes. 
 
CLÍNICA   
Dolor intenso en hombro afectado 
Signo de la charretera. 
Asocia lesión del nervio axilar. ==> anestesia en borde lateral del hombro 
 
DIANGÓSTICO 
Clínico, radiográfico (AP y escápula en Y) 
 
TRATAMIENTO 
Reducción urgente, maniobra de "tracción contracción" es la menos traumática y lesione smenos, maniobra truamática no se 
recomienda.    Cabestrillo + AINES 
 
LUXACIÓN DE CADERA 
DEFINICIÓN 
Por mecanismo de alta energía. 
Localización más frecuente es posterior. 
 
CLÍNICA 
Acortamiento de miembro afectado 
Signo del pudor: rotación interna y flexión de cadera. 
Dolor intenso 
Asocia lesión del nervio ciático.   
 
DIAGNÓSTICO 
Clínico y radiográfico: AP de pelvis.   
 
TRATAMIENTO 
Reducción urgente bajo antesia por gran riesgo de daño vascular con luxación de cadera y posición de pudor.   
 
 
MUJER DE 68 AÑOS DIABÉTICA CONTROLADA QUE ACUDE AL SERVICIO DE URGENCIA POR DOLOR INTENSO EN MT DERECHO AL SUFRIR 
CAÍDA DE 4 ESCALONES Y AL TRATAR DE SOSTENERSE, PRESENTA DOLOR EN HOMBRO DERECHO E INCAPACIDAD FUNCIONAL.   
A LA EF SE OBSERVA CON ASIMETRÍA DEL HOMBRO DERECHO CON RESPECTO AL CONTRALATERAL, PALPACIÓN DE MASA ANTERIOR NO 
MÓVIL Y FIJA A PLANOS PROFUNDOS, MOVILIDAD DEL HOMBRO ABOLIDA POR DOLOR Y ANESTESIA EN SUPERFICIE ANTEROLATERAL 
DEL HOMBRO, NEUROVASCULAR A NIVEL DE LOS DEDOS CONSERVADA.   
 
POR LA PRESENTACIÓN CLÍNICA USTED PENSARÍA QUE EL PACIENTE SUFRE DE: 
A) OSTEOBLASTOMA 
B) FRACTURA EN HOMBRO DERECHO 
C) LUXACIÓN GLENOHUMERAL 
D) LESIÓN DEL PLEXO BRAQUIAL POR DISTENSIÓN 
 
El caso NO nos cuenta traumatismo directo en el MT, sino un mecanismo de rotación externa y abducción forzadas del hombro siendo 
este el mecanismo de lesión que ocasiona luxación.   
 
A LA EF CÓMO INTERPRETARÍA LA ASIMETRÍA ENTRE AMBOS HOMBROS: 
A) VACIAMIENTO DE LA CAVIDAD GLENOIDEA Y DESPLAZAMIENTO HACA EL PLANO ANTERIOR 
B) MASA PROBABLEMENTE DE ORIGEN TUMORAL BENIGNO 
C) FRAGMENTOS DESPLAZADOS DE UNA PROBABLE FRACTURA 
D) HIPOTROFIA MUSCULAR POR UNA PROBABLE LESIÓN NERVIOSA   
 
¿CÓMO DIAGNOSTICARÍA USTED UNA LESIÓN DEL NERVIO AXILAR? 
A) ANESTESIA ESQUINA POSTEROLATERAL DE LA AXILA 
B) ANESTESIA BORDE LATERAL DEL HOMBRO 
C) ANTESIA PRIMER ESPACIO INTERDIGITAL 
D) ANTESTESIA BORDE CUBITAL PALMAR DE LA MANO 
 
RESPUESTA C, A, B   
 
HALLUX VALGUS 
DEFINICIÓN 
Desviación lateral de la falange proximal (puntas hacia afuera de la línea media), y desviación medial del primer metatarsiano (punta de 
los metatarsianos acercándose a línea media). más común en mujeres. 
 

 
CLÍNICA 
Bunion o juanete: prominencia medial dolorosa correspondiente a la cabeza del primer metatarsiano.   
Superposición del primer dedo sober el segundo dedo.   
 
PARACLÍNICA 
Rx pie  AP   
  lateral con apoyo   
  blicua   
 
TRATAMIENTO 
No quirúrgico: calzado en horma ancha, plantilla con soporte longitudina interno. 
Quirúrgico: la única indicación es el dolor, nunca estética.   
 
 
PACIENTE MASCULINO DE 58 AÑOS DE EDAD CON ANTECEDENTE DE HIPERURICEMIA CONTROLADA QUE ACUDE A CONSULTA POR 
SUFRIR DOLOR DE LARGA EVOLUCIÓN EN EL PRIMER DEDO (DEDO GORDO) DEL PIE IZQUIERDO. EL DLOR ES CONTANTE Y AUMENTA 
CON EL USO DE ZAPATOS ANGOSTOS, NO MEJORA CON MEDICAMENTOS ANALGÉSICOS. 
A LA EF SE OBSERVA AUMENTO DE VOLUMEN EN SUPERFICIE INTERNA DE LA ARTICULACIÓN METATARSOFALÁNGICA DEL PRIMER 
DEDO DEL PIE, LIGERAMENTE HIPERÉMICA Y SIN AUMENTO DE TEMPERATURA, MOVILIDAD CONSERVADA NO DOLOROSA. ÁC. URICO 
5.1 MG/DL, GLUCOSA DE AYUNA 148 MG/DL, TA 180/106, FC 80 FR 21 
 
CON LOS DATOS CLÍNICOS OTORGADOS USTED INTEGRA EL DIAGNÓSTICO DE:   
A) ATAQUE AGUDO DE GOTA 
B) GOTA INTERCRÍTICA 
C) JUENETE O HALUX VALGUS 
D) HIPERURICEMIA 
 
COMO TRATAMIENTO DE PRIMERA ELECCIÓN EN ESTE PACIENTE USTED INDICARÍA: M 
A) COLCHICINA 
B) ZAPTOS DE HORMA AMPLIA, AINES Y APARATO ADUCTOR DEL PRIMER DEDO. 
C) COLCICINA MÁS INDOMETACINA 50MG BID 
D) PROBENECID 500MG DOS VECES AL DÍA. 
 
RESPUESTA C, B 
 
 
LESIONES COMUNES EN RODILLA 
 
Las lesiones más cmunes son ligamentarias y de meniscos.   
 
Lesiones ligamentarias: hay dos ligamentos colaterales, el interno (medial) y el externo (lateral).     
  Mecanismo de lesión:    valgo forzado ==> lesión de LCI 
        varo forzado==> lesión de LCE 
  Clínica: no derrame articular, dolor a la palpación medial o lateral, bostezos lateral y     
  medial positivos. Flexión y extensión completas con leve dolor.   
  Rx y RM n indicadas.   
  Tx: Conservador con inmovilización rígida, hielo, AINES. 
 
Ligamento cruzado anteior 
  Mecanismo de lesión: flexión, rotación y valgo forzado.   
  Clínica: derrame articular importante mediato con sangre 
  No dolor a la flexión y/o extensión 
  Prueba de Lachman, cajón anterior y pivot shift positivo.   
  Rx solo para busca fx asociadas.   
  RM Solo si ya no hay inflamación (luego de 10 días aprox) 
  Tx inmovilizar con rodillera rígida, AINES, hielo, analgésicos.   
 
Meniscos: menisco lateral y medial.   
  Mecanismo de lesión: torsión interna o externa con flexión de la rodilla 
  Clínica: derrame articular moderado, tardío, sin sangre, dolor al palpar línea articular. 
  Chasquidos y dolor al flexionar y extender. 
  Rx no indicada 
  RM de primera elección.   
  Tx inmovilizar con rodillera rígida, hielo, AINES. 
 
PACIENTE MASCULINO DE 32 AÑOS DE EDAD SIN ANTECEDENTES DE IMPORTANCIA QUE ACUDE A CONSULTA POR DOLOR EN RODILLA 
IZQUIERDA DE 20 DÍAS DE EVOLUCIÓN. 
AL ESTAR JUGANDO FUTBOL REFIERE HACER UN "QUIEBRE" Y GIRAR HACIA EL LADO EXTERNO CON EL PIE FIJO AL SUELO, EL CUAL 
OCASIONÓ UN AUMENTO DE VOLUMEN LIGERO QUE NO LO IMPOSIBILITÓ PARA JUGAR, SIN EMBARGO UNAS HORAS DESPUÉS REFIRIÓ 
MÁS AUMENTO DE EVOLUMEN Y DOLOR PARA DOBLAR LA RODILLA.   
ACTUALMENTE REFIERE QUE AL BAJAR ESCALERAS O AL BAJAR UNA PENDIENTE LA RODILLA "SE LE VA" A LA EF PRESENTA LIGERO 
AUMENTO DE VOLUMEN, PRUEBAS ESPECIALES PARA MENISCO NO VALORABLES POR DOLOR, ÚNICAMENTE CAJÓN POSITIVO.   
 
EL MANEJO INMEDIATO POSTERIOR A LA LESIÓN SERÍA: 
A) COLOCACIÓN DE CLORURO DE ETILO, FÉRULA TIPO CALZA A LA RODILLA, AINES Y HIELO EN LAS PRIMERAS 2 4HRS.   
B) DEBIDO AL AUMENTO DE VOLUMEN UNA PUNCIÓN DE LÍQUIDO ARTICULAR, INMOVILIZACIÓN RÍGIDA CON RODILLERA Y HIELO. 
C)COLOCACIÓN DE RODILELRA RÍGIDA CON VARILLAS LATERALES, AINES, ANALGÉSICOS Y CALOR LOCAL PARA RELAJAR LA 
MUSCULATURA. 
D) INFILTRACIÓN DE BETAMETASONA Y COLORACIÓN DE RODILLERA RÍGIDA. 
 
POR EL MECANISMO DE ELECCIÓN Y LA SEMIOLOGÍA DEL CASO, USTED PENSARÍA QUE EL PACIENTE TENGA UN DIAGNÓSTICO DE:   
A) LIGAMENTO COLATERAL EXTERNO 
B) MENISCO EXTERNO 
C) FRACTURA CONDRAL 
D) LIGAMENTO CRUZADO ANTERIOR 
 
Estos pacientes puede referir datos de inestabilidad con movimientos de flexión, recuerda que la rodilla " se le va".   
 
¿CUÁL SERÍA EL ESTUDIO DE PRIMERA ELECCIÓN QUE USTED SOLICITARÍA? 
A) RADIOGRAFÍA AP, LATERAL Y OBLICUA DE RODILLA 
B) RESONANCIA MAGNÉTICA DE RODILLA 
C) ARTROCENTESIS DE RODILLA PARA ESTUDIO CITOQUÍMICO 
D) RESONANCIA MAGNÉTICA HASTA DESPUÉS DE 2 SEMANAS.   
 
El estudio de primera elección siempre será la radiografía ya que ante cualquier traumatismo siempre se está obligado a descartar 
alguna fractura, el estudio gold standard es la RM y se indica cuando la inflamación ya disminuyó.   
 
RESPUESTA A, D, A 
PIE PLANO 
DEFINICIÓN 
Pie con arco plano, asocia valgo de talón.   
 
 
Flexible: más frecuente es una variación de la psotura normal del pie. Común en niños pequeños, asintomácitos, rengos de mov 
normales, al elvar dedos del pie muestran una inversión del talón que corrige la deformiad.   
  Tx 
  Mejoran sin tramiento la mayoría. 
  Fisioterapia. 
 
Rígido: aplanamiento persistente de arco con restricción del movimiento. Asocido a enf neuromusculares.   
  Rx: AP (desviación del astrágalo) 
  Lateral, verticalización (flexión plantar) del astrágalo.   
  Tx: corrección quirúrgic.a   
 
PACIENTE MASCULINO DE 6 AÑOS DE EDAD ASINTOMÁICO SIN ANTECEDENTES DE IMPORTANCIA, QUE ACUDE PORQUE LA MADRE 
REFEIRE QUE SU HIJO SE CANSA MUCHO AL CORRER Y QUE METE LOS PIES.    A LA EF CON SV NORMALES, SE OBSERVA UN VALGO DE 
RETRO PIE Y LIGERA TENDENCIA AL EQUINO QUE CORRIGE A LOS 90° DE FLEXIÓN DORSAL, AL ELEVAR LOS DETOS DEL PIE EL VALGO DE 
RETROPIE SE CORRIGE. LAS RADIOGRAFÍAS NO MUESTRAN DATOS PATOLÓGICOS.   
 
EL DIAGNÓSTICO QUE USTED INTEGRARÍA CON LOS DATOS BRINDADOS ES:   
A) PIE PLANO VALGO FLEXIBLE   
B) PIE EQUINO VARO ADUCTO CONGÉNITO (PEVAC) 
C) PIE PLANO VALGO RÍGIDO 
D) COALICIÓN TARSAL.   
 
EL TRATAMIENTO QUE USTED INDICARÍA AL PACIENTE SERÍA:   
A) NADA, Y EXPLICA A LOS PADRES QUE DESPUÉS DEL SEGUNDO DECENIO DE LA VIDA ESTE PADECIMIENTO SE CORRIGE.   
B) CORRECCIÓN QUIRÚRGICA CON FIJADOR EXTERNO Y ALARGAMIENTO DE TENDÓN DE AQUILES. 
C) EJERCICIOS DE ESTIRAMIENTO DEL TENDÓN DE AQUILES Y FORTALECIMIENTO DEL TIBIAL ANTERIOR Y EXPLICAR A LOS PADRES QUE 
DESPUÉS DEL SEGUNDO DECENIO DE LA VIDA ESTE PADECIMIENTO SE CORRIGE 
D) PLANTALLAS CON SOPORTE LONGITUDINAL INTERNO 
 
RESPUESTA A, C 
 
Halux valgus 

 
 
 

 
 
 
 
Talus valgus    Talus varus 

 
 
DISPLASIA DEL DESARROLLO DE LA CADERA 
 
DEFINICIÓN 
Anormalidad de la articulación coxofemoral del niño, involucra el borde acetabular y tercio proximal del fémur, ocasionando displasia y 
luxación durante el desarollo fetal o infantil.   
 
FR: H1:M3 (principal), primípara, pélvico, historia de hiperlaxitud familiar, oligohidramnios, Sx Down, artrogriposis, menos importante 
gemelar.   
El lado izquierdo es el más afectado.   
 
CLÍNICA 
Diagnóstico clínico se divide en 3 etapas 
Del 1er día de nacido a los 10 meses: Maniobra de Ortolani (reducción/Clonk/abducción!) y Barlow    (luxación/aducción) reducción de 
ángulo de movimiento abductor del lado afectado, asimetría en pliegues.   
10‐12 meses: Galeazzi (acostado, médico en los pies, flexiona las rodillas y una se ve más abajo), signo de Pistón (hace movimiento), 
limitación de la abducción, asímetría en miembros pélvicos.   
>12 meses: Signo de Trendelemburg (insuficiencia del glúteo medio), si reviso izquierda pido que pierna izquierda la deje abajo y 
levanté la derecha= se vascula.    Si cae el lado derecho, está luxado el izquierdo.   
 
PARACLÍNICOS 
USG es el método de elección diagnóstica en los primeros 3 meses de vida (muy bueno pero operador dependiente). 
 
Rx gold estándar y primera elección    en términos generales, especialmente en <3 meses.   
Posición de von rosen y neutra, de pelvis.   
 
Linea de Perkins (borde óseo acetabular externo) 
Línea de Hilgenreiner (línea per pendicular a Perkins que una cartílagos trirradiados) 
Línea de Shenton (línea continua que va del cuello femoral al agujeto obturador) 
Ángulo acetabular: normal RN 20°, 6m 20‐25°,      >2 años 30° 

 
 
 
TRATAMIENTO 
Conservador los primeros 6 meses de vida.   
Arnés de Pavlik: durante 3‐5 mses y hasta que se alcance abducción completa.   
Quirúrgico: solo en caso de falla de método conservador.   
>1 año no es candidato a tx quirúrgico 
 
PACIENTE FEMENINO DE 1 SEMANA DE VIDA G1P0C1 POR PRESENTACIÓN PÉLVICA, PRODUCTO DE 39.5SDG, PESO 2.8KG, TALLA 46CM, 
APGAR 9 A LOS 5MIN, NO REQUIRIÓ REANIMACIÓN NEONATANAL. CON ANTECEDENTE DE PRECLAMPSIA, QUE ACUDE PARA REVISIÓN. 
EF TIENE LIGERO CAPUT SUCCEDANEUM EN PARIETAL DERECHO, ESTÁ DORMIDA PERO RESPONDE A LOS ESTÍMULOS. ESTÁN 
PRESENTES TODOS LOS REFLEJOS INCLUYENDO EL DE SUCCIÓN, HAY LANUGO Y PLIEGUES PROFUNDOS EN LAS PALMAS Y PLANTAS. NO 
PRESENTA ALTERACIONES EN CABEZA, CUELLO O ABDOMEN. EL CORDÓN UMBILICAL SIN DATOS DE SANGRADO. EL PRIMER REPORTE 
QUE SE LE DIO A LA MADRE ES QUE SU NIÑA ESTÁ BIEN.   
 
EN SU EF USTED PERCIBE UN CLUNCK EN LA CADERA MIENTRA REALIZA UN MOVIMIENTO DE ADUCCIÓN Y PRESIÓN SOBRE LA 
SUPERFICIE INTERNA DEL MUSLO. ¿CUÁL ES EL NOMBRE DE LA MANIOBRA? 
A) BARLOW   
B) ORTOLANI 
C) GALEAZZI 
D) TRENDELENBURG 
 
Aducción luxa.   
Abducción reduce.   
 
EN SU EXPLORACIÓN USTED PERCIBE UN CLUNCK EN LA CADERA IZQUIERDA MIENTRAS REALIZA UN MOVIMIENTO DE ABDUCCIÓN Y 
PRESIÓN SOBRE LA SUPERFICIE DEL TROCÁNCER MAYOR. ¿CUÁL ES EL NOMBRE DE ESTA MANIOBRA? 
A) BARLOW   
B) ORTOLANI 
C) GALEAZZI 
D) TRENDELENBURG 
 
LA PRUEBA DE TRENDELENBURG POSITIVA TRADUCE: 
A) INSUFICIENCIA GLÚTEO MAYOR 
B) INSUFICIENCIA GLÚTEO MEDIO 
C) INSUFICIENCIA GLÚTEO MENOR 
D) INSUFICIENCIA DEL TENSOR DE LA FASCIA LATA 
 
SON SIGNOS POSITIVOS EN PACIENTES CON LUXACIÓN DE CADERA DIAGNOSTICADA A LOS 5 MESES DE EDAD: 
A) BARLOW, ASIMETRÍA DE PLIEGUES, ORTOLANI, TRENDELENBURG. 
B) BARLOW, ASIMETRÍA DE PLIEGUES, ORTOLANI, PISTÓN 
C) BARLOW, ASIMETRÍA DE PLIEGUES, ORTOLANI, GALEAZZI.   
D) BARLOW,    ASIMETRÍA DE PLIEGIUES, ORTOLANI, LIMITACIÓN DE LA ABDUCCIÓN.   
 
ESTE SIGNO SE CARACTERIZA POR UNA ALTURA DISTINTA A LA QUE SE ENCUENTRAN LAS RODILLAS CUANDO SE JUNTAN LOS PIES DEL 
NIÑO EN DECÚBITO SUPINO, ESTANDO FLEXIONADAS LAS CADERAS Y RODILLAS.   
A) BALOW   
B) ORTOLANI 
C) GALEAZZI 
D) TRENDELENBURG 
 
EL SIGUIENTE NO ES UN FACTOR DE RIESGO PARA DDC: 
A) VARÓN 
B) OLIGOHIDRAMNIOS 
C) ECLAMSPIA 
D) PARTO GEMELAR 
 
EL ESTUDIO DE ELECCIÓN PARA VALORAR LA CADERA NEONATAL ES: 
A) RADIOGRAFÍA DE PELVIS EN POSICIÓN NEUTRA 
B) USG 
C) RADIOGRAFÍA AP DE CADERA 
D) RADIOGRAFÍA LATERAL DE CADERA 
 
EL ESTUDIO DE ELECCIÓN PARA VALORAR LA CADERA A PARTIR DE LOS 3 MESES ES: 
A) RADIOGRAFÍA DE PELVIS EN POSICIÓN NEUTRA 
B) US 
C) RADIOGRAFÍA EN AP DE CADERA 
D) RADIOGAFÍA LATERAL DE CADERA 
 
RESPUESTAS A, B, B, D, C, C?, B, A 
 
 
 
PATOLOGÍA DEL OÍDO 
 
Respecto al tatamiento amoxi es de primera elección cefalosporina de 2da generación es segunda eleccion, y cloranfenicol es de 
tercera. En otits media S.pneumonae tiene 50% de resistencia. TMP/SMX no ha    mostrado ser mejor que amoxicilina, 
 
OTITIS EXTERNA 
ETIOLOGÍA 
Etiología bacteriana en el 90% de los casos, 10% por hongos.   
Patógens más frecuentes: Pseudomonas aeruginosa, S. aureus, estreptococos, bacilos gram negativos. Hongos: Aspergilluis, Candida 
albicans y Candida krusei.   
 
FR: pérdida de la integridad de la piel, alt de la barrera protectora de cerumen, nadadores, conducto estrecho.   
 
DIAGNÓTICO 
Prurito en CAE, rascado, otalgia moderada‐intensa, edema, oclusión del lumen, plenitud ótica, hipoacusia conductiva superficial. Si los 
restos dérmicos==> secreción purulenta. Si persiste pude afectarse la región preauricular, auricular y cervical.   
 
TRATAMIENTO 
a) Limpieza médica 
b) AB con o sin esteroides aplicado cada 3‐4 gts cada 6 hrs en oido afectado.   
De elección son las fluoroquinolonas (ciproflox, ofloxa). También buenos resultados con neumocina ótica + dexametasna + ác. acético.   
Si más infección más allá del CAE cambiar a AB oral (fluoroquinolonas, cefalosporinas) 
 
 
HOMBRE DE 27 AÑOS DE EDAD QUE ACUDE A CONSULTA POR PRESENTAR OTALGIA DERECHA SEVERA. EL PACIENTE REFIERE QUE EL 
DOLOR SE IRRADIA HACIA LA REGIÓN TPM Y TAMBIÉN TIEN PRURITO. NIEGA ANTECEDENTES DE IMPORTNAICA. REFIERE REALIZAR 
NATACIÓN 3 VECES POR SEMANA. FC 67, FR 13, T37.5 TA 114/76. TIENE ERITEMA Y EDEMA DIFUSO EN CONDUCTO AUTITIVO 
DERECHO.   
¿CUÁL ES EL TRATAMIENTO MÁS ADEUCADO PARA ESTE PACIENTE? 
A) AMOXICILINA/CLAGVULANATO ORAL + GOTAS ÓTICAS + AC. AC´TICO 
B) NEOMICINA ÓTICA + DEXAMETASONA + AC ACÉTICO 
C) CIPROFLOXACINO ORAL + AINES 
D) GOTAS ÓTICAS CON DEXAMETASONA Y ÁC. AC´TICO 
 
RESPUESTA B 
Ciertos factores se asocian con la presencia de otitis externa, que es el diangóstico de este caso: la alteración del pH del conducto 
(secundario a exposición a agua como por ejemplo antación), trauma local (cotonetes), limpieza agresiva entre otros. La manifestación 
clínica de esta patología es la hiperestesia locorregional a la manipulación de pabellón auricular (o también conocido como signo del 
trago). La exploración física por medio de otoscopia demuesta un entorno inflamatorio (ritema, edema). De acuero a las guias del 
CENETEC, el tratamiento es a base de gotas óticas que combinan esteroide o antibiótico + ac acético. El uso de ác acético sólo es menos 
efectivo que la combinación previamente mencionada. recordar que los agentes etiológicos más frecuentes son: peusomonas 
aeruginosa, S. aureus y el tratmiento tópico es de elección. 
 
OSTEOMIELITIS DE LA BASE DE CRÁNEO 
DEFINICIÓN 
También llamada ostitis externa maligna o necrosante. Es una infección bacterian de la base del cráneo, más frec en diabético e 
inmunodeficientes.   
Puede probocar septicemia y muerte.   
 
PATOGENIA 
Progresa a celulitis==> condritis==> osteítis==> osteomielitis   
Conforme progresa a lo largo de la base del cráneo afecta pares craneales: V, VI, VII, IX, X, XI, XII.   
Microorganismo más frecuente P. aeruginosa y en segunda Aspergillus.   
 
CLÍNICA 
Otalgia intensa, otorrea, tejido de granulación en piso de CAE, edema, linfadenopatía preauricular, Trismus. Nervio facial es el más 
afectado. Progresión a trombosis del seno venso==> meningitis==> septicemia==> muerte.   
 
DIAGNÓSTICO 
Clínicos, rastreo con TC‐99.    Para seguimiento Galio 67, indio 111. 
 
TRATAMIENTO 
Tx de elección: AB parenterales a largo plazo. Aminoglucósidos, betalactámicos antipseudomonas, ceftazidima. 
 
PACIENTE MASCULINO DE 54 AÑOS DE EDAD, DIABÉTICO MAL CONTROLADO, INCIA SU PADECIMIENTO ACTUAL HACE 
APROXIMADAMENTE 4 SEMANAS CON OTALGIA DERECHA INTENSA Y OTORREA, HACE 2 DÍAS PRESENTA PARÁLISIS FACIAL IPSILATEAL. 
A LA EF ENCUENTRAS DOLOR A LA MOVILIZACIÓN DEL PABELLÓN AURICULAR, ASÍ COMO SIGNO DE TRAGO POSITIVO. LA OSTOSCOPIA 
REVELA EDEMA, HIPEREMIA DEL CONDUCTO AUDITIVO EXTERNO PRÁCTICAMENTE IMPIDE LA VISUALIZACIÓN DE LA MEMBRANA 
TIMPÁNICA, ADEMÁS OBSERVAS TEJIDO DE GRANULACIÓN EN EL PISO DEL CONDUCTO.   
 
EL DIAGNÓSTICO MÁS PROBABLE EN ESTE PACIENTE SERÁ: 
A) OTITIS EXTERNA MICÓTICA 
B) OTITIS EXTERNA AGUDA 
C) OTITIS EXTERNA CRÓNICA 
D) OTITIS EXTERNA MALIGNA U OSTEOMIELITIS DE BASE DE CRÁNEO.   
 
EL TRATAMIENTE DE ELECCIÓN SERÁ: 
A) ANTIBIÓTICOS TÓPICOS 
B) TRATAMIENTO PARA PSORIASIS 
C) ANTIBIÓTICOS ANTIPSEUDOMONAS IV Y CONTROL METABÓLICO 
D) ANTIMICÓTICOS TÓPICOS. 
 
LA CAUSA MÁS PROBABLE DEL PADECIMIENTO ACTUAL SERÁ: 
A) ALTERACIÓN DE LAS BARRERAS DE DEFNSA DE LA PIEL DEL CONDUCTO AUDITIVO EXTERNO 
B) DESCONTROL METABÓLICO 
C) USO CRÓNICO DE ANTIBIÓTICOS TÓPICOS Y DESCONTROL METABÓLICO 
D) PSORIASIS, ECCESMA U OTRA PATOLOGÍA DE BASE. 
 
El descontrola metabólico provoca microangipatía en el oído, aumentando con ello este tipo de infecciones.   
 
RESPUESTA D, C, B 
 
 
OTITIS MEDIA AGUDA 
DEFINICIÓN 
Infección del oido medio <3 semanas. Se define como recurrente si 3 episodios en 6 meses o    4 en 1 año con curación completa entre 
episodios.   
Puede ser supurativa o no supurativa. 
 
EPIDEMIOLOGÍA 
Máxima incidencia en los 2 primeros años de vida, a los 5 años, 90% ha tenido OMA. 
FR: alteraciones craneofaciales, IVRS recurrentes, alergias, estancia en guardías, alimentación en decúbito supino, tabaquismo positivo 
o tabaquismo pasivo, historia de desórdenes inmunol´gicos, RGE, hipertrofia adenoidea. 
 
PATOGENIA 
Disfunción de trompa de Eustaquio.   
Patógenos más frec, S. penumonia 30%, H. influenzae 20‐25, M. catarrhalis 10‐15, SBHGB, S. aureus 
 
CLÍNICA   
Otalgia, plenitud aural, hipoacusia, acúfeno, fiebre. EF membrana timpánica engrosada, hiperémica abombada, niveles hidroaereos, Si 
es supurativa se verá perforación y otorrea.   
 
AAP ha propuesto: 1)presentación aguda, 2)presencia de exudado en cavidad media (abombamiento timpánico, neumatoscopia 
patológica u otorrea, 3)Signos o síntomas inflamatorios como otalgia o enrojecimiento del tímpano. 
 
  DIAGNÓSTICO 
OMA confirmada 
1)Otorrea en últimas 24‐48 hrs 
2)Otalgia en últimas 24‐48 hrs + abombamiento con o sin enrojecimiento 
 
OMA probable 
1)Sin otalgia, exudado + enrojecimiento + catarro 
2)Sin otoscopia, otalgia explícita en niño, predominio nocturno, horas en cama, en lactante catarro reciente.   
 
 
TRATAMIENTO 
Sintomático: para o ibu 
AB: espontánea en 80‐90% 
Afección leve‐moderada 
  Amoxi 80‐90mgkgd div cada 8 x 5‐7d 
       Si fracaso luego de 48‐72 hrs==> amoxi clav 80mg/kg/d div c 8 x5‐10d 
  Si > 2 años sin factores de mal pronóstico, analgésico y revalorar en 48 hrs. 
Afección intensa (>39° o <6meses) 
  AmoxiClav 80mgkgd 
  Si fracaso 48‐72hrs ==> timpanocentesis 
Posible OMA 
  Si leve o moderado expectante 
  Si intenso==> valorar amoxi 
 
Fracaso de tx amoxi==> amoxiclav==> ceftriaxona IM 50mgkgd x3==> trimpanocentesis, cultivo, gram, antibiograma. 
Si alergia no anafiláctica a peni: cefpodoxima proxetilo o cefuroxima axetilo x    5‐10d 
Si alergia anafilactica (tipo 1): azitro o claritromicina. 
 
Si alergia a peni, macrólidos 
Miringotomía si otalgia severa en pacietes tóxicos. 
 
PRONÓSTICO 
Mastoiditis aguda simple puede ser asintomática, dx solo x imagen. 
Mastoiditis aguda con osteitis o periostitis: edema retroauricular (dx x imagen, especialmente TAC) si osteitis quirúrgica 
Parálisis facial: 2da compliación más frec. Buen px, evoluciona con el Tx de la OMA. 
Laberintitis: excepcional. 
meningitis: excepcional, vía hematógena    o por proximidad (rara). 
Absceso cerebral: por extensión local. 
        
 
 
[OTITIS MEDIA] 
MUJER DE 17 AÑOS QUE INICIA SU PADECIMIENTO HACE 7 DÍAS CON LA PRESENCIA DE CEFALEA Y OTALGIA IZQUIERDA. EL DÍA DE AYER 
SE AGREGA FIEBRE DE 38.5°C, HIPOACUSIA. A LA EXPLORACIÓN SE ENCUENTRA MEMBRANA TIMPÁNICA IZQUIERDA INTEGRA, 
HIPERÉMICA, ABOMBADA E INMÓVIL. 
EL DIAGNÓSTICO CLÍNICO MÁS PROBABLE ES: 
   
A) OTITIS MEDIA AGUDA. 
B) OTITIS MEDIA CRÓNICA. 
C) OTITIS SEROSA. 
D) OTITIS NECROTIZANTE. 
 
EL FACTOR DESENCADENANTE MÁS COMÚN EN ESTE TIPO DE PACIENTES ES: 
A) NADAR SIN PROTECCIÓN. 
B) LA ACUMULACIÓN DE CERUMEN. 
C) LAS INFECCIONES DE VÍAS RESPIRATORIAS SUPERIORES. 
D) LA PERFORACIÓN DE LA MEMBRANA TIMPÁNICA. 
 
  ADEMÁS DE ADMINISTRAR ANALGÉSICOS Y ANTIPIRÉTICOS SE DEBERÁ AGREGAR: 
A) TRIMETOPRIMA MÁS SULFAMETOXAZOL. 
B) CIPROFLOXACINA. 
C) AMOXICILINA. 
D) PENICILINA PROCAÍNICA. 
 
RESPUESTA A,C,C 
Otitis media aguda: principal agente etiológico 1) S. pneumoniae, 2)H. influenzae no tipable. Es una enfermedad íntimente relacionada 
con resfraido común del que es complicación habitual. La infección vírica de vías respiratorias altas provoca una toxicidad sobre el 
epitelio respiratorios, causante d discinesia ciliar en la trompa de Eustaquio, con alteración subsecuente del aclaramiento de moco. 
La otitis serosa es muy frecuente en la infancia es una condición muy frecuente, se caracteriza por la pesencia de líquido detrás del 
tímpano. Generalmente se asocia a resfraido o a otitis aguda previa, aunque no significa que el oido esté infectado. Mejora 
espontáneamente en unos meses. 
La osteomilitis de la base del cráneo también conocida como otitis externa maligna o necrotizante. Comineza como infección del 
conducto auditivo externo, lugo del temporal, meningitis, septicemia,    muerte.    Se observa en inmunocomprometidos (diabéticos de 
edad avanzada). Etiología: pseudomonas aeruginosa. Clínica: a parte de los síntomas óticos puede haner hipoacusia, trismus, parálisis 
facial o datos de septicemia. (en sus inicios hay síntomas de otitis externa aguda). 
 
OTITIS MEDIA CRÓNICA 
 
DEFINICIÓN 
Exudado persistente o intermitente de una membrana timpánica perforada o con tubo de ventilación por más de 6 semanas.   
Se divide en:    OMC colesteatomatosa 
    OMC no colesteatomatosa.   
 
La OMC colesteatomatosa: son quistes de inclusión epidérmica en el oido medio o mastoides que contienen productos de descamación 
del epitelio. Un colesteatoma puede ser congénito o adquirido. Dx de colesteatoma por otoscopia, endoscopia o microscopia. 
 
ETIOLOGÍA 
Las bacterias más frecuentes en la OMC son P. aeruginosa, S. aureus, Klebsiella y bacterias anaerobias com Bacteroides sp. 
 
TRAMIENTO   
OMC Colesteatoma. El manejo de colesteatoma congénito o adquirido debe ser mediante resección quirúrgica (mastoidectomía). 
OMC no colesteatomatosa: antibióticos tópicos. Si más de 3 meses ligres de enfermedad sin colesteatma==> timpanoplastia curativa.     
OMC no colestomatomatosa con otorra recurrente; timpanomastoidectmía. 
 
 
 
PACIENTE MASCULINO DE 40 AÑOS DE EDAD QUIEN ACUDE A CONSULTA POR PRESENTAR EPISODIOS DE OTORREA IZQUIERDA 
INTERMITENTE DESDE LA INFANCIA, ADMEÁS REFIERE PÉRDIDA DE LA AUDICIÓN Y ACÚFENO IPSILATERAL.   
A LA EF OBSERVAMOS CAE PERMEABLE, MEMBRANA TIMPÁNICA IZQUIERDA CON RETRACCIÓN DE LA PARS FLÁCIDA Y PRESENCIA DE 
MASA BLANQUECINA DE ASPECTO GRUMOSO EN PARED POSTERIOR QUE EROSIONA CONDUCTO Y AL PARECER TAMBIÉN LOS 
HUESECILLOS. 
 
EL DIAGNÓSTICO MÁS PROBABLE EN ESTE PACIENTE SERÁ: 
A) OTITIS MEDIA CRÓNICA NO COLESTEATOMATOSA 
B) OTITIS MEDIA CRÓNICA COLESTEATOMATOSA 
C) CARCINOMA EPIDERMIOID DE CONDUCTO AUDITIVO EXTERNO 
D) GRANULOMA DE COLESTEROL 
 
DENTRO DE TU PROTOCOLO DIANGÓSTICO DECIDES SOLICITAR UN ESTUDIO DE GABINTE. EL MÁS INDICADA PARA ESTE PADECIMIENTO 
SERÍA: 
A) RESONANCIA MAGNÉTICA 
B) TOMOGRAFÍA SIMPLE DE OIDO 
C) TOMOGRAFÍA CONTRASTADA DE OIDO 
D) GRAMMAGRAFÍA CON TC 99 
 
En caso de infecciones crónicas de oido el estudio de elección sería la tomografía simple, cuando sospechamos de una etilogía tumoral 
entonces está indicado el contraste.   
 
EL TRATAMIENTO INDICADO EN ESTE PACIENTE SERÁ: 
A) RESECCIÓN EN BLOQUE DE HUESO TEMPORAL 
B) MASTOIDECTOMÍA 
C) ANTIBIÓTICOS TÓPICOS Y VIGILANCIA 
D) COLOCAGIÓN DE TUBO DE VENTILACIÓN TIMPÁNICA   
 
El tratamiento de otitis media crónica colesteatomatosa es la resección del colesteatoma, el procedimiento para realizarlo es la 
mastoidectomía en cualquiera de sus variedades. 
 
RESPUESTAS B, B, B 
 
HEMATOMA AURICULAR 
Acumulación de sangre en espacio subpericóndrico, por lo general por traumatismo contuso.   
El cartílago auricular carece de riego sanguíneo, se alimenta del pericontrio. Cuando un traumatismo produce un hematoma dificulta la 
nutrición del cartílago por obstruir la difusión desde el pericondrio.    Necrosis==> infecciones.   
 
PACINETE FEMENINO DE 6 AÑOS DE EDAD, QUE INICIA SU PADECIMIENTO ACTUAL POSTERIOR A CAÍDA DESDE SU BASE DE 
SUSTENTACIÓN, PRESENTANDO UN TRAUMATISMO EN HEMICARA DERECHA, PRESENTA AUMENTO DE VOLUMEN EN PABELLÓN 
AURICULAR DERECHO, LA PIEL SE OBSERVA EDEMATOSA Y CON COLOR ROJO, AL TACTO PRESENTA FLUCTUACIÓN. LA MADRE NIEGA 
FIEBRE, OTORREA U OTORRAGIA.   
 
EL DIAGNÓSTICO MÁS PROBABLE DE LA PACIENTE SERPA: 
A) HEMATOMA 
B) PERICONDRITIS 
C) PERICONDRITIS SUPUADA 
D) ERISIPELA 
 
LA ETIOLOGÍA MÁS PROBABLE EN ESTE CASO CLÍNICO SERÁ_ 
A) INFECCIÓN POR ESTREPTOCOCO BETA HEMOLÍTICO DEL GRUPO A 
B) INFECCIÓN POR PSEUDOMONAS AEURIGINOSA 
C) TRAUMATIMOS CON ACÚMULO SANGUINOLENTO EN ESPACIO SUBMUCOPERICNDRICO 
D) INFECCIÓN POS PSEUDOMONAS MAL TRATADA 
 
LA TERAPÉUTICA INDICADA PARA ESTA PACIENTE ES: 
A) OBSERVACIÓN 
B) DRENAJE DEL HEMATOMA Y FERULIZACIÓN 
C) QUINOLONAS VÍA ORAL 
D) BETALACTÁMICOS 
 
RESPUESTA A, C,B 
 
 
 
 
HIPOACUSIA 
DEFINICION 
El ser humano puede oir desde la 27 sdg, 
ETIOLOGÍA 
Hipoacusia congénita: 50% adquirida, 50% genética. 
 
Infeciosas: TORCH, parotiditis, meningitis. 
Hipoxia neonatal: insuf placentaria y compresión del cordón producen lesión de cóclea y núcleos del tronco. 
Parto prematuro: produce hemorragia y atrofia de la estría vascular 
Hiperbilirrubinemia: se deposita bilirrubina en cóclea y ganglios basales.   
Hereditaria: lo más frecuente (70%)es que se presente de forma aislada.   
Autoinmune:    anticuerpos y linfocitos vs células ciliadas y estría vascular. Puede formar parte de granulomatosis de Wegener, LES, AR, 
hipotiroidismo.   
Trauma acústico 
Tumores (Schwanoma vestibular) 
Fármacos (aminoglucósidos, diurétcos de asa, quimioterápicos. 
 
 
SALUD PÚBLICA 
Discapacidad auditiva es la pérdida de audición mayor a 25 dB.   
25% es genético, 25% no genético, 50% indeterminado.   
En México la hipoacusia es la anormalidad congénita más común en el RN. 3‐1000 NV. 
Hipoacusia congénita: FR: UCI >5 días, ventilación asistida, ototóxicos, diruéticos de ASA, hiperbilirrubinemia, infección intauterina 
(CMV, herpes, rubeola, sífilis, toxo) Malf cráneofaciales, neurofibromatosis, osteopetrosis, Usher, Waardenburg, Alport, Pendrer y 
Jervell y Lange‐Nielson 
 
PATOGENIA 
Investigar prematurez, consanguinidad, hipoxia, suf fetal, hiperbilirrubinemia, historia familiar de niños con pérdida auditiva 
neurosensorial, infevviones in úter (rubeola, CMV, sífilis, toxoplasmosis, herpes). APGAR al minuto de 1‐4, a los ,min de 0‐6, ventilación 
mecánia x más de 5 días, meningitis bacteriana, anomalías cránefaciales, Sx TORCH, ototóxicos.   
 
DIAGNÓSTICO 
Tamiz auditivo. 
Emisiones otoacústicas: identifica la pérdida más no la severidad. Si sale alterado se repite en dos meses (el paciente podría tener 
líquido en oido medio. Si nuvamente sale alterado realizar PAETC 
Potenciales evocados auditivos de tallo cerebal: registra actividad neural en la cóclea. Si se corrobora hipoacusia neurosensorial se 
indican auxiliares bilaterales. A los 6 meses de edad deberá estar adaptado con auxiliares auditivos si lo requiere.    Si al año no logra 
oir, en candidato a implante coclear.    ¿Al año del auxiliar o al año de edad? 
 

 
 
TRATAMIENTO 
Se recomienda implante coclear multicanal bilateral en aquellos que no se beneficien de auxiliar auditivo convencional debido a su 
severidad. 
Candidatos a implante coclear: 
  Pérdida auditiva >90 dB medidos por PPATC en frec de 2000‐4000. 
  Umbrales superiores 65 dB SPL (sound pressure level) en campo libre con uso de audífonos 
  Reconocimiento del lenguaje >40% en condiciones de amplificación. 
  Función y anatomía de oído medio normal 
  Ausencia de cirugía previas en oído medio 
  No alt etrococleares en la vía auditiva 
  Insatisfacción comprobada a audífonos al menos durante 6m. 
  + 
  Prelingüísticos entre 1‐3 años 
  Antecedente de educación oralista y uso de auxiliar auditivo x 3meses. 
  >3 años y hasta preadolescencia solo si ya se sometió a educación oralista y adquisiciones lingüísticas. 
No se recomienda implantar si: 
  >6 años prelingüísticos 
 
Descartar con estudios de imagen: anomalias del VII par, dilatación del acueducto coclear, monitoreo del VII par transoperatorio. 
 
CONTENIDO ADICIONAL CLAVE 
EO si no pasa ==> 2da EO si no pasa===> 2 y 3er nivel==>ORL/audiólogo==> PPTAC 
 
PACIENTE MASCULINO DE 1 AÑO 6 MESES DE EDAD, LOS PADRES ACUDEN A CITA PORQUE REFIEREN QUE EL NIÑO SOLO DICE 
ALGUNAS PALABAS, PAPA, MAMA, AGUA, LA    MADRE ADEMÁS, MENCINA QUE PARECE QUE EL NIÑO TODO EL TIEMPO ESTÁ 
IRRITABLE Y NO LE HACE CASO. CUANDO INTERROGAS A LOS PAPÁS ELLOS TE COMENTAN QUE EL PACINTE NACIÓ A LOS 7 MESES, CON 
SUFRIMIENTO FETAL Y ESTUVO EN LA TERAPIA INTENSIVA DURANTE 15 DÍAS. SOSPECHAS QUE EL PACIENTE TIENE HIPOACUSIA Y POR 
ELLO LA FALTA DE DESARROLLO DEL LENGUAJE.   
 
PRIMER ESTUDIO QUE SOLICITAMOS EN ESTE PACIENTE SERÍA: 
A) ELECTROCOCLEOGRAFÍA 
B) POTENCIALES AUDITIVO EVOCADOS DE TALLO CEREBRAL 
C) AUDIOMETRÍA TONAL 
D) TAMIZ NEONATAL EXTENDIDO 
 
EL ESTUDIO TE REVELA UNA HIPOACUSIA SENSORIAL SEVERA BILATERAL. LO MÁS RECOMENDADO PARA ESTE PACIENTE SERÁ: 
A) OBSERVAR Y ESPERAR MEDURACIÓN DEL SISTEMA AUDITIVO 
B) ADAPTACIÓN DE AUXILIARES AUDITIVOS 
C) ADAPTACIÓN DE AUXILIARES AUDITIVOS Y TERAPIA DE LENGUAJE 
D) COLCACIÓN DE IMPLANTE COCLEAR 
 
Por la edad requiere adaptación inmediata de auxiliar auditivo además de terapia de lenguaje. Si no logra oir==> implante coclear 
 
LA ETIOLOGÍA MÁS PROBABLE DE LA HIPOACUSIA EN ESTE PACIENTE SERÁ: 
A) CONGÉNITA SINDRÓMICA 
B) ENFERMEDAD DE MNDINI 
C) MULTIFACTORIAL 
D) OTOTOXICIDAD 
 
RESPUESTA B, C, C 
 
 
[HIPOACUSIA][r] 
  MUJER DE 48 AÑOS DE EDAD, QUE ACUDE A LA CONSULTA EXTERNA AL PERCIBIR DISMINUCIÓN DE LA CAPACIDAD AUDITIVA. 
PARA SOLICITAR E INTERPRETAR UNA AUDIOMETRÍA, ES NECESARIO SABER QUE LA ESTRUCTURA QUE INTERVIENE EN EL AJUSTE DE LA 
IMPEDANCIA DEL SONIDO ES: 
A) LA MEMBRANA TENTORIA. 
B) LOS HUESECILLOS. 
C) EL MÚSCULO ESTAPEDIO. 
D) EL ÓRGANO DE CORTI. 
 
RESPUESTA B 
Las huesecillos tienen la función de ajustar la impedancia aire‐liquido para transmisión del impulso auditivo.   
El músculo estapedio se inserta en el cuello del estribo y jala de él, no contribuye por si solo a la impedancia.   
Una vez que el impulso sonoro llega a la ventana oval en el oido interno se produce movimiento de la perilinfa 
La lámina tectoria esta por encima del órgano de Corti, las células pilosas externas está embebida en el lámina tectoria, constituye la 
formación del estímulo acústic cuando se produce deflexión en ella 
El órgano de corti está en la cóclea y transforma vibraciones en impulsos nerviosos. 

 
 

 
 
 
 
 
 
 
 
PRESBIACUSIA 
DEFINICIÓN 
Pérdida auditiva por cambios degenerativos de la edad. Tiene un compente periférico y central.   
Es un diagnóstico de exclusión. 
 
CLÍNICA 
Edad 60‐70 años, 
Pérdida de audición y regresión fonémica. 
Bilateral, lentamente progresiva.   
Alteraciones del equilibrio (presbiestasia) 
 
DIAGNÓSTICO 
Audiometría tonal. 
Logo audiometría: es la prueba más importante donde se obsrva una grave deterioro del umbral de inteligibilidad    y del umbal de 
máxima discriminaci´nde la palabra.   
 
TRATAMIENTO 
No existe médico ni quirúrgico efctivo.   
Protésico en caso necesario.   
 
CONTENIDO ADICIONAL CLAVE 
 
        Weber      Rinne    Schawabach 
Hipoacusia neurosensorial  Lateraliza al normal  Positiva    Acortado 
Hipoacusia de conducción  Lateraliza al enfermo  Negativa  Normal‐Prolongada 
 
PACIENTE MASCULINO DE 65 AÑOS DE EDAD, QUIEN INICIA SU PADECIMIENTO ACTUAL DESDE HACE 5 AÑOS CON HIPOACUSIA 
BILATERAL, MENCIONA QUE LE CUESTA TRABAJO ESCUCHAR EN AMBIENTES RUIDOSOS, TAMBIÉN PRESENTA ZUMBIDO DE OÍDOS. 
REFIERE LEVE EXPOSICIÓN A RUIDO LABORAL Y QUE ES INGENIERO, NO PRESENTA ANTECEDENTES FAMILIARES DE PÉRDIDA DE 
AUDICIÓN, NI INFECCIONES OTOLÓGIAS DE REPETICIÓN.   
 
DIAGNÓSTICO MÁS PROBABLES: 
A) OTOSCLEROSIS 
B) TRAUMA ACÚSTICO 
C) PRESBIACUSIA 
D) ACÚFENO IDIOPÁTICO 
 
¿CÓMO ESPERARÍAS ENCONTRAR LA ACUAMETRÍA EN ESTE PACIENTE? 
A) WEBER CENTRAL, RINNE NEGATIVO BILATERAL 
B) WEBER CENTRAL, RINNE POSITIVO BILATERAL 
C) WEBER CENTRAL, RINNE SIN CAPTAR BILATERAL 
D) WEBER SIN CAPTAR, RINNE SIN CAPTAR. 
 
Es simétrica y neurosensorial.   
 
EL ESTUDIO DE ELECCIÓN PARA CONFIRMAR EL DIAGNÓSTICO SERÁ: 
A) AUDIOMETÍA TONAL 
B) AUDIOMETRÍA TONAL CON LOGOAUDIOMETRÍA 
C) AUDIOMETRÍA DE BEKESY 
D) ÚNICAMENTE LOGOAUDIOMETRÍA 
  
RESPUESTA C, B, B 
 
OTOSCLEROSIS 
DEFINICIÓN 
Enfermedad metabólica ósea primaria de la cápsula ótica y la cadena osicular que causa fijación de los huesecillos con la resultante 
hipoacusia. Enf de transmisión genética AD con penetrancia incompleta y expresión variable. 
 
EPIDEMIOLOGÍA 
M2:H1 esta relación se explica porque el embarazo acelera el proceso de otosesclerosis. 2/3 de los pacientes tienen historia familiar de 
hipoacusia.   
Presentación clínica común: “Mujer en la segunda o tercera década de la vida, con hipoacusia conductiva lentamente progresiva, 
bilateral (80%) y asimétrica, así como con acufeno (75%) 
 
CLÍNICA 
Hipoacusia progresiva, algunos refieren mejoría de la audición en ambiente ruidoso, fenómeno conocido como Paracusia de Willis. 
 
PARACLÍNICA 
Timpanometría: pico bajo en el oído medio con rango de presión de aire normal se denomina patrón tipo AS, es característico de 
otoesclerosis. 
Reflejos acústicos: reflejo estapedial disminuye progresivamente. 
Audiometrías con tonos puros: nicho de Carhart que se caracteriza por disminución del umbral de conducción ósea de 
aproximadamente 5Db A 500Hz, 10Db A 1000HZ, 15Db A 2000 Hz y 5db a 4000Hz? 
 
TRATAMIENTO 
No quirúrgico: auxiliar auditivo, la ventaja es que se evita el riesgo de hipoacusia sensorioneural postquirúrgica. Los pacientes que se 
somenten a tratamiento quirúrgico tienen significativamente mayor satisfacción de audición. 
Quirúrgico: candidato aquel con buen edo de salud, audiometría socialmente aceptable, prueba de rinne negativa, buen 
discriminación, deseo de operarse. Estapedectomía. 
 
 
 
PACIENTE FEMEINO DE 35 AÑOS DE EDAD, QUIEN INCIA POSTERIOR A EMBARAZO CON HIPOACUSIA PROGRESIVA DERECHA, 
ACOMPAÑADA DE ACUFENO, REFIERE QUE OCASIONALMENTE PRESENTA INESTABILIDAD Y MAREO. MENCIONA QUE CUANDO ESTÁ EN 
LUGARES RUIDOS EN OCASIONES ESCUCHA MEJOR QUE EN LUGARES SIN RUIDO. UNA DE SUS HERMANAS TAMBIÉN PADECE 
HIPOCUSIA. LA PACIENTE REFIERE QUE EN ALGUNA OCASIÓN EN LA INFANCIA PRESENTÓ OTITIS.   
 
CON ESTE CUADRO CLÍNICO EL DIAGNÓSTICO MÁS PROBABLE SERÁ: 
A) OTITIS MEDIA AGUDA 
B) ENFERMEDAD DE MENIERE 
C) OTOSCLEROSIS 
D) HIPOACUSIA CONGÉNITA DE PRESENTACIÓN TARDÍA. 
 
¿QUÉ ESPERARÍAS ENCONTRAR A LA EXPLORACIÓN FÍSICA? 
A) OTOSCOPIA NORMAL, WEBER DERECHO, RINE NEGATIVO DERECHO Y POSITIVO IZQUIERDO. 
B) OTOSCOPIA CON PERFORACIÓN DE MEMBRANA TIMPÁNICA, WEBER DERECHO RINNE NEGATIVO DERECHO Y POSITIVO IZQUIERDO 
C) OTOSCOPIA NORMAL, WEBER IZQUIERDO, RINE POSITIVO BILATERAL 
D) OTOSCOPIA NORMAL, WEBER IZQUIERDO, RINE NO CAPTANTE 
 
LOS ESTUDIOS AUDIOLÓGICOS QUE CONFIRMAN TU SOSPECHA DIAGNÓSTICA SON: 
A) EMISIONES OTOACÚSTICAS 
B) AUDIOMETRÍA TOTAL CON LOGOAUDIOMETRÍA 
C) AUDIOMETÍA TONAL, TIMPANOMETRÍA Y REFLEJOS ESTAPEDIALES 
D) AUDIOMETRÍA AUTOMÁTICA 
 
RESPUESTA C, A, C 
 
 
 
PATOLOGÍA DE LA FARINGE Y LARINGE 
FARINGOAMIGDALITIS AGUDA 
 
VÉRTIGO GENERALIDADES 
Vértigo periférico: fatigable, latencia, se agota, horizontal o rotatorio 
Vértigo central: Vertical, cambiante, asocia síntomas neurológicos 
 
Segundos: VPPB, persistencia de canal semicircular superior y fístula perilinfática. 
Síntomas otológicos: plenitud, acufeno e hipoacusia. 
VPPB no presenta síntomas otológicos, persistencia de canal semicircular y fístula perilinfática sí.   
Persistencia de canal semicircular presenta hipoacusia conductiva y la fístula perilinfática neurosensorial. 
 
Minutos a horas: Enf de Meniere 
 
Horas a días: neuritis vestibular (presenta solo vértigo), laberintitis (presente vértigo + hipoacusia, infección del oído medio oído 
interno) 
 
VÉRTIGO POSTURAL PAROXÍSTICO BENIGNO 
Vértigo objetivo: se mueven los objetos 
Vértigo subjetivo: se mueve el sujeto 
ETILOGÍA 
Puede deberse a lesión cefálica, laberintitis viral u oclusión vacular. Más comunmente idiopático.   
 
DIAGNÓSTICO 
Maniobra de Dix‐Hall‐Pike. Una posición provoca el vértigo más fuertemente, lo que indica que la lesión está en el lado al que está 
girada la cabeza. El nistagmo rotatorio puede observarse durante la maniobra después de un periodo de latencia de 2‐10 seg. Por lo 
gral el nistagmo cambia de dirección cuando el paciente se sienta.    La enfermedad es de resolución espontánea y el paciente se 
recupera por completo en plazo de tres meses.   
 
TRATAMIENTO 
Maniobras de reposicionamiento (Epley) son el tx de elección.   
 
NEURONITIS VESTICULAR 
EPIDEMIOLOGÍA 
50% tiene antecedente de IVRS semanas previas. 
Puede estar presente en epidemias.   
 
CLÍNICA 
  Se presenta con vértigo grave y súbito (puede durar varios días). Náuseas y vómito. Puede persistir incluso pOr meses y de modo 
especial en ancianos, inestabilidad sin vértigo. 
 
TRATAMIENTO   
EsteriOdes y frenadores laberíntis en fase aguda, en fase de recuperación ejercicios de rehabilitación vestibular. 
 
SX DE MENIERE 
PATOGENIA 
 
CLÍNICA 
Planitud aural, hipoacusia neurosensorial fluctuante, plenitud aural, tinitus y vértigo. Alcanza máxima intensidad a los pcos minutos, 
cede lento en varias hrs. 
En la fase de extinción origina hipoacusia permanente grave pero cese del vértigo.   
 
TRATAMIENTO 
Dieta hiposódica, vasodilatadores, diuréticos. 
 
PACIENTE FEMENINO DE 30 AÑOS DE EDAD QUIEN INICIA SU PADECIMIENTO ACTUAL EL DÍA DE HOY DE MANERA SÚBITA PRESNTA 
SENSACIÓN DE GIROS, TE MENCIONA QUE DESDE LA MÑANA DESPERTÓ Y N SE HA TENIDO LA SENSACIÓN DE VÉRTIGO, PRESENTA 
NÁUSEA Y VÓMITO, NIEGA OTORREA, OTALGIA, ACÚFENO, HIPOACUSIA. A LA EF REALIZAS DIAPASONES OBTENIEDO WEBER CENTRAL, 
RINNE POSITIVO BILATERAL, LA PACINET PRESENTA NISTAGMUS ESPONTÁNEO Y EVOCADO CON MIRATA HORIZONTAL CON FASE 
RÁPIDA A LA DERECHA, NO PUEDES REALIZAR MANIOBRA DE DIX HALLPIKE POR LA CONDICIÓN DE LA PACINETE. ROMBERG POSITIVO, 
SIN ASIMETRÍAS NI DISDIADOCOCINESIS. FUDUKA A LA IZQUIERDA, HALMAGYI POSITIVO A LA IZQUIERDA. TE RIFERE QUE TUVO UNA 
GRIPE MALTRATADA HACE 2 SEMANAS.   
 
EL DIAGNÓSTICO MÁS PROBABLE EN ESTA PACIENTE SERÁ DE: 
A) CEREBELITIS 
B) NEURONITIS VESTIBULAR 
C) ENFERMEDAD DE MINERE 
D)TUMOR DE ÁNGUL PONTOCEREBELOSO 
 
El equilibrio está dado por visión, audición y propiocepción. Por eso en este paciente al hacer la prueba de Romberg da positivo (se 
suprime la visión al cerrar los ojos, la audición por tener vértigo y la propiocepción que está indemne no es suficiente para dar un 
Romberg negativo. En un paciente con cerebelitis se anula la visión, la propiosepcion (por daño cerebeloso) y la audición queda 
indemne pero no es suficiente para hacer un Romber negativo) 
 
Fukuda y Halmagyi traducen paresia vestibular. 
 
EL TRATAMIENTO DE ELECCIÓN PARA ESTE PADECIMIENTO ES: 
A) DIURÉTICOS 
B) ESTEROIDES Y FRENADORES LABERÍNTICOS 
C) CIRUGÍA 
D) MANIOBRAS DE REPOSICIONAMINETO OTOLÍTICO 
 
No usar frenadores laberínticos fuera de la agudización porque retrasamos la compensación. 
 
LA ETIOLOGÍA MÁS COMÚN EN ESTE PADECIMIENTO ES: 
A) VIRAL 
B) HIDROPS ENDOLINFÁTICO 
C) COMPRESIÓN DEL NERVIO VESTIBULAR 
D) VASCULAR 
 
Principal etiología es viral por Herpes virus 
RESPUESTA    B, B, A 
 
 
 
PATOLOGÍA DE LA FARINGE Y LARINGE 
FARINGOAMIGDALITIS AGUDA 
DEFINICIÓN 
Infección de faringoamigdalar caracterizada por garganta roja de más de 5 días. 
 
SALUD PÚBLICA 
FR: tabaquismo, RGE, SAOS, inmunodepresión, clima artificial, uso de la voz. 
Causa frecuente EBHGA 
 
DIAGNÓSTICO 
Criterios de Centor modificados:    Criterios  Puntos 
0 pts  1‐2.5%  Temperatura >38  1 
2 pts  11‐17  Exudado amigdalar  1 
3 pts  28‐35  Ausencia de tos  1 
>4  51‐53  Adenopatías  1 
  laterocervicales doloras 
  3‐14 años  1 
  15‐44  0 
  >45  1 
 
Faringitis 85% por virus, 15 por EBHGA. Habitualmente 1) exudado blanquecino en amígdalas, 2) adenomatía cervical, ausencia de 
rinorrea, 3)tos, 4) fiebre >38°C. Dx clínico con 3 de esos datos, tiene S 75%. 
 
Rinorrea, tos húmeda, disfonía, vesículas, conjuntivitis sugiere origen viral. 
Gold stándar: cultivo de exudado faringeo 
 
TRATAMIENTO 
Aceptable dar tx empírico si 4 datos y aspecto tóxico.   
Bencilpenicilina benzatínica compuesta de 1,200,000 UI, aplicar intramuscular cada 12 hrs por 2 dosis. Seguida de 3 dosis de 
Bencilpenicilina procaínica 800,000 UI cada 12 hrs IM. 
Si alergia: eritromicina 500mg. Tomar 1 tableta cada 6 hrs por 10 días o TMP/SMX 160/600. Tomar 1 tableta cada 12 hrs por 10 días.   
Tx erradicación inicia 9 dias después de la enf aguda. Se aconseja cultivo 7 dias después del tratamiento en agudo. 
Dado que S. pyogenes no es productor de beta lactamasas no amerita añadir clavulanato, además este aumenta la hepatotoxicidad.   
¿Cuándo demonios iniciar tx de erradicación? a todos a solo a cultivos positivos post AB inicial? 
 
REHABILITACIÓN 
Si exudado purulento, bicarbonato. 
Abundantes liquidos, dieta sin irritantes. 
 
PRONÓSTICO 
Síntomas de obstrucción de vía aérea superior: estridor, disnea y cianosis, relacionados con epiglotitis, en relación con asbceso 
periamigdalino, y retrofaringeo. 
 
COMPLICACIONES 
Supurativas: Otiitis media, isnusitis, absceso periamigdalino, mastoiditis, absceso retofaringeo, síndrome de shock tóxico 
estreptocócico. 
No supurativas: fiebre reuma´tica, glomerulonefritis postestretocócica, artritis reactivas, escarlatina. 
 
 
CONTENIDO ADICIONAL CLAVE 
Los cultivos positivos a Staphylococcus aureus, se encuentran en    personas sanas asintomáticas. Los intentos de erradicación son a 
menudo inútiles e innecesarios, excepto entre el personal que trabaja con inmunodeprimidos y pacientes con heridas abiertas. 
 
Criterios de amigdalectomía: 
  Faringoamigdalitis de repetición: 7 x 1 año, 5 x2 años, 3 x 3 años.   
  Antecendente de abscesos periamigdalinos 
  Amígdalas tumorales 
  Amigdalitis por corynebacterium 
 
 
PACIENTE MASCULINO DE 17 AÑOS DE EDAD, QUE INCIA SU APDECIMIENTOA CTUAL HACE 48 HRS CON ODINOFAGIA INTENSA, BOCIO 
EN PAPA CALIENTE, FIEBRE DE 40° CON ATAQUE AL ESTADO GENERAL. A LA EF OBSERVAMOS AMÍGDALAS GRADO 2 CON EXUDADO 
PURULENTO Y ABOMBAMIENTO EN PALADAR BLANDO Y DESPLAZAMIENTO DE AMÍGDALAS IZQUIERDA HACIA LÍNEA MEDIA.   
 
EL CUADRO CLÍNICO CORRESPONDE A: 
A) MONONUCLEOSIS INFECCIOSA 
B) DIFTERIAE 
C) ABSCESO PERIAMIGDALINO 
D) FARINGOAMIGDALITIS VIRAL 
 
EL AGENTE CAUSAL DE ESTA ENFERMEDAD ES: 
A) S. PNEUMONIAE 
B) S. AUREUS 
C) CORYNEBACTERIUM DIFTERIAE 
D) S. PYOGENES 
 
EL TRATAMIENTO INDICADO PARA ESTA PACIENTE ES: 
A) SINTOMÁTICO (PARACETAMOL, ANTIHISTAMÍNICO, ANTIINFLAMATORIO) 
B) ANTIBIÓTICOS IV 
C) DRENAJE POR PUNCIÓN Y ANTIBIÓTICOS 
D) AMIGDALECTOMÍA DE URGENCIA 
 
RESPUESTAS C, D, C 
 
 
 
ABSCESO PERIAMIGDALINO 
DEFINICIÓN 
Lesión difusa (celulitis o flemón) o localizada (absceso propiamente dicho). 
 
SALUD PÚBLICA 
30‐100 mil habitantes.   
Morbilidad grave: fascitis necrotizante, mediastinitis, mortalidad. 
 
PATOGENIA 
2 teorías: Infección de glándulas de Weber, celulitis y absceso. 
 
DIAGNÓSTICO 
Dolor severo, disfagia, otalgia ipsilateral referida, trismus, edema de paladar blando,    desplazamient de amígdalas y úvula.   
Cultivo: generalmente polimicrobiano.   
USG intraoral S 89%, E 100%, TAC S 100%, E 75%, clínica S 75%, E 75% 
 
TRATAMIENTO 
La tendencia es realizar drenaje y continuar manejo ambulatorio. 
Peni procaínica si no precisa hospitalización,    AMOXI CLAV para casos de resistencia. 
 
ABSCESO PERIAMIGDALINO 
MASCULINO DE 28 AÑOS CON ANTECEDENTE DE RINITIS ALÉRGICA, INICIA SU PADECIMIENTO HACE 11 DÍAS CON ODINOFAGIA, FIEBRE, 
DISFAGIA, TRISMUS Y OTALGIA IZQUIERDA. RECIBIÓ TRATAMIENTO CON AMOXICILINA POR 7 DÍAS SIN PRESENTAR MEJORÍA. A LA 
EXPLORACIÓN PRESENTA AMÍGDALAS HIPEREMICAS CON DESPLAZAMIENTO MEDIAL DE LA AMÍGDALA IZQUIERDA. 
 
EL DIAGNÓSTICO CLÍNICO MÁS PROBABLE ES: 
A)  AMIGDALITIS ESTREPTOCOCICA. 
B)  ABSCESO PERIAMIGDALINO. 
C)  FARINGITIS CRÓNICA. 
D)  MONONUCLEOSIS INFECCIOSA. 
 
EL PROCESO FISIOPATOLÓGICO QUÉ MAS PROBABLEMENTE DIO ORIGEN A ESTA ENFERMEDAD ES: 
A)  ACUMULACIÓN DE CASEUM EN CRIPTAS AMIGDALINAS. 
B)  INFLAMACIÓN REACTIVA DE TEJIDO LINFOIDEO. 
C)  EDEMA FARÍNGEO. 
D)  COLECCIÓN PURULENTA PERIAMIGDALINA. 
 
EL TRATAMIENTO INDICADO SERÍA: 
A)  ANTIBIÓTICO INTRAVENOSO. 
B)  DRENAJE QUIRÚRGICO. 
C)  COLUTORIOS CON ANTISÉPTICO. 
D)  ACICLOVIR. 
 
RESPUESTA B,D, B 
 
LARINGITIS SIMPLE O CATARRAL 
GENERALIDADES 
Las laringitis generalmente son víricas. 
 
CLÍNICA 
Prutito faringolaríngeo, tos irritativas, disfonia progresiva en horas. Evoluciona a curación espontánea en pocos días.   
 
DIAGNÓSTICO 
Clínico + laringoscopía indirecta (mucosa congestiva, brillantes y edematosa). 
 
TRATAMIENTO 
Aliviar los síntomas con fluidificantes de la secreción laringotraqueal, humidificación con nebulización, antitusígenos, analgésicos y 
antitérmicos.   
 
LARINGOTRAQUEOBRONQUITIS 
GENERALIDADES 
Suele inciar como una laringits subglótica, posteriormente se agrega sintomatología traqueobronquial, tiraje intercostal, roncus y 
sibilancias.   
 
DIAGNÓSTICO 
Laringoscopia directa (rodete subglótico y pseudomembranas fibrinosas gran adherencia que aumentan la obstrucción respiratoria. 
 
TRATAMIENTO 
Fluidificación, aspiración de secreciones y pseudomembranas, mediante laringotraqueobroncoscopia, AB y traqueotomía para faciliar 
las aspiraciones.   
PACIENTE FEMENINO DE 50 AÑOS DE EDAD, REFIERE INICIAR SU PADECIMIENTO ACTUAL HACE TRES MESES CON SENSACIÓN DE 
GLOBUS FARINGEO, DISFONIA MATUTINA QUE MEJORA AL PASO DEL DÍA, REFIERE QUE SU SINTOMATOLOGÍA MEJORA AL ACLARAR 
LAS SECRECIONES DE LA GARGANTA. OCASIONALMENTE PRESENTA PIROSIS. REFIERE ANTECEDENTE DE TABAQUISMO OCASIONAL.   
 
LO QUE ESPERAS ENCONTRAR A LA EXPLORACIÓN FÍSICA DE ESTA PACIENTE ES: 
A) ADEMA INTERARITENOIDEA Y DE BANDAS VENTRICULARES 
B) MASA LARÍNGEA 
C) LEUCOPLAQUEA 
D) PLACAS BLANQUECINA COMPATIBLE CON INFECCIÓN MICÓTICA 
 
EL DIAGNÓSTICO MÁS PROBABLE SERÁ: 
A) CÁNCER LARINGEO 
B) LARINGITIS CRÓNICA VIRAL 
C) LARINGITIS ALÉRGICA 
D) LARINGITIS CRÓNICA POR REFLUJO 
 
EL TRATAMIENTO RECOMENDADO PARA ESTE PACIENTE ES: 
A) RESECCIÓN QUIRÚRGICA 
B) ANTIHISTAMÍNICOS 
C) ESTEROIDES VÍA ORAL 
D) MEDIDA ANTIREFLUJO E INHIBIDORES DE BOMBA DE PROTONES 
 
RESPUESTA A, D, D 
 
TUMORES LARINGEOS BENIGNOS 
GENERALIDADES 
Puedes ser pólipos, nódulos y granulomas.   
Los pólipos se dan por uso y mal uso de voz. Frec entre 20‐40 años. Síntoma principal: disfonía. Este es agudo. Reabsorción 
espontánea Quirúrgico 
Los nódulos se dan en el tercio medio de la cuerda vocal, gritan o hablan mucho. Principal síntoma es la disfonia. Dx por laringoscopia 
indirecta o fibrolaaringoscopia.  terapia foniátrica si no  microcirugia laringea. 
Papilomas laringeos: por VPH. Masa vegetante blancuzca. Gris rosácea y sésil. Tx es quirúrgico. 
 
TUMORES LARINGEOS MALIGNOS 
GENIA 
Los más comúnes son epidermoides. 
FR: tabaquismo, alcoholismo, ERGE, exposición a sustancias, VPH 
 
Puede ser supraglóticos, glóticos o subglóticos. 
Temprano si no hay fijación de cuerda vocal. Se trata con cordectomía o RT 
Avanzados si hay fijación. Se trata con laringectomía, disección de cuello, QT y RT como ayuda.   
 
QUISTE DE CONDUCTO TIROGLOSO 
GENIA 
 
QUISTES FARINGEOS 
GENIA 
 
 
PATOLOGÍA DE LA NARIZ Y SENOS PARANASALES 
EPISTAXIS 
PATOGENIA 
Origen traumático (digital frecuentemente) 
Fármacos tópicos y cocaína 
Otras: Rinitis, rinosinutisis crónicas 
 
DIAGNÓSTICO 
Exploración armada define si anterior o posterior, si se sospecha neoplasia ameritará endoscopia.   
 
TRATAMIENTO 
1 Presión nasal con vasocontrictor local (oximetazolina, felilefrina 0.25%, nafazolina) detiene sangrado en 70%.   
2 Taponamiento anterior 1‐5 d c lubricación 
3 Taponamiento nasal con gasa lubricada, material expansible (Merocel o espojas Kennedy) y sondas inflables (Foley, Epistat, Rush) 
resuelven en 60‐80% de casos díficiles.   
Sellador de fibrina sustituye de forma efectiva taponamiento nasal y cauterización quimica o eléctica y con menos efectos secundarios.   
 
Quirúrgico 
Ligadura microscópica transnasal de arteria esfenopalatina          93% 
Ligadura carótida externa        93 
Ligdura Maxina interna transnasal      91 
Embolización percutánea de arteria maxilar interna    88 
Cauterización endoscópica          83 
 
Embolizaciones tienen mayor frecuencia de complicaciones que ligaduras. 
 
 
SINUSITIS AGUDA Y CRÓNICA 
DIAGNÓSTICO 
Dos mayores, un mayor y dos menoes.   
Mayores: rinorrea purulenta, obstrucción nasal, hiposmia, anosmia, fiebre. 
Menores: cefalea, tos, halitosis, fatiga, otalgia, plenitud ótica, dolor dental.   
 
Estudios de imagen rutinarios no si clínica franca. 
Si cefalea frontal donde quede la duda de si hay sinusitis: Caldwell y lateral de cráneo 
 
TRATAMIENTO 
Amoxi 500c 8 x 10‐14 d. 
Si alergia TMPSMX 160/800 c 12 x 10‐14 d. 
Oximetazolina 
Paracetamol 500c 6 
Naproxeno 250 c 12 hrs 
Sol salina 
 
CATARATAS 
CATARATAS 
DEFINICIÓN 
Opacificación del cristalino 
 
SALUD PÚBLICA 
Catarata senil, causa número 1 de ceguera reversibl.   
FR: DM, esteroides crónicos, >50 años, inmunosuprsión, agonisa alfa 1a 
 
DIAGNÓSTICO 
a)Disminución progresiva de agudeza visual   
b)deslumbramiento (fotofobia?) en condiciones de baja luminosisdad 
 
Oftalmoscopio directo: opacidad en el reflejo rojo, si hay duda hacemos midriasis en no hipertensos con fenilefrinas.   
 
TRATAMIENTO 
Quirúrgico   
¿Cuál es la técnica de elecicón? 
No se debe esperar a que madure la catarata, si hay déficil visual que puede mejorar con cirugía y no hay contraindicaciones, se opera. 
 
PRONÓSTICO 
Si se deja evolucionar aumentar riesgo de glaucoma facomórfico.   
 
CONTENIDO ADICIONAL 
Agonista α1    Vasoconstricción, midriasis  Fenilefrina, Oximetazolina  Estimulan actividad de fosfolipasa C 
Agonista α2  Vasodilatador, sedantes  Clonidina      Inhibe adenil ciclasa, inhubie SNS 
Agonista β1  Estimulación cadíaca    Dobutamina, isprotenerol  Estimulan adenil ciclasas y abren canales de Ca 
Agonista β2  Broncodilaación    Salbutamol, fenoterol isoprot  Estimulan adenilciclasa, abre canales de Ca 
 
 
ENFERMEDADES DE LA RETINA 
RETINOPATÍA HIPERTENSIVA 
DEFINICIÓN 
Hipertensión arterial==>estrechamiento vascular de arterias retinianas (a)contricción, b)extravasación, c) arteriosclerosis. 
Dichas 3 lesiones determinarán estadios de enfermedad retiniana. 
  Cruces arteriovenosos 
  Exudados duros y algodonosos 
  Trombosis y embolias 
  Hemorragias parenquimatosas retininana 
  Desprendimiento seroso de retina 
  Edema de papila 
  Neuropatía óptica isquémica 
 
PATOGENIA 
Ateroesclerosis: arterias grandes con depósitos de gasa en íntima y fibrosis. 
Arterioesclerosis: arterias pequeñas con fibrosis.    Arterioesclerosis moderada===> alambre de cobre, grave==>plaa. 
 
DIAGNÓSTICO 
Retinopatía hipertensiva (daño a órgano blanco) en el contexto de una crisis: 
  Edema de papila 
  Exudados 
  Hemorragias retinianas 
 
Hemorragias retinianas, microaneurismas, exudados algodonosos, asocian 2‐4 veces riesgo de EVC. 
 
Retinopatía hipertensiva de larga evolución: 
 
TRATAMIENTO 
control presor 
 
CONTENIDO ADICIONAL 
 
 
 
 
 
 
RETINOPATÍA DIABÉTICA 
DEFINICIÓN 
Daño microvascular del diabgético:   
  Arrosariamiento venoso 
  Alt microvasculares 
  Manchas algodonosas 
  Microaneurismas 
  Neovasos 
  Exudados duros   
  Tej fibroso. 
 
SALUD PÚBLICA 
Principal causa de ceguera en México. 
Prevalencia 31.5% 
Las formas de RD que afectan potencialmente la visión la EMCS (edema macular clínicamente significativo) y la RDP (retinopatía 
diabética proliferativa). Ambas se pueden tratar con fotocoagulación.   
21‐39% tiene retinopatía al diagnóstico, casi 100% luego de 20 años. 
Primera causa de ceguera prevenible. 
 
FR: alcoholismo, mal control metabólico, >5 años de ser diabético, embarazo, disliidemia LDL >100, TA 130/80, >30 IMC, <60 de 
depuración de cretinina, pubertad. 
 
DIAGNÓSTICO 
D1: 12 años de edad, 5 año de diagnóstico si pubertad, 3 años de diagnóstico si postpúber. Cada año. 
D2: al diagnóstico, y cada año 
D1‐2 y embarazo:    c 3 meses si leve, cada mes si severa, 
 
Clínica: baja agudeza visual 
EF. exploración bajo midriasis farmacológica., con lámpara de endidura 
Si hemorragia vítrea u opacidad ==> USG 
 
[Microaneurismas, microhemorragias y microexudados en un solo cuadrante es leve, en todos los cuadranes es moderada] 
Es clínico en no proliferativa. Flurangiografía en proliferativa.   
 
 
1. Retinopatía diabética NP leve:   
  Al menos un microaneurisma. 
2. Retinopatía diabética NP moderada:   
  Hemorragias y/o microaneurismas >2A foto estándar; y/o 
  EB, RV o AMIR.   
3. Retinopatía diabética NP severa: 
  EB, RV y AMIR, todos presentes en al menos 2 campos de las 4 a las 7 ó 
  2 de 3 lesiones, en al menos 2 de los campos a través de 4‐7 y hemorragias con microneurismas en los 4 campos >2A ó 
  AMIR en todos los campos de las 4 a las 7 y >8A foto estándar    en al menos 2 de ellos.   
 
4. Retinopatía diabética proliferativa de bajo riesgo.   
  Neovasos 
5. Retinopatía diabética proliferativa de alto riesgo 
  Neovasos 
  Hemorragia en vítreo 
  Hemorragia subhialoidea 
  Proliferación neovascular 

 
Proliferativa con AR los neovasos están dentro del disco óptico. 
 
 
EB Exudados blandos 
RV Rosarios venosos 
AMIR Anormalidades microvasculares intrarretinaninas 
 
TRATAMIENTO 
RDNP leve o moderada    NO son candidatos a láser. Control glucémico.   
RDNP severa:    Fotocoagulación si: 1)Mal control metabólico, 2) no cumple con revisiones de retina habituales 
      3)Cursa con catarata que pude limitar la aplicación láser a futuro. 
      4)Presenta isquemia generalizada, 5)Embarazada 
 
Tratamietno con fotocoagulación de retina periférica o transretiniana(en breve): 
RDP sin AR 
RDP con AR 
NV en iris 
 
Aplicacion fotocoagulación con láser focal o en rejilla si: 
RDNP moderada y EMCS 
RDNP severa y EMCS 
RDP sin AR y EMCS 
RDP con AR y EM 
 
REHABILITACIÓN 
Envío a 3er nivel: hemorragia vítrea, desprendimienot de retina traccional o regmatógeno. 
 
CONTENIDO ADICIONAL 
Exudados duros son un signo de edema macular actual y antiguo.   
El edema macular diabético se define como engrosamiento de la retina: esto require de una evalcuación 3D que se realiza bajo 
dilatación pupilar por examen de biomicroscopia con lámpara de hendidura y fotografía estereoscópica de fondo.   
 
[RETINOPATÍA DIABÉTICA][r] 
HOMBRE  DE  63  AÑOS  DE  EDAD  CON  DIAGNÓSTICO  DE  DIABETES  MELLITUS  DESDE  18  AÑOS.  ACUDE  A  CITA  DE  CONTROL 
OFTALMOLÓGICA. A LA EXPLORACIÓN DEL FONDO DE OJO SE OBSERVA LA SIGUIENTE IMAGEN. 
 
 
 
CON BASE EN LA IMAGEN DE FONDO DE OJO MOSTRADA SE PUEDE CONCLUIR QUE EL PACIENTE PRESENTA: 
   
A) RETINOPATÍA NO PROLIFERATIVA. 
B) UNA HEMORRAGIA VÍTREA. 
C) RETINOPATÍA PROLIFERATIVA. 
D) EDEMA DE PAPILA. 
 
EL TRATAMIENTO DE ELECCIÓN EN ESTE CASO INCLUYE: 
   
A) CONTROL METABÓLICO + APLICACIÓN DE LÁSER. 
B) UNICAMENTE CONTROL METABÓLICO. 
C) CONTROL METABÓLICO + DERIVACIÓN DE LÍQUIDO CEFALORRAQUÍDEO. 
D) CONTROL METABÓLICO + REPOSO EN SEMIFOWLER 
 
RESPUESTA A, B 
En la retinopatía diabética podemos encontrar varias lesiones: microaneurismas, hemorragias, exudados duros, manchas algodonosas, 
alteraciones microvasculares, arrosaraiamiento venoso, neovasos, tejido fibroso.   
Se clasfica en RDP y RDNP.   
La RDNP se caracteriza por aneurismas vasculares retinianos, manchas hemorrágicas, dilatación venosa y exudados algonosos.   
RDNP mínima: microanerusismas 
RDNP moderada: microaneurismas + exudados duros + engrosamiento de capilares 
RDNP avanzada: Más de 20 hermorragias retinianas x cuadrante o Arrosaramiento venoso en 2 o más cuadrantes o AMIR en un cuadrante. 
 
MUJER DE 48 AÑOS DE EDAD, QUE ACUDE A CONSULTA DE CONTROL Y QUE CUENTA CON EL DIAGNÓSTICO DESDE HACE UNA SEMANA 
DE HERPES ZÓSTER OFTÁLMICO. REFIERE, NO HABER TOMADO EL MEDICAMENTO INDICADO (ACICLOVIR) AL NO PODERLO COMPRAR. 
 
USTED LA REVISA, Y LE COMENTA QUE EN ESTE CASO SU RIESGO DE PERDER LA VISTA ES:   
A) DEL 80 AL 100%. 
B) DEL 50 AL 70%. 
C) DEL 30 AL 40%. 
D) DEL 10 AL 20%. 
 
Sin terapia antiviral 50‐70% puede desarrolar enfermedad crónica, 20% desarrollar uveítis y perder la vista.   
RESPUESTA D 
 
 
 
 
RETINOPATÍA HIPERTENSIVA 
Keith Wagener y Barker 
Hilos de cobre estadio III 
Hilos de plata: estadio IV 
 

 
GLAUCOMA 
GLAUCOMA DE ÁNGULO ABIERTO 
 
DEFINICIÓN 
Neuropatía óptica crónica y progresiva    (se excava el nervio óptico) no siempre relaciodado a hipertensión ocular (de ángulo abierto= 
 
PATOGENIA 
GPAA: NO está más sensible a los cambios de PIO 
Procesos ciliares==> humor acuoso==> camara posterior==> pupila==> cámara anterior==> malla trabecular*==>canal de Schlemm==> 
plaexo coroides===> sistema venoso. 
*Es el que más influye en la PIO 
 
 
 
 
DIAGNÓSTICO 
Criterios para GPAA: 
  PIO sin manejo >21mmHg con aunsencia de causa secundaria 
  Ángulo abierto 
  Paquimetria 
  Cabeza del nervio óptico 
    Anillo neuroretiniano con pérdida de la relación SINT 
    Excavación de la papila siendo superior el eje vetical 
  Campo visual 
    Campo visual pre‐perimétrico 
    Escotoma de Bjerrum 
    Escotoma paracentral profundo 
    Escalón nasales 
    Depersiones concéntricas 
 
Hemorragias en astilla del nervio óptico 
Atrofia peripapilar con presencia de anillos alfa y beta. 
 
TRATAMIENTO 
1)análogos de prostaglandinas[Latanaprost, bimatoprost] (si fracasa adicionar 2)) 
Si contraindicación beta bloqueador [timolol hemihidrato, metilpranolol] 
2)inhibidores de anhidrasa carbónica tópica, alfa agonistas o pilocarpina [acetazolamida, dorzolamida] 
 
Indicaciones de tx quirúrgico:   
  PIO no alcanzada 
  Etapa de claucoma: avanzado 
Indicaciones de implante valvular: 
  Glaucoma neovascular en etapa de ángulo abierto 
  Glaucoma secundario a silicón en cámara anterior 
Catarata más GPAA: 
  Inicial y controlado con 1‐2 medicamentos==> facoemulsificación + LIO 
  Moderado, avazado o descontrolado==>famoemulsificación + LIO + trabeculectomía 
 
REHABILITACIÓN 
Objetivo si daño leve: PIO 20‐30% de basal. 
  si daño avanzado: PIO 40% o más de basal. 
 
CONTENIDO ADICIONAL CLAVE 
Ángulo de 1‐2 está cerrado 
Ángulo de 3‐4 está abierto 
 
GLAUCOMA DE ÁNGULO CERRADO? 
DEFINICIÓN 
Un ataque agudo conlleva pérdida d ela visión sino se maneja a tiempo. 
 
PATOGENIA 
Grado 0  [0°]    Ocluible 
Grado I  [10°]  Alto riesgo 
Grado II  [20°]  Cierre poco proba ble 
Grado III  [25‐35°]  Abierto 
Grado IV  [35%‐45%] Cuerpo ciliar se visualiza con facilidad 
 
Cierre angular primario agudo: el iris contacta con malla trabecular, el ángulo se cierra, PIO aumenta mucho, edema corneal (visión 
borosa, halos multicolores), congesión vascular, dolor ocular o cefalea, náuseas, vómito,   
Cierre angular primario crónico: puede no haber síntomas, hay sinequias.   
 
TRATAMIENTO 
  

 
 
 
 
UNA MUJER DE 61 AÑOS, HIPERMÉTROPE Y CON CATARATAS, ACUDE POR DOLOR INTENSO EN OJO IZQUIERDO, DE UNAS HORAS DE 
EVOLUCIÓN. PRESENTA UNA TENSIÓN OCULAR DE 40 MMHG, REACCIÓN HIPERÉMICA, MIDRIASIS Y EDEMA CORNEAL. ¿CUÁL SERÍA LA 
ACTITUD INMEDIATA MÁS CORRECTA? 
 

 1.   MANITOL INTRAVENOSO ASOCIADO A TRATAMIENTO MIÓTICO Y CORTICOIDES TÓPICOS 

 2.   CORTICOIDES TÓPICOS ASOCIADOS A TRATAMIENTO MIDRIÁTICO 

 3.   DILATAR ADECUADAMENTE EL OJO PARA EXPLORAR LA RETINA Y DESCARTAR TUMOR INTRAOCULAR 

 4.   OPERACIÓN DE URGENCIA LA CATARADA DE ESE OJO, DESENCADENANTE DEL CUADRO 
 
RESPUESTA 1 
 
 
 
 
 
 
 
 
 
 
 
 
GLAUCOMA 
  GLAUCOMA CRÓNICO SIMPLE  GLAUCOMA PRIMARIO DE ÁNGULO ESTRECHO 
 
FR  Miopía levada, antecedentedes familiares, DM.      Cribado >40 años o con FR.              Hipermetropía, ángulo iridocorneal estrecho, cámara anterior pequeña, ojos pequeños, >50 años. 
 
PATOGENIA  Aumento de la presión daña la papila óptica.                                                                  Bloqueo pupilar==>    protrusión anterior del iris==> cierre angular==> elevación dela PIO       
  Se produce en midriasis media. (Oscuridad, ansiosos, uso de simpaticommiméticos [atropina])             
 
SÍNTOMAS  Escotoma pericentral (área de Bjerrum) y escotoma priférico nasal d Ronne.  Si >30mmHg: edema corneal, pérdida de la transparencia corneal, dificultad para ver el iris. 
  (excavación 0.4‐0.5)  Halos de colores alrededor de las luces 
Isla de visión central o isla temporal residual (excavación 0.8‐0.9)    Si 40‐50mmHg: dolor intenso, puede irradiarse al territorio del trigémino, blefaroespasmo, 
Agudeza se pierde en estadíos avanzados.  lagrimeo, blefaroespasmo, epífora, bradicardia, hipotensión arterial, náuseas, vómito. 
Papila >5/10 sospechar glaucoma  Si 80‐90mmHg: colapso de arteria central de la retina, dolor más intenso, disminución de la agudeza 
  visual, dureza del globo ocular al tacto. Si no se resuelve pronto==> atrofia retiniana irreversible.   
EXPLORACIÓN  Excavación de la papila (E/P <0.2 normal, 0.3‐0.6 sospechosa, >0.7patológica o  Dureza pétrea, midriasis media arreactiva, hiperemia mixta o ciliar, nubosidad corneal. 
  asimetría >0.2 
Tonometría PIO (<20 normal, 21‐24 sospechosa, >25 patológica) 
Campimetría (primer estudio diagnóstico) 
Tomografía de coherencia ópitca 
TRATAMIENTO  Permanente  Diuréticos osmóticos [manitol, acetazolamida] 
Análogos de prostaglandinas F2: aumentan la salida de humor acuoso por la vía  Corticoides tópicos: limitan componente inflamatorio 
úveo‐escleral. [latanaprost]  Mióticos: rompen bloqueo pupilar [pilocarpina] 
B bloqueantes: disminuyen producción. Cuidado en IC descompensada y bloqueo  B bloquentes, a2 agonistas (solo selectivos) 
AV, broncópatas y asmáticos. [timolol]  Iridotomía quirúrgica o con láser. 
Agonistas a2 adrenérgicos: disminuyen producción. [apraclonidina, brimonidina]   
Inhibidores de anhidrasa carbónica: disminuyen producción. [dorzolamina]  No hay uno de primera elección, generalmente da todos. Pero nunca es de primera instancia 
Estimuladores adrenérgicos: aumentan la salida. Producen midriasis.  boqueante a2. 
Contraindicados en cámara anterior estrecha, hipermetropes ya que   
podríanpercipitar glaucoma agudo.    Prohibido dar análogos de prostaglandinas.   
Pilocarpina: aumenta salida de humor acuoso, produce miosis. Poco usado prque 
ocasiona miopización del ojo y catarata. 
Trabeculoplastía con láser 
Trabeculectomia quirúrgica (más eficaz). 
 
Glaucoma neovascular 
Causa No 1 : DM 
Clínica: mala visión, rubeosis (neovasos en iris) + PIO elevada 
Tratamiento: panfotocoagulación + válvula de Ahmed 
Pronóstico: malo para la función. 
MUJER DE 75 AÑOS DE EDAD, QUE ACUDE A LA CONSULTA CON SINTOMATOLOGÍA COMPATIBLE CON GLAUCOMA. 
PARA ENTENDER LA FISIOPATOLOGÍA DEL GLAUCOMA, ES IMPORTANTE SABER QUE EL DAÑO GLACOMATOSO SE DA EN LA SIGUIENTE 
PORCIÓN DEL NERVIO ÓPTICO: 
   
A) INTRAOCULAR. 
B) ORBITARIA. 
C) INTRACANALICULAR. 
D) INTRACRANEAL. 
 
RESPUESTA A 
El glaucoma es una neuropatía óptica crónica y progresiva, hay pérdida de fibras nerviosas de la retina, excavación y palidez progresivas 
de la papila y defectos del campo visual. Va asociado a hipertensión intraocular pero NO en todos los casos. De hecho hay glaucoma 
normotenso, y hay hipertensión intraocular (>21mmHg) sin glaucoma.    La papila corresponde a la parte intraocular del nervio óptico. 
El  glaucoma  de  ángulo  abierto  (GPAA)  es  el  más  freucente  con  un  60%.  El  ángulo  esta  abierto  y  suele  ser  simétrico.    Etiología 
desconocida.    Pero el principal defecto fisiopatológico es la resistencia al paso del humor por la malla trabecular.   
Factores de reisgo: 1) PIC, 2) antecdenetes familiares (4‐16% de riesgo de padercerla si son de primer grado, 3)>40 años. 
 
Epitelio  de  cuerpo  ciliar===>  humor  acuoso  en  cámara  posterior==>  pupila==>  cámara  anterior  ==>ándulo  iridocorneal==> 
trabeculum===> canal de Schlemm==>vaso espiesclerales==> circulación sanguínea general (eliminación del 90% del humor acuoso) 
 
Solo 10% se eliminará vía uveoescleral atravesando directamente el cuerpo ciliar. 
 
En el glaucoma de ángulo cerrado el origen está en un bloqueo pupilar relativo a una mayor aposición del iris y el cristalino que dificutla 
el paso del humor acuoso de la cámara posterior a la anterior a través de la pupila (empujando a la periferia el iris).   
 
Independientemente de la causa, habrá un pérdida progresiva de la capa de fibras nerviosas ya sea de forma mecánica directa o por 
compromiso vasculaar.   
Tratatamiento farmacológico: 
1)betabloqueadores: disminuyen la producción del humor acuoso al actuar sobre los receptores beta de los procesos ciliares [timolol, 
carteolol, betaxolol (selectivo B1)] 
2)Mióticos parasimpaticomiméticos    y    anticolinesterásicos: disminuyen la resistencia de la malla trabecular. [pilocarpina y aceclidina] 
3) Simpaticomiméticos A y B adrenérgicos:    [adrenalina] aumentan el flujo de salida del humor acuoso y disminuyendo la producción en 
el cuerpo ciliar. [NO en ángulo cerrado] 
4) Agonistas alfa 2 adrenérigcos: disminución de la prodccuón de humor acuoso 
5)  Prostaglandinas:  [latanaprost  al  0.005%]  aumenta  la  salida  de  humor  acuoso  a  través  de  vía  uveoescleral.  [NO  en  glaucoma 
inflamatorio] 
6) Inhibidores de anhidrasa carbónica 
 
Tratamiento láser: trabeculoplastia con lárser de argón en GPAA y trabeculectomía, iridotomía periférica con láser argón o lsa Ng‐YAG en 
GPAC. 
Tratamiento quirúrgico: trabeculectomía simple o combinada cirugía de catarata (facoemulsificación) 
 
¿CUÁL ES LA CAUSA MÁS FRECUENTE DE UNA EXCAVACIÓN PAPILAR CON RECHAZO NASAL DE LOS VASOS EMERGENTES DE LA MISMA? 
A) HIPERTENSIÓN INTRACRANEAL 
B) UVEÍTIS POSTERIOR 
C) CONJUNTIVITIS CRÓNICA 
D) GLAUCOMA CRÓNICO SIMPLE 
 
RESPUESTA D 
No entendí la pregunta :( 
 
UNA  MUJER  DE  64  AÑOS,  HIPERMÉTROPE  Y  CON  CATARATAS  EN  AMBOS  OJOS,  ACUDE  A  CONSULTA  CON  DOLOR  INTENSO  EN  OJO 
IZQUIERDO,    DE UNAS HORAS DE EVOLUCIÓN. LA EXPLORACIÓN DE ES OJO PONE DE MANIFIESTO UNA TENSIÓN OCULAR DE 40MMHG, 
REACCIÓN HIPERÉMICA CILIO‐CONJUNTIVAL, MIDRIASIS Y EDEMA CORNEAL. ¿CUÁL SERÍA LA ACTITUD INMEDIATA MÁS CORRECTA DE 
LAS QUE SE ENUMERAN A CONTINUACIÓN? 
A) MANITOL INTRAVENOSO, ASOCIADO A TRATAMIENTO MIÓTICO Y CORTICOIDES SISTÉMICOS 
B) PRACTICAR TRABECULECTOMÍA 
C) CORTICOIDES TÓPICOS, ASOCIADOS A TRATAMIENTO MIDRIÁTICO 
D) OPERAR CON URGENCIA LA CATARATA DE ESE OJO, DESENCADENANTE EL CUADRO. 
 
RESPUESTA A 
 
ACUDE ACONSULTA UN PACIENTE PORQUE DICE QUE EN LOS ÚLTIMOS MESES LE HA CAMBIADO EL COLOR DEL OJO IZQUIERDO. EN LA 
ANAMENSIS DEBE INSISTIR EN POSIBLE OCNSUMO DE ALGIUNA DE LAS SIGUIENTES MEDICAMENONES POR VÍA TÓPICA: 
A) COLIRIOS PARA GLAUCOMA 
B) AINE 
C) CORTICOIDES 
D)  ACICLOVIR,  PUES  ESTÁ  DEMOSTRADO  QUE  PUEDE  PRODUCIR  HIPERPIGMENTACIÓN  EN  EL  IRIS  CUANDO  SE  MANTIENE  EL 
TRATAMIENTO DURANTE UN PERIODO MUY LARGO DE TIEMPO 
 
RESPUESTA A 
Hay que recordar que los análogos de prostaglandinas F2 pueden producir como efectos secundarios cambios en la coloración del iris 
(hiperpigmentación) e hipermia y aumento del tamaño de las pestañas.    A pesar de ello son los medicamentos mejor tolerados y eficacez 
en el manejo de glaucoma de ángulo abierto. 
 
A 93‐YEAR OLD WOMAN, COMES TO THE EMERGENCY ROOM COMPAINING ABOUT ACUTE PAIN AND REDNESS IN HER RIGHT EYE. TEN 
YEARS AGO SHE WAS SUBMITTED TO CATARACT SURGERY OF HER LEFT EYE. HOWEVER THEY "DIDN'T OPERATE MY RIGHT EYE, BECAUSE 
IT IS A LAZY EYE". VISUAL ACUITY IS COUNTING FINGERS IN HER RIGHT    EYE AND 8/20 IN HER LEFT EYE. IOP IS 48MMHG IN HER RIGHT 
EYE  AND  15MMHG  IN  HER  LEFT  EYE.  SLIT  LAMP  EXAMINATION  SHOWS  CORNEAL  EDEMA,  HYPERMATURE  CATARACT  AND  A  DEEP 
ANTERIOR  CHAMBER  WITH  INTENSE  FLARE,  AND  GROSS  TYNDALL.  AGGREGATES  OF  WHIT  MATERIAL  ARE  PRESENT  OVER  THE 
ENDOTHELIUM AND THE ANTERIOR FACE OF THE LENS. FUNDUS EXAMINATION REVEALS THE PRESENCE OF DRUSEN IN HER LEF EYE, BUT 
IT IUS NOT POSSIBLE IT IN HER RIGHT EYE, DUE TO THE DENSITY OF THE CATARACT. THE MOST PROBABLE DIAGNOSIS IS: 
A) ACUDE ANGLE‐CLOSURE GLAUCOMA 
B) PHACOLYTIC GLAUCOMA 
C) PHACOMORPHIC GLAUCOMA 
D) CHOROIDAL MELANOMA 
 
THE MOST APROPIATE TREATMENT FOR THIS PATTIEN IS:   
A) OCULAR HYPOTENSIVE DRUGS, TOPICAL STEROIDS AND THE CATARACT SURGERY ON HER RIGHT EYE. 
B) OCULAR HYPOTENSIVE DRUGS, AND TOPICAL STEROIDS ON HER RIGHT EYE 
C) ENUCLEATION OF HER RIGHT EYE 
D) EVISCERATION OF HER RIGHT EYE 
 
ANSWERS 
B, A 
 
A 45‐YEAR‐OLD HEALTHY MAN HAS STARTED TREATMENT WITH LATANAPROST AND TIMOLOL 0.5% FOR A RECENTLY DIAGNOSED OPEN 
ANGLE GLAUCOMA. HE TELLS YOU THAT HE CAN'T KEEP ON WITH HIS FRIENDS WHEN HE IS PLAYING PADDLE. WICH OF THE FOLLOWING 
WOULD BE THE WISER ATTITUDE? 
A) CHANGE LATANAPROST FOR A DIFFERENT DRUG 
B) CHANGE TIMOLOL FOR A DIFFERENT DRUG 
C) CHANGE BOTH DRUGS 
D) SEND HIM T THE CARDIOLOGIST 
 
ANSWER B 
No entendí la pregunta pero es poco factible que latanaprost tenga que suspenderse. Timolol debe usarse con cuidado en cardiopatas y 
broncópatas. Probablemente este paciente tenga alguna contraindicación.   
 
 
 
 
 
 
 
 
 
 
 
 
 
 
 
 
 
 
 
 
 
 
 
 
PATOLOGIA DE LAS VÍAS VISUALES 
 
 
DEFINICIÓN 
La vía visual puede interrumpirse dando lugar a tres síndromes: 1)Sx del nervio óptico, 2)Sx quiasmático, 3)Sx retroquiasmático. 
 
PATOGENIA 
El conocimiento de estos tres síndromes permite inferir el sitio del proceso patológico (tiumor, traumatismo, EVC, aneurisma), en 
función de los datos clínicos. 
La vía óptica se inicia en la retina (ahí se encuentran cels bipolares [1eras] y cels ganglionares [2das]), los axones de las cels ganglinares 
se unen y forman el nervio óptico que penetra el agujero craneal. La unión de ambos nervios ópticos constituye el quiasma óptico. Las 
fibras retinianas nasales se decusan pero las temporales continuan su trayecto. Posterior al quiasma hay cintillas hasta llegar al cuerpo 
geniculado lateral. En el cuerpo geniculado se desprenden las 3eras neuronas. Estas 3era neuronas constituyen las radiacione ópticas y 
parten a la corteza visual. 
[Corregir cuadrito, el "b" es bitemporal no binasal] 
a) Hemianopsia derecha, b)hemianopsia bitemporal [heterónima] c)hemianopsia nasal unilateral derecha, d)hemianopsia homónima 
izquierda, e)cuadrantopsia homónima inferior izquierda. f) hemianopsia homónima izquierda. 
 
 

 
 
 
 
 
DIAGNÓSTICO 
Papiledema es la "hinchazón" de la papila secundaria a hipertensión intracraneal (por hemorragia IC o aumento de LCR. Una vez que se 
leva el LCR entre 1‐5 días habrá edema de papila. Si es por hemorragia IC tardará 2‐8hrs.   
Papiledema desarrollad tarda de 6‐8 semanas en recuperarse. 
Síntomas oscurecimiento con conservación de agudeza (en papilitis la agudeza disminuye desde el comienzo). Episodios de 10‐30 seg 
frecmente provocados por cambios posturales. Diplopia si parálisis uni o bilateral del VI par.    Cefalea que empeora en la mañana o con 
Valsalva. Náuseas y vómitos. 
Signos: reflejos pupilar normal, FO con afectación bilateral gralmente... 
  Precoz. Papila borrosa, rimero superior e inferior, luego nasal y al final temporal.   
  Desarrollado: hemorragia retinianas, papilares, peripapilares, focos blancos algodonosos, pérdida de plsación benosa 
espontánea. 
  Crónico: tarda en desarrollarse entre 6‐8 sems, atrofia ópica. 
 
Neuritis óptica o papilitis es la tumefacción papilar asociada a pérdida visual debida a desmilinización del NO o inflamación. 
Etiología: esclerosis múltiple, infecciones virales, inflamaciones de vecindad (senos, meninges, órbitas), inflamación intraocular. 
Síntomas: dismiunución progresiva de agudeza visual 2‐5 días. Adultos 75% unilateral, dolor con movimientos oculares, fénomeno de 
Pulfrich (alt en la prcepción de objetos en movmiento), Signo de Uhtoff (empeoramiento con ejericcio o fiebre). Algunos puedne tener 
síntomas neurológicos.   
Signos: Reflejo pupila alterado, FO no se diferencia del papiledema si existe papilitis. 
 
Sx quiasmático generalmente se deben a tumores extrínsecos (adenoma hipofisario, meningiomas, craneofaringiomas. 
Defecto campimétrico típico==> hemianopsia bitemporal. 
                       
Sx retroquiasmático produce alteraciones campimétricas homónimas. 
  Cintillas ópticas y cuerpo geniculado externo: edema cerebral o alt vascualres la pueden causar. 
  Radiaciones ópticas y córtex visual: tumores, inflamación, trauma, vascular. 
                      
 
 
TRATAMIENTO 
Papiledema: tratar la causa de HIC, si esclerosis múltiple, sino esclerosis múltiple... 
  Hacer RMN, si no tiene alteraciones, tiene bajo riesgo de esclerosis múltiple, se pueden dar esteroides..   
  Si áreas de desmilinización: bolos de esteroides para disminuir recurrencia de neuritis ópitca.   
Sx quiasmáticos: dependerá de la causa.   
 
CONTENIDO ADICIONAL CLAVE 
Corteza visual primaria [Área 17] recibe la radiación óptica del núcleo geniculado lateral del tálamo. La lesión del área 17 produce 
ceguera completa. Su estimulación produce destellos alucinacines visuales a manera de destellos brillantes.   
 
Corteza visual secundaria [área 18, 19] su estimulación produce alucinaciones visuales realistas, su daño produce incpacidad para 
recnocer rostros familiares (prosopagnosia), pérdida del color en ciertas partes del campo visual. 
´ 
 
 
 
ENFERMEDADES DE LOS PÁRPADOS, APARATO LAGRIMAL Y CONJUNTIVA 
CONJUNTIVITIS GENERALIDADES 
DEFINICIÓN 
Inflamación de conjuntiva tarsal y bulbar atribuida a distantas causas y que ocasionea escozor ocular, sensación de cuerpo extraño, 
presencia de folículos o papilas, hiperemia, epífora, fotofobia, secreción serosa, fibrinosa o purulenta.   
 
SALUD PÚBLICA 
FR: disfunción de glándula de Meibomio, deficiencia de película lagrimal ==> conjuntivitis papilar gigante 
FR: trauma, def severa de    película lagrijmal, mal posición palpebral==> conjuntivitis mucopurulenta 
FR lente de contacto, medicamentos tópicos, laxitud palpebral===> conjuntivitis mecánica irritativa 
Fr: pediátrico con sinusitis, OMA, faringitis, obstrucción de conducto nasolagrimal==> conj. mucopurulenta 
 
DIAGNÓSTICO 
Mucosa==> alérgico 
Mucopurulenta==>bacterina 
Acuosa==> irritativa o def. de película lagrimal. 
 
Aguda: bacteriana, UV, por lente o trauma 
Recurrente: atópica 
Crónica: def de plícula lagrila, laxitud palpebral. 
 
Unilateral: mecánica o químia 
Bilateral: bacteriana, UV def de película lagrima, alérgica.   
 
TRATAMIENTO 
Conjuntivitis alérgica: [PATANOL 0.1% gotas ofálmicas] hidrocloridrato de olopatadina 0.1% Aplicar 1 gota cada 12 hrs, terminar un 
frasco y continuar con [ALERCROM 4% gotas oftálmicas] Cromoglicato de sodio durante período de recurrencia. 
Prednisolona 5mg/ml. Aplicar 1 gta cada 8hrs por 5 días máximo (riesgo de cataratas e hipertensión ocular secundaria= 
Conjuntivitis mecánica irritativa: Hipromelosa 0.5% gotas oftálmicas. Aplicar 1 gta cada 2 hrs por 5 días. Si disfunción de película 
lagrimal de por vida.   
Conjuntivitis mucopurulenta: Cloranfenicol 5mg/ml gotas ofálmicas . Aplicar 1 gta cada 4 hrs pr 7 días. Si alergia o hipersensibilidad 
Neomicina, polimixina, gramicidina 1 gota cada 4 hrs por 7 días. 
 
 
CONJUNTIVITIS BACTERIANA DEL ADULTO 
 
DEFINICIÓN 
Enfermedad común y autoilimitada 
 
ETIOLOGÍA 
Grampositivos: S. aureus, S. epidermidis, H.influenzae, Moraxella. 
N. gonorrhoeae: secreción mucopurulenta, invasión de epitelio sano.   
 
CLÍNICA 
Comienzo agudo, bilateral, ojo rojo, secreción purulenta, sensación de cuerpo extraño, prurito, lagrimeo, fotofobia, disminución de la 
agudeza visual, edema palpebral, papilas, folículos, secreción en fondo de saco, hiperemia conjuntival, infiltrados cornales. 
 
PARACLÍNICOS 
Si la secreción es abundante: cultivo agar chocolate o medio Thayer Martin, por sospechar de N.honorrhoeae 
 
DIAGNÓSTICO DIFERENCIALES 
Conjuntivitis viral 
Dacriocistitis 
Obstrucción del conducto nasolagrimal 
 
TRATAMIENTO 
Cloranfenicol, polimixina B,    bacitracina. 
Aminoglucósidos: gentamicina, neomicina, netilmicina. 
Quinolonas: ciprofloxacino, gatifloxacino y moxifloxacino. 
Suspender uso de LDC 
Medidas de higiene 
 
 
PACIENTE MASCULINO DE 18 AÑOS, QUIEN ACUDE A CONSULTA POR REFERIR SENSACIÓN DE CUERPO EXTRAÑO, DOLOR OCULAR, Y 
SALIDA DE SECRECIÓN MUCOPURULENTA EN AMBOS JOS DE 5 DÍAS DE EVOLUCIÓN.    A LA EF SE OSBERVA AGUDEZA VISUAL 
OD:20/80, OI 20/30 (DIFÍCIL EVALUAR POR DOLOR Y SECRECIÓN). PÁRPADOS EDEMATOSOS, CONJUNTIVA HIPERÉMICA,FONDOS DE 
SACO CON SECRECIÓN PURULENTA ABUNDANTES, CÓRNEA TRANSPARENTE Y POLO POSTERIOR NORMAL. 
 

 
 
¿CUÁL ES LA PRINCIPAL SOSPECHA DIAGNÓSTICA? 
A) CONJUNTIVITIS VIRAL 
B) CONJUNTIVITIS BACTERIANA POR S. AUREUS 
C) CONJUNTIVITIS BACTERIANA POR N. GONORRHOEAE 
D) CONJUNTIVITIS ALÉRGICA COMPLICADA 
 
Siempre que veas un caso de hipermia conjuntivitis    importante con abundante exudado hay que pensar en Neisseria gonorhoea. Las 
conjuntivitis virales muchas veces se presentaran como unilaterales. 
Cuando es S. aureus tiene poca secreción y es bilateral.   
Una conjuntivitis alérgica presentará prurito y tendrá la caracterísita de presentar remisión‐exacerbación 
 
CONDUCTA A SEGUIR EN ESTE PACIENTE: 
A) INICIAR TRATAMIENTO ANTIBIÓTICO EMPÍRICO 
B) TOMA DE CULTIVO Y POSTERIOR INICIO DE MANEJO ANTIMICROBIANO 
C) LAVADOS PROFUSOS DE LA SECRECIÓN 
D) EXPECTANTE YA QUE ES AUTOLIMITADA 
 
Recuerda que si la sospecha es de conjuntivitis vírica o simple bacteriana es diagnóstico es clínico, pero si se sospecha de gonorrea se 
indica cultivo antes de dar tratamiento.   
 
MÉTODO DIAGNÓSTICO MÁS ESPECÍFICO PARA ESTE PADECIMIENTO: 
A) CULTIVO THAYER MARTIN 
B) GIEMSA 
C) GRAM 
D) CULTIVO AGAR SANGRE 
 
TRATAMIENTO INDICADO EN ESTE PACIENTE: 
A) CEFTRIAXONA IV O IM 25‐50MG/KG 1 SOLA DOSIS. 
B) ERITROMICINA (12.5MG/KG VO O IV CADA 8 HRS POR 14 DÍAS) 
C) MEDIDAS DE HIGIENE SIN SUSPENDER EL USO DE LENTE DE CONTACTO 
D) CLORANFENICOL 1 GOTA CADA 4 HRS POR 10 DÍAS.   
 
No olvidar que generalmente gonococo y clamidia van juntas, así que habrá que valorar manejo conjunto para este microorgansmo. 
Podriamos dar como coadyunvante eritromicina (para clamidia) 
 
PRINCIPAL RIESGO EN ESTE PACIENTE: 
A) DOLOR AGUDO INCONTROLABLE 
B) TRATAMIENTO PROLONGADO 
C) AUMENTO DE PRESIÓN INTRAOCULAR 
D) PERFORACIÓN CORNEAL 
 
N. gonorrhoeae tiene la capacidad de penetrar el tejido epitelial sano, por lo tanto el principal riesgo y la importancia de la premara y 
agresividad del tratamiento es evitar una perforación corneal con endoftalmitis secundaria.   
 
RESPUESTA C, B, A, D 
 
A parte de la cantidad de exudado, ¿existen diferencias adicionales entre conjuntivitis bacteriana simple y gonocócica? 
La única diferencia será el exceso de secreción. No hay forma de confundirla. 
 
CONTENIDO ADICIONAL CLAVE 
Unilateral viral 
Foliculos viral 
Papilas: bacteriana y alérgica 
Infiltrado numular: viral 
Úlcera en escudo: alérgica 
Ulcera corneal: bacteriana 
 
Los principales en bacteriana: eritromicina, ciprol cloranfenicol. En la práctica se puede dar esteroide, para fines de ENARM la respuesta 
más adecuada es solo antibiótico. Para fines de ENARM cloranfenicol está bien, pero en la práctica tiene mucha resistencia y ya no se 
utiliza. Mejo tobra, eritro, cipro.   
 
 
 
CONJUNTIVITIS NEONATAL 
 
 
 
 
 
 
 
 
 
 
 
 
 
 
 
 
 
 
 
 
 
 
 
 
 
 
 
 

 
 
 
 
 
 
 
 
 
 
 
 
 
 
 
 
 
TOXOPLASMOSIS 
Causa No 1 de uceítis posterior 
Transmisión: ingesta de carne contaminada, leche no pasteurizada, heces de gato, transplacentario. 
Clínica: uveítis granulomatosa, retininas, viritis. 
Típico: imagen en faro de niebla (reactivación). 
Dx: anticuerpos IgG (memoria), IgM (actividad) vs toxoplasma 
Tx: clindamicina, TMP/SMX 
 
 
TRACOMA 
Serotipo A, B, C son los más frecuentes. 
 
 
 
 
 
 
ORZUELO Y CHALAZIÓN 
 
ETIOLOGÍA 
Obstrucción e infección bacteriana aguda de las glándulas sebáceas palpebrales, con mayor frecuencia a partir del glándulas de 
meibomio (internas) y Zeiis (externas) 
Causa No1: S. aureus 
 
CLÍNICA 
Tumoración palpebral rojiza, caliente, dolorosa, puede haner punto de drenaje visible, solitario, unilateral, único o múltiple. 
 
DIAGNÓSTICO 
Clínico 
 
DIAGNÓSTICO DIFERENCIAL 
Celulitis preseptal, carcinoma de células sebáceas, granuloma piógeno, chalazión. 
 
TRATAMIENTO 
Compresas calientes + masaje + ungüento antibiótico (eritromicina, ciprofloacno) en fondo de saco.   
Antibiótico + corticoesteroide 
Crticoide intralesional 
 
PRONÓSTICO 
bueno, pero si se hace crónico ==> chalazión 
 

 
 
PACIENTE FEMENINO DE 1 AÑO QUIEN INICIA SU PADECIMIENTO CON AUMENTO DE VOLUMEN EN PÁRPADO SUPEROMEDIAL, 
DOLOROSO A LA PALPACIÓN, DE 1 CM. BLANDO HIPERÉMICO DE 4 DÍAS DE EVOLUCIÓN. 
 
 
 
SU PRINCIPAL SOSPECHA DIAGNÓSTICA ES:   
A) CHALAZIÓN 
B) ORZUELO 
C) CELULITIS PRESEPTAL 
D) BLEFARITIS 
 
Piensa en orzuelo si es agudo, localizado, blando, doloroso. Piensa en chalazión si >3 semanas, indurado, no doloroso.   
 
LA CAUSA DE DICHO PADECIMIENTO ES: 
A) INFECCIOSO 
B) OBSTRUCTIVO 
C) INFLAMATORIO 
D) METABÓLICA 
 
EL TRATAMIENTO INDICADO POR EL TIEMPO DE EVOLUCIÓN ES:   
A) CURETAJE 
B) EXPECTANTE 
C) COMPRESAS TIBIAS + ANTIBIÓTICO 
D) REFERIR AL OFTALMÓLOGO POR GRAVEDAD DEL CUADRO 
 
El curetaje está inicado en chalazión.   
 
RESPUESTA B, B, C 
 
 
CELULITIS PRESEPTAL Y ORBITARIA 
 
DEFINICIÓN 
Inflamación de tejido subcutáneo (los párpados) que no se extiende por detrás del tabique orbitario, no existe afección ocular ni 
orbitaria.   
Habitualmente no existen datos de respuesta inflamatoria sistémica (fiebre, taquicardia, afección del estado general). 
 
ETIOLOGÍA 
Agentes: S. aureus, antecedente de traumatismo periorbitario y cutáneo.   
Diseminación por infección local: sinusitis, dacriocistitis, infección dérmica.   
A partir de infección a distancia: IVRA 
 
CLÍNICA 
Tumefacción palpebral 
Enrojecimiento 
Ptosis 
Dolor 
Fiebre baja 
Eritema palpebral doloroso 
Edema palpebral   
Agudeza Visual normal 
Motilidad ocular normal e indolora 
No hay proptosis 
Conjuntiva y esclera sin inflamación 
 
ABORDAJE 
HC oftalmológica 
TC orbitaria y de senos par*anasales 
BH, hemocultivo y cultivo.* 
No se piden de forma sistemática. 
 
DIAGNÓSTICO DIFERENCIAL 
Celulitis orbitaria: disminución de agudeza visual + dolor intenso ocular    + limitacióon de mov extraoculares + fiebre + ataque a edo 
gral + leucocitosis 
Inflamaciójn orbitaria idiopática 
Dacriocistitis y conjuntivitis.   
Traumatismo. 
 
TRATAMIENTO 
Amoxicilina + ác. clavulánico 
Cefaclor   
Antibióticos tópicos: eritromicina 
Drenar absceso palpebral en caso de estar presente.   
 
PACIENTE MASCULINO DE 9 AÑOS, QUIEN ACUDE AL HOSPITAL INQUIETO, CON AUMENTO DE VOLUMEN PALPEBRAL, ERITEMA 
PALPEBRAL DERECHO DE 5 DÍAS DE EVOLUCIÓN. A LA EF SE OBSERVA AGUDEZA VISUAL AO: 20/40, MOTILIDAD OCULAR NORMAL Y 
CONSERVADA, PÁRPADOS ERITEMATOSOS, DOLOROSOS A LA PALPACIÓN Y CONJUNTIVA HIPERÉMICA.   

 
SU PRINCIPAL SOSPECHA DIAGNÓSTICA ES: 
A) CELULITIS ORBITARIA 
B) CELULITIS PRESEPTAL 
C) DACRIOCISTITIS CRÓNICA AGUDIZADA 
D) CONJUNTIVITIS BACTERIANA SEVERA 
 
EL ESTUDIO DIAGNÓSTICO A SOLICITAR DE FORMA INICIAL ES:   
A) TAC 
B) BHC 
C) RM 
D) ECOGRAFÍA OCULAR 
 
EL MANEJO INICIAL EN ESTE PACIENTE ES: 
A) INICIO DE ANTIBIÓTICO TÓPICO EN UNGÚENTO 
B) INICIO ANTIBIÓTICO INTRAVENOSO 
C) REFERIR AL OFTALMÓLOGO 
D) HOSPITALIZACIÓN 
 
Por ser pediátrico, el manejo no puede hacerse de forma ambulatoria y debe haerse seguimiento por un especialista en oftalmología, 
por las posibles complicaciones.   
 
LA COMPLIACIÓN MÁS FRECUENTE DE DICHA PATOLOGÍA ES: 
A) CELULITIS ORBITARIA 
B) CELULITIS PRESEPTAL 
C) TROMBOSIS DEL SENO CARVERNOSO 
D) DISMINUCIÓN PERMANENTE DE LA AGUDEZA VISUAL 
 
La compliación más frecuente es la celulitis orbitaria, y la más grave es la trombosis del seno cavenoso.     
Si el diagnóstico del paciente fuere celulitis orbitaria, la complicación más frecuente sería la tombosis del seno cavernoso.   
 
RESPUESTA B, B, C,   
 
 
PERLAS 
Niño, masa aguda, dolorosa          Orzuelo 
Niño, masa crónica, no dolorosa          Chalazión 
Aumento de vol + visión normal + mov normales      Celulitis preseptal 
Aumento de vol + agudeza visual disminuidas + limitación de mov oculares  Celulitis orbitaria 
 
Principal causa de obstrucción de vía lagrimal baja en niños: obstrucción de válvula de Hasner 
Tratamiento de obstrucción    de vía lagrimal baja en niños: masaje, no cirugía. 
 
Tumor benigno más común en niños: quiste dermoide 
Tumor maligno más común en niños: retinoblastoma 
 
Tumor más freuente de la órbita en niños: hemangioma capilar 
Tumor benigno más común en adulto: hemangioma cavernoso 
Principal causa de proptosis en adulto: orbitopatía distiroidea (no pongan ningún tumor) 
 
 
PATOLOGÍA DE COLON ANO Y RECTO 
FISURA ANAL 
[La Salle] 
 
 
GENERALIDADES 
Ocurren en línea media, 90% son posteriores, 10% son anteriores, 1% mixtas. Nunca deben ser en otro sitio, de ser así considerar 
enfermedad inflamatoria o cáncer. 
La división entre úlcera anal aguda y crónica es de 3 semanas.   
 
La úlcera crónica hace borde fibrosos y tríada patognomónica: úlcera + papila centinela + papila hipertrófica 
 
ETIOPATOGENIA 
Resultan de la dilatación busca del canal anal 
Ruptura del anodeermo 
Exposición del esfínter anal interno 
Espasmo muscular 
Ciclo vicioso, el pacinete n quiere defectar, con almacenamiento de más heces y endurecidas, rasgarán nuevamente.   
Isquemia del anodermo y no cura. 
 
CLÍNICA 
Dolor desgarrante o quemante. 
Sangrado: GOTEO, no mezclado con las heces.   
Estreñimiento, raras veces son indoloras.   
 
PARACLÍNICOS 
BH, QS, ES, TP/TTP en caso de que sea candidato a cirugía. 
Anoscopia o sigmoidoscopia: no forman parte del examen inicial, descarta malignidad o enferemdad inflamatoria intestinal, biopsia en 
cualquier fisura en línea media que no sane, sospecha de Crohn, malignidad, manometría anal para corroborar al aumento de la 
presión. 
 
Diagnóstico diferencial: sospechar si no está sobre la línea media. (Crohn, TB, malignidad, absceso, fístula, CMV; hepes, Chlamydiosis, 
sífilis, SIDA, actinomicosis)   
 
TRATAMIENTO MÉDICO 
Ablandar heces (32 grs diarios de fibra), anaglésicos e inflamatorios locales, baño de asiento (solución templada 2 cucharadas de 
bicarbonato, 2 cucharadas de árnica líquida c 12 hrs por 10 días), pomada con 1 tableta molida de isosorbide (10mg/kg) aplicar cada 12 
hrs después de cada baño de asiento,    90% curan, toxina botulínica, nitroglicerina 0.2% 
 
 
TRATAMIENTO QUIRÚRGICO 
Úlceras crónicas o recurrentes.   
Fistulectomía + Esfinterotomía lateral parcial 
 
 
SE TRATA DE PACIENTE DE LA TERCERA DÉCADA DE LA VIDA, CON PRESENCIA DE DOLOR INTENSO AL MOMENTO DE DEFECTAR, REFIERE 
ADEMÁS DE SANGRADO, QUE SE HACE EVIDENTE AL EVACUAR CON    PRESENCIA DE GOTEO, TAMBIÉN LA INTENSIDAD DEL DOLOR 9 
DE 10, CON TIEMPO DE EVOLUCIÓN DE MÁS DE 6 SEMANAS.   
 
A LA EXPLORACIÓN FÍSICA USTED BUSCA: 
A) TRÍADA DE PAPILA HIPERTRÓFIA, CENTINELA Y ÚLCERA. 
B) ÚLCERA AGUDA CON FONDO SANGRANTE 
C) ORIFICIO EXTERNO EN REGIÓN GLÚTEA 
D) NINGUNA DE LAS ANTERIORES 
 
La tríada clásica de prentación de un fisura crónica es la presencia de úlcera, así como de papila hipertrófica y centinela.   
El orificio extrno en región glútea es la presentación clínica de una fístula.   
Úlcera aguda con fondo sangrante, es la presentación de fisura aguda.   
 
LA CIRUGÍA INDICADA PARA EL CASO EXPUESTO ES: 
A) HERROIDECTOMÍA 
B) FISTULECTOMÍA 
C) DRENAJE 
D) FISURECTOMÍA MÁS ESFINTEROTOMÍA LATERAL PARCIAL.   
 
La fístula no produce dolor.   
Drenaje es el tx de absceso.   
Es necesario hacer estinterotomía lateral parcial para tomer la hipertensión del esfínter y el cículo vicioso.   
 
RESPUESTAS A, D 
 
     
 
[FISURA ANAL][r] 
MUJER DE 33 AÑOS CON ANTECEDENTE DE MIGRAÑA, ACUDE A CONSULTA POR FISURA ANAL. 
EL  SIGUIENTE  MEDICAMENTO  PARA  EL  TRATAMIENTO  DE  LA  FISURA  ANAL  ESTARÁ  ESPECIALMENTE  CONTRAINDICADO  EN  ESTA 
PACIENTE: 
   
A) TRINITRATO DE GLICERILO TÓPICO. 
B) DILTIAZEM TÓPICO. 
C) TOXINA BOTULÍNICA LOCAL. 
D) LIDOCAÍNA CON HIDROCORTISONA TÓPICA. 
 
RESPUESTA A 
El trinitratato de glicerilo tópico cura las fisuras anales mejor que un placebo independientemente de sus dosis. Ocurre curación en un 
60% de los pacientes. Se asocia a cefalea en el 25% de los pcaintes por lo que está especialmente controindicado en paciente con migraña. 
Diltiazem es tan eficaz como el glicerilo pero con menos efectos secundarios razón por la cuál es de primera elección en el manejo de 
fisura anal.   
La toxina botulínica es tan efectiva que el glicerilo pero con costos más elevados, esta indicada en fisuras resistenten a tratamiento con 
trinitrato de glicero y ditiazem. 
El tratamiento con anestésicos y corticoides de forma asilada o complementaria están indicados en la enfermedad hemorroidal.   
 
 
  ABSCESO Y FÍSTULA ANO‐RECTAL 
[La Salle] 
 
 
GENERALIDADES 
Cuando no está relacionado con una enfermedad sistémica, el absceso entonces se considera criptoglandular.   
Son más difíciles de tratar los abscesos que están proximales o circunferenciales al plano interesfinteriano.   
Pueden ser: 
  Perianales 
  Isquiorrectales 
  Interesfinterianos 
  Supraelevadores 

 
La exploración del absceso es difícil y debe hacerse bajo antestesia.   
 
Las fístulas son compliaciones frecuentes (50%), estas son: 
    Interesfinterianas   
    transesfinterianas (menos frecuente)   
    extraesfinterianas. 
SALUD PÚBLICA 
Abscesos: H2:M1,    20‐60 años. 
 
 
CLÍNICA 
Absceso anorectal 
  Dolor intenso, punzante que empora al deambular y con esfuerzos.   
  Fiebre 
  Retención urinaria 
  Sepsis 
Fístula   
  Dolor intenso 
  Descarga hematopurulenta que disminuye el dolor 
  Descarga mucopurulenta crónica 
 
EF: tumoración blanda perianal o rectal, la fístula se logra palpa el conducto.   
 
LABORATORIO Y GABINETE 
BHC, QS, ES, EGO. 
No estudios de imagen en enfermedad no complicada   
 
Enferemedad compliada: la Tomografía con doble contraste es la más adecuda para determinar su extensión. Otros estudios: 
USG transrectal, RMI en enfermedad recurrente o compleja.   
 
DIANGÓSTICO DIFERENCIAL 
Complicaciones de Crohn, hidrosadeinitis supurativa, TB, actinomycosis, trauma, fisuras, carcinoma, lesión por radiación.   
En pacinetes con DM o inmunocompromiso hay que prestar especial atención. 
 
TRATAMIENTO   
Absceso: 
Antibióticos solo alcanzan el 50% de la resolución.   
Drenaje es el principal tratamiento, curación diaria, cierre secundario. 
Baños de asiento si absceso en fase de celulitis.   
AB sistémicos solo si celulitis circundante, bacteriemia o sisgnos de sistemico, inmunodeprimidos. (amoxi clav o dicloxa) 
Fístula:   
Interesfinterianas ==> resección del trayecto 
transesfinterianas ==> setón 
extraesfinterianas==> resección del trayecto 
 
CASOS CLÍNICOS 
SE TRATA DE PACIENTE MASCULINO DE 58 AÑOS, EL CUAL ES DIABÉTICO DE LARGA EVOLUCIÓN, CONTROLADO CON SITAGLIPTINA, 
QUIEN REFIERE PRESENCIA DE DOLOR DE 5 EN ESCALA DE 10 A NIVEL DEL ANO, CON SENSACIÓN DE CALOR LOCAL, SIN PRESENCIA DE 
TUMORACIÓN O INDURACIÓN, PERO QUE AL ESTAR INCLUSO SENTADO CONDICIONA MAYOR DOLOR. A LA EF NO ENCUENTRA ZONAS 
DE INDURACIÓN SOLO AUMENTO DE LA TEMPERATURA LOCAL. 
 
DECIDE REALIZAR: 
A) TACTO RECTAL 
B) ANOSCOPIA 
C) PROCTOSIGMOIDISCOPIA 
D) NINGUNA DE LAS ANTERIORES 
 
El cuadro clínico sugiere la presencia de absceso anal de la región del supraelevador, por lo cual es completamente necesario, 
incialmente realizar tacto rectal y posteriormente anoscopia. Característicamente cuando tenemos absceso del supralevador no 
encuentramos áreas de induración o fluctuación perianales ni glúteas, es por es que precisa tacto rectal o incluso anoscopia.   
 
LA PRINCIPAL COMPLICACIÓN EN ESTE TIPO DE ABSCESO ES:   
A) SEPSIS PERINEAL 
B) SEPSIS PÉLVICA 
C) FOURNIER 
D) NINGUNA DE LAS ANTERIORES 
 
El absceso supraelevador se encuentra relacionado    con mayor frecuencia a apertura hacia hueco pélvico, por lo que su rápido 
diagnóstico y drenaje es imperioso, ya que además se presenta con más frecuencia en inmunodeprimidos. Este absceso es el más 
riesgoso.   
 
La sepsis perineal se presenta como complicación de absceso perianal,Isquierrectal o Interesfinteriano. 
 
RESPUESTA A, B 
  
MUJER DE 33 AÑOS DE EDAD, DE OCUPACIÓN SECRETARIA, POR LO QUE LA MAYOR PARTE DE SU DÍA LABORAL LA PASA EN POSICIÓN 
SENTADA. USTED CONFIRMA EL DIAGNÓSTICO DE ABSCESO PERIANAL AGUDO 
EL MECANISMO FISIOPATOLÓGICO QUE MÁS PROBABLEMENTE ANTECEDÍO A LA FORMACIÓN DEL ABSCESO PERIANAL DE LA PACIENTE 
FUE: 
   
A) UNA FISURA ANAL. 
B) UNA FÍSTULA PERIANAL. 
C) INFLAMACIÓN CRIPTOGLANDULAR. 
D) COLITIS ISQUÉMICA.   
 
LA COMPLICACIÓN ESPERADA DEL ABSCESO PERIANAL DE LA PACIENTE EN CASO DE NO TENER UNA RESOLUCIÓN PRONTA SERÍA:   
A) FISURA ANAL 
B) FÍSTULA PERIANAL 
C) SEPSIS ABDOINAL 
D) PROLAPSO RECTA.   
 
RESPUESTAS C, B 
 
El Absceso anal    frecuente entre 20‐60 años, en ambos sexos. Causas específicas:    cuerpos extraños, traumatismos, enfermedades 
inflamatorias intestinales (Crohn) procesos infecciosos específicos, tumores, secuelas de tratamientos radioterápico y otras enf anales 
como la fisura anal. Causas inespecíficas: como la enf criptoglandular.    La teoría criptoglandular es la base fisiopatológica más aceptada 
del absceso perianal. 
 
La Fisura anal    es una úlcera longitudina por debajo de la línea dentada. Generalmente posterior, menos habitual anterior y en 3% 
pueden  coexistir  las  dos.    Incidencia  igual  en  ambos  sexos,  frec  en  edad  media  de  la  vida.  Generalmente  idiopáticas.  Etiología  más 
probable traumatismo agudo del conducto anal durante la defecación.    Si fístulas múltiples pensar en tuberculosis, sífilis, Crohn o sx de 
inmunodeficiencia.    Si es crónica se verá como úlcera con bordes indurados, acompañada de pliegue cutáneo indurado en el extremo 
distal (hemorroide  centinela)  y  una  papila hipertrófica  en  el  borde  proximal  (pólipo  de  Lane).  La  físura puede  acompañar  al  absceso 
perianal agudo e incluso favorecerlo, pero no forma parte de la fisiopatología.   
 
La Fístula perianal es un conducto que comunica una cripta anal o el lumen del recto con la piel. Las fístulas simples son normalmente 
interesfinterianas  [B]  o  transesfinterianas  bajas  [C],  generalmente  de  trayecto  único.    La  fístula  compleja  tiene  un  trayecto  que 
comprende más del 30‐50% del esfínter externo (transesfintérica alta [C] , supraesfintérica [D] o extraesfintérica[E]), es de localización 
anterior en mujeres.    Se presentan en 50%    de los pacientes con absceso anorrectal desarrollan una fístula.    Los antecedentes de 
importancia  en  este  caso  son:    patología  intestinal  asociada,  eventos  que  comprometan  el  esfínter,  cirugía  anorrectal,  trauma 
obstétrico,  procesos  infecciosos  anorrectales.  Presentan  como  descarga  a  través  del  orificio  externo,  en  ocasiones  dolor.  EF  orificio 
externo con descarga y se palpa trayecto con fibrosis. La fístula perianal es más bien una complicación del absceso.   
 
La Colitis isquémica es la forma de presentación más frecuente de isquemia intestinal (70‐75%). Afecta princilamente a adultos mayores, 
y en jovenes asocia DM2, LES o crisis de células falciformes. La pancreatitis se ha asociado como factor al ocluir en ocasiones los vaso 
mesocólicos.    La  fisiopatlogía  de  la  colitis  isquémica  yace  en  una  insuficiencia  venosa  o  arterial  previa.  La  colitis  isquémica  derecha 
aislada de relaciona con    cardiopatía crónica, en especial estenosis aórtica. Se manifiesta como un dolor abdominal agudo, hemorragia 
gastro intesitnal y diarrea aguda. No se relaciona con abscesos perianales, patología distinta.   
 
  Causas de sepsis abdominal:   
  Colecistopancreatitis      27.5% 
  Perforación de colon    17.2 
  Heridas penetrantes abd.  13.7 
  Apendicitis aguda    13.7 
  Politraumtizado    10..3 
  Abscesos peripancráticos  6.9 
  Trombosis mesentéricas  3.4 
 
El prolapso rectal se asocia a: demensia senil, parasitosis (esquistosomiasis, tricocefalosis, amebiasis), enf neurológias (espina bífida), 
multiparidad, histectomía, estreñimiento de larga evolución, hipermotilidad sigmdea.   
 

 
 
 
  
 
 
 
 
FÍSTULA ANAL 
DEFINICIÓN 
Fístula simple solo un conducto entre orificio primario y secundario. 
Fístula compleja aquella que precisa sección de aparato esfinteriano. 
 
SALUD PÚBLICA 
50% de los abscesos fistulizan.   
95% de los abscesos son de origen criptoglandular o inespecífico. 
5% de los abscesos son específicos o secundarios (ej Crohn, tb, carcinoma rectal, caricnoma anal, actinomicosis) 
 
DIAGNÓSTICO 
Clínica. Dolor cíclico x acumulación de material en el trayecto. 
 
USG endoanal estudio inicial. 
RM estándar de oro para fístulas completas y recidivantes. 
Fistulografía solo útil si extraesintéricas 
TAC solo si secundaria a Crohn. 
 
TRATAMIENTO 
Fístula:   
Interesfinterianas ==> resección del trayecto 
transesfinterianas ==> setón 
extraesfinterianas==> resección del trayecto 
 
Fístula anal simple: fistulotomía, marsupialización.   
Fístula anal completa: determinación por Parks, no existe manejo estándar. 
  Con riesgo de incontinencia: legrado, cierre de orificio primario, colocar sello de fibrina y colgajo de mucosa. 
  Sin riesgo de incontinencia setón y fistulotomía 
Fístula anal por Crohn: si asintomático no manejo, con síntomas fistulotomía, setón no cortante, si compleja setón para drenaje.   
 

  
 
[FÍSTULA ANAL] 
  MUJER  DE  38  AÑOS  QUE  ACUDE  A  URGENCIAS  AL  PRESENTAR  SALIDA  DE  MATERIAL  PURULENTO  POR  EL  ANO.  HACE 
APROXIMADAMENTE 25 DÍAS PRESENTÓ UN ABSCESO GLÚTEO QUE DRENÓ Y CURÓ ESPONTÁNEAMENTE. 
 
EL DIAGNÓSTICO CLÍNICO MÁS PROBABLE EN ESTE PACIENTE ES: 
A) ENFERMEDAD DE CROHN. 
B) COLITIS ULCERATIVA CRÓNICA. 
C) DIVERTICULITIS COMPLICADA. 
D) FÍSTULA PERIANAL 
 
EL TRATAMIENTO INDICADO EN ESTE PACIENTE SERÁ CON: 
   
A) FISTULOTOMÍA CON MARSUPIALIZACIÓN. 
B) ANTIBIÓTICOS Y COLOSTOMÍA. 
C) INMUNOSUPRESORES Y ANTIBIÓTICOS. 
D) DRENAJE ABIERTO. 
 
RESPUESTA D, A 
Las fistulas anorrectales se pueden explicar por las cuatro patologías mencionadas, pero el antecedente de absceso 25 días antes soporta 
la fisiopatología de la fístula perianal y no de las demás condiciones.   
La enfermedad de Crohn    es una trastorno inflamatorio crónico transmural que frecuenemnte afecta íleon, ciego y colon. Se caracteriza 
por diarrea crónica, dolor abdominal, pérdida de peso, fiebre y sangrado rectal.    Una de las características de esta enfermedad es la 
presencia de fístulas que derivan a colon, vejiga, vagina o piel.    Una característica primordial de Crohn es su capacidad para fistulizar. 
1/3 parte tiene afección perianal formando fisuras anales, abscesos perianales y fístulas.   
La colitis ulcerativa se caracteriza por inflamación de la mucosa. La mayor parte de los casos se limita al recto o rectosignmoides, casos 
poco frecuentes afectarían al colon o íleon. Recordar que por lo general CUCI no produce fístulas.   
La enfermedad diverticular tiene clínica variada, como dolor abdominal crónico en cuadrante inferior izquierdo o sangrado diverticular, 
trastorno de la motilidad. En le diverticulitis se presenta inflamación de los divertículos, es un cuadro agudo caracterizado por dolor, 
sensibilidad,  masa  en  cuadrante  inferior  izquierdo,  fiebre  y  leucocitosis.  La  diverticulitis  complicada  se  define  como  aquella  que  se 
acompaña de absceso, fístula, obstrucción, perforación.   
La fístula anal se presenta en 50% de los casos que ha desarrollado absceso anorrectal. Todo absceso no resuelto tiende a fistulizarse ya 
que el material purulento que contiene busca un trayecto de salida. El manejo Qx incluye fistulotmía, fistulectomia con o sin reparación 
de esfínteresm sedal, colocación de setón de corte o de drenaje, avance de colgado de mucosa retal y aplicación de fibrina.    El setón de 
corte es una técnica quirúrgica para quellas fístulas perianales en las que se encuentra comprometido alguno o los dos esfrínteres anales. 
Ciertamente el setón es un drenaje abierto de la fístula pero se reserva a quellos casos en que las fístulas son complicada o el abortaje pr 
fistulostomía no ha sido suficiente.   
 
 
Diverticulitis complicada==> antibióticos y colostomía? 
Fístula perianal==> fistolotomía con marsupialización 
Diverticulitis complicada==>   
Colitis ulcerativa crónica==> inmunosiupresores y antibióticos
 
  ENFERMEDAD DE CROHN [EC]  CUCI 
Definición  Es un trastorno autoinmunitario que genera inflamación persistente en íleon, sin embargo puede afectar boca,  Inflamación mucosa difusa de colon y recto. Progresión proximal. 
intestino delgado, grueso y recto.    Afección más grave: fístulas.    Afeccion más grave: colon tóxico 
Epidemiología  Adultos 15‐35 años.    Bimodal 15‐30 años, 55‐80 años. 
FR: AHF, judío, tabaco, apendicectomía,    FR. HLD‐DR2 y DR1, DR3 y DQ2 con afección extensa. 
5.6 por cada 100,000 habitantes, mayor afectación en mujeres.  FP: apendicetomía, lactancia materna, tabaquismo.   
Clínica  Formas de presentación: 1) inflamatoria, 2) fistulizante, 3) fibroestenótica (quirúrgica)    Distal (proctitis o proctosigmoiditis) 
Dolor abdominal tipo cólico (oclusión / suboclusión)  Extensa: colitis izquierda (ángulo esplénico), colitis extendida (ángulo hepático) y pancolitis. 
Diarrea acuosa persistente, Fiebre, Pérdida de peso, Anemia y/o hemorragia digestiva  95% tiene afección a recto, 50% a recto y sigmoides, 30% colitis izquierda, 20% pancolitis.   
Inapetencia, fatiga, dolor con las deposiciones    Diarrea sanguinolenta (cardinal), dolor abdominal tipo cólico, urgencia, tenesmo, distensión, 
Otros: Úlceras bucales, gingivtis, dolor articular, úlceras en piel, hepatitis, estreñimiento, fisuras, fístulas y abscesos  evacuaciones con moco y sangre (diarrea exudativa). 
perianales. (Fístulas gralmente en área rectal), pasa esto porque afecta de la mucosa a la serosa. Rectorragia.    Sistémicas: fiebre, malestar gral, astenia, adinamia, pérdida de peso, en niños retraso en el 
neurodesarrollo.   
Extraintestinales:    artritis colítica o esponditilis anquilosante entre 16‐.26%, eritema nodoso y 
pioderma gangrenoso en 5%, conjuntivitis, uveítis y epiescleritis en 4%, hepatobiliares 
(pericolangitis, colangitis esclerosante) y pulmonares. 
Paraclínica  BHC (anemia), Coproparasitoscópico/ coprocultivo (descarta infección)  Laboratorio 
VSG y PCR, PFH (causa hepatitis), VIH (por diarrea crónica), Sangre oculta en heces (causa sangrados), Perfil tiroideo  Descatar C. difficele, p‐ANCAs presentes en 60‐65%, ASCAS: positivos 5% 
(hipotiroidismo autoinmune puede asociar EC). ANCAs: presentes en 20‐25%% ASCAS 47‐76%  Imagen 
Tránsito intestinal estudio inicial. Empedrado.(valora extensión, búsqueda de estenosis?)  Colon x enema: imagen granular.   
Panendoscopia o colonoscopia (según dia el tránsito)    Gold estándar (puede haber afección alta)  Gold: endoscopia + biopsia (la primera opción a considerar es la RSM por ser menos invasiva. 
Ileocolonoscopia    (úlceras aftosas, úlceras profundas, pérdida de patrón vascular, lesiones en parche, imagen en  Leves a moderadoscolonoscopia 
empedrado, granularidad, eritema, friabilidad, pseudopólipos, estenosis) Generalmente respeta recto.  Moderados a severos rectosigmoidoscopia (reduce riesgo de perforación intestinal). 
Toma de biopsias (granulomas no caseificantes 15‐60%, afección transmural metaplasia foveolar, fisuras, ausencia  (Eritema, friabilidad en mucosa, pérdida de patrón vascular, granularidad de mucosa, úlceras, 
de infección o neoplasias, inflamación crónica [linfocitos y cels plasmáticas], abscesos en criptas.  pseudopólipos, estenosis rara y sugiere malignidad) Recto generalmente afectado. 
  Patología 
Anticuerpos antisarcomises: positivos en 41‐76%  Biopsia: inflamación de mucosa y submucosa con infiltrado neutrófilo y abscesos en criptas. 
Criptas atróficas. 
Diagnóstico  No existe gold standard para diagnóstico de EC, se usan métodos clínicos, endoscópicos, histológicos y radiológicos.  Clínico + rectosigmoidoscopia o colonoscopia + biopsia + no infección en heces.   
Para diagnóstico de EC es necesaria: ileocolonoscopia (entra por recto), endoscopia alta (entra por boca), y examen 
histopatológico.   
En la presentación aguda son útiles: Tránsito intestinal, USG abdominal, TAC 
Tratamiento  Inducción: prednisona, antiinflmatorios 5‐ASA (mesalazina y sulfazalasina) si enfermedad leve que no desea  Antiinflmatorios 5‐ASA: mesalazina, sulfazalasina, para leves y moderadas. 
médico  glucocorticoide.  Severas: esteroides. 
Mantenimiento: azatrioprina  Azatioprina sirve para desescalar: estroides azatioprina mesalazina. 
CDAI si buena respuesta continuar azatioprina y valorar suspensión en 4 años. Si mala respuesta incrementar   
dosis de azatioprina.  Azatioprina, 6    y    ciclosporina, 
CDAI en 2 meses si buena respuesta continur azatioprina y valorar suspensión en 4 años. Si mala respuesta 
realizar TORCH, PPD, iniciar antiTNF. 
Inducción: infliximab o adalimumab 
Mantenimiento: infliximab o adalimumab. 
CDAI c 8 sem si buena respuesta continuar, si mala respuesta infliximab + azatioprina 
En agudo si no remite con esteroides también puedo dar metotrexate /anti‐tnf 
Tratamiento  Enfermedad ileal estenosante.  Prococolectomía total no restaurativa si: Perforación, colitis tóxica, hemorragia, fracaso de tx 
quirúrgico  Estenosis < 4cm alcanzable con endoscopio dilatación si falla resección quirurgica o estricturiplastia  médico, prevención de carcinoma, retraso de crecimiento infantil. 
Estenosis <4cm NO alcanzable con endoscopio y con síntomas resección quirúrgica o estricturiplastia.     
Estenosis 4‐10cm  estricturoplastia convencional (Heineke Miculicz) 
Estenosis >10cm estricturoplastia no convencional (Finney) 
Enfermedad ileocolónica 
Con síntomas obstructivos: resección quirúrgica, hemicolectomía con anastomosis si es viable.   
Sin síntomas obstructivos: tratamiento médico.   
Enfermedad colónica 
Estenosante o fistiluzante: resección del segmento afectado.   
Enfermedad perianal 
Si asocia absceso: setón de drenaje + a manejo médico; Si no asocia absceso: manejo médico 
 
 

 
 
INCONTINENCIA FECAL 
PATOGENIA 
Muchas causas locales, espinales y cerebales.   
Anticolinérgicos pueden lesionar plexo mientérico    y producir u esfinter laxo, y reflejo anormal de nervios sacros. 
 
DIAGNÓSTICO 
Manometría anorrectal: arma diagnóstic amás utilzida,    valora ausencia de reflejo rectoanal inhibitorio (daño intramural),    y ausencia 
de reflejo reactoanal cotráctil(daño nervioso distal). 
USG endoanal: útil si la IF es secundaria a trauma o iatrogénicas. 
Elctromiografía valora disfunción neurouscular. 
Estudios de conducción nerviosa: útil si alt neurológicas mínimas.   
 
TRATAMIENTO 
Cambio en la dieta. 
codeína, loperamida, difenoxilato de atropina, formadores de bolo fecal en algunos, aminotriptilina.   
 
Quirúrgico si fracasa el médico. (reto quirúrgico) 
 
 
HEMORROIDES 
[La Salle] 
GENERALIDADES 
Son cojincillos muy vascularizados que forman abultamientos circunscritos de submucosa que contiene vasos sanguíneos, musculo liso 
y tejido elástico conectivo.   
Se localizan en cuadrante lateral izquierdo y lateral derecho (anterior y posterior) 
Su función es la participación de la continenia anal.    Pueden ser internas o externa acode a la linea dentada, las superiores son 
internas y vestidas de epitelio intestinal imperceptible al dolor y las extrnas por debajo d la línea dentada con epitelio similar ala piel y 
muy sensible. 
 
Pueden ser mixtas cuando las internas cmprometen a la externas. La aparición de la patología hemorroidal interna es más frecuente. Se 
ingurgitan al aumentar la presión intrabdominal como embarazo, obesidad, estreñimiento e hipertensión portal.   
Las hemorrodes internas son sintomáticoas cuando existe ingurgitación crónica o prolapso hacia el canal anal.   
Las hemorroides externas son sintomáticas, cuando hay trombosis, al erservese dejan un colgajo de piel.   
 
SALUD PÚBLICA 
5% de la pob presenta síntomas hemorroidales. 
50% a partir de los 50 años. 
50‐75% presenta síntomas en algún momento de su vida.   
Causa más común de hemorragia de tubo digestivo bajo.   
 
CLASIFICACIÓN 
Grado 1: sangrado indoloro 
Grado 2: sangrado y prolapso que reduce espontáneamente 
Grado 3: sangrado y prolapso que reduce manualmente 
Grado 4: sangrado y prolapso no reductible. Aquí entran las trombosadas. 
 
 
 
 
CUADRO CLÍNICO 
Internas: sangrado rojo rutilante, sensación de ámpula rectal llena, descarga mucosa, indoloras, rara vez trobosan.   
Externa: dolor perianal intenso, colgajo de piel, trombosis (violáceas o negras, edema, induración, hipersensibilidad), sangrado por 
necrosis de la piel. 
 
Cuando la hemorragia es constante con cada evacuación hay que hacer dx diferencial con ca colorrectal. 
 
 
PARACLÍNICO 
Estudios siguientes en caso de duda diagnóstica: Colon por enema==> rectosigmoidiscopia o colonoscopia completa.   
PROEDUMED dice que anoscopia confirma el diagnóstico. 
DIAGNÓSTICO DIFERENCIAL 
Sangrado indoloro: cáncer, enfermedad inflamatoria intestinal, enf diverticular, pólipos adenomatosos.   
Sangrado doloroso: úlcra rectal o fisura anal, maceración perianal, condiloma acuminado.   
 
TRATAMIENTO MÉDICO 
Hemorroides grado 1, 2: dieta, líquidos, ejercicio, menos tiempo sentado, baños de asiento cada 12 hrs (bicarbonato y árnica) 
PROEDUMED, corticoides y anestésicos tópicos (no más de 5‐7 días). 
 
TRATAMIENTO QUIRÚRGICO 
Falla del tratamiento médico, ligadura con banda elástica, escleroterpia, hemorroidetomía.   
Si trmbosadas: trombectomia NO hemorroidectomía.   
Externas se envía a 2 do nivel 
 
CASO CLÍNICO 
SE TRATA DE PACIENTE FEMENINO DE 41 AÑOS DE EDAD, LA CUAL CUENTA CON ANTECEDENTE DE 3 GESTACIONES, IMC 34, CON 
PRESENCIA DE MOLESTIAS A NIVEL ANAL, CON LEVE DOLOR ADEMÁS MOLESTIAS, PRURITO, DESCARGA ANAL DE TIPO MUCOSO, 
SENSACIÓN DE CUERPO EXTRAÑO Y AL REALIZAR HIGIENE DE REGIÓN PERSENTA HEMRRAGIA QUE NO SE MEZCLA CON LAS HECES, A LA 
EXPLORACIÓN FÍSICA ENCUENTRA ABULTAMIENT DE LA ZONA CON PRESENCIA DE TUMOR VIOLÁCEO, INTENSAMENTE DOLOROSO, 
QUE IMPOSIBILIDAD LA REALIZACIÓN DE EXPLORACIÓN MANUAL. 
 
SU PRINCIPAL SOSPECHA CLÍNICA ES:   
A) HEMORROIDEES MIXTAS 
B) HEMORROIDES INTERNAS 
C) HEMORROIDES EXTERNAS 
D) FISURA ANAL 
 
EN EL CASO DE ESTA PATOLOGÍA LO MÁS CONVENIENTE ES: 
A) HEMORROIDECTOMÍA 
B) FISTULETOMÍA 
C) FISURECTMÍA 
D) TROMBECTOMÍA 
 
La hemorroidectomía está contraindicada en caso de trombosis 
 
RESPUESTA A, D 
 
 
      
 
HOMBRE DE 43 AÑOS, QUE SE PRESENTA A LA CONSULTA EXTERNA REFIRIENDO SANGRADO RUTILANTE AL FINAL DE CADA EVACUACIÓN. 
EL  SANGRADO  SE  PRESENTA  ALGUNAS  VECES  EN  GOTEO  Y  OTRAS  VECES  ABUNDANTE,  ACOMPAÑANDOSE  DE  LA  PROTRUSIÓN 
INTERMITENTE A TRAVÉS DEL ANO DE UNA MASA INDOLORA DE 1.5 CM DE DIÁMETRO QUE SE REDUCE ESPONTÁNEAMENTE. 
 
EL DIAGNÓSTICO CLINICO MÁS PROBABLE ES: 
A) HEMORROIDES GRADO I. 
B) HEMORROIDES GRADO II. 
C) HEMORROIDES GRADO III. 
D) HEMORROIDES GRADO IV. 
 
EN CASO DE NO RESPONDER AL MANEJO CONSERVADOR EL TRATAMIENTO INDICADO SERÍA: 
A) HEMORROIDECTOMÍA. 
B) LIGADURA CON BANDA DE HULE. 
C) HEMORROIDOPEXIA CON ENGRAPADORA. 
D) FLEBOTÓNICOS POR VÍA ORAL. 
 
Ligadura con banda de hule    Hemorroides I, II y III     
Hemorroidectomía      Hemorroides IV   
Hemorroidopexia con engrapadora  Grado III que requiera incorporación a la vida diaria rápido (tiene más recidiva) 
RESPUESTA B, B 
 
 
 
 
 
 
 
 
 
 
 
CÁNCER DE COLON Y RECTO 
DEFINICIÓN 
Son adenocarcinomas, del ciego al recto, no incluye intestino delgado ni ano.   
Los adenomas son lesines premalignas que acorde al grado de displasia o de patrones histológicos (tubular, tubulovelloso) asocian un 
aumento de potencial maligno. 
 
SALUD PÚBLICA 
Colon derecho más frec en mujeres, recto más frec en hombres.   
FR: Colitis ulcerativas (Croh, por radiación, esquistosómica), ureterocolostomía, tabaco, obesidad, poca fibra, alcohol.   
FP: ejercicio, AINES, vitamina C, consumo de ac de pescad dietético ric en omega 3 (inhibe la sx de PG a partir del ac araquinónico) 
 
PATOGENIA 
A más grasas dietética, más sx de colesterol y ác biliares===> intestino ==> bacterias convierten esto en ac biliares secundarios==> 
carcinogénesis. 
 
Vías de diseminación.  
  Extensión directa:    colon descendente tiene un lumen más pequeño razón por la cual es más factible que se debute como 
obstrucción intestinal. El tumor crece de forma radial y le lleva aproximadamente un año en cubrir 66% de la luz intestinal. Penetra las 
capas del intestino y se extiende por contiguidad a hígado, curvatura mayor del estómago, duoden, intestino delgado, páncreas, bazo, 
vejigad, riñones, ureteresl,pared abdominal. 
  Metástasis  hematógnas:  cels  viajan  por  el  sistema  porta  hepático  y  hace  mets  hepáticas,  vertebras  lumbares,  pulmones, 
ovarios, 
 
DIAGNÓSTICO 
Dolor abdominal, ambio hábitos intestinales, pérdida de peso, sangrado rectal (rectorragia, hematoquezia, melena), anemia microcítica 
hipocrómica.   
 
Gold standar: para ca de colorrectal y pólipos adenomatosos es la colonoscopia con toma de biopsia. 
Indicado como estudio inicial en pacientes con alto riesgo.   
 
Sigmoidoscopia flexible detecta Ca colo rectal y los politos adenomatosos a nivel de la inserción del endoscopio (40 y 60cm), solo 
examina el colon izquierdo. Se debe realizar cada 5 años independiente de la sangre oculta en heces. 
 
Colon por enema está inciado cuando no se puede realizar colonoscopia cada 5 años. 
Colonoscopia virtual cuando los procedimiento anteriores no son factibles. 
 
Colonoscopia cada 10 años. 
Sigmoidoscopia flexible cada 5 años.   
Colon por enema con doble contraste cada 5 años.   
Colonografía virtual TC variable. 
 
Si hay bajo riesgo para el método de cribado es guayaco o inmunoquímica para buscar sangre.   
 
 
 
 
DIVERTICULOSIS 
DEFINICIÓN 
Son adenocarcinomas, del ciego al recto, no incluye intestino delgado ni ano.   
 
PATOGENIA 
ds 
 
TROMBOSIS MESENTÉRICA 
 
GENERALIDADES 
Interrupción aguda o crónica circulación arterial o del drenaje venoso,    la más frecuente es por afección de la vía arterial, en la que los 
pacientes con alteraciones del ritmo en el corazón son los afectados en primer lugar, (50% de frecuencia), siendo la arteria mesentérica 
superior la más afectada en 50% de los casos.   
 
FISIOPATOLOGÍA   
Venosa: cualquier alteración de la tríada de Virchow. Puede tener estasis venosa, depleción del volumen intravascular, lesión en el 
endotelio de vasos venosos. Ej. Deshidratación quemados. 
Arterial: ateroesclerosis, alteraciones del ritmo.   
 
CLÍNICA 
Arterial:    desarrollo agudo, dolor, distensión abdominal, diaforesis, imposibilidad para evacuaciones, canalización de gases. 
Venosa: desarrollo progresivo. dolor, distensión, diaforesis, evacuaciones diarreicas (en grosella), alt electrolíticas, afecciones salteadas, 
más específicas, menos extensa. 
 
PARACLÍNICOS 
En la aguda los estudios iniciales: BHC, QS, Es, TP/TTP, GA (a más acidosis mayor isquemia, riesgo de perforación), PCR ultrasensible, 
CPK, tele de tórax, placa de abdomen de pie y decúbito, TAC si obstrucción venosa, arteriografía si es arterial. 
 
La arteriografía es la prueba diagnóstica y terapéutica más sensible ya que además de darnos diagnóstico, permite administración de 
vasodilatadores, trombolítico y anticoagulantes, además de iniciar con antibiótico de amplio espectro por la posible fuga intestinal.   
 
TRATAMIENTO 
Teniéndose como límitacion máxima para efectuar trombectomía con catéter de Fogarty 6 hrs, después de esas 6 hrs instalar un catéter 
e iniciar trombolíticos, vasodilatadores y anticoagulantes.   
 
Luego de 6 hrs está indicado tratamiento contra el síndrome de reperfusión. Luego de 24 hrs se deberá realizar arteriografía y observar 
la recanalización del vaso afectado. Si recanaliza quito catéter y cambio medicamentos a vía oral. Negativo el estudio deberá proceder a 
intervención quirúrgica, se usará fluoresceína y luz negra para visualizar áreas mal perfundidas y cortar selectivamente. 
 
 
EN EL SERVICIO DE URGENCIA, RECIBE A UNA PACIENTE FEMENINO DE 68 AÑOS DE EDAD, LA CUAL, INICIA PAEDCIMIENTO ACTUAL 
DESDE HACE 10 DÍAS, CON PRESENCIA DE EVACUACIONES DISMINUIDAS EN CONSISTENCIA DE GRAN CANTIDAD, SIN MOCO, SIN 
ANGRA, DOS DÍAS DE EVACUACIONES EN NÚMERO DE 10‐11, LÍQUIDAS COMPLEMTAMENTE, CON TRATAMIENTO ANTIDIARREICO Y 
ANTIBIÓTICO, SIN MEJORÍA, PERO QUE CEDE AL 4TO 5TO DÍA, PERSISTIENDO CON DISTENSIÓN ABDOMINAL, MAL ESTADO GENERAL, 
POLIDIPSIA, SE AGREGA NÁUSEA, VÓMITO SOMNOLENCIA HASTA HACE 24 HRS, CON PRESENCIA DE DOLOR ABDOMINAL QUE SE 
TORNÓ SÚBITO, INTENSO, LOCALIZADO EN MESOGASTRIO, EL DOLOR SE HA PRESENTAOD MÁS INTENSO.   
CON PRESENCIA DE DOLOR HASTA PARA MOVERSE, PROVOCANDO QUE TENGA QUE ADOPTAR POSICIÓN ANTIÁLGICA.   
 
SI DIAGNÓSTICO DE SOSPECHA ES: 
A) SÍNDROME DE INTESTINO IRRITABLE 
B) GASTROENTERITIS PROBABLEMENTE INFECCIOSA 
C) ENFERMEDAD ÁCIDO PÉPTICA COMPLIADA 
D) ISQUEMIA MESENTÉRICA 
 
¿QUÉ ESTUDIO INICIAL DE LABORATORIO DE ACUERDO A SU DIAGN´SOTICO FINAL SE DEBE REALIZAR? 
A) BHC, QS, ES 
B) QS, ES, GASA 
C) TOXINA A Y B 
D) REACCIONES FEBRILES 
 
DE CORROBORARSE SU DIAGNÓSTICO EN EL ESTUDIO SOLICITADO, DÓNDE SE EVIDENCIA LA DISMINUCIÓN SEVERA DE PH, APARTE DE 
ELEVACIÓN DE LEUCOCITOS Y DATOS DE ÍLEO,    ¿CUÁL ES EL ESTUDIO SIGUIENTE A REALIZAR? 
A) TAC ABDOMINAL 
B) RM 
C) USG ABDOMINAL 
D) PLACA DE TORAX Y ABDMEN 
 
EL PACIENTE ES MANEJADO CON RESTITUCIÓN DE VOLUMEN INTRAVASCULAR, ASÍ COMO CORRECCIÓN DE ESTADO 
HIDROELECTROLÍTICO, SIN EMBARGO NO PRESENTA MEJORÍA, POR LO TANTO, ¿QUÉ ESTUDIO DEBERÁ REALZIARSE ANTE LA 
SITUACIÓN? 
A) TAC CON CONTASTE INTRAVENOSO 
B) TMOGRAFÍA CON CONTRASTE ORAL 
C) RESONANCIA MAGNÉTICA 
D) ULTRASONIDO DOPPLER. 
 
EL ESTUDIO SOLICITADO MUESTRA DE ACUERDO A SU CERTEZA DIAGNÓSTICA: UNA ZONA DE INTERRUPCIÓN EN LA IRRIGACIÓN 
VASCULAR, ¿CUÁL SERÁ EL PASO A SEGUIR? 
A) ARTEROGRAFÍA CON CATETERISMO 
B) VENOGRAFÍA CON CATETERISMO 
C) CIRUGÍA URGENTE 
D) ENDARTERECTOMÍA 
 
La deshidratación provoca aumenta de la viscosidad sanguínea (hipercoagulabilidad) 
La GASA nos reportará acidosis metabólica con un anión GAP elevado por desviación de las rutas metabólicas.   
 
Imagen en ileo (liquido libre o imagen en vidrio despulido), si hay aire libre en abdomen o espacio subdiafragmático. Trombosis 
necrosis perforación.   
 
Si es venosa TAC 
Si es arterial RM  generalmente es crónica.   
 
Recordar que es más frecuente la arterial. 
 
La arteriografía es buena opción terapéutica apero si fuera arterial, en este caso sabemos que el paciente tuvo depleción del volumen 
intravascular, estamos y trombosis venosa. La respuesta sería venografía. 
 
RESPUESTA D, B, D, A, B 
 
 
MASCULINO  DE  65  AÑOS  DE  EDAD  CON  DIAGNÓSTICO  DE  POSTOPERADO  DE  RESECCIÓN  INTESTINAL  SECUNDARIO  A  TROMBOSIS 
MESENTÉRICA. LA MAYOR PARTE DEL INTESTINO RESECADO FUE ILEON. 
 
A PARTIR DE AHORA EL PACIENTE TENDRÁ PRINCIPALMENTE DIFICULTAD PARA LA ABSORCIÓN DE: 
   
A) VITAMINA B1. 
B) EL COLESTEROL. 
C) LA GLUCOSA. 
D) HIERRO. 
 
RESPUESTA B 
En el íleon se absorben las grasas, vitamina B12 y sales biliares. 
La mayor parte de glucosa se absorbe en duodeno y yeuno terminal. 
El hierro se absorbe principalmente en duodeno y yeyuno. 
 
El hierro se absorbe principalmente en duodeno y yeyuno. 
La mayor parte de glucosa se absorbe en duodeno y yeuno terminal. 
En el íleon se absorben las grasas, vitamina B12 y sales biliares. 
 
 
 
CÁLCULOS URETERALES Y UROPATÍA OBSTRUCTIVA 
CÁLCULOS URINARIOS 
 
DIAGNÓSTICO 
Generalmente debutan como dolor lumbar que irradia a la fosa ilíaca ipsilateral (a labio de vulva o escroto). 
sintomagolotía irritativa urinaria baja (urgencia, disuria, hematurina). Expulsión del lito 
==> alivio del dolor.     
 
Urografía excretora ha sido el    estándar de oro para el diagnóstico==> TC helicoidal NO contrastada ha demostrado se supeiror.   
USG renal ureteral y vesical junto con rx simple de abdomen ofrecen evaluación aleranativa. 
Pielografía retrógrada en situaciones especiales.   
 
TRATAMIENTO 
Manejo del dolor: 1era. Diclofenaco, indometacina, ibuprofeno, 
2da Hidromorfina, hidrocloruro de atrpina, metamizol, pentazocina, tramadol.   
 
Bloqueadores alfa 1 facilia el paso de cálculos. 
 
<4mm    80% paso espontáneo 
6‐10mm    10‐53% 
 
Quirúrgico:   
litotropsia extracorporea 
El más efectivo si la LEC falla es la ureteroscopía flexible.   
Nefrolitotomía percutánea. 
 
 
 
 
CÁLCULOS URINARIOS 
MUJER DE 48 AÑOS DE EDAD, DIABÉTICA DE 10 AÑOS DE EVOLUCIÓN, CON ANTECEDENTE DE INFECCIONES URINARIAS DE REPETICIÓN. 
SE REPORTA UN ÚLTIMO UROCULTIVO HACE 5 DÍAS CON AISLAMIENTO DE PROTEUS VULGARIS PARA LO CUAL SE INICIA TRATAMIENTO 
ESPECÍFICO. ACUDE HOY AL SERVICIO DE URGENCIAS POR NÁUSEAS, DOLOR INTENSO EN REGIÓN LUMBAR DERECHA QUE SE IRRADIA 
HACIA  INGLE  DEL  MISMO  LADO.  EL  ULTRASONIDO  REPORTA  LIGERA  DILATACIÓN  URETERAL  EN  TERCIO  SUPERIOR  LO  QUE  SUGIERE 
URETEROLITIASIS. 
      
LO MÁS PROBABLE ES QUE EL CÁLCULO PRESENTE EN ESTA PACIENTE ESTÉ FORMADO POR: 
A) CALCIO. 
B) FOSFATO DE AMONIO‐MAGNESIO. 
C) ÁCIDO ÚRICO. 
D) OXALATO DE CALCIO. 
       
EL TRATAMIENTO ESPECÍFICO EN ESTE CASO DEBERÁ SER CON: 
A) HIDROCLOROTIAZIDA + ORTOFOSFATOS 
B) FOSFATO + ÁCIDO ASCÓRBICO 
C)PENICILINA ORAL + ALOPURINOL 
D) ÁCIDO ACETOHIDROXÁMICO + DOXICICLINA. 
 
RESPUESTAS   
B,D 
 
 
HOMBRE DE 34 AÑOS DE EDAD SIN ANTECEDENTES DE IMPORTANCIA. ACUDE A CONSULTA POR PRESENTAR DESDE HACE 1 HORA DOLOR 
INTENSO  EN  REGIÓN  LUMBAR  IZQUIERDA  QUE  SE  IRRADIA  HACIA  TESTÍCULO  DEL  MISMO  LADO,  ACOMPAÑADO  DE  VÓMITO  Y 
DIAFORESIS. A LA EXPLORACIÓN MARCHA CLAUDICANTE, TAQUICARDIA, PALIDEZ EN PIEL Y GIORDANO IZQUIERDO. 
 
EL SIGUIENTE ESTUDIO ES CONSIDERADO ESTÁNDAR DE ORO PARA REALIZAR EL DIAGNÓSTICO DEL PACIENTE: 
A) EXAMEN GENERAL DE ORINA. 
B) UROGRAFÍA EXCRETORA. 
C) TOMOGRAFÍA HELICOIDAL. 
D) ULTRASONIDO ABDOMINAL. 
 
SE REPORTA OBSTRUCCIÓN A NIVEL DEL TERCIO MEDIO DEL URETER, CON BASE EN ESTE RESULTADO, LA PROBABILIDAD DE EXPULSIÓN 
DEL CÁLCULO SERÁ DEL: 
A) 25%. 
B) 45%. 
C) 65%. 
D) 85%. 
 
El estándar de oro es la urogrofía excretora. La TAC helicoidal es    más sensible y específica pero es más cara, razón por la cual no es 
estándar de oro. La TAc helicoidal puede mostrar cálculos de ácido úrico y xantinas que son radiolúcidos en la Rx y habitualmente no se 
ven.    Grammagrafía si contraindicación de material de contraste intravenoso.     
En embarazadas con sospecha de litiasis el USG es de primera elección como estudio de imagen, a menos que el caso sea severo y el USG 
no concluyente deberá hacerse resonancia magnética o urografía excretora.   
 
Probabilidades de expulsión espontánea 
<4mm    80%     
6‐10mm    10‐54%    ==> indicación formal de extracción instrumentada 
 
Uréter proximal  25% 
Uréter medio    45 
Uréter distal  70 
RESPUESTA B, B 
  
 
PACIENTE FEMENINA DE 27 AÑOS CON ANTECEDENTE DE INFECCIÓN DE VÍAS URINARIAS RECURRENTES, POR CULTIVO SE IDENTIFICÓ 
PROTEUS. HACE 24 HORAS CURSA CON UN LITO EN VÍA URINARIA. EL COMPONENTE MÁS PROBABLE DE ESTE LITO ES: 

 1.   ESTRUVITA 

 2.   FOSFATO DE CALCIO 

 3.   PIROFOSFATO DE CALCIO 

 4.   HIDROXIAPATITA 
RESPUESTA 1 
 
 
 
 
 
UROLITIASIS ADD 
 
TRATAMIENTO 
Fármaco de elección para el manejo del dolor nefrítico es diclefenaco, se puede agregar metamizol. 
Tratamiento médico expulsivo para litiasis: tamsulosina 0.4mg o alfuzosina 10mg/dia x 30 días.   
 
     
         
COMPOSICIÓN  SALES CÁLCICAS  ESTRUVITA (FOSFATO  ÁCIDO ÚRICO  CISTINA 
AMÓNICO 
MAGNÉSICO) 
 
FRECUENCIA  55‐60% Oxalato cálcico    10‐15%  5‐10%  1‐3% 
10‐15% Fosfato cálcico 
GENERO  Masculino  Femenino  Masculino  Ambos 
ETIOLOGÍA  Idiopática,  Infección por gérmenes  50% Gota  Cistinuria 
hipercalciuiria  productores de ureasa  <50% idiopática 
idiopática,  (Proteus)  Hiperuricemia 
hiperuricosuria    secundaria 
 
PRECIPITAN  Alcalino (pero al oxal el  Alcalino  Ácido  Ácido 
pH le da igual) 
RADIOLOGÍA  Radioopacos  Radioopacos  Radiotransparentes  Radiolúcidos 
MORFOLOGÍA    Formas pristmáticas  Aglomerados de  Cristales hexagonales 
molimórficas (en  cristales  en prismas o láminas 
ataud)  desorganizados   
Pueden ser   
coraliformes. 
TRATAMIENTO  Hipercalciuria  Ácido priopiónico y  Alcalinizar la orina  Forzar diuresis (ingesta 
idiopática= tiazidas  ácido  (UROCLASIO NF Citrato  hídrica) 
Hiperoxaliuria  acetohidroxámico  de potasio solución  Alcalinizar la orina 
primaria= piridoxina  (¿México?)  oral. Tomar 5‐10ml  D‐Penicilamina (si no 
Hiperoxaliuria  Antibioticoterapia  cada 8 hrs. Deberá  hay respuesta) 
secundaria=  Cirugía  disolverse en una taza 
colestiramina  (ocasionalmente).  de agua o jugo).   
     
Importante quitar toda  Alopurinol si 
la piedra para quitar  hiperuricemia 
todas las bacterias.   
Dieta de bajo 
contenido protéico 
 
¿radiotranspartente y radiolúcido no es lo mismo? 
TAMSULOSINA EN LITIASIS? 
 
Tratamiento de bacteriuria asintomática: embarazo, procedimiento, Proteus. 
 
INCONTINENCIA URINARIA 
INCONTINENCIA URINARIA 
Contenido en GYO 
 
PATOLOGÍA DE PRÓSTATA 
CÁNCER DE PRÓSTATA 
DIAGNÓSTICO 
Del epitelio 
 
SALUD PÚBLICA 
Si un familiar de primera línea lo tiene, el riesgo aumenta al doble, si 2 o más 5‐11 veces. 
Fenómenos de anticipación de 7 años.   
 
PATOGENIA 
La proliferación NO cancerosa predominentemente se da en la zona transicional.   
 
 
Aumenta riesgo con ác. linoleico o carbohidratos aromáticos policíclicos (se forman al cocinar carnes rojas.) 
 
DIAGNÓSTICO 
La detección basada en APE reduce mortalidad del 20% pero se asocia a alto riesgo de sobrediagnóstico.   
No se recomienda tamizaje si asintomático 50‐69 años basado en APE y TR. 
Si historia familiar de Ca de próstata, tamiz antes a los 40,45, 50 años dependiente asociación.   
 
APE es órgano específico pero no cáncer específico.   
70% de Ca se detecta con APE >4ng/ml. 
 
Si APE 4‐10 utilizar:    relación APE fracción libre‐total <20%. 
    Velocidad de APE >0.75ng/ml/año. 
 
USG transrrectal indicado para biopsias, no para rutina.   
Imagen clásica: zona hipoecóica en zona periférica valorada por USG transrectal (63% de los pacintes se observa, en 37 la lesión es 
isoecóica) 
Biopsia transrectal de próstata:   
  a) sospecha clínica + APE elevado 
  b)APE 4‐10, disminución del 20% de la fracción libre, densidad > 0.15, velocidada >0.75ng/ml año y tiempo    de 
duplicación 3m 
  c) APE >10 
  d)hallazgo anormal al TR aislado. 
 
TAC: para estadificación. No útil si el paciente está asintomático    y PSA <20. 
Gramagrama óseo: supera a todo para val metástasis. Recomendado si alto riesgo. 
  
CONTENIDO ADICIONAL 
PSA total    El que habitualmente se mide 
PSA libre    Cantidad no unida a proteínas. El riesgo de Ca aumenta si la relación PSA L : PSA total <20%   
    Ca tiene menos PSA libre, HPB tienen mas PSA libre. 
Velocidad PSA  Es PSA total durante el tiempo   
Densidad PSA  A    más densidad mayor riesgo de Ca. Relación entre PSA    y volumen prostático.   
 
TRATAMIENTO 
Referencia a 2do nivel si: 
  Tacto rectal con sospecha. 
  APE >10 
  APE 4‐10 si la rel APE L y APE total    <20% o APE >0.75ng/ml/año. 
 
Prostatecomía retropúbica radical si esperanza de vida >10 años.   
 
PRONÓSTICO 
T1  Tumor clínicamente no aparente 
T2  Tumor locacalizado a próstata 
T3  Tumor con extensión etracapsular 
T4  Tumor invade estructurae sdistintas a las v. seminales, cuello de vejiga, esfínter externo, recto, músculos elevadores, pared 
  pélvica. 
 
Agresividad la definite la escala de Gleason. 
Grado 1 o bien diferenciado [2‐4] 
Grado 2 o moderadamente diferenciado [5‐6] 
Grado 3 o pobremente diferenciado [7‐10] 

 
 
 
 
 
 
HOLA PROEDUMED. Me encuentro leyendo Tema Patología de la próstata, Subtema Cáncer de Próstata y tengo una duda respecto al 
contenido,    en el subtítulo Diagnóstico dice   
 
"No se recomienda la realización de tamizaje de cáncer de próstata a población masculina asintomática entre 50 a 69 años de edad 
empleando la prueba de APE y el TR, debido a que se tiene evidencia de que no es efectivo en la reducción de la mortalidad por cáncer 
de próstata; a que su implantación representaría un elevado impacto presupuestal y a que se expondría a los pacientes a riesgos 
innecesarios." 
 
Y en el diagrama de flujo del final dice “que está indicado realizar TR y APE en pacientes ASINTOMÁTICOS sin factores de riesgo a partir 
de los 50 años y 40 con factores de riesgo” 
 
Mi duda es, ¿se recomienda o no se recomienda realizar el tamiz? 
Saludos cordiales, gracias por atender mi mensaje.   
 
EPIDEMIOLOGÍA 
Riesgo de cáncer a lo largo de la vida es de 16%. 
Menos de 2% se diagnostica antes de los 50.   
Escritidio 40‐50 años: APE + tacto rectal.   
 
FR: historia familiar (familiar = más de un familiar afectado, hereditario= más de tres o más de dos antes de los 55 años), raza negra, 
casos esporádicos 85%, edad.   
 
CLÍNICA 
Asintomático en estadios temprano 
Si avanzado: dolor óseo, fracturas patol´gicas, crecimiento local, obstrucción urinaria o ureteral.   
 
DIAGNÓSTICO 
Biopsia prostática (transrectal o perineal). 
Zonas hipoecoicas mayor riesgo de cáncer.   
Técnica: sextantes extendidos ( al menos 10, usualmente 12)de la zona periférica. 
Histología Gleason (2‐10) según patrón primario y secundario. 
  Riesgo bajo <6, intermedio 7, alto >8. 
 
La mayoría adenocarcinoma acinar + o ductal y neuroendocrinos (indifernciados). 
 
LABORATORIOS 
Antígeno prostático específico.   
  APE >4ng/ml. Sospecha 
  Zona gris entre 4‐10ng/ml (20‐30% con cáncer. 
  APE >100ng/ml prácticamente todos tienen mets. 
 
Elevación de azoados (creatinina), fosfatasa alcalina y calcio (enf ósea). 
 
Extensión: 
TAC de abdomen y pelvis con contraste, RMN (enf extracapsular) 
Gammagrama óseo si APE >20 (lesiones blásticas) 
 
TRATAMIENTO 
Tratamiento curativo: prostatectomía radical y radioterapia.   
El bloqueo androgénico se hace con: ketoconazol, dietilestilbestrol (riesgo trombótico), agonistas de LHRH que son los más 
utilizados(leuprolide, goserelina), antagonistas (degarelix), antiandrogénicos (bicalutamida, flutamida). Orquiectomía simple bilateral 
(estándar).   
Caazitaxel, abirateron, enzalutamida, docetaxel: Ca refractario a castración.   
Ac. zolendrónica, denosumab, radio‐223: enfermedad mestásica ósea, prevención de fracturas. 
 
Para no metastásicos dar tratamiento curativo con prostatectomía o radioterapia, si queda Ca residual luego de cirugía hay que radiar 
el lecho prostático. 
Hormonterapia es para casos metastásicos. 
 
MASCULINO DE 65 AÑOS DE EDA, SIN COMORBILIDADES, NIEGA ALÉRGICOS Y QUIRÚRGICOS PREVIOS, ACUDIÓ HACE 6 MESES PARA 
ESTUDIOS GENERALES Y REPORTARON ANTÍGENO PROSTÁTICO DE 10NG/ML, ACUDIÓ CON MÉDICO PARTICULAR Y RECIBIÓ 
LEVOFLOXACINO POR UN MES. HACE UN MES, NUEVAMENTE SE REALIZA LOS ESTUDIOS Y PRESENTA UN ANTÍGENO PROSÁTICO DE 12 
NG/ML. AL INTERROGATORIO SÓLO REFIERE CHORRO DÉBIL OCASIONALMENTE. TACTO RECTAL CON PRÓSTATA AUMENTADA DE 
TAMAÑO, SIN NÓDULOS PALPABLES. GENITALES SIN ALTERACIONES. EGO SIN ALTERACIONES. UROCULTIVO NEGATIVO.   
MANDA A BIOPSIA Y REPORTAN: ADENOCARCINOMA DE PRÓSTATA GLEASON 4+4 EN 3 DE 12 FRAGMENTOS.   
 
¿QUÉ ESTUDIOS MANDA PARA TERMINAR ABORDAJE? 
A) TAC DE ABDOMEN Y PELVIS CONTRASTADA, RX DE TÓRAX, GAMMAGRAMA ÓSEO Y PRUEBAS DE FUNCIÓN HEPÁTICA. 
B) USG HEPÁTICO, FOSFATASA ÁCIDA, SERIE ÓSEA COMPLETA, TAC SIMPLE DE TÓRAX 
C) GAMMAGRAMA ÓSEO, TAC DE ABDOMEN SUPERIORES, RX DE TÓRAX. 
D) RESONANCIA MAGNÉTICA DE PELVIS, SERIE ÓSEA 
 
SI LA ENFEREMDAD SE ENCUENTRA LOCALIZADA DESPUÉ SDE COMPLETAR SUS ESTUDIOS, LA CONDUCTA A SEGUIR ES: 
A) RESECCIÓN TRANSURETRAL DE PRÓSTATA. 
B) PROSTATECTOMÍA RADICAL 
C) BLOQUEO HORMONAL 
D) RADIO 123 
 
EL APE NADIR DE 0.5NG/ML NOS HACE SOSPECHAR DE PERSISTENCIA DEL CÁNCER, QUE DECIR HACER:   
A) BLOQUEO ANDROGÉNICO 
B) RADIOTERAPIA ADYUVANTE 
C) PROSTATECTOMÍA RADICAL 
D) VIGILAR HASTA ELEVACIÓN DEL ANTÍGENO 
 
EN CASO DE HANER DEMOSTRADO ENFERMEDAD METASTÁSICA: 
A) QUIMIOTERAPIA 
B) BLOQUEO ANDROGÉNICO 
C) RADIOTERAPIA DIRIGIDA A SITIOS METASTÁSICOS 
D) CUIDADOS PALIATIVOS. 
 
EN CASO DE TENER METÁSTASIS ÓSEAS, ¿CUÁLES SON LAS MÁS COMUNES? 
A) OSTEOBLÁSTICAS 
B) OSTEOCLÁSTICAS 
C) NO SON FRECUENTES EN CÁNCER DE PRÓSTATA 
D) B Y D SON CORRECTAS.   
 
RESPUESTA A, B, B, B, A 
 
 
 
[CÁNCER DE PRÓSTATA] 
HOMBRE DE 64 AÑOS DE EDAD QUE ACUDE A LA CONSULTA PORQUE DESDE HACE 3 MESES PRESENTA DISMINUCIÓN DEL CALIBRE DEL 
CHORRO URINARIO, GOTEO TERMINAL, POLAQUIURIA Y DISURIA. A LA EXPLORACIÓN SE REALIZA TACTO RECTAL ENCONTRANDOSE LA 
PRÓSTATA CON UN NÓDULO PÉTREO EN LA PARTE POSTERIOR DE ESTA GLÁNDULA. 
PARA CONFIRMAR EL DIAGNÓSTICO SE DEBERÁ INDICAR EL SIGUIENTE ESTUDIO: 
A) ULTRASONIDO PROSTÁTICO. 
B) FOSFATASA ALCALINA EN SUERO. 
C) NIVELES DE ANTÍGENO PROSTÁTICO ESPECÍFICO. 
D) BIOPSIAS TRANSRECTALES DE PRÓSTATA. 
 
RESPUESTA D 
No se recomienda tamizaje para próstata a población asintomática entre 50‐69 años    de edad empleando la prueba de APE y TR (ya que 
no asocia reducción de mortalidad). 
La detección basada en APE demostró reducción del 20% pero se relaciona con importante riesgo de sobrediagnóstico.   
La mayor parte de cánceres se presentan en la parte posterior de la glándula en forma de nódulos indoloros y pétreos.   
La biopsia de próstata guida por USG es la única prueba confirmatoria, idelmente tres de la derecha, tres de la izquieda y si procede 
clínicamente 1 de la zona transicional. No se recomienda si prostatitis, a menos que haya recibido un ciclo de antibioticoterapia.   
TR VPP 21%. 17% de pacientes con PSA 2.5‐4ng/ml y datos normales en TR tiene cáncer.   
 
 
[r] 
HOMBRE  DE  60  AÑOS  DE  EDAD  QUE  ACUDE  A  CONSULTA  CON  SINTOMATOLOGÍA  URINARIA  IMPORTANTE.  USTED  SOSPECHA  LA 
POSIBILIDAD DE CÁNCER DE PROSTATA. 
EN ESTE CASO DEBERÁ IDENTIFICAR EL SIGUIENTE FACTOR DE RIESGO: 
   
A) CONSUMO DE DIETA CON ELEVADO CONTENIDO DE GRASAS. 
B) CONSUMO OLIGOELEMENTOS. 
C) EL CONTACTO PERMANENTE CON SUBSTANCIAS DERIVADAS DE LA ACTIVIDAD INDUSTRIAL. 
D) LA RAZA O GRUPO ÉTNICO 
 
RESPUESTA D 
Factores ambientales: zona geográfica, dieta alta en grasa, exposición a humo de automóviles, polución del aire, cadmio, fertilizantes, 
sustnacias de la industria de la goma, imprenta y pintura.   
Factores genéticos: gen HPC 1. 
Factores infecciosos: ITS. 
Factores hormonales: andrógenos (en eunucos no aparecen). 
 
El consumo elevado de grasa si es FR pero no el más importante, los oligolementos no tienen nada que ver.   
La sustnacias de la industria de pueden generar riesgo pero la respuesta es vaga, no especifica que sea de pintura y la goma.   
La raza negra es el mayor factor de riesgo de neoplasia prostática intraepilial multifocal y con gran inestabilidad. Quizás por altos niveles 
de testosterona. 
En términos generales son los factores de riesgo más imprtantes para Ca prostático: Edad avanzada, antc familiares, raza negra.   
 
 
HOMBRE DE 35 AÑOS DE EDAD ACUDE A CONSULTA CON MÉDICO GENERAL. EL PACIENTE SE MUESTRA CONSTERNADO DEBIDO A QUE 
DOS DE SUS TÍOS LES DETECTARON CÁNCER DE PRÓSTATA A LOS 50 Y EL OTRO A LOS 65 AÑOS, NIEGA SINTOMATOLOGÍA ASOCIADA Y 
NO CUENTA CON ANTECEDENTS DE IMPORTANCIA. AL TACTO RETAL SIN PRSENCIA DE NÓDULOS O ALTERACINES. EL APE ES DE 2.3NG/DL.   
¿CUÁL ES LA RECOMENDACIÓN MÁS APROPIADA PARA EL PACIENTE? 
A) DETERMINACIONES BIENALES DE APE A PARTIR DE LOS 65‐85 AÑOS 
B) DETERMINAICONES ANUALES Y BIENALES DE APE DE LOS 35‐75 AÑOS 
C) DETERMINACIONES ANUALES DE APE DE LOS 40‐75 AÑOS 
D) DETERMINACIONES ANUALES DE APE DE LOS 50‐75 AÑOS 
 
RESPUESTA C 
Ca de próstata tiene fenómeno de anticipación de 7 años. El paciente en cuestión tiene factores de riesgo y por lo tanto el cribado se 
inicia con APE a los 40 años, de no tenerlos sería    a los 50.    [info del proedumed] 
 
El antígeno prostático específico es el examen más importante según las guías de la Secretaría de Salud, a pesar de que hay varios países 
que lo han retirado de sus guías de práctica clínica, porque aunque detecta    el cánr no modifica la motalidad. 
un gram de cáncer produce 3ng/ml de antígeno.   
los valores normales van de 0‐4ng/ml. La velocidad de crecimiento anual de más de 1ng/ml por año se considera anormal. El porcentaje 
de antígeno libre también se puede detectar, y si este es menor a un 10% en general es debido a áncer y es más de 25% probablemnte 
se trate de un adenoma. El estándar de oro para el diag nóstico es la biopsia. Se sugiere realizar determinaciones deanuales de APE, pero 
denbido al crecimiento lento del cáncer, se podría realizar también en intervalos de 2 años iniciando desde los 40‐45 años hasta los 75‐
65 añls si el pacient presentó determinaciones bajas persistentes entre 0.5‐1ng/dl. [reto enarm] 
 
 
HOMBRE DE 67 AÑOS DE EDAD QUE ACUDE A CONSULTA PORQUE, HACE ALGUNOS MESES, DEBE PUJAR CON ÁS FUERZA PARA PODER 
ORINAR Y AUN ASÍ, EL CHORRO URINARIO HA DISMINUIDO DE CALIBRE. AL INTERROGATORIO DIRIGIDO REFIERE QUE SE PARA A ORINAR 
MÁS DE DOS VECES POR LA NOCHE. NIEGA DISURIA, HEMTURIA O ALGUNA CONDICIÓN ASOCIADA. TIENE DIAGNÓSTICO DE DM2 HACE 
10 AÑOS, EN TRATAMIENTO REGULRA. FC 77, FR 29 T 36 TA 115/67. A LA EXPLORACIÓN FÍSICA SE PALPA PRÓSTATA SIN NÓDULOS O 
INDURACIONES PERO AUMENTADA DE TAMAÑO. LOS ESTUDIOS DE LABORATORIO REPORTAN APE 4‐5NG/DL. SE REALZÓ UN USG QUE 
DEMOSTRÓ UNA PRÓSTATA DE 45 CC. SIN NÓDULOS NI CALCIFICACIONES. 
¿CUÁL ES EL MANEJO INICIAL DE ESTE PACIENTE? 
A) CLONIDINA + FINASTERIDE 
B) DANAZOL + DOXAZOSINA 
C) PRAZOSINA + DANAZOL 
D) DOXAZOSINA + FINASTERIDE 
 
RESPUESTA D 
La hiperplasia prostática benigna se reifere a una detección microscópica de la hiperplasia (una proloferación de estroma y epitelio), 
crecimiento de próstata detectado por examen rectal digital o por medio de ultrasonido además de presentar síntomas asociados a esta 
hiperplasia. Se recuerda que el tamaño de la próstata no siempre correlaciona con los síntomas. La prevalencia aumenta en forma lineal 
con la edad, teniendo una prevalencia de 90% en pacientes de 85 años o más. La clásica sintomatología obstructiva es pujo miccional, 
dismiinución del calibre de la orina, nicturia, etre otros. Entre los métodos diagnósticos está la USG con medici´n de orina residual. Hay 
una relación directa entre lso niveles de APE y el volumen prostático pero no tienen mayor riesgo de desarrollar cáncer de próstata. El 
tratamiento farmacológico evita la morbilidad asociada a la cirugía. En pórstata de más de 40cc el tratamiento combiada (alfablqoueador 
e inhibidor de la 5 alfa redutasa) es la mejor opciójn. La clonidina es una gonista alfa 2.   
 
 
HIPERPLASIA PROSTÁTICA BENIGNA 
DEFINICIÓN 
El diagnóstico y el término es histológico exclusivamente. 
Multifactorial, edad (>40 años), dihidrotestosterona.   
 
Testosterona    dihidrotestosterona 
  5 alfa reductasa tipo 2 
 
SALUD PÚBLICA 
Afecta a hombres >45 años, clínica por lo regular 60‐65. 
Obesidad aumenta >10% el riesgo de HPB clínica.   
 
DIAGNÓSTICO 
Almacenamiento (urgencia, frecuencia, nicturia), vaciamiento (pujo, intermitencia, disminución del chorro, tenesmo, otros: doble 
micción, goteo terminal). 
 
Clínica:    >50 años 
  Pujo 
  Intermitencia 
  Nicturia 
  DIsminución del calibre 
  DIsminución de la fuerza 
 
I‐PSS Leve 0‐7, Moderado 8‐19, severo 20‐35. 
 
Datos que predicen retención aguda de orina, necesidad de cirugía y que además ameritan tratamiento. 
  Vol prostático >30ml 
  Flujo urinario débil 
  APE >1.4ng/ml 
 
‐Si clínica solicitar EGO, y glucosa/creatinina en caso selectos.   
‐TODO PACINETE que ingresa a protocolo de tratamiento de HPB deberá tener USG vesical y prostático con medición de orina residual. 
  ‐Si >50 % de orina residual postmiccional, elevación de creatinina, ant de litiasis, IVUs de repetición, hematuria macro, incontinencia x 
rebosamiento==> USG renal.   
 
HPB no aumenta el riesgo de Ca de prostata.   
 
TRATAMIENTO 
1)Medidas generales: Disminuir cafeína, alcohol y líquido libre por la tarde noche.   
2)Alfabloqueadores relajan próstata y cuello de vejiga, son los mas efectivos para los STUI su eficacia se mantiene luego de 6‐12 meses 
de tratamiento. NO REDUCEN TAMAÑO DE PRÓSTATA NI MODIFICAN EVOLUCIÓN DE LA ENFERMEDAD. 
Alfuzosina 10mg c 24 
Tamsulosina 0.4mg c 24 
Terazosina 2‐5mg c 24 
Doxazosina 2‐4mg c 24 
3)Inhibidores de 5 AR, reduce el tamaño prostático, reduce riesgo de RAO en 50% y el riesgo de cirugía indicada si STUI y >40cc de vol 
prost, en estos casos es mejor tx combinado   
Finasterinde 5mg c 24 
Dutasteride 0.5mg c 24 
4) Inhibidores de la fosfodiesterasa. Se indica si hiperplasia prostática + disfunción eréctil. 
5) Quirúrgico: indicado si complicaciones (hidronefrosis, IR, RAOR, IVUR, hematuria persistente, síntomas que afecten su calidad de 
vida, fracaso a manejo farmacológico, litiasis vesical, 
a)RTUP (resección transuretral del próstata) es el tx más efectivo pero no el de primera elección.   
  Complicaicones:    eyaculación retrógrada  65‐70% 
      disfunción eréctil    6.5 
      Transfusión    2‐5 
      estenosis uret, contractur ves  3.8 
      incontinencia urinria de esf  1.8 
      Mortalidad    0.25 
      Sangrado 
Sx postRTUP (hiponatremia)  alt de edo de consciencia 
Ameritarán monitoreo anual con TR y APE 
b) Prostatectomía abierta si >80cc o >80‐100gr, complicaciones asociadas, litiasis vesical. Tiene más complicaciones. 
 
Otras: resección transuretral con láser, vaporicazión con láser verde. Puede usarse en anticoagulados.   

 
 
 
 
 
 
PACIENTE MASCULINO DE 55 AÑOS DE EDAD, TABAQUIMSO NEGADO, SIN CIRUGÍA PREVIAS, PREVIAMENTE SANO, CON CUADRO DE 2 
AÑOS DE EVOLUCIÓN CARACTERIZADO POR FRECUENCIA URINARIA, NICTURIA, DISMINUCIÓN DEL CHORRO URINARIO, NIEGA DISURIA.   
A LA EF AFEBRIL, ABDOMEN CON PERISTALSIS PRESENTE, SIN DOLOR A LA PALPACIÓN SUPERFICIAL NI PROFUNDA, GIORDANO 
NEGATIVO, PRÓSTATA NO DOLOROSA Y SIN NÓDULOS, CREAT 1.1MG/DL, APE 3 NG/ML EGO 1 LEUCOCITOS, ERITROCITOS 1, 
BACGERIAS 30, NITRITOS NEGATIVOS.   
 
EL DIAGNÓSTICO PARA ESTE PACIENTE ES: 
A) ESTENOSIS DE URETRA 
B) INFECCIÓN DE VÍAS URINARIAS 
C) CRECIMIENTO PROSTÁTICO OBSTRUCTIVO 
D) PROSTATITIS CRÓNICA 
 
ESTUDIOS QUE COMPLEMENTAN SU ABORDAJE:   
A) USG VESICAL Y PROSTÁTICO 
B) CISTOURETROGRAFÍA 
C) UROCULTIVO 
D) CULTIVO DE EXUDADO PROSTÁTICO 
 
EL TRATAMIENTO QUE USTED ELEGIRÍA ES: 
A) ALFA BLOQUEADORES 
B) FLUOROQUINOLONAS 
C) DILATACIÓN URETRAL 
D) TMP/SMX 
 
RESPUESTA C, A, A 
 
 
PROSTATITIS 
DEFINICIÓN 
No aumentan riesgo de Ca de próstata. 
 
PATOGENIA, DIAGNÓSTICO, TRATAMIENTO. 
Prostatitis aguda bacteriana: generalmente gramnegativas (E coli, Klebsiella p,    Proteus mirabilis, Pseudomona aeroginosa, Stap h 
aureus, enterococos, y anerobios (bacteroides spp). ITS com gonorrea, clamidia, trichomonas. Disuria, polaquiuria, urgencia, dolor 
lumbosacro, perineal, en pene, en recto,fiebre, mialgias, malestra gral. Tx AB, AINEs, liquidos.   
Prostatitis crónica: clínica similar pero gralmente sin fiebre, mismos gérmenes además Clamidia tachomatis.   
Prostatitis no bacterianas: es la mas frec, se sospecha sea por bacterias no comunes,    (Clamidia, Ureaplasma urealyticum, 
Micoplasma hominis) o sea    inmunológica.    en orina y fluidos prostátitos cno hay evidencia de células inflamatorias.   
Prostatodinia: igual que la no bacteriana pero sin céls infalmatorias en líquidos. Tx alfa1bloqueadores. 
Las prostatitis elevan    PSA 
 
 
PROSTATITIS AGUDA 
 
PATOGENIA 
Ascendente, reflujo de orina a conductos prostáticos. Vía linfática y hematógena.   
 
CLÍNICA 
Fiebre, síntomas irritativos, obstructivos, dolor suprapúbico o perineal, dolor intenso al tacto rectal.   
Tacto rectal vigoroso o masaje prostático pueden ocasionar septicemia, no deben hacerse.   
 
PARACLÍNICOS 
Leucocitosis, piuria, bacteriuria, hematuria.   
Urocultivo: gramnegativos (E. coli y Pseudomonas), menos frec. gram positivos (Enterococcus) 
 
DIAGNÓSTICO DIFERENCIAL 
Pielonefritis, epididimitis, diverticulitis.   
 
CLASIFICACIÓN DE PROSTATITIS 
Categoría I: prostatitis bacteriana aguda 
Categoría II: prostatitis bactriana crónia 
Categoría III: prostatitis no bactriana/ síndome pélvico doloroso 
  IIIA‐ Inflamatorio 
  IIIB‐ No inflamatorio 
Categoía IV: Prostatitis asintomática (diagnóstico histológico o andrológico) 
 
TRATAMIENTO 
Hospitalización vía parenteral (ampicilina y aminoglucósidos) hasta obtener sensibilidades. Luego de 24‐48 hrs afebril, cambiar    vía 
oral y completar ciclo de 4‐6 semanas.   
Si retención aguda de orina: cistostomía suprapúbica o sondeo uretral.   
 
Complicaciones: absceso prostático, prostatitis crónica bacteriana, retención urinaria.   
 
PROSTATITIS CRÓNICA BACTERIANA 
CLÍNICA 
Muchos asintomáticos, antecedente de IVU.   
Próstata    blanda, indurada, molestia o dolor sordo mal localizado en periné suprapúbico, síntomas irritativos.   
 
PARACLÍNICOS 
EGO normal, a excepción de cistitis secundaria. Secreción prostática con aumento en número de leucocitos (>10/HPF), especialmente 
macrófagos.     
 
El cultivo de secreciones prostáticas u orina post masaje es necesario para el diagnóstico.   
 
DIAGNÓSTICOS DIFERENCIALES 
Uretritis crónica, cistitis, enf anales. 
 
TRATAMIENTO 
AB de adecuada penetración a próstata (TMP/SMX, quinolonas o efalexina, eritromicina) por 6‐12 semanas, se puede dar tx sintomático 
con antiinflamatorios y baños de asiento.   
 
PROSTATITIS NO BACTERIANA 
ETIOLOGÍA 
Es la más común de los sx prostáticos, etiología desconocida.   
Pudiera obedecer a Clamydia, micoplasmas, ureaplasmas, virus. Enf autoinmune, inflamación no infecciosa. Diagnóstico    de exclusión.   
 
No antecedente de IVU. 
 
CLÍNICA 
Se usa NIH‐CPSI para valorar los síntomas 
Síntomas irritativos, molestia perineal o suprapúbica. 
 
PARACLÍNICOS 
Aumento de número de leucocitos en secreciones prostáticas, cultivos negativos.   
 
DIFERENCIALES 
Prostatitis crónica bacteriana 
 
TRATAMIENTO 
Ensayo de tx para atípicos con eritromicina.   
Tratamiento sintomático y baños de asiento.   
 
PROSTATODINIA 
Dolor prostático sin inflamación, jóvenes y mediana edad, etiología diversa como disfuncción miccional y disfunción de los músculos 
pélvicos.   
Síntomas igual que prostatitis crónica. Puede haber intermitentecia o dificultad para orinar.   
 
EF normal o molesto al examen 
EGO nrmal, secreciones prostáticas normales. 
 
Tratamiento: alfabloqueadores, ejercicios del piso pélvico, baños de asiento.   
 
MASCULINO DE 60 AÑOS DE EDAD, ANTECEDENTE DE DIABETES MELLITUS DESDE HACE    5 AÑOS EN TRATAMIENTO CON 
METFORMINA; HIPERPLASIA PROSTÁTICA BENIGNA EN TRATAMIENTO CON TAMSULOSINA DESDE HACE 2 AÑOS. HACE 4 MESES TUVO 
INFECCIÓN DE VÍAS URINARIAS TRATADA EMPÍRICAMNETE CON ÁCIDO NILIDÍXICO/FENAZOPIRIDINA CON LO CUAL CEDIERON LAS 
MOLESTIAS.   
INICIÓN HACE 3 DÍAS, CON DISURIA LEVE Y EXACERBACIÓN DE LOS SÍNTOMAS COMO PUJO, TENEMOS, FRECUENCIA E INTERMITENCIA 
URINARIAS.    AL DÍA SIGUIENTE PRESENTÓ UN PICO DE FIEBRE DE 39° Y AYER DE FORMA    PERSISTENTE QUE CEDÍA TEMPORALMENTE 
A LA ADMINISTRACIÓN DE PARECETAMOL.   
A LA EXPLORACIÓN: FC 96, FR 20, T 38.5, TA 110/70, DIAFORESIS, PALIDEZ DE TEGUMENTOS, ABDOMEN DOLORSO A LA PALPACIÓN 
PROFUNDA EN HIPOGASTRIO, TESTÍCULOS Y PENE SIN ALTERACIONES, DOLOR INTENSO A LA REALIZACIÓN DEL TACTO RECTAL, NO SE 
PALPAN SONAS DE FLUCTUACIÓN, HIPERTÉRMICO. BIOMETRÍA HEMÁTICA LEUC 18MIL, NEU 80%, HB 15, HTO 45, PLAQ 300, CREAT 1.1, 
GLUC 105, EGO 20, ERI30 BACT ABUNDANTES, NITRITOS POSITIVOS.   
 
POR EL CUADRO CLÍNICO ACTUAL, LA SOSPECHA DIAGNÓSTICA ES:   
A) PROSTATITIS AGUDA 
B) PROSTATITIS CRÓNICA BACTERIANA 
C) CISTITIS AGUDA COMPLICADA 
D) URETRITIS 
 
EL TRATAMIENTO A ELEGIR ES: 
A) COLOACIÓN DE SONDA FOLEY A DERIVACIÓN 
B) OBTENER CULTIVO DE ORINA E INICIAR TRATAMIENTO DE AMPLIO ESPECTRO INTRAVENOSO 
C) RESECCIÓN TRANSURETRAL DE PRÓSTATA 
D) OBTENER PRUEBA DE STAMEY‐MEARES (4MUESTRAS) 
 
La prueba de Stamey es solo para localizar la infección en caso de sospecha de prostatitis crónica.   
 
TRES DÍAS DESPUÉS NO SE OBSERVA MEJORÍA CON EL TRATAMIENTO PREVIO, USTED DECIDE: 
A) TOMOGRAFÍA COMPUTADA DE ABDOMEN Y PELVIS 
B) CULTIVO DE SECRECIONES PROSTÁTICAS 
C) CULTIVOS DE MICRORGANISMOS ATÍPICOS EN SANGRE Y ORINA 
D) CISTOSTOMÍA SUPRAPÚBICA 
 
Se realiza ante la sospecha de abseso prostático, otro estudio adecuado es el USG transrectal.   
 
SEGÚN LO ANTERIOR, USTED ESPERA ENCONTRAR: 
A) DIVERTÍCULO VESICAL GIGANTE CON AUMENTO DE ECOGENICIDAD EN SU INTERIOR 
B) ASILAMIENTO DE MICOORGANISMOS ATÍPICOS 
C) GRANGRENA DE FOURNIER 
D) ABSCESO PROSÁTICO 
 
EL TRAMIENTO A ELEGIR ES:   
A) DIVERTICULECTOMÍA TRANSVESIAL 
B) ANTIBIÓTICO DIRIGIDO A MICROORGANISMO AISLADOS 
C) DRENAJE POR PUNCIÓN TRANSRECTAL 
D) DEBRIDACIÓN EXTENSA DE TEJIDO NECÓTICO Y ANTIBIÓTICOS. 
 
El drenaje del absceso se puede transuretral o transrectal guido por USG 
RESPUESTA A, B, A, D, C 
 
EPIDIDIMITIS AGUDA 
ETIOLOGÍA 
Infecciosa vs no infeccionsa 
>80% es bacteriana 
<20%: sarcoidosis, Behcet, medicamentos (amiodarona) 
 
Aguda vs crónica (>6 semanas) 
 
Menores de 40 años==> Chlamydia o N. honorrhoeae 
Adultos mayores ==> IVU y prostatitis (gram negativos). 
Epidimitis crónica infecciosa: Micobacteriua, Brucella.   
 
PATOGENIA 
Flujo retrógrado de orina a conductos eyaculatorios. Desde vejiga, uretra o próstata.   
FR: instrumentación de vía urinaria, obstrucción al flujo de salida, anormalidades genitourinarias.    En niños pensar anomalías 
genitourinarias o etiolgía viral.   
 
CLÍNICA 
Dolor y aumento de tamañ del epidídimo, disminuye con la elevación escrotal (Prehn positivo). 
Acompaña dolor en conducto deferente. 
Fiebre, síntomas de almacenamiento (cistitis) o de uretritis son comunes.   
Si avanza, la infección puede afectar al testículo (orquiepididimitis). 
 
PARACLÍNICOS 
Leucocitosis, EGO (piuria, bacteriuria, hematuria)., cultivo de secreción uretral y cultivo de orina.   
 
DIAGNÓSTICOS DIFERENCIALES 
Tumos testicular, torsión testicular (USG con duda diagnóstica). 
 
TRATAMIENTO 
Antibióticos 2‐4 semanas (en caso de ITS tratar a la pareja sexual) 
Ceftriaxona, doxiciclina y levofloxacino. 
Sintomático: analgésicos, antiinflamatorios, reposo y elevación escrotal.   
Complicaciones: absceso, orquiepididimitis 
 
GPC recomienda ceftria o azitro + doxi si tienen prácticas sexuales, 2 da opción levo.   
Si no tienen prácticas sexuales:    TMP/SMX o levo. 
Si cateterismo o instrumentación: TMP/SMX o ciproflox. 
 
 
Respecto a epididimitis. ¿Cuál es el manejo de las complicaciones (orquiepididimitis y absceso)?   
¿Que tan frecuentes son?   
Epididimitis >6 semanas puede hacer abscesos, casi no vemos epididimitis aisladas.   
 
MASCULINO DE 25 AÑOS DE EDAD, IVSA 18 AÑOS DE EDAD, 8 PS, USO IRRGULAR DE MÉTODO DE BARRERA, VIDA SEXUAL ACTIVA. 
QUIRÚRGICOS: ORQUIDOPEXIA DERECHA AL AÑO DE EDAD POR CRIPTORQUIDIA. REALIZA ACTIVIDAD FÍSICA 3 VECES A LA SEMANA. 
HACE 5 DÍAS INICIÓN CON DOLOR CON TESTÍCULO DERECHO, DESDE HACE DOS DÍAS NOTÓ AUMENTO DE VOLUMEN IPSILATERAL, SE 
OBSERVA CON COLORACIÓN ERITEMATOSA. TESTÍCULO CONTRALATERAL NORMAL. TEMPERATURA 37.5, DOLOR A LA PALPACIÓN DEL 
TESTÍCULO DERECHO, AUMENTADO DOS A TRES VECES EL VOLUMEN DEL TESTÍCULO IZQUIRDO, AUMNTA AL ESTAR DE PIE, CEDE 
LEVEMENTE AL SOSTENER EL TESTÍCULO. 
 
EL DIAGN´STICO DE SU ELECCIÓN ES: 
A) TORSIÓN TESTICULAR, 
B) HIDROCELE 
C) TUMOR TESTICULAR 
D) ORQUIEPIDIDIMITIS 
 
LA MANIOBRA DE ELEVAR EL TESTÍCULO Y DISMINUCIÓN SUBSECUENTE DEL DOLOR SUGIERE: 
A) ORQUIEPIDIDIMITIS 
B) TORSIÓN TESTICULAR 
C) HIDROCELE 
D) TUMOR TESTICULAR 
 
AL CONFIRMAR SU DIAGNÓSTICO, USTED DECIDE:   
A) DESTORSIÓN TESTICULAR QUIRÚRGICA Y ORQUIDOPEXIA 
B) TRATAMIENTO ANTIBIÓTICO 
C) HIDROCELECTOMÍA 
D) ORQUIECTOMÍA RADICAL 
 
LA ETIOLOGÍA MÁS PROBABLE DE ESTA PATOLOGÍA ES: 
A) INFECCIÓN RETRÓGRADA DE EPIDÍDIMO Y TESTÍCULO POR BACTERIAS, PROBABLEMENTE DE TRANSMISIÓN SEXUAL. 
B) ROTACIÓN DEL TESTÍCULO SOBRE SU EJE LONGITUDINAL POR IMPLANTACIÓN ANÓMALA DE LA TÚNICA VAGINALIS 
C) AUMENTO PROGRESIVO DE LÍQUIDO ENTRE LAS CAPAS DE LA VAGINALIS. 
D) TESTÍCULO NO DESCENDIDO FUE EL FACTOR PREDISPONENTE DE NEOPLASIA INTRATUBULAR Y DESARROLLO DE SEMINOMA.   
 
RESPUESTA D, A, B, A 
 
Webinars 
Ca de próstata  afección periférica (por eso casi no da síntomas) 
HPB zona de transición (está cerca de la uretra por eso da síntomas) 
La velocidad de crecimiento del antígeno no es indicativo de biopsia, tiene mayor peso si eleva >4. 
 
Energía monopolar no usas NaCl, de hecho sería un factor de riesgo para Sx postRTUP 
Sx postRTUP : bardiardia, convulsiones, náuseas, hiponatremia. 
____ 
Ca de próstata RT si hay mets a hueso y compresión medular.   
Si ya se salió de la próstata no se opera. 
La próstata no se radia. 
________________ 
<12 años orquidopexia   
>12 años orquiectomía 
Si antes menor riesgo malignidad o hipofertilidad. 
 
______________________________________‐ 
 
Mejor studio para litiasis TAC simple. 
Litos renales >2cm cirugía (nefrolitotomía percutánea), <2cm ureteroscopia flexible o litrotripsia extracorpórea. 
Lito ureteral: >7mm casi imposible que pase por uretera, romperá la piedra o catéter doble J. 
    Cuando no hacerle algo a la piedra? Dolor controlable, función renal normal, lito unilateral <7mm. 
 
 
Calcio es lo más frecuente, hipercalciuria es frecuente. 
Indicaciones de litotripsia extrcorporea: piedras <1cm (funciona mejor) renales o uréter superior. En obesos no es muy adecuada, es 
contraindicación relativa.   
______________ 
IVU pediátrico: cefalosporina, penicilinas. 
 
________ 
Reflejo cremastérico generalmente no tiene en torsión, aquí hay horizonatalización.   
 
DISFUNCIÓN ERÉCTIL 
DEFINICIÓN 
No aumentan riesgo 
 
SALUD PÚBLICA 
>50años    4% 
>60  17% 
>70  75% 
 
Cirugía prostática puede generar lesión de nervios cavernosos, mala oxigenación de cuerpos cavernosos, insuficiencia vascular. 
 
DIAGNÓSTICO 
Prueba de tumescencia y rigidez peneana nocturna mediante Rigiscan, estudios vasculares (inyección intracavrnsa de fármacos 
vasoactivos, eco doppler de arterias cavernosas, carvernosografía/carvenosometría con infusión, arteriografia de art pudencia interna), 
estudios neurológicos (latencia del reflejo bulbocavernoso, estudios de conducción nerviosa), estudios endocrinológicos. 
 
Prueba de tumescencia normal: 60% de rigidez, >10 min. 
USG doppler normal: flujo sistólico pico >30cm/s y un índice de resistencia >0.8 (si es normal no son necesarios más estudios). 
  Si USG anormal==>arteriografia y prueba de cavernosografia/cavernosmetria si candidato a qx vascular. 
 
TRATAMIENTO 
Pequeños cambios de vida pueden solucionar el problema. 
Si causa hormonal aplicar testosterona, contraindicada si antecedente de Ca de próstata o síntomas de prostatismo. 
 
Tx de primera línea: inhibidores de 5‐PDE aumentan los efects de NO. 
Sildenafil (viagra): comienza entre 30‐60min, comida con abundante grasa puede reducir absorción.    La eficacia puede durar hasta 12 
hrs. 
 
Vardenafil (levitra): hace efecto a los 30min, grasas reduce su efecto, dosis de 5‐10‐20mg. 
 
Tadalafil (cialis): efecto a partir de los 30min, eficacia máxima a las 2hrs, eficacia mantenida durante máximo 36hrs. Los alimentos NO 
afectan su eficacia, dosis de 10 ‐20mg. 
 
Vardenafil y sildenafil asociarn visión borrosa. 
Nitratos contraindicados con todos. Si un paciente desarrolla angina mientras usa PDE5 puede usar otros antianginosos o o esperar 24 
hrs si tomó sildenafilo o vardenafilo, 48 con tadalafi. El perfil de efectos adversos no empeora nisiquiera si toma múltiples 
anithirtensivos.   
 
Vardenafilo + alfabloqueante NO. 
Sildenafil + alfabloqueante: solo 4 hs despúes del alfa 
Tadalafilo + alfabloqueante NO, solo si el alfa es tamsulosina.   
 
Dispositivo de constricción al vacío: Adecuado en ancianos. 
 
Tx de segunda línea: Alprostadil intracavernoso 
Tx de tercera línea: prótesis peneana. 
 
 
Testosteronaa, pedir LH, FSH, proalctina, estróngenos. 
 
MASCULINO DE 55 AÑOS DE EDAD, DM E HIPERCOLESTEROLEMIA DE RECIENTE DIAGNÓSTICO. TABAQUISMO POSITIVO DESDE LOS 18 
AÑOS (2 CIGARRILLOS/DÍA), CIRUGÍAS NEGADAS. TATAMIENTOS: METFORMINA Y PRAVASTATINA (APEGO IRREGULAR) 
ACUDE POR DISMINUCIÓN DE FIRMEZA DE LAS ERECCIONES Y DETUMESCENCIA TEMPRANA, DOS AÑOS DE EVOLUCIÓN, AUMENTANDO 
DE FORMA PROGRESIVA, EN MÁS DE LA MITAD DE LAS OCASIONES NO PERMITE LA PENETRACIÓN. INTERROGATORIO DIRIGIDO 
TAMBIÉN NOTA PÉRDIDA DE ERECCIONES NOCTURNA Y DEL LIBIDO.   
EXPLORACIÓN GENITAL NORMAL. PRÓSTATA DE 25 GR, SIN NÓDULOS.   
 
ESTUDIO INICIAL DE ABORDAJE: 
A) PERFI HORMONAL, LÍPIDOS, GLUCOSA. 
B) USG DOPPLER PENEANO 
C) RMN PELVIS 
D) TESTOSTERONA, HORMONAS TIROIDEAS. 
 
EL ESTUDIO QUE PIDIÓ NO ENCONTROL ALTERACIONES Y DECIDE INICIAR CON TRATAMIENTO MÉDICO, LA PRIMERA ELECCIÓN ES: 
A) TESTOSTERONA 
B) INHIBIDORES DE 5 ALFA REDUCTASA 
C) INHIBIDORES DE FOSFODIESTERASA 5 
D) ALFA BLOQUEADORES 
 
EJEMPLOS DE FÁRMACOS: 
A) VARDENAFIL, ALPROSTADIL 
B) TADALAFIL, VARDENAFIL 
C) ALPROSTADIL, PAPAVERINA 
D) FENTOLAMINA, DOXAZOSINA 
 
MECANISMO ACCIÓN: 
A) BLOQUEA DEGRADACIÓN DE GMPc Y RELAJACIÓN DE MÚSCULO LISO CAVERNOSO 
B) BLOQUEO DE RECEPTORES ALFA, VASODILATACIÓN Y AUMENTO DE FLUJO SANGUÍNEO CAVERNOSO. 
C) ACTIVA LOS RECEPTORES ALFA, CONSTRICCIÓN DE LOS VASOS SANGUÍNEOS D ELOS CUERPOS CAVERNOSOS.   
D) INHIBE LA DEGRADACIÓN DE LA DIHIDROTESTOSTERONA. 
 
CONTRAINDICACIONES 
A) USO SIMULTÁNEO DE BETA BLOQUEADORES, CRISIS ASMÁTICA 
B) CÁNCER DE PRÓSTATA 
C) HIPOTENSIÓN ORTOSTÁTICA, BETABLOQUEADORES, DM INSULINO DEPENDIENTE 
D) USO DE NITRATOS, ANGINA INTETABLE, INFARTO AGUDO AL MIOCARDIO RECIENTE. 
 
RESPUESTAS A, C, B, A, D 
 
TORSION TESTICULAR 
DEFINICIÓN 
Torisón de cordón espermático y pérdida de flujo sanguíneo. Más frecuente en adolescentes y neonatos. 
 
DIAGNÓSTICO 
Duración menos a 6 hrs hace posible el salvamiento del testículo.   
Torsión extravaginal constituye 5% (70% es prenatal), asocia alto peso al nacimiento. La torsion intravaginal se observa normalemente 
en menores de 30 años.   
 
Tiempo de evolución  Salvamiento de testículo 
<6 hrs    90‐100% 
12‐24hrs    20‐50% 
>24hrs    0‐10% 
 
Complicaciones: infarto testicular, pérdida de testículo, infeccion, malignidad.   
 
CLÍNICA 
Comienzo súbito (ej después del ejercicio, trauma o espontáneo=). 
33% náusea y vómito.   
Ausencia de reflejo cremastérico, testículo en posición anormal, 
En neonatos el testículo duro, no doloroso, fijo y con cambios de coloración en la piel. 
 
Puede haber hdirocele reactivo o edema escrotal. Testículo horizontal, Prehn negativo‐, eritema es escroto.   
DIAGNÓSICOS DIFERENCIALES 
Torsión de apéndice testicular o de epidídimo, epididitimisi, orquitis, idrocele, turmos, edema escrotal idiopático, truama.   
 
PARACLINICOS 
USG cuando existen una sospecha baja de torsión. 
USG Doppler: ausensi de flujo sanguineo, disminución de la velocidad de flujo en arterias intratesticulares, aumento en los índices de 
resistencia.   
S y E 90‐100% 
 
TRATAMIENTO 
Exploración quirúrgica, está indicada de forma inmadiata. Si fracaso orquiectomía.  
Destorsión quirúrgica manual, orquidopexia. 
 
MASCULINO DE 25 AÑOS DE EDAD, PREVIAMENTE SANO, INICIÓ CON DOLOR TESTICULAR IZQUIERDO INTENSO HACE 4 HRS DE FORMA 
SÚBITA, SIN ASOCIACION A TRAUMATISMO, NO CEDIÓ CON ANALGÉSICOS NI DISMINUE CON LOS CAMBIOS DE POSICIÓNM 
IRRADIACIÓN A REGIÓN SUPRAPÚBICA, NO TIENE SÍNTOMAS URINARIOS, NO HAY SECRECIÓN URETRAL. 
TEMP 37, FC 80, FR 18, IMC 20, ABDOMEN SIN DATOS PATOLÓGICOS, NO HAY ALTERACIONES EN PENE, TESTÍCULO IZQUIERDO 
ELEVADO, SIN AUMENTO DE VOLUMENM NO SE PALPAL ADENOPATÍAS INGUINALES, SIGNO DE PREHN NEGATIVO, REFLEJO 
CREMASTÉRICO ABOLIDO. 
 
SOSPECHA DIAGNÓSTICA: 
A) TORSIÓN TESTICULAR 
B) ORQUIEPIDIDIMITIS 
C) VARICOCELE IZQUIERDO 
D) TUMOR TESTICULAR 
 
LE SOLICITA UN ULTRASONIDO TESTICULAR PARA CORROBORAR SU DIAGNÓSTICO, USTED ESPERA ENCONTRAR:   
A) AUSENCIA DE FLUJO SANGUÍNEO TESTICULAR 
B) AUMENTO DE FLUJO SANGUÍNEO TESTICULAR, EPIDÍDIMO ENGROSADO 
C) PLEXO PAMPINIFORME DIALTADO >3MM 
D) TUMORACIÓN INTRATESTICULAR CON AUMENTO DE VASCULARIDAD Y ÁRES QUÍSTICAS. 
 
TRATAMIENTO DE ELECCIÓN: 
A) DESTORSIÓN MANUAL INMEDIATA 
B) DESTORSIÓN QUIRÚRGICA INMEDIATA Y ORQUIDOPEXIA BILATERAL 
C) ANALGÉSICOS Y VARICOCELECTOMÍA IZQUIERDA 
D) ORQUIECTOMÍA 
 
La orquidopexia bilateral se hace para prevenir futuros cuadros. 
 
FISIOPATOLOGÍA DEL TRASTORNO: 
A) TORSIÓN INTRAVAGINAL DEL TESTÍCULO POR IMPLANTACIÓN ALTA DE LA VAGINALIS, EL EJE DEL CORDÓN ESPERMÁTICO SE 
TUERCE E ISQUEMIA SUBSECUENTE.   
B) INFECCIÓN DEL EPIDÍDIMO Y POSTERIOMRENTE DEL TESTÍCULO DE FORMA RETRÓGRADA Y PROBABLMENTE POR 
GÉRMENES DE TRANSMISIÓN SEXUAL. 
C) FLUJO DE VENA GONADAL IZQUIERDA HACIA LA VENA RENAL IZQUIERDA EN ÁNGULO RECTO, MALFUNCIONAMIENTO DE 
VÁLVULAS VENOSAS Y CONGESTIÓN VASCULAR 
D) DISPLASIA INTRATUBULAR QUE PROGRESIÓ AL DESARROLLO DE TUMOR TESTICULAR 
 
RESPUESTA A, A, B, A 
 
CRIPTORQUIDIA 
DEFINICIÓN 
Ausencia de testículos en escroto por alteración en el descenso. Alrededor de 85% es unilateral, frecuentemente es el derecho. Por lo 
general se encuentra en el canal inguinal 
 
ETIOLOGÍA 
Multifactorial. Fr prematuros, bajo peso, talla baja, exposición intrauterina a estrógenos. Peso al ancer es el principal determinante 
asociado.   
En niños a término el descenso de puede completar durante los primeros 6 meses de vida, en prematuros este descenso se peude 
retrasar hasta los 12 meses. 
 
Puede ser espontánea y aislada, asociar AHG o otras anomalías (tumor de Willms, Prader Willi, Beckwith‐Wiederman, Klinefelter, alt de 
cromosoma Y).   
 
PARACLÍNICOS 
Solo en caso de criptorquidia bialteral, ambigüedad genital.  cariotipo, gonadotropinas y testosterona basales. 
Laparoscopia útil para visualizar testículo no palpable, sirve para Dx y Tx.   
 
TRATAMIENTO 
Hormonal: Hcg (éxito de 6‐21), GnRH (éxito 6‐38) 
Cirugía: éxito 70‐90% 
Orquidopexia es la de primera elección.   
 
 
Si el paciente tiene >32 con testículo no descendido observamos porque ya pasó lo crítico.   
Si tiene <32 años orquiectomía. 
PACIENTE MASCULINO DE 2 AÑOS DE EDAD, ANTECEDENTE DE PREMATUREZ (33 SEMANAS DE GESTACIÓN), QUIEN ES LLEVADO POR 
SU PADRES DEBIDO A QUE NO PALPAN TESTÍCULO DERECHO EN BOLSA ESCROTAL. A LA EXPLORACIÓN FÍSICA, SE ENCUENTRA 
TESTÍCULO IZQUIERDO DENTRO DE LA BOLSA ESCROTAL Y EL TESTÍCULO DERECHO AUSENTE EN EL ESCROTO, Y SE IDENTIFICA EN EL 
CANAL INGUINAL, NO TIENE OTRAS ANOMALÍAS EN GENITALES. 
 
DIAGNÓSTICO: 
A) TESTÍCULO CRITPRQUÍDICO 
B) TESTÍCULO EVANESCENTE 
C) TESTÍCULO RETRÁCTIL 
D) TORSIÓN TESTICULAR 
 
DIAGNÓSTICO DIFERENCIAL 
A) HIPERPLASIA SUPRARRENAL CONGÉNITA 
B) DISGENESIA GONADAL MIXTA 
C) TESTÍCULO RETRACTIAL 
D) PRADER‐ WILLI 
 
TRATAMIENTO 
A) ORQUIDOPEXIA 
B) VIGILANCIA 
C) TRATAMIENTO HORMONAL 
D) ORQUIECTOMÍA 
 
RIESGO: 
A) MIGRACIÓN ABDOMINAL 
B) SEMINOMA 
C) ALTERACIÓN EN FERTILIDAD 
D) B Y C SON CORRECTOS. 
 
 
RESPUESTA A, C, A, D 
 
 
 
 
 
 
TRAUMA CRÁNEO‐ENCEFÁLICO 
TCE 
DIAGNÓSTICO 
Factores de riesgo para compliaciones: >65 años,historia de hemorragia, alt de la coagulación, uso de anticoagulantes. 
Mecanismos de TCE peligroso: caída >1metro o cinco escalones, carga axial, atropellamiento por vehículo de motor en movimiento, 
colisión entre vehículos a >100km/h, accidente automovilístico con vuelco, salir despedido de un vehículo.   
Signos de alarma prehospitalaria:   
[MAS CAAFI VIP] 
Múltiples fracturas 
Alt pupilares,   
Sat O2 <80%,   
Cefalea intensa,   
Amnesia anterógrada,   
Alt conductuales, irritabilidad,   
Focalización, crisis convulsivas postrauma. 
hIpotensión, 
Vómito en 2 ocasiones o más,   
Intoxicación 
Pérdida de    2 o más puntos Glasgow adulto o pediaátrica en mediciones sucesivas,   
 
Apertura ocular 
Espontánea      4 
Estímulo verbal      3 
Estímulo doloroso      2 
No hay        1 
 
Respuesta verbal 
Orientada        5 
Desorientada      4 
Palabras inap      3 
Incomprensibles      2 
No hay        1 
 
Respuesta motora 
Obedece órdenes      6 
Localiza dolor      5 
Retirada al dolor      4 
Flexiona al dolor (decorticación)    3 
Extiende al dolor (descerebración)  2 
Sin respuesta      1 
 
Si glasgow 15 en 2 mediciones consecutivas:   
  Cada 30 min x 2 hrs. 
  Cada 60 min x 4 hrs 
  Cada 2 hrs. 
 
 
TCE grave==> UCI 
Rx de cráneo: contusión o laceración de piel cabelluda, profundida de la herida hasta hueso, longitud de la herida >5cm.   
TCE + dolor o rigidez cervical + >65 años o mecanismo traumático peligroso: AP, lateral y trans‐oral. 
Si FR + signo de alarma: TAC 
Si alt de edo alerta: glucosa sérica 
 
Criterios de alta TCE no complicado: ausencia de datos de alarma luego de 24 hrs de observación, TAC normal, familiar que puede 
acompañar al pacinete.   
 
TRATAMIENTO 
Si hiperventila y hace PCO2 de 25 tiene mal pronóstico a 3‐6 meses en comparación con el que tiene 35mmHg. 
 
Indicación para intubación antes del traslado:   
  deterioro consciencia, 
  fractura bilateral mandibular,   
  convulsiones,   
  signos de fractura de base del cráneo.   
Inmovilización cervical en TCE:   
  Glasgow <15 en cualquier momento 
  Dolor o rigidez de cuello 
  Parestesias en extremidades 
  Def neurológico focal 
  Traumatismo de alta energia 
  Sospecha de daño cervical 
Indicación de intubación inmediata: 
  Glasgow 8 o menos 
  Pérdida de reflejo laringeo 
  IRA 
  Hipoxia >65 
  Hiprcapnia >45 
  Hiperventilación espontánea (PCO2 <30) 
  Respiración irregular. 
 
Mejor pronóstico si uso de manitol que no requirió cirugía descompresiva 
Mejor pronóstico si suo de Sol cloruro de sodio al 7.5% en vez de Ringer lactato. 
Sol salina 7.5% es más efectiva que manil para manejo de hipertensión intracraneal. 
Evitar el uso de esteroides en el amejo de trauma craneal en cualquier nivel de gravedad.   
Profilaxis con fenitoina tan pronto se pueda (reduce riesgo de crisis posttrauma las cuales aparecen en los primeros 7 días). 
 
PRONÓSTICO 
Hipotensión aumenta moralidad. 
Presión arterial baja, Sat O2 baja en fase pre‐hospitalaria indican mal pronóstico.   
Si no se pude intubar: mascarilla laringea 
Dosis altas de manitol pre‐operatorias mejoró el pronóstico.   
 
 
 
 
[CASO CLÍNICO RETO ENARM] 
MUJER DE 25 AÑOS DE DE EDAD. LLEADA AL SERVICIO DE URGENCIAS POR PERSONAL DE LA CRUZ ROJA POR PRSENTAR CAÍDA DESDE 
UN CABALLO EN MOVIMIENTO (GALOPE) CON GOLPE CONTUSO EN REGIÓ OCCIPITAL DEL CRÁNEO. LOS FAMILAIRES REFIERE QUE A LA 
CAÍDA ELLA ESTABA CONSCIENTE, CON DOLOR DE CABEZA INTENSO. A LA LLEGADA DE LOS PARAMÉDICOS ELLA INICIÓ CON VÓMITO 
EN PROYECTIL. A SU LLEGADA URGENCIAS TIENE VÍA AÉREA PERMEABLE, ADEUCADA RESPIRACIÓN Y VENTILACIÓN, SIN HEMORAGIA 
EVIDENTE, COM APERTURA OCULAR ANTE ESTÍMULO DOLOROSO, RESPUETA VERBAL INAPROPIADA, RESPONDE A MOVIMIENTO DE 
FLEXION AL ESTÍMULO DOLOROS. FC 82 FR 22 T 36.3 SO2 92%, TA 120/60, GLUCOSA CAPILAR 89. 
 
¿CUÁL ES EL MANEJO ADECUADO PARA LA VÍA AÉREA DE LA PACIENTE? 
A) PUNTAS NASALES 
B) AIRE AMBIENTE 
C) INTUBACIÓN OROTRAQUEAL   
D) MÁSCARA LARINGEA 
R: C 
 
 
[TRAUMA CRANEOENCEFÁLICO][r] 
FEMENINA DE 40 AÑOS DE EDAD, CON TRAUMATISMO CRANEOENCEFÁLICO POR CAÍDA DE APROXIMADAMENTE UN METRO DE 
ALTURA. ES LLEVADA A URGENCIAS. A LA EXPLORACIÓN CON HEMATOMA EN REGIÓN FRONTOPARIETAL, OJOS CERRADOS QUE AL 
ESTÍMULO DOLOROSO LOS ABRE, QUEJUMBROSA, CON SONIDOS INCOMPRENSIBLES Y LAS EXTREMIDADES PRESENTAN FLEXIÓN 
INVOLUNTARIA. 
EL MANEJO QUE REQUIERE DE INMEDIATO LA PACIENTE ES: 
   
A) SOLUCIÓN SALINA AL 0.9% Y MANITOL. 
B) CRANEOTOMÍA DESCOMPRESIVA. 
C) INTUBACIÓN OROTRAQUEAL E HIPERVENTILACIÓN. 
D) CORTICO ESTEROIDES INTRAVENOSOS. 
 
RESPUESTA C 
Recordar la escala de Glasgow e indicación de intubación. Tiene 2 de respuesta ocular, 2 de respuesta verbal, 3 de respuesta motora. 
Con 8 puntos o menos basta para intubar. La hiperventilación está indicada en deterioro neurológico agudo, debe usarse con cautela ya 
que a largo plazo puede favorecer isquemia.     
El segundo objetivo de la reanimación de la paciente es precisamente el control hemodinámico y el manitol. los objetivos son PAM 
90mmHg, PPC60mmHg (Presión de Perfusión Cerebral, PPC= PAM‐PIC o PVY dependiendo de cual se mayor). 
 
Indicaciones de creaneotomía:     
A) Hematoma epidural: hematoma con vol >30cc, grosor >15mm, desviación de línea media >5mm, localización temporal, compresión 
de cisternas mesencefálicas   
B) Hematoma subdural: grosor >10mm, desviación de línea media >5mm, diferencia de grosor y desviación >5mm, compresión de 
cisternas mesencefálicas 
 
No se ha observado ningún efecto benéfico de esteroides en el control de la PIC en pacientes con TCE grave. 
 
 
Motora      Verbal      Ocular       
Orden    6  Normal    5  Espontánea  4   
Localiza dolor  5  Desorientada  4  Orden    3 
Retira al dolor  4  Cambiar palabras  3  Dolor    2 
Decorticación  3  Incompresible  2  No los abre  1 
Descerebración  2  No habla    1 
Sin repsuesta  1 
 
Se considera que está en coma cuando la sume es 8 o menos.   
TCE leve  14‐15 
TCE mod  9‐13 
TCE sev  8 o menos   Hay hipoxemia requiere O2 rápido y corregir hipotensión,   
 
 
HOMBRE DE 33 AÑOS DE EDAD, QUE ES ASALTADO AFUERA DE SU DOMICILIO, RECIBIENDO GOLPE CONTUSO CON TUBO EN CRÁNEO. A 
SU LLEGADA AL HOSPITAL USTED LO ENCUENTRA DESPIERTO, CON GLASGOW QUE FLUCTÚA ENTRE 14 Y 15. DURANTE LA 
EXPLORACIÓN USTED PALPA UNA FRACTURA HUNDIDA DE CRÁNEO. 
 
EL SIGUIENTE HALLAZGO DURANTE EL SEGUIMIENTO Y EVALUACIÓN DEL PACIENTE, JUSTIFICARÍA LA INDICACIÓN DE 
HIPERVENTILACIÓN: 
   
A) APARICIÓN DE PARESIAS Y PARESTESIAS. 
B) DETERIORO NEUROLÓGICO AGUDO. 
C) INTEGRACIÓN DE LA TRIADA DE CUSHING. 
D) PREVENCIÓN DE CRISIS CONVULSIVAS SECUNDARIAS. 
 
RESPUESTA 
La presencia de parestesias o hemiparesia, junto con dilatación pupilar o pérdida del estado de alerta forman parte del deterioro 
neruológico agudo.   
Triada de Cushing es hipertesión, bradicardia y respiración irregular. Es una respuesta del SNC a la hipertensión intracraneana. Son 
indicadores de deterioro neurológico.   
La HIC puede manifestarse por: cefalea, vómito, edema de papila, paresia del recto externo por compesión del VI par craneal o 
abducens 
 
Se prefiere mantener normocarbia en la mayoría de los pacientes (35‐45mmHg).   
La hiperventilación deberá ser utilizada solo en "deterioro neurológico agudo" puesto que se hay que recordar que la reducción de 
PaCO2 genera vasoconstricción y puede producir isquemia especialmente si está debajo de 30mmHg. 
Es aceptable una PaCO2 ebtre 25‐30mmHg por periodos breves para tratar un deterioro neuroógico agudo. El efecto de la 
hiperventilación es transitorio (menos de 6 hrs) 
 
 
[r] HOMBRE DE 18 AÑOS DE EDAD, QUE ES LLEVADO POR SU MADRE AL SERVICIO DE URGENCIAS AL PRESENTAR HERIDA SANGRANTE 
EN REGIÓN PARIETAL DERECHA, SECUNDARIA A TRAUMATISMO CRANEOENCEFÁLICO CON UN PALO DURANTE UNA RIÑA. DURANTE LA 
EXPLORACIÓN NEUROLÓGICA RÁPIDA USTED LO ENCUENTRA CON GLASGOW DE 15. 
LA INDICACIÓN DE UNA TAC DE CRÁNEO EN ESTE PACIENTE SE JUSTIFICARÁ CON LA IDENTIFICACIÓN DEL SIGUIENTE HALLAZGO: 
   
A) HERIDA MAYOR DE 5CM. 
B) POLITRAUMATIZADO. 
C) HIPOTENSIÓN SEVERA. 
D) AMNESIA. 
 
RESPUESTA 
Criterios para TAC en TCE 
Pérdida del edo de alerta > 5min 
Amnesia 
Cefalea grave 
Glasgow <15 
Focalización 
 
[r] FEMENINA DE 42 AÑOS DE EDAD, CON TRAUMATISMO CRANEOENCEFALICO AL CAER DE UNA ESCALERA DE 2 METROS. CON 
PERDIDA DEL ESTADO DE ALERTA DURANTE 2 MINUTOS, YA CONCIENTE REFIERE CEFALEA HOLOCRANEANA. Y A LA LLEGADA AL 
HOSPITAL SE ENCUENTRA CONFUSA, CON PUPILA DERECHA MIDRIÁTICA Y HEMIPARESIA IZQUIERDA. 
EL DIAGNÓSTICO MÁS PROBABLE EN ESTA PACIENTE ES: 
   
A)HEMATOMA SUBDURAL. 
B) HEMATOMA EPIDURAL. 
C) CONTUSIÓN CEREBRAL. 
D) HEMORRAGIA INTRAPARENQUIMATOSA. 
 
RESPUESTA B 
El dato pivote en este caso es la fluctuación en el estado de alerta (pérdida del estado de alerta 2 minutos, lucidez y confusión). Esta es 
la presentación del hematoma epidural en 50% de los casos. La hemiparesia y la midriasis, incluso el mecanismo de trauma se puede 
compartir con hematoma subdural. 
Las contusiones cerebrales forman el 20‐30% de las lesiones cerebrales graves, pueden coalecer en horas o días y formar un hematoma 
intracerebral.    Esto ocurre en 20% de los pacientes por lo que se aconseja repetir TAC en 12‐24 hrs, En la TAC se ven imágenes hipo‐
hiperdensas intracerebrales y edema. 
 
 
 
 
 
CONTENIDO ADICIONAL CLAVE 
 
  HEMATOMA EPIDURAL  HEMATOMA SUBDURAL 
Epidemio  Incidencia 2.5‐5%, frecuente entre 2da y 3era década de  Más frec que el epidural [20‐30% de TCE severos] 
la vida.    20‐40 años accidente automovilístico (56%) 
Accidente automovilístico 50%  65años caída accidental (56%) 
Caída accidental 30%   
Traumatismo por objeto contundente 10% 
Patogenia  Ruptura de arterias y/o vena meíngea media y en fos  Ruptura de las venas puente entre la corteza cerebral y 
posterior por ruptura de senos transversos y  senos venosos. Se acumula entre duramadre y 
sigmoideos.    aracnoides. El hematoma se distribuye a lo largo de toda 
El hematoma se circunscribe a las suturas ya que la  la superficie cerebral ya que la aracnoides no está 
duramadre se adhiere a ellas.  adherida a las suturas. 
Clínica  La mayoría son agudos.  Agudos <72hrs 
Pérdida del edo de alerta seguido de lucidez mental y  Subagudos de 4 días a 3sem/4m. 
deterioro rápidamente progresivo del edo neurológico.  Crónicos >3sem. 
[50%]   
Coma sin periodo de lucidez [20‐50%]  Pérdida del edo de alerta, clínica de HIC, lesión de 
Sin coma [30%]  nervios craneales (III par con alt del diámetro pupilar 
Anisocoria [50%]  con anisororia en 50% por midriasis, II par papiledema 
Lesión del III con midriasis.  en 15%, paresia del VI en 5%, hemiparesia en 50%] 
Imagen  Gold estándar TAC [Biconvexa] adyacente a un sutura  Gold estándar TAC [Concavidad interna] 
por lo regular.   
Tratamiento  Qx  Qx 
Pronóstico  Mortalidad 20‐55% [en agudo]  Mortalidad del 50‐90% [en agudo] 
Midriasis ipsilateral es reversible y asocia buen  Desviación de la línea media, HS con grosor >18mm, 
pronóstico si evacuación en 70 min.  contusión cerebral, lesiones extracraneantes agregadas. 
Midriasis contralateral o bilateral indica mortalidad del  DLM <10mm y HS con grosos <10mm asocia SV de 85‐
74%.  90%. 
 
 
 
TRAUMA DE CUELLO 
ESGUINCE CERVICAL 
 
DEFINICIÓN 
Estiramiento de ligamentos por mecanismo de extensión/flexión.   
Es muy importante hacer diagnóstic diferencial con fractura de vértebras cervicales, meingitis, hemorragia subaracnoidea, hernia discal 
cervical. 
 
DIAGNÓSTICO 
Síntomas agregados al esguince cervical, cefalea, mareo, vértigo, fosfenos, acúfenos, disfagia, dolor mandibular, dolor en hombro o 
dorso, inestabilida emocional [44% GI, 20% GII, 16% GIII] 
 

 
 
Clínica neurológica 
 
  PARESIA  REFLEJOS  HIPOESTESIA 
C5  Deltoidea  Bicipital  Cara externa del hombro y brazo 
C6  Extensores de muñeca  Supinador largo  Cara externa de antrebrazo, pulgar y dedo medio 
C7  Flexores de la muñeca  Tríceps  Cara externa dedo medio 
C8  Flexores de los dedos    Mitad distal antebrazo, dedo anular y meñique. 
 
 
Grado I/II: No precisa estudio 
Grado III:  potenciales evocados y electriomiografía. Estudios radiológicos simples.   
 
1) Buscar factores de alto riesgo: >65, parestesia extremidades, mecanismo peligroso (igual que en TCE), si hay)))> Rx de columna 
cervical. 
2) Buscar factores de bajo riesgo: simple colisión, estar sentado mucho tiempo, caminar todo el tiempo, el dolor se retrasó, estado 
ambulatorio, aunsencia de relajación de columan cervical. ==> si puede rotar 45% el cuello a los costados no precisa rx 
 
TRATAMIENTO 
Fase I: Reducir dolor, disminución de cargas   1‐4d 
Fase II: Incrementar actividad, aumentar cargas  4‐21d 
Fase III: Aumentar actividad, más carga    3s‐6s 
Fase IV: Capacidad funcional total    6s‐3m 
 
GradoI/II:   
naproxeno 250 c 12 + Paracetamol 500 1‐2 c/8 
Piroxicam 20mg cada 24 + paracetamol 500 1‐2 c/8 
Diclofenaco tab.100mg c 24 + paracemol 1‐2 c /8 
Ranitidiina 150mg c 12 si enf ác péptica. 
 
Frío local primeras 72hrs 
Calor superficial después de 72hrs 
Evitar uso de collarin en Grado I 
Evitar reposo absoluto 
Evitar cirugía salvo compicaciones grado IV 
Evitar almohadas "cervicales" 
Evitar inyecciones intratecales 
 
Grado III o IV:   
Referencia 
 
 
 
HOMBRE DE 53 AÑOS DE EDAD, DEDICADO ACTUALMENTE AL CUIDADO DE ANCIANOS Y NIÑOS ENFERMOS. DURANTE SUS VACACIONES 
Y  TRAS  PRACTICAR  BUCEO  PRESENTA  DOLOR  EN  CUELLO  DE  TIPO  INCAPACITANTE.  A  SU  LLEGADA  AL  SERVICIO  DE  URGENCIAS  SE 
SOLICITAN RADIOGRAFÍAS EN 2 PROYECCIONES DE CUELLO. 
 
EL FACTOR DE RIESGO MÁS IMPORTANTE PARA PRESENTAR LESIÓN CERVICAL EN ESTE PACIENTE ES: 
A) SER HOMBRE. 
B) BUCEAR. 
C) LA EDAD. 
D) SER CUIDADOR DE PERSONAS. 
 
RESPUESTAS    B 
 
 
 
 
 
TRAUMA DE TÓRAX 
TRAUMA DE TÓRAX 
 
DIAGNÓSTICO 
Rx torax AP y lateral, cervical lateral, pelvis. 
TAC si dudas y estabilidad hemodinámica.   
Ecocardiograma trasnesofágico: para dx de lesión aórtica 
Ecocardiograma transtorácico: taponamiento cardiaco 
 
Neumotórax abierto: 
Herida abierta hace efecto válvula y provoca neumo a tensión si es progresivo, simple si no progresa. 
Clínica: dolor, disnea, hiperresonancia a la percusión, hipoventilación. Si supera 20 del volumen pulmonar==> clapso 
Tx. cerrar orificio de entrada con 3/4 bordes (deja salir aire pero no entrar). Tubo de drenaje en sitio diferente. Revisión y reperación 
qx. 
 
Neumotórax a tensión:   
Fisiopatología: esplazamiento de mediastino a contralateral (compromiso bipulmonar), compromiso de retorno venoso (IRA y shock).   
Clínica: distención de venas en cuello ( salvo que haya hipovolemia), hipoventilación uni o bilateral, hiperresonancia a la percusión del 
lado afectado.   
Tx: inserción de cateter 14 en 2do pescio intercostal, línea MC. Rx de control si todo bien==> tubo de tórax en 4‐5 espacio IC línea AM. 
 
Hemotórax masivo:   
>1500ml en espacio pleural, frec en trauma abierto 
Clínica: Shock, insuf respiratoria, hipoventilación, matidez a percusión, distensión de venas de cuello pese a hipovolemia.   
Tx: Reposiciion de volumenes (cristalides, coloides, sangre), drenaje torácico con tubo 32F en 4‐5 espacio IC LMA. 
Toracotomía si >1500ml en los primeros momentos, especiamente si paciente inestable o si 200‐400ml/h en las primeras 4 hrs. 
 
PRONÓSTICO   
Mortalidad 25‐30% 
 
CONTENIDO ADICIONAL 
Cristaloides: sol fisiológica (salino al 0.9%), ringer‐lactato, sol salina hiptónica, sol salina hipertónica, sol glucosalina, solución dextrosa o 
glucosa a 5‐10%. 
Coloides: albúmina, gelatina y almidones. 
 
 
  TAPONAMIENTO  NEUMOTÓRAX  NEMOTÓRAX A  HEMOTÓRAX 
CARDIACO  ABIERTO  TENSIÓN 
SÍNTOMAS  Triada: hipotensión + pulso  Dolor, disnea  Disnea  Insuficiencia respiratoria 
paradógico + ingurgitación 
yugular.  ¿? 
Triada de Beck: 
hipotensión + ingurgitación 
yugular + velamiento de 
ruidos cardiacos. 
SIGNOS  Ruidos cardiacos  Hiperresonancia,  Ingurgitación yugular,  Shock, hipoventilación, 
apagados, rocepericárdico.  hipoventilación (si  hiperresonancia (si  matidez, ingurgitación yugular 
grave  grave colapso),  pese a hipovolemia. 
colapso),enfisema  hipoventilación, 
subcutáneo?  enfisema subcutráneo 
TRATAMIENTO  Punción subxifoidea  Parche 3/4 + tubo de  Cateter #14 en 2do  Reposición de volumen + 
Intubar empeora gasto  drenaje + revisión  espacio IC LMC. + Rx +  drenaje torácico tubo 32F en 4‐
cardiaco  reparación.  tubo en 4‐5 espacio IC  5 espacio IC LMA 
  LMA  Si >1500ml en primer 
momento o 200‐00ml/hr en 
primeras 4 hrs==> 
toracotomía. 
   
*el aire del enfisema subcutáneo pude ir del medio ambiente a los tejidos, o de los pulmones a los tejidos.   
 
 
[PONER ALGORITMO DE GPC TRAUMA DE TÓRAX] 
 
Claro pulmonar o Resonancia normal. Normalmente es un sonido de intensidad fuerte, de tono bajo y de duración prolongada. 
 
Timpanismo. Sonido que se obtiene cuando se percute sobre un órgano que contiene aire como el estómago e intestino.    Es más 
resonante que el sonido claro pulmonar, y su intensidad, tono y duración son variables, su característica acústica es la de ser musical y 
rico en sobretonos. 
 
Hiperresonancia o hiperclaridad.Se obtiene cuando se percuten zonas cuyo contenido de aire está aumentado, por ejemplo cuando se 
aumenta la entrada del aire al pulmón en el caso patológico del neumotórax; el sonido se caracteriza por ser más fuerte, más grave y 
más prolongado que el sonido claro pulmonar, pero desprovisto de carácter musical. 
 
Sonido  mate.Se  obtiene  cuando  se  percuten  órganos  sin  aire,  músculos  y  vísceras  macizas  como  hígado  y  bazo.  Es  un  sonido  débil, 
apagado, de tonalidad alta y duración breve. El sonido mate es la completa y absoluta ausencia de resonancia y se oye con dificultad y 
sólo a cierta distancia. 
 
Sonido submate. Es el sonido que se obtiene cuando se percute un órgano macizo que se encuentra cubierto parcialmente con aire, 
como sucede con el hígado cubierto por el borde inferior del pulmón —se escucha al golpear aproximadamente entre el quinto y sexto 
espacio intercostal derecho—. Su tono es menos elevado que la matidez. 
 
 
[TRAUMA TORÁCICO] 
MUJER DE 24 AÑOS DE EDAD ES LLEVADA AL SERVICIO DE URGENCIAS POR SUFRIR ACCIDENTE AUTOMOIVILÍSTICO EN CARRETERA, NO 
TRAÍA CINTURÓN DE SEGURIDAD, SALE EXPULSADA DEL AUTOMÓVIL. CON PÉRDIDA MOMENTÁNEA DEL CONOCIMIENTO, A SU LLEGADA. 
REFIERE DOLOR TORÁCICO Y ESTÁ DISNEICA. SE COLOCAN PUNTAS NASALES Y SE ESTABILIZA LA COLUMNA CERVICAL. RESPIRACIÓN Y 
VENTILACIÓN, CON AUSENCIA DE RUIDOS RESPIRATORIOS EN HEMITÓRAX IZQUIERDO, HIPERRESONANCIA A LA PERCUSIÓN DEL MISMO. 
NO HAY HEMORRAGFIAS EVIDENTES. SE ENCUENTRA CONFUSA. SE EXPONE POR COMPLETO A LA PACIENTE. TA 60/40, FC138, FR 29. 
SIMULTÁNEMANETE A LA VALORACIÓN PRIMARIA . ¿CUÁL ES EL PROCEDIMIENTO DE MAYOR IMPORTANCIA QUE SE LE DEBE REALIZAR 
A ESTA PACIENTE? 
A) RX PÓRTATIL DE TÓRAX 
B) TORACOTOMÍA ABIERTA 
C)    PUNCIÓN CON AGUJA GRUESA EN SEGUNDO ESPACIO INTERCOSTAL, LÍNEA MEDIA CLAVUCULAR. 
D) PUNCIÓN CON AGUJA FINA EN SEGUNDA ESPACIO, A NIVEL COSTOVERTEBRAL 
 
RESPUESTA C 
Lesiones  torácicas  graves  que  deben  ser  reconocidas  y  tratadas  durante  la  valoración  primaria.  Incluyen  neumotórax  a  tensión,   
neumotórax abierto, tórax intestable y neumotórax masivo.    El neumotórax a tensión va a colapsar el pulmón afectado, desplaza al 
mediastino hacia el lado opuesto, reduciendo el retorno venoso y comprimiento el pulmón contralateral. La situación de choque se debe 
a disminucion del retorno venoso. Este es un diagnóstico clínico, no radiológico. Por lo tanto no debe demorarse su tratamiento a la 
confirmación  radiológica.  este  se  basa  en  descomprimir  con  punción  de  aguja  gruesa  en  el  segundo  espacoi  intercostal,  línea  media 
clavicular en el hemitórax afectado.   
No olvidar datos pivote de hiperresonancia, ausencia de ruidos respiratorios y TA baja eso es igual a neumotórax a tensión. 
 
 
MASCULINO DE 27 AÑOS DE EDAD, SUFRE ACCIDENTE AL CHOCAR SU MOTO CONTRA EL MURO DE CONTENCIÓN, A SU LLEGADA AL 
HOSPITAL  SE  ENCUENTRA  CON  TEMPERATURA  DE  38°c,  TA  90/40,  FC  130,  FR  24.  SE  ENCUENTRA  DESORIENTADO  CON  PALIDEZ  DE 
TEGUMENTOS, AUSENCIA DE RUIDOS RESPIRATORIOS, MATIDEZ A LA PERCUSIÓN, CREPITACIÓN DEL CUARTO AL OCTAVO ARCO COSTAL 
EN HEMOtÓRAX DERECHO. 
EL DIAGNÓSTICO PRINCIPAL DEL PACIENTE ES: 
A) TAPONAMIENTO CARDIACO 
B) ROTURA TRUAMÁTICA DE DIAFRAGMA 
C) TÓRAX INESTABLE 
D) HEMOTÓRAX MASIVO 
 
EL TRATAMIENTO QUE REQUIERE DE INMEDIATO EL PACIENTE ES CON: 
A) TORACOSTOMÍA CON SONDA 
B) PLASTIA DIAFRAGMÁTICA CON ABORDAJE ABDOMINAL 
C) PERICARDIOCENTESIS 
D) SONDA ENDOPLEURAL   
 
RESPUESTA D, A 
El neumotórax masivo se pdocuce por la rápida acumulaciónde sangre en el espacio pleural, como ya sabemos su principal causa es la 
lesión del hilio pulmonar o de los vasos sistémicas de la reja costa. el paciente se puede presentar con intestabildiad hemodinámica y 
ausencia de ventilación en un hemitórax, el cuadro clínico es secundario    a la hipovelamia y a la alteración de V/Q que lleva a shock y 
dificultad respiratoria severa. Clíniamente estos pacientes presentan "datos de bajo gasto", con hipotensión, taquicardia, palidez y pulsos 
disminuidos, a la EF ese se tacracteriza por la ausencia de murmullo vesicular y matidez a la percusión del tórax yal y como presenta 
nuestro apciente. La ausencia de ruidos respiratorios y matdezen torax sustentan el diagnóstico.   
El tratamiento del hemotórax consiste en el manejo del shock, descompresión del espacio pleural con tubo de toracotomía, luego de una 
rápida estabilización se lleva a cirugía y no solo con sonda endopleural.   
 
 
 
FEMENINA DE 32 AÑOS DE EDAD, QUE SUFRE ACCIDENTE AUTOMOVILÍSTICO, RECIBE GOLPE DIRECTO DEL VOLANTE SOBRE EL TÓRAX, 
PRESENTA TAQUICARDIA, HIPOTENSIÓN, ESTERTORES BILATERALES Y CIANOSIS IMPORTANTE DE FORMA AGUDA. 
EN BASE AL CUADRO CLÍNICO EL DIAGNOSTICO PROBABLE ES: 
   
A) TAMPONADE CARDIACO. 
B) CHOQUE HEMORRÁGICO. 
C) NEUMÓTORAX A TENSIÓN. 
D) CHOQUE NEUROGÉNICO 
 
RESPUESTA D 
 
 
 
MASCULINO  DE  24  AÑOS  DE  EDAD,  SUFRE  CAÍDA  DE  UNA  MOTOCICLETA  A  GRAN  VELOCIDAD,  ES  LLEVADO  A  URGENCIAS.  A  LA 
EXPLORACIÓN CON TA 90/60, FRECUENCIA CARDIACA 95 POR MINUTO, FRECUENCIA RESPIRATORIA DE 26 POR MINUTO, TEMPERATURA 
DE  36.4  °C.  CONCIENTE,  ORIENTADO,  CON  DISNEA  INTENSA  DE  PEQUEÑOS  ESFUERZOS,  INGURGITACIÓN  YUGULAR,  ENFISEMA 
SUBCUTÁNEO EN CARA ANTERIOR DEL TÓRAX QUE SE EXTIENDE AL CUELLO Y ABDOMEN DEL LADO DERECHO, AUSENCIA DE RUIDOS 
RESPIRATORIOS EN HEMITÓRAX DERECHO E HIPERCLARIDAD A LA PERCUSIÓN. 
     
CON LOS DATOS CLÍNICOS QUE PRESENTA EL PACIENTE, EL DIAGNOSTICO PROBABLE ES: 
A) NEUMOTÓRAX A TENSIÓN. 
B) HEMOTÓRAX MASIVO. 
C) CONTUSIÓN PULMONAR. 
D) TAPONAMIENTO CARDIACO. 
       
DE ACUERDO AL DIAGNOSTICO QUE TIENE EL PACIENTE, SE DEBE REALIZAR INMEDIATAMENTE UNA: 
A) PUNCIÓN CON AGUJA DE GRUESO CALIBRE. 
B) INTUBACIÓN ENDOTRAQUEAL. 
C) PERICARDIOCENTESIS. 
D) TORACOSTOMÍA CON SONDA. 
 
RESPUESTAS    A, A 
 
 
A 27‐YEAR‐OLD MALE PRESENTS TO THE EMERGENCY ROOM WITH A STAB WOUNG TO THE LEFT SIDE OF HIS CHEST. HE IS DYSPNEIC AND 
COMBATIVE, AND HAS A BLOOD PRESSURE THAT CHANGES WITH RESPIRATION. EXAMINATION REVEALS A DISTENDED JUGULAR VEIN 
ON HIS NECK AND DISTANT, MUFFLED HEART SOUNDS. BASED ON THESE CLINICAL FINDINGS, YOU SUSPECT THE PATIENT MAY HAVE 
CARDIAC TAMPONADE. WHICH OF THE FOLLOWING IS THE STRONGEST INDICADOR OF CARDIAC TAMPONADE? 
A)  HYPOTENSION 
B)  PULSUS PARADOXUS 
C)  JUGULAR VENOUS DISTENSION 
D)  DYSPNEA 
 
RESPUESTA B 
Recordar que la triada de Beck de tamponamiento cardíaco es Hiipotensión + ingurgitación yugular + disminución de los ruidos cardiacos. 
Sin embargo en la medicina y en el trauma de tórax mismo hay muchas causas de hipotensión y disnea razón por la cual esas respuestas 
son  fácilmente  descartables.  Neumotorax  abierto,  hemotórax  también  son  presentaciones  de  trauma  de  tórax  y  ambas  hacen 
ingurgitación yugular. El pulso paradójico si bien no es exclusivo de tamponamiento cardiaco si es más el datos más característico. Otras 
causas de pulso paradójico son la embolia pulmonar masiva, choque hemorrágico grave, enfermedad EPOC. 
 
 
MASCULINO DE 37 AÑOS DE EDAD, CHOFER DE TAXI, SUFRE IMPACTO EN SU AUTO, RECIBIENDO TRAUMATISMO CONTUSO EN TÓRAX. 
ES LLEVADO A URGENCIAS. A LA EXPLORACIÓN FÍSICA CON TEMPERATURA DE 36.2°C, TENSIÓN ARTERIAL 90/50, FRECUENCIA CARDIACA 
110  X  MINUTO,  FRECUENCIA  RESPIRATORIA  18  X  MINUTO.  SE  ENCUENTRA  CONSCIENTE,  CON  PALIDEZ  DE  TEGUMENTOS,  DOLOR 
INTENSO EN CARA ANTERIOR DEL TÓRAX, DIAFORÉTICO, CON INGURGITACIÓN YUGULAR, LACERACIONES EN HEMITÓRAX DERECHO Y 
ESTERNÓN. RUIDOS CARDIACOS DISMINUIDOS EN INTENSIDAD 
 
EL DIAGNÓSTICO PROBABLE DEL PACIENTE ES DE 
A) DERRAME PERICÁRICO 
B) HEMOTÓRAX 
C) NEUMOTÓRAX 
D) TAPONAMIENTO CARDIACO 
 
EL TRATAMIENTO QUE DEBE RECIBIR EL PACIENTE INMEDIATAMENTE ES:   
A) TORACOTOMIA 
B) COLOCACIÓN DE SONDA ENDOPLEURAL   
C) PERICARDIOCENTESIS 
D) URESIS FORZADA 
 
RESPUESTAS D, C 
 
  
FEMENINA DE 40 AÑOS DE EDAD, SUFRE CAÍDA DESDE UNA ESCALERA DE 2 METROS RECIBIENDO GOLPE DIRECTO EN LA REGIÓN COSTAL, 
POR LO  QUE ES LLEVADA A URGENCIAS. A LA EXPLORACIÓN CON TENSIÓN ARTERIAL DE 110/70, FRECUENCIA CARDIACA DE  90 POR 
MINUTO,  FRECUENCIA  RESPIRATORIA  DE  40  POR  MIN.  SE  ENCUENTRA  AGITADO,  RESPIRACIONES  RÁPIDAS  Y  SUPERFICIALES,  A  LA 
PERCUSIÓN  CLARO  PULMONAR,  RUIDOS  RESPIRATORIOS  PRESENTES  EN  AMBOS  HEMITÓRAX,  TRAQUEA  CENTRAL,  PRESENTA 
CREPITACIÓN  EN  REGIÓN  COSTAL  DERECHA.  ASIMETRÍA  DE  LOS  MOVIMIENTOS  RESPIRATORIOS.  SE  LE  ADMINISTRA  OXÍGENO 
SUPLEMENTARIO A 10 LTS/MIN. Y SOLUCIONES CRISTALOIDES INTRAVENOSAS. RADIOGRAFÍA DE TÓRAX NO CONCLUYENTE. 
 
ESTA PACIENTE CURSA CON EL DIAGNOSTICO DE: 
A) TÓRAX INESTABLE. 
B) CONTUSIÓN PULMONAR. 
C) HEMOTÓRAX. 
D) NEUMOTÓRAX A TENSIÓN. 
 
LA PACIENTE SE DETERIORA A PESAR DEL TRATAMIENTO MÉDICO IMPLEMENTADO; SE TOMA GASOMETRÍA ARTERIAL PRESENTANDO 
HIPOXEMIA E HIPERCAPNEA, POR LO QUE LA MEDIDA TERAPÉUTICA DE URGENCIA ES REALIZAR: 
A) TORACOSTOMÍA CON SONDA. 
B) AUMENTAR EL APORTE DE O2 Y ANALGESIA. 
C) DESCOMPRESIÓN CON AGUJA DE GRUESO CALIBRE. 
D) INTUBACIÓN ENDOTRAQUEAL CON VENTILACIÓN ASISTIDA. 
 
El tórax inestable se define como la fractura de 3 o 4 costillas en más de dos segmentos de su longitud casionando un fenómen paradójico 
en la caja torácica colapsándolo en inspiración y expandiéndolo en espiración.    Es importante reconocer que pude asociarse a otras 
condiciones como neumotórax, hemotórax, contusión pulmonar. 
El estudio de imagen inicial es Rx portátil de tórax, si hay compromiso multiorgánico ocn gran severidad del trauma se sugiere realizar 
tomografía torácica y abdominal simple y con contraste. Aunque realmente el diagnóstico es clínico.   
Cuando se lesionan las dos primeras costillas se debe sospehcar lesión de grandes vasos,    y cuando se lesionan las últimas se  debe 
descartar lesión diafragmática o intrabdominal.. 
 
La paciente presenta hipoemia, hipercapnea como datos de insfueciencia respiratoria.   
Los critrios de ventilación mecánico son: 
  PO2 <60 
  Frecuencia respiratoria <8, >35 
  Asociación con shock 
  Lesions graves asociadas 
  TCE severo 
  Enf pumonar previa 
  Más de 8 constillas fracturadas 
  ISS (índice de severidad del trauma) >23 
  >65 años   
  PAFI <250 
  Shunt >15% 
  Diferencia alveolo arterial >350 
   
El inicio de ventilación mecánica disminuye 68% la mortalidad hasta el 7%. 
RESPUESTA A, D 
 
 
A 56‐YEAR‐OLD WOMAN WITH A 6‐YEAR HISTORY OF STAGE II CANCER OF THE LEFT BREAST PRESENTED WITH PROGRESSIVE 
SHORTNESS OF BREATH AND FATIGUE. PHYSICAL EXAMINATION REVEALED HYPOTENSION, TACHYCARDIA, JUGULAR VENOUS 
DISTENTION, PULSUS PARADOXUS, AND DISTANT HEART SOUNDS; THE BLOOD PRESSURE WAS 63/44 MM HG, AND THE HEART RATE 
110 BEATS PER MINUTE. RADIOGRAPHY OF THE CHEST PERFORMED WITH THE USE OF PORTABLE EQUIPMENT REVEALED THE WATER‐
BOTTLE SIGN (IN WHICH THE ROUNDED AND ENLARGED CARDIAC SILHOUETTE ON FRONTAL CHEST RADIOGRAPHY MIMICS THE 
APPEARANCE OF AN OLD‐FASHIONED WATER BOTTLE), WHICH IS SUGGESTIVE OF PERICARDIAL EFFUSION (PANEL A). TRANSTHORACIC 
ECHOCARDIOGRAPHY REVEALED A LARGE PERICARDIAL EFFUSION WITH SWINGING OF THE HEART FREELY IN AN ANTERIOR–
POSTERIOR FASHION (SEE VIDEO) AND COLLAPSE OF THE RIGHT AND LEFT ATRIA IN END DIASTOLE, WHICH IS CONSISTENT WITH 
CARDIAC TAMPONADE. LEFT VENTRICULAR SYSTOLIC FUNCTION WAS MODERATELY IMPAIRED. SUCH CHANGES IN THE POSITION OF 
THE HEART RESULT IN BEAT‐TO‐BEAT VARIATION IN BOTH THE AMPLITUDE AND THE AXIS OF THE QRS COMPLEX (ELECTRICAL 
ALTERNANS) SEEN ON ELECTROCARDIOGRAPHY (PANEL B, ARROWS). PERICARDIOCENTESIS RESULTED IN DRAINAGE OF 1.3 LITERS OF 
BLOODY FLUID, WHICH WAS SUBSEQUENTLY ATTRIBUTED TO A MALIGNANT EFFUSION. REPEAT ECHOCARDIOGRAPHY OVER THE NEXT 
6 HOURS REVEALED REACCUMULATION OF EFFUSION. THE PATIENT SUBSEQUENTLY HAD CARDIAC ARREST AND DIED. 
 
 

 
 
 
 
 
 
TAPONAMIENTO CARDIACO 
 
PATOGENIA 
Aumento de presión intrapericárdica, bajo gasto cardiaco, restricción de llenado. Compensación pr SRAA. (aumento de resistencias 
vasculares periféricas) ¿? 
 
DIAGNÓSTICO 
Tríada clásica: hipotensión arterial + pulso paradójico + ingurgitación yugular.   
 
Hipotensión arterial: Si shock. Si hay respuesta adrenérgica compensadora ==> hipertensión 
Pulso paradóiico: Caída >10mmgHg con inspiración. 
Ingurgitación yugular: signos de presión venosa elevada con datos de IC derecha.   
 
Oliguria (IR por bajo GC). 
Frialdad, cianosis acra (aumento de RVP) 
Taquicardia (compensa bajo GC) 
Fiebre (si sepsis) 
Ruidos apagados, roce pericárdico. 
 
EKG: 
  Alternancia eléctrica QRS (2:1) [PONER IMAGEN QUE ENVIÉ A GIGI] 
  Bajo voltaje de QRS   
  Taquicardia 
  Disociación eletromecánica   
Tele de tórax: 
  cardiomegalia cuando >250ml, con silueta en garrafa 
ECOCardioTranstorácico 
  Derrame pericárdico 
  Colapso de cavidades (más las derechas) 
  Variaciones respiratorias exageradas de flijos AV y venas suprahepáticas (seno Y) 
  Vena cava inferior dilatada con abolición del colapso 
ECOCardioTrasnesofágico 
  útiles en ocasines, derrames ocultos, mets, coágulos. 
 
 
TRATAMIENTO 
Drenaje pericárdico, PEEP en intubados.   
 

 
TRAUMA ABDOMINAL 
TRAUMA ABDOMINAL CERRADO Y ABIERTO 
PATOGENIA 
Trauma es el intermbio de energía entre un objeto externo y un organismo. Daño tisular proporcional a la energía intercambiada.   
Trauma cerrado==> vísceras sólidas 
Trauma penetrante==> visceras huecas 
 
DIAGNÓSTICO 
A    Vía aérea 
B    ventilación 
C    circulación 
D  Déficit neurológico 
E  Examen completo desnudo 
 
Indicación de laparotomía: 
‐Inestabilidad hemodinámica: paciente hipotenso que no responde a volumen o que respondión y se inestabilizó nuevamente.   
Laparotomía==> hemostasia. 
‐Hematemesis o enterorragia==> lesión intestinal 
‐Herida transabdominal x arma de fuego==>95% tendrá lesión visceral 
‐Signos de irritación peritoneal 
‐Evisceración (excepto epiplón) 
 
‐Trauma abdominal cerrado con hipotensión y evidencia de hemorragia intraperitoneal 
‐Trauma abdominal cerrado con LPD o ECO positivos. 
‐Hipotensión con herida penetrante 
‐Hemorragia de esómago, recto, aparato GU x trauma penetrante 
‐Neumoperitoneo, aire en retroperitoneo, ruptura de diafragma. 
 
 
Trauma cerrado: impacto lateral del chofer (trauma hepático), lado del acompañante (trauma esplénico). 
Equimosis periumbilical (Signo de Cullen), en flanco (signo de Turner) ==> hemorragia retroperitoneal.   
Se debe hacer examen rectal ¿Próstata alta o no palpable? ¿sangre? 
 
 
 
 
Rx de pelvis: solo en paciente politraumatizado o glasgow <12 o falta de puesta a estímul doloroso. 
Lavado peritoneal diagnóstico: hipotenso, taquicardia, anemia inexpliada, examen dudoso herido con punzocortante, examen físico no 
valorable por TCE o déficit neurológico, examen físico dudoso por facturas costales, pelvicas, trauma de columna.   
  Contraindicaciones absolutas: 
    Indicación de lapartotomía 
    Inestabilidad hemodinámica 
    Evisceración 
    Abdmen agudo 
    Neumoperitoneo   
    Heridas de diafragma 
    Lesiones de tubo digestivo en tomo 
USG: búsqueda de líquido en cavidad pericárdica, espacio de Morison (espacio hepatorrenal), receso esplenorrenal, saco de Douglas. 
Es de primera elección frene a TAC    y LPD. 
TAC: solo en pacientes estables, valora bien retroperitoneo a diferencia de LPD. 

 
 
 
 
 
[TRAUMA ABDOMINAL] 
MUJER DE 24 AÑOS DE EDAD. TRAS TIROTEO QUE SE PRESENTA POR LAS CALLES ES ALCANZADA POR PROYECTIL DE ARMA DE FUEGO 
EN REGIÓN ABDOMINAL ES LLEVADA POR FAMILIARES AL SERVICIO DE URGENCIAS. LEVEMENTE ANSIOSA, COOPERADORA. RESPONDE 
VERBALMENTE A LA EXPLORACIÓN FÍSICA CON HERIDA EN REGIÓN ABDOMINAL (MESOGASTRIO) CON ORIFICIO DE ENTRADA, SIN 
ORIFICIO DE SALIDA, EN EL TRAYECTO CON EVIDENTE PENETRACIÓN A LA FASCIA. AUSENCIA DE RUIDOS INTESTINALES, ABDOMEN 
BLANDO DOLOROSO A LA PALPACIÓN PROFUNDA, REBOTE NEGATIVO, SIGNOS VITALES TA 100/60, FR 22, FC 89. 
¿SEGÚN LAS CONDICIONES DEL PACIENTE, CUÁL ES EL MANEJO ADECUADO? 
A) FAST 
B) LAPAROTOMÍA EXPLORADORA 
C) RX DE ABDOMEN 
D) TAC ABDOMINAL 
 
RESPUESTA B 
En este tipo de heridas está indicada laparotomía temprana, ya que cuando existe pentración a peritoneo la incidencia de una lesión 
intraperitoneal se acerca al 98%.   
Lo órganos más afectados en heridas por arma de fuego son: 1) intestino delgado, 2) colon, 3)hígado, 4) estructuras vasculares 
abdominales.   
Sin importar la estabilidad hemondinámica del paciente, o que no haya signos de irritación peritoneal, debe someter al paciente a 
laparotomía.   
 
 
 
MUJER DE 48 AÑOS DE EDAD, LLEGA A SERVICIO DE URGENCIAS TRAS SUFRIR ACCIDENTE POR DESACELERACIÓN EN VEHÍCULO 
AUTOMOTOR HACE 20 MINUTOS. A LA EXPLORACIÓN FÍSICA, TA 100/70, FC 94X’, TEMP 36.3, ABDOMEN DISTENDIDO, HIPERESTESIAS E 
HIPERBARALGESIAS PRESENTES, DOLOR A LA PALPACIÓN MEDIA Y PROFUNDA, REBOTE POSITIVO. SE SOSPECHA TRAUMA ABDOMINAL 
CERRADO CON PROBABLE HEMORRAGÍA ABDOMINAL. 
PARA CONFIRMAR EL DIAGNÓSTICO DEBERÁ REALIZARSE DE PRIMERA ELECCIÓN: 
   
A) LAVADO PERITONEAL DIAGNÓSTICO 
B) ULTRASONIDO ABDOMINAL. 
C) TAC HELICOIDAL. 
D) LAPARATOMÍA EXPLORADORA 
 
RESPUESTA B 
 
El USG es el método de elección para valorar trauma abdominal ya que es rápido y preciso para la determinación de líquido o sangre 
intraabdominal. El paciente se encuentra estable, sin datos de choque. La presencia de líquido por USG tras un trauma abdominal sea 
cual sea su origen y características, obligará a realizar una exploración quirúrgica en la mayoría de los casos sin necesidad de lavado 
peritoneal.     
Si bien es cierto es lavado peritoneal es rápido, de bajo costo, muy sensible para determinar si hay sangre en peritoneo, pero tiene la 
desventaja de que no localiza el sitio de lesión y que es invasivo.   
La TAC helicoidal no se usa de primera elección por ser de alto costo, tarda e implica el traslado del paiente.   
[Según curso de gigi, si es un trauma cerrado está indicado USG FAST    si hay datos que sugieren ser positivo y el paciente está estable 
se hace una TAC;    pero si está inestable se hace laparotomía]. 
La LAPE tiene 3 indicaciones: 
  Hemorragia intraabdominal en curso 
  Presencia de lesiones que se asocien a lesiones intrabdominales (ej. penetración a peritoneo). 
  Trauma cerrado con peritonitis (poco común) 
¿Qué hacer luego de un 
 
 
 
HOMBRE DE 43 AÑOS DE EDAD, INGRESA A QUIRÓFANO PARA LAPARATOMÍA DE URGENCIA POR TRAUMA ABDOMINAL CERRADO Y 
CHOQUE HIPOVOLÉMICO. DURANTE EL PROCEDIMIENTO DE ENCUENTRA HEMORRAGIA ACTIVA PROVENIENTE DE HÍGADO, SE 
PROCEDE A COLOCACIÓN DE APÓSITO PERIHEPÁTICO CON COMPRESIÓN MANUAL. 
 
  SI LA HEMORRAGIA HEPÁTICA PERSISTE A PESAR DE LA COMPRESIÓN MANUAL ESTÁ INDICADO: 
A) EL PINZAMIENTO DEL PEDÍCULO HEPÁTICO. 
B) LA LIGADURA CON CLIPS DE LA VENA HEPÁTICA. 
C) LA COLOCACIÓN DE ESPONJAS CON TROMBINA. 
D) EL TAPONAMIENTO ABDOMINAL. 
 
Control de hemorragia hepática: 1) compresión manual, apósito perihepático, ==> 2)maniobra de Pringle o pinzamiento de pedículo 
hepático). 
RESPUESTA A 
 
 
HOMBRE DE 24 AÑOS DE EDAD, QUE DURANTE UNA RIÑA RESULTA HERIDO POR PROYECTIL DE ARMA DE FUEGO EN EL ABDOMEN. SE 
DESCONOCE CALIBRE Y TIPO DE ARMA. ES LLEVADO INMEDIATAMENTE AL SERVICIO DE URGENCIAS DEL HOSPITAL MÁS CERCANO, EL 
CUAL SE ENCUENTRA A 30 MINUTOS DEL LUGAR. A SU LLEGADA SE INGRESA DE INMEDIATO A QUIRÓFANO PARA CIRUGÍA DE CONTROL 
DE DAÑOS.   
 
CONFORMAN LA “TRIADA LETAL” EN PACIENTES CON ESTE TIPO DE HERIDAS: 
A) INESTABILIDAD HEMODINÁMICA, HIPOTERMIA Y ACIDOSIS 
B) SEPSIS, COAGULOPATÍA Y FIEBRE 
C) ACIDOSIS, HIPOTERMIA Y COAGULOPATÍA 
D)HEMORRAGIA, INESTABILIDAD HEMODINÁMICA E HIPOTENSIÓN   
 
LO MÁS PROBABLE ES QUE LA CIRUGÍA DE CONTROL DE DAÑOS FUE INDICADA DE INMEDIATO PORQUE A SU INGRESO EL PACIENTE 
PRESENTÓ: 
A) FRACTURA INCOMPLETA DEL ANILLO PELVIANO 
B) ESTABILIDAD HEMODINÁMICA 
C) ORIFICIO EN EL CUADRANTE SUPERIOR DERECHO 
D) SÍNDROME COMPARTIMENTAL ABDOMINAL   
 
La tríada de la muerte es: acidosis, hipotermia y coagulopatía.   
Criterios para someter a un paciente a cirugía de control de daños: 
  Déficit de base <‐6 
  Lactato sérico >2.5mEq(L 
  Trayecyto transabdominal del misil 
  Ubicación del orificio de entrada o de salida en cuadrante superior derecho 
 
RESPUEESTA C, C 
 
 
HOMBRE DE 32 AÑOS DE EDAD. ACUDE A URGENCIAS TRAS SE IMPACTADO CON AUTOMÓVIL; SE ENCUENTRA ESTABLE, AL COLOCAR 
LA SONDA VESICAL SE OBSERVA FRANCA HEMATURIA. 
 
DE ACUERDO A LO VALORADO, ¿QUÉ ESTRUCTURA ÓSEA ES LA MÁS PROBABLEMENTE AFECTADA? 
A) RAMA ISQUIPÚBICA 
B) SACRO ILIACA 
C) ACETÁBULO 
D) CABEZA DE FÉMUR 
 
LA RADIOGRAFÍA MUESTRA FRACTURA NO DESPLAZADAS DE LA LESIÓN DIAGNOSTICADA. EL TRATAMIENTO SIGUIENTE ES:   
A) COHIBIR HEMORRAGIA 
B) OSTEOSÍNTESIS CON FIJADOR EXTERNO 
C) PRÓTESIS DE RAMA ISQUIPÚBICA 
D) CONSERVADOR, REPOSO Y REHABILITACIÓN 
 
El paciente tiene fractura isquipúbica. En pacientes con accidentes en automóvil frontales las estructuras más frecuentementeafectas 
son cabeza femoral y el acetábulo. En este caso clínico por la hematuria franca hay que pensar en la porción que cause daño a órganos 
intrapélvicos (en este    caso es lesión en uretra), por lo tanto tiene una fractura isquiopúbica. En las fracturas de cadera, lo primero a 
evaluar es la estabildiad hemodinámica, de no existir compromiso y ser una fractura no desplazada el tatamiento de elcción es 
conservador; dejando los demás para los casos complicados (ancianos, lesión de otros órganos, hemoragia activa). 
 
¿Por que no    cohibir la hemorragia? 
 
 
CHOQUE HIPOVOLÉMICO TRAUMÁTICO 
 
[CHOQUE] [r] 
MUJER INDÍGENA DE 50 AÑOS DE EDAD, HIPERTENSA Y DIABÉTICA CONTROLADA. AL SALIR DE SU CASA SUFRE CAÍDA AL IR 
CAMINANDO. POSTERIOR A LA CUAL PRESENTA DOLOR INTENSO EN PELVIS Y EXTREMIDAD INFERIOR DERECHA. EL HOSPITAL MÁS 
CERCANO QUEDA A DOS HORAS DE SU CASA, POR LO QUE A SU LLEGADA AL HOSPITAL SE REPORTAN LOS SIGUIENTES DATOS AL 
MOMENTO DE SU VALORACIÓN: FC DE 130X´, PRESIÓN ARTERIAL 90/50 MMHG, FR DE 35X´, GASTO URINARIO DURANTE EL TRASLADO 
APARENTEMENTE DISMINUIDO Y NEUROLÓGICAMENTE ANSIOSA CON TENDENCIA A LA CONFUSIÓN. 
 
  CON BASE EN LOS DATOS CLÍNICOS USTED CLASIFICARÍA EL CHOQUE DE LA PACIENTE COMO: 
A) GRADO I. 
B) GRADO II. 
C) GRADO III. 
D)GRADO IV. 
 
CON BASE EN LOS DATOS CLÍNICOS USTED CALCULA QUE LA PÉRDIDA DE SANGRE ES DE: 
A) HASTA 750 ML. 
B) DE 750 A 1500 ML. 
C) DE 1500 A 2000 ML. 
D) MÁS DE 2000 ML 
 
CON BASE EN LOS DATOS CLÍNICOS USTED ESPERARÍA QUE EL GASTO URINARIO DE LA PACIENTE SEA DE: 
A) > DE 30ML/HR. 
B) 20 A 30 ML/HR. 
C) 5 A 15 ML/HR. 
D) DESPRECIABLE. 
 
SU TRATAMIENTO DE REPOSICIÓN DE LÍQUIDOS BASADOS EN LA CLASIFICACIÓN DEL GRADO DE CHOQUE DEBERÁ CONSISTIR EN: 
A) NO REQUIERE REPOSICIÓN. 
B) EXCLUSIVAMENTE CRISTALOIDES. 
C) CRISTALOIDES MÁS TRANSFUSIÓN DE SANGRE. 
D) TRANSFUSIÓN DE SANGRE. 
 
RESPUESTA C,C,C,C 
En fracturas de fémur la principal causa de muerte es la hemorragia incontrolable. 
 
Grado  ml pérdida  % pérdida  FC  FR  TA  Presión de pulso*  GU ml/hr  Estado mental  Tratamiento 
I  <750  15  <100  14‐20  Normal  Normal‐  >30  Ligeramnete  No requiere reposición, llenado 
aumentada  ansioso  capilar se restaura en 24 hrs o 
Sol fisiológica/Hartman 
II  750‐1500  15‐30  <100  20‐30  Normal  Disminuida  20‐30  Medianamente  Cristaloides** 
ansioso 
III  1500‐2000  30‐40  >120  30‐40  Baja  Disminuida  5‐15  Ansioso,  Catéter corto y ancho + tipar y 
confuso  cruzar + cristaloides*** 
IV  >2000  >40  >140  >35  Muy  Disminuida  <5  Confuso  Sangre O‐, inmediatamente 
baja  letárgico.  después Sol salina 
 
*La presión de pulso disminuye ya que la diástole aumenta por efecto de las catecolaminas. 
**Esporádicamente transfusión 
***En ese órden 
 
Choque III= IC a UCI 
Choque II‐III Venas permeables en ambas extremidades + catéter venoso central 
 
 

 
INSUFICIENCIA ARTERIAL AGUDA 
[Resumen la Salle] 
DEFINICIÓN 
 
 

 
 
¿Cuál es su impresión diagnóstica en este caso? 
a) Insuficiencia arterial aguda 
b) Insuficiencia arterial crónica 
c) Trombosis venosa profunda 
d) Celulitis de miembro pélvico izquierdo 
 
En el caso de este paciente, ¿Cuál es el factor de riesgo más importante para el desarrollo de su patología actual? 
a) Diabetes mellitus descontrolada 
b) Dislipidemia mixta 
c) Fibrilación auricular 
d) Obesidad 
El tratamiento indicado para esta patología es: 
a) Anticoagulación oral 
b) Antiagregación plaquetaria 
c) Quirúrgico a la brevedad 
d) Control de factores de riesgo. 
 
¿Cuál es la complicación sistémica esperada más frecuente una vez practicado el tratamiento? 
a) Síndrome compartimental 
b) Requerimiento de amputación 
c) Síndrome de respuesta inflamatoria sistémica 
d) Síndrome de reperfusión 
Respuestas a, c, c 
 
¿Cuál es el INR esperado en un paciente con fibrilación auricular manejado con acenocumarina? 
2‐3 
 
DEFINICIÓN 
Interrupción abrupta de irrigación 
 
EPIDEMIOLOGÍA 
Incidencia 14:1000,000 para pélvicos y 2.4:100,000 para torácicos 
 
ETIOLOGÍA 
Embolismo de origen cardiaco el más común 
Trombosis in situ (ateroesclerosis) 
Traumatismo (se secciona o se estira) 
 
En el caso de los miembros pélvicos el 41% es por trombosis, 38% por embolismo. 
En el caso de miembros escapulares el 90% es por emboismo. 
 
CLÍNICA   
Pain 
Poiquilotermia 
Palidez 
Pérdida de pulso 
Parestesias* 
Paralisis* 
 
*Signos de irreversibilidad 
 
DIAGNÓSTICO 
Clínico 
USG doppler 
Angiografía 
Angiotmografía 
Estos estudios se hacen en caso de duda, no se justifica retrasar tratamiento por hacer estudios. 
 
 
Clasificación de Rutherford 
Categoría  Descripción  Hallazgos  Señal doppler 
    Déficit sensitivo  Déficit motor  Arterial  Venosa 
I  Viable  Ninguno  Ninguno  Audible  Audible 
II  En riesgo         
  IIa Requiere tx  Mínimo  Ninguno  Ocasionalmente  Audible 
oportuno  Distal  audible 
Iib Requiere tx  Leve‐moderado    Leve a moderado  Generalmente no  Audible 
inmediato  Proximal  audible 
III  Irreversible  Anestesia  Parálisis  Ausente  Ausente 
 
 
TRATAMIENTO 
Tenemos de 6‐8 hrs para resolver, de lo contrario habrá datos de irreversibilidad. 
Tx es quirúrgico:    Tromboembolectomía es el manejo más frecuente (catéter Fogarty) 
    Trombólisis farmacológica, sobre todo en IIA 
    Trombólisis farmacomecánica (tratamiento endovascular) 
 
 
 
ENFERMEDAD ARTERIAL PERIFÉRICA 
[Resumen la Salle] 
 
DEFINICIÓN 
Es una de las manifestaciones clínicas de la ateroesclerosis que afecta a la aorta abdominal y sus ramas terminales; se caracteriza por 
estenosis u oclusión de la luz arterial por placas de ateroma originadas en la íntima que proliferan hacia la luz arterial provocando 
cambios hemodinámicos que se traducen en disminución de la presión de perfusión y dan lugar a isquemia de los tejidos. 
 
EPIDEMIOLOGÍA 
Prevalencia pob gral 3‐10% y aumenta hasta el 20% en mayores de 70 años. Sólo el 12.5% están diagnosticados. 
 
FISIOPATOLOGÍA 
Hay varias teorías. Todas coinciden en que hay daño endotelial crónico por diversos factores: tabaquismo, hipertensión, DM, 
dislipidemia, hiperhomocisteinemia. 
 
CUADRO CLÍNICO 
Solo 25‐30% son sintomáticos, de los cuales la mayoría se presentan con claudicación intermitente y solo 1‐3% con isquemia crítica.   
Claudicación intermitente: dolor generalmente de característica punzante, opresivo o tipo calambre en un determinado grupo 
muscular que se desencadena con un grado específico de actividad física (reproducible) y que remite con el reposo en un tiempo 
constante. (TASC II:10min) 
 
Isquemia crítica: se define como el dolor severo en reposo que requiere de administración de opioides con una duración mínima de 2 
semanas (con presión en tobillo de <50mmHg o <70mmHg cuando se asocia a lesiones cutáneas) o un ITB <0.3 
A la EF se puede presentar ausencia de pulsos (no es indispensable ni patognomónica), hipotrofia de la extremidad afectada, así como 
pérdida de los anexos cutáneos (ausencia de vello). 
 
DIAGNÓSTICO 
Índice tobillo‐brazo: es una prueba diagnóstica, se recomienda su realización en pacientes de 50‐69 años asintomáticos con factores de 
riesgo y en todos los pacientes >70 años. Si es <0.9 es diagnóstico de enfermedad arterial periférica. Prueba en banda sin fin: A 3.2km/h 
con inclinación de 12° por 5 minutos, si hay disminución de >15% del ITB basal se considera diagnóstica. 
Otros estudios útiles: USG dúplex, determinaciones de presiones segmentarias, angioTAC, angioRM, angiografía. Los estudios invasivos 
se consideran en pacientes que se someten a cirugía.   
Diagnóstico diferencial: claudicación neurógena (radioculopatías, lumbociatalgia), claudicación osteomuscular, gonartrosis, coxartrosis, 
síndrome compartimental crónico, atrapamiento de la poplítea, miosisits, quiste de Baker. 
 

 
 
TRATAMIENTO 
Abandono definitivo del tabaquismo. 
Farmacológico para claudicación intermitente: cilostazol 100mg VO c 12 hrs, como alternativa pentoxifilina 400mg VO c 8 h. 
No farmacológico para claudicación intermitente: marcha metódica progresiva (duración 30‐60 min al día 3 veces por semana), 
medidas de higiene arterial. 
Prevención secundaria: ASA 80‐150mg VO c 24 hrs o clopidogrel 75mg VO c 24hrs. Estatinas. 
Iniciar antibiótico: (dicloxacilina 500mg c 6 hrx 10 días) en caso de presentar úlceras con evidencia de infección. 
Quirúrgico: revascularización abierta o endovasular está indicado en pacientes con claudicación incapacitante o isquemia crítica. 
 
PRONÓSTICO 
En claudicadores mortalidad a 5 años de 10‐15% (la mayoría por causas cardiovasculares) y hasta 10% progresarán a isquemia crítica. 
En pacientes con isquemia crítica a 1 año tienen mortalidad de 25. 30% requerirán amputación mayor. 
 
 
SE PRESENTA EN SU CENTRO DE SAUD PACIENTE MASCULINO DE 73 AÑOS DE EDAD, CUANTA CON ANTECDENTE DE DIABETES 
MELLITUS TIPO 2 E HIPERTENSIÓN ARTERIAL SISTÉMICA DE MÁS DE 10 AÑOS DE DIAGNÓSTICO EN TRATAMIENTO IRREGULAR NO 
ESPECIFICADO, ASÍ COMO TABAQUISMO INTENSO DE 40 AÑOS DE EVOLUCIÓN A RAZÓN DE 20 CIGARRILLOS DÍA.   
REFIERE QUE DESDE HACE 6 MESES PRESENTA DOLOR TIPO “CALAMBRE” EN GASTROCNEMIO DERECHO AL CAMINAR 
APROXIMADAMENTE 5 CUADRAS, DICHA SINTOMATOLOGÍA CEDE AL DETERNSE Y PERMANCER EN REPOSO POR LO MENOS 10 
MINUTOS.   
 
EL PACIENTE REIFERE QUE EL CUADRO HA INCREMENTADO PREOGRESIVAMENTE Y ACTUALMENTE SE PRESENTA AL CAMINAR 
ÚNICAMENTE DOS CUADRAS A LA EXPLORACIÓN FÍSICA USTED ENCUENTRA HIPOTROFIA DE MIEMBROS PÉLVICOS CON AUSENCIA DE 
VELLO EN 1/3 DISTAL DE TOBILLOS, DISMINUCIÓN DE PULSO POPLÍTEO Y AUSENCIA DE TIBIAL ANTERIOR, POSTEIOR Y PEDIO EN 
MIEMBROS PÉLVICO DERECHO. 
 
DE ACUERDO AL CUADRO CLÍNICO PRESENTADO, ¿CUÁL ES SU IMPRESIÓN DIAGNÓSTICA? 
A) INSUFICIENCIA ARTERIAL AGUDA 
B) INSUFICIENCIA ARTERIAL CRÓNICA 
C) INSUFICIENCIA VENOSA CRÓNICA 
D) TROMBOSIS VENOSA PROFUNDA 
 
EL DOLOR CARACTERÍSTICO QUE PRESENTA EL PACIENTE SE CONOCE COMO: 
A) DOLOR NEUROPÁTICO 
B) DOLOR ISQUÉMICO AGUDO 
C) CLAUDICACIÓN INTERMITENTE 
D) MIALGIA CRÓNICA 
 
LA CARACTERÍSTICA FISIOPATOLÓGICA EN EL DESARROLLO DE LA PATOLOGÍA PRESENTADA ES: 
A) DESARROLLO DE PLACA DE ATEROMA 
B) EMBOLISMO DE ORIGEN CARDIACO 
C) SÍNDROM DE TROUSSEAU 
D) TRÍADA DE VIRCHOW 
 
UNA VEZ REALIZADO EL DIAGNÓSTICO CLÍNICO, USTED COMO MÉDICO DE PRIMER CONTACTO DEBERÁ: 
A) REALIZAR ÍNDICE TOBILLO‐BRAZO 
B) SOLICITAR USG DÚPLEX 
C) SOLICITAR ANGIOTOMOGRAFÍA 
D) REFERIR AL SEGUNDO NIVEL 
 
De todas las preguntas quizás quedará duda de la última. Recordar que el índice tobillo brazo, el USG dúplex son pruebas de segundo 
nivel de atención. Desde el primer nivel acorde a GPC habrá que referir. La angioTAC se realiza en los que se someterán a cirugía.   
No olvidar que el índice tobillo brazo NO se hace con estetoscopio, se hace con USG. 
RESPUESTA B, C, A 
 
HOMBRE  DE  64  AÑOS  DE  EDAD,  TABAQUISMO  CRÓNICO,  DIABÉTICO  E  HIPERTENSO.  ACUDE  A  CONSULTA  POR  DOLOR  EN  LAS 
EXTREMIDADES INFERIORES. A LA EXPLORACIÓN FÍSICA FC 80 FR 19 T 37 TA 130/78, PULSOS PEDIOS, TIBIAL POSTERIOR DISMINUIDOS Y 
DOLOR DEL PIE. REALIZA PRUEBA DE ÍNDICE TOBILLO‐BRAZO.   
¿CUÁL ES EL VALOR APROXIMADA QUE PROBABLMENTE TENDRÁ EL PACIENTE EN ESTA PRUEBA? 
A) 1.2 
B) 1.5 
C) 0.9 
D) 0.6 
 
RESPUESTA D 
El  paciente  tiene  enfermedad  arterial  periférica.  La  prevalencia  de  enfermeda  arterial  periférica  aumenta  en  pacientes  >70  años, 
pacientes entre 50‐69 años con historia de tabaquismo o diabetes,    como en    este paciente . Se concen otros factores de riesgo como 
historia familiar de ateroesclerosis, sexo masculino, hipertensión, hiperlipidemia, entre otros. Hay una pruba simple que se puede realizar 
para orientar el diagnóstico de esta entidad, y se le llama índice tobillo brazo, el cual es la presión sistólica del tobillo dividido entre la 
presión sistólica braquial.    El valor normal es >0.9 por lo que en este caso encontraremos valores menores a 0.9. Valores mayores a 1.3 
sugieren la presencia de vasos calcificados por lo que se necesitarán estudios adicionales.   
 
 
 
SÍNDROME DE REPERFUSIÓN 
[Resumen la Salle] 
 
DEFINICIÓN 
Conjunto de signos y síntomas sistémicos debidos a la interrupción súbita en la irrigación en un órgano o extremidad y su posterior 
restablecimiento.   
 
FISIOPATOLOGÍA 
  
   
CUADRO CLÍNICO 
Puede presentarse con inicio súbito posterior al restablecimiento de la irrigación a un órgano o tejido y se caracteriza por: edema 
tisular (incluido edema cerebral), lesión renal aguda, rabdomiólisis, síndrome de distres respiratorio agudo, síndrome compartimental 
hasta arritmias cardíacas, traslocación bacteriana y falla orgánica múltiple. Cabe destacar que la gravedad y las manifestaciones clínicas 
dependen del órgano afectado, en el caso de extremidades existe relación con la cantidad de músculo afectado 
 
DIAGNÓSTICO 
Se debe sospechar en todo paciente con isquemia en el que se reperefunde el órgano o extremidad afectada Solicitar QS; ES, EGO, GA 
en los que se pueden encontrar elevación de creatinina (lesión renal aguda), elevación de enzimas musculares (CPK), elevación de DHL, 
hiperkalemia, mioglobinuria, acidosis metabólica. En el caso de encontrar las enzimas musculares elevadas >5‐10veces de su límite 
superior se corrobra el diagnóstico. Pueden encontrase arritmias cardíacas (TV, FA) en el EKG. 
 
TRATAMIENTO 
Se han experimentado con    múltiples alternativas desde barredores de radicales libres (ej manitol), inhibidores de la xantina oxidasa 
(alopurinol), antioxidantes (vitamina E y C), sin embargo, hasta el momento no se encuentra con un grado de evidencia que suporte su 
utilidad. Por lo que se limita el manejo de soporte y la corrección de las alteraciones (falla renal: hidratación con cristaloides, 
rabdomiólisis: cristaloides, diurético, alcalinización de la orina y en casos severos hemodiafiltración, SDRA: oxígeno suplementario o 
ventilación mecánica).   
 
El alopurinol y el manitol tienen un nivel de evidencia III (Recomendación por expertos) 
 
PRONÓSTICO 
Está ligado a la causa inicial de la isquemia, así como, el órgano afectado, puede tener mortalidad >50% en caso de miembros pélvicos. 
 
 
MASCULINO DE 72 AÑOS, POSTOPERADO EN SU DÍA UNO DE EMBOLECTOMÍA DERECHA SECUNDARIA A INSFUCIENCIA ARTERIAL 
AGUDA QUE SE LLEVA A CABO SIN COMPLICACIONES NI INCIDENTES. AL MOMENTO DEL PASE DE VISITA USTED ENCUENTRA QUE EL 
PACIENTE PRESENTA EDEMA CON GODETTE +++ DEL MIEMBRO PÉLVICO AFECTADO, ASÍ COMO DOLOR INTENSO A LA PALPACIÓN, LOS 
PULSOS PERIFÉRICOS SE ENCUENTRAN PRESENTES; CALCULA UNA URESIS HORARIA DE 0.25ML/KG/H Y OBSERVA VENTILACIONES 
RÁPIDAS Y SUPERFICIALES. 
 
DE ACUERDO A LOS DATOS CLÍNICOS PROPORCIONADOS, ¿CUÁL SERÍA SU IMPRESIÓN DIAGNÓSTICA EN ESTE CASO? 
A) TROMBOSIS VENOSA PROFUNDA 
B) SÍNDROME DE REPERFUSIÓN 
C) INFECCIÓN DE TEJIDOS BLANDOS 
D) NUEVO EVENTO DE EMBOLIZACIÓN ARTERIAL 
 
¿QUÉ EXÁMENES DE LABORATORIO Y/O GABINETE SOLICITARÍA EN ESTE CASO PARA INCIAR SU PROTOCOLO DIAGNÓSTICO? 
A) ARTERIOGRAFÍA DE MEIBMRO PÉLVICO DERECHO 
B) USG DOPPLER VENOSO MPD 
C) QS, ES, EGO, GA 
D) BH, VSG, PCR 
 
¿CUÁL ES EL FACTOR QUE ESPERARÍA ENCONTRAR PARA CORROBORAR SU SOSPECHA DIAGNÓSTICA? 
A) INCREMENTO DE LA CREATININA SÉRICA >0.3MG/DL EN LAS SIGUIENTES 48 HRS 
B) ACIDOSIS METABÓLICA COMENTASADA CON ALCALOSIS RESPIRATORIA 
C) FIBRILACIÓN AURTICULAR DE RECIENTE INICIO 
D) ENZIMAS MUSCULARES ELEVADAS >5‐10 VEECES DE SU LÍMITE SUPERIOR 
 
EL TRATAMIENTO DE ELECCIÓN EN ESTE CASO SERÍA: 
A) NUEVA EMBOLECTOMÍA ARTERIAL DE MIEMBRO PÉLVICO CERECHO 
B) ANTICOAGULACIÓN ORAL 
C) ANTIBIOTICOTERAPIA DE AMPLIO ESPECTRO 
D) MEDIDAS DE SOSTÉN COMO SOLUCIONES CRISTALOIDES PARA MANTENER UN BALANCE HÍDRICO POSITIVO Y 
OXIGENOTERAPIA.   
 
RESPUESTA B, C, D, D 
 
SÍNDROME COMPARTIMENTAL 
[Resumen la Salle] 
 
DEFINICIÓN 
Conjunto de signos y síntomas ocasionados por la alteración en la perfusión debido al aumento en la presión en un compartimento del 
cuerpo. 
 
FISIOPATOLOGÍA 
La presión de perfusión en las extremidades está determinada por la diferencia en la presión hidrostática capilar y la presión 
hidrostática en las vénulas poscapilares; al presentarse un incremento en la presión intracompartimental (normal 10‐12mmHg), ya sea 
por el aumento de volumen de su contenido o reducción del volumen del compartimento,    se compromete    inicialmente el flujo en 
las vénulas y vasos linfáticos ocasionando edema, posteirormente se compromete la perfusión arteriolar y con ello la perfusión de los 
tejidos dentro del compartimiento, presentándose isquemia. Por últmo, la presión comprime las arterias de quepeño calibres y medio, 
conasionado isquemia completa del segmento. 
 
ETIOLOGÍA 
Existen múltiples causas del síndrome compartimental: vasculares (síndrome de isquemia‐reperfusión, trauma arterial y venoso, 
flegmasía cerúlea dolens). Hemorrágicas no vasculares (fracturas, lesiones por aplastamiento por extravasación de fluidos, 
inmovilización (aparatos de yeso), en paciente crítico con SIRS y reanimación hídrica masiva.   
 
CUADRO CLÍNICO 
Dolor desproporcionado de la magnitud de la lesión (generalmente a la movilización pasiva o compresión), aumento de volumen de la 
extremidad, aumento de la tensión del compartimento, hipoestesias, parestesias y ausencia de pulsos.   
Es importante destacar que la sintomatología puede ser inicialmente insidiosa y posteriormente rápidamente progresiva, se debe tener 
en cuenta que en pacientes con alteraciones del estado de conciencia o con alteraciones neurológicas (sensitivas o motoras) es difícil 
que nos refieren síntomas tempranos. 
 
DIAGNÓSTICO 
Para el diagnóstico se debe tener un alto índice de sospecha por antecedente y que a la exploración física tiene una senbilidad de 13‐
19% con VPP 11‐15% y especificidad de 97, VPN 98%, es más útil para descartar que para confirmar el diagnóstico. Sin embargo la 
presencia de 3 signos clínicos puede incrementar la probabilidad de diagnóstico hasta 93%. En caso de pacientes pediátricos con 
alteraciones neurológicas es de utilidad la medición de la presión intracompartimental la cual en caso de encontrarse >30mGh es 
diagnóstica. 
 
TRATAMIENTO 
Es necesario llevar a cabo fasciotomías para descompresión del compartimento afectado de manera pronta para evitar la pregresión y 
las secuelas. 
 
PACIENTE MASCULINO DE 36 AÑOS, SIN ANTECEDENTE DE IMPORTANCIA QUE PRESENTA HACE 6 HRS HERIDA POR PROYECTIL DE 
ARMA DE FUEGO EN CARA ANTEROMEDIAL DE MUSLO DERECHO, POR LO QUE ES LLEVDO A URGENCIAS DONDE SE DIAGNOSTICA 
TRUAMA VASCULAR A NIVEL DE ARTERIA Y VENO FEMORAL SUPERFICIALES, SE REALIZA TRATAMIENTO QUIRÚRGICO CON REPARACIÓN 
DE AMBOS VASOS, SIN COMPLICACIONES. A LAS 3 HRS INICIA CON EDEMA ++++ DE PIERNA, DOLOR INTERNSO, 10/10 COMPARABLE AL 
DOLOR DE INICIO ACOMPAÑADO DE PARESTESIAS TIPO HORMOGUEO. 
A LA EXPLORACIÓN FÍSICA SE ENCUENTRA CON TA 130/85MMHG, FC 98, FR 22, T 37, MIEMBRO PÉLVICO DERECHO CON PRESENCIA DE 
HERIDA QUIRÚRGICA LIMPIA, SIN DATOS DE SANGRADO, A NIVEL DE PIERNA AUMENTO DE VOLUMEN, EDEMA ++++, AUMENTO DE 
TENSIÓN DE MASAS, DOLOR INTESO A LA PALPACIÓN SUPERFICIA EN CARA ANTERIOR DE PIERNA Y A NIVEL VASCULAR CON PULSO 
FEMORAL, POPLÍTEO, TIBIAL ANTERIOR, POSTERIOR Y PEDIO 2/2, LLENADO CAPILAR 4S, FUERZA MUSCULAR 4/5 POR DOLOR, 
SENSIBILDIAD SUPERFICIAL CON PARESTESIAS EN L5‐S1. 
SE SOLICITAN EXÁMENES DE LABORATORIO ENCONTRANDO COMO RELEVANTE CPK EN 5348. 
 
¿CUÁL ES EL DIAGNÓSTICO MÁS PROBABLE EN ESTE PACIENTE? 
A) TROMBOSIS VENOSA PROFUNDA 
B) SÍNDROME COMPARTIMENTAL 
C) CÉLULITIS DE MIEMBRO PÉLVICO DERECHO 
D) LINFEDEMA DE MIEMBRO PÉLVICO DERECHO 
 
Respecto al diagnóstico estamos ante un cuadro agudo con dolor desproporcionado a la lesión. No hay datos de respuesta inflamatoria 
propiamente por lo que descartamos celulitis, descartamos linfedema por ser crónico. Descartamos TVP por que el dolor es más 
característico de Sx compartimental así como la presencia de tensión de masas. 
 
LA ETIOLOGÍA DEL SÍNDROME COMPARTIMENTAL QUE RPESENTA ESTE PACIENTE ES: 
A) FLEGMASÍA CERÚLEA DOLENS 
B) SÍNDROME DE ISQUEMIA REPERFUSIÓN 
C) INMOVILIZACIÓN PROLONGADA CON APARATO ORTOPÉDICO 
D) REANIMACIÓN HÍDRICA MASIVA 
 
Todas son causas de Sx compartimental pero el antecedente de cirugía previa de revascularización y CPK elevada, nos hace pensar que 
hay un síndrome de isquemia‐reperfusión 
 
EL DIAGNÓSTICO DE ESTE PACIENTE SE REALIZA POR MEDIO DE: 
A) USG 
B) ARTERIOGRAFÍA   
C) CLÍNICA 
D) TAC DE MEIBMRO PÉLVICO DERECHO 
 
EL TRATAMIENTO DE ELECCIÓN EN EL CASO PRESENTADO SERÁ: 
A) FASCIOTOMÍA INMEDIATA 
B) TERAPIA DESOCMPRESIVA COMPLEJA 
C) ANTCOAGULACIÓN CON HEPARINA Y CUMARÍNICO 
D) ANTIBIOTICOTERAPIA DE AMPLIO ESPECTRO 
 
 
Recordar que el linfedema generalmente es indoloro. Respecto a los estudios diagnósticos, la terapia descompresiva compleja es 
tratamiento pero del mismo linfedema. 
RESPUESTA B,B 
CRITERIOS DE AMPUTACIÓN 
[Resumen la Salle] 
 
EPIDEMIOLOGÍA 
En E.U se realizan 60 000 amputaciones mayores (por arriba del tobillo) al año.   
México 75 000 amputaciones mayores y menores de miembrosl pélvicos. 
FR DM y enferemdad arterial periférica.   
Los objetivos de amputar: retirar todo el tejido infectado, necrosado o isquémico preservado la mayor longitud funcional del miembro 
afectado; es importante evitar las amputaciones repetidas y buscar cicatrización no complicada del muñón. 
 
INDICACIONES DE AMPUTACIÓN:   
Isquemia aguda:   
  Presente cuando hay cambios irreversibles (anestesia, parálisis, rigidez) 
  Cuando no hay opciones de revascularización 
  Posterior a intento fallido de revascularización 
  Isquemia crónica crítica 
  Posterior a múltiples intentos fallidos de revascularización 
  Cuando no hay opciones de revascularización 
  Cuando se presentan comorbilidades seberas que ponen en riesgo la vida. 
  Estatus funcoinal paurpérrimo. 
  Expectativa de vida limitada 
  Gangrena extensa o infección en las cuales no es posible el salvamiento de la extremidad. 
  Infección ascociada a DM2 
  Infección extensa qu eimpide salvamiento 
  Infección extensa asociada a sepsis 
  Lesiones traumáticas extensas   
  Tumores malignos óseos o de tejidos blandos 
  Escala MESS (Mangled Extremity Severity Score) >7 indica amputación 
 
 
Determinación del nivel de amputación 
Siempre se debe considerar preservar la mayor longitud del miembro que sea funcional.   
Si la causa es por isquemia deberá realizars a el nivel hsta donde se presenten pulsos o temperatura de la extremidad, es decir si 
presenta pulso femoral puede realziar a nivel supracondíleo, si presenta pulso poplíteo a nivel infracondíleo, si presenta pulsos distales 
puede hacerse amputaciones menores (dedos, mtransmetatarsiana, , tarso‐metatarsiana [Lisfranc], transtarsal [Chopart]). En ocasiones 
pueden realziars eáun en ausencia de pulsos cuando se tiene una determinación de oxígeno transcutánea >80% con lo que la 
probabilidad de cierre del muñón es mayor.   
 
Si la causa es infecciosa deberá retirarse todo el tejido infetado, es importante considerar que por la respuesta a la infección los tejidos 
circundantes pueden presentarse edematosos, lo cual dificulta el cierre primario. Puede decidirse el manejo en 2 tiempos para 3evitar 
absceso de muñón, esto es, realziar amputación retirando, todo el tejido afectado y diferir el cierre para un segundo momento cuando 
la infección se encuentre controlada.   
 
 
PACIENTE MASCULINO DE 69 AÑOS DE EDAD, CON ANTECEDENTE DE HAS, DM2 DE MÁS DE 20 AÑOS DE DIAGNÓSTICO Y TABAQUISMO 
DE LARGA EVOLUCIÓN, QUIEN ACUDE AL SERVICIO DE URGENCIAS POR PRESENTAR CUADRO CLÍNICO QUE INICIÓ HACE 24 HRS CON 
DOLOR INTENSO, INCAPACITANTE, EN MIEMBRO PÉLVICO DERECHO A PARTIR DEL TERCIO INFERIOR DEL MUSLO, ASÍ COMO 
HIPOTERMINA, PALIDEZ EN REGIÓN MENCIONADA, AUSENCIA DE SENSIBILIDAD Y MOVILIDAD. 
A LA EF MIEMBRO PÉLVICO DERECHO, USTED CORROBORAR QUE EL PACIENTE CURSA CON POIQUILOTERMIA, PULSO FEMORAL 2/2, 
POPLITEO, TIBIAL ANTERIOR, POSTERIOR Y PEDIO AUSENTES, COLORACIÓN MARMOREA DE LA PIERNA, ANTESTESIA A NIVEL DE L4 Y S1 
Y FUERZA MUSCULAR 0/5 EN ESCALA DE DANIELS, ASÍ COMO LLEGADO CAPILAR DE >6S   
 
EL CUADRO CLÍNICO PRESENTADO CORRESPONDE A: 
A) INSUFICIENCIA ARTERIAL AGUDA CON EXTREMIDAD VIABLE 
B) INSUFICIENCIA ARTERIAL AGUDA CON ISQUEMIA IRREVERSIBLE 
C) INSUFICIENCIA ARTERIAL CRÓNICA 
D) SEPSIS POR INFECCIÓN DE TEJIDOS BLANDOS 
 
Hay que recordar las 6P de insuficiencia arterial aguda, los datos de irreversibildiad son parestesias (o anestesia) y parálisis. También el 
tiempo de ventana para revascularizar es de 6‐8 hrs, razón por la cual ya no es viable.   
 
DE ACUERDO AL CUADRO CLÍNICO MENCIONADO Y UNA VEZ CORROBORADA LA EXISTENCIA DE OCLUSIÓN A NIVEL DEL CANAL DE LOS 
ADUCTORES (CANAL DE HUNTER) DERECHA, SU CONDUCTA TERAPÉUTICA SERÁ: 
A) ANTICOAGULACIÓN ORAL 
B) ANTIBIOTICOTERAPIA DE AMPLIO ESPECTRO 
C) EMBOLECTOMÍA FEMOROPOPLÍTEA 
D) AMPUTACIÓN 
 
Si no está viable se somete a amputación. Si se intentar revascularizar a parte de no ser exitosa, aumentaría el riesgo de complicaciones 
como Sx de reperfusión severo y sepsis.   
 
A QUÉ NIVEL SE REALIZARÁ LA AMPUTACIÓN EN ESTE CASO PARTICULAR: 
A) SUPRACONDILEA 
B) INFRACONDILEA 
C) DESARTICULACIÓN DE LA EXTREMIDAD 
D) AMPUTACIÓN DE SYME 
 
Si el pulso más distal que se palpa es el femoral se hace amputación supracondilea, si el más distal en palparse es poplíteo es 
infracondíela.   
 
RESPUESTA B, D, A 
 
 
ENFERMEDAD CAROTÍDEA 
[Resumen la Salle] 
 
DEFINICIÓN 
Es una manifestación de la enferemdad ateroesclerosa que afecta a las carótidas, que puede progresar a la complicación (trombosis o 
hemorragia) dando lugar a eventos isquémicos cerebrales transitorios o irreversibles. 
 
EPIDEMIOLOGÍA 
EVC es la 2da causa    a nivel mundial, primera de discapacidad en EU. Aumenta con si >60 años, se duplica el riesgo por cada década 
después de los 55a.   
 
FISIOPATOLOGÍA 
Al igual que otras enferemdadesde vaculares la lesión endotelial es el principal mecanismo. Se generan células espumosas, que 
fagocitan LDL oxidasas, producen estado inflamatorio y placa fibrolipídica. La placa prolifera y obstruye.    Es frecuente encontrar dichas 
placas en la bifurcación de la carótida interna y la externa.   
Si la placa se rompe se acumulan depóstios plaquetarios.   
 
CUADRO CLÍNICO 
Es responsable del 16‐20% de todos los EVC. 
Es responsable del 50% de todos los EVC isquémicos secundarios a embolismos.   
Asíntomática en varias ocasiones y solo diagnosticada con USG doppler en >65 años.   
Se puede presentar como AIT u EVC. 
Cuando ocurre un EVC por enferemdad carotíeda se afecta la arteria cerebral media. 
  Oclusión de ACM: alt motoras (hemiplegia, hemiparesia), alt sensitivas del himicuepo contralateral al afectado. 
  Alteraciones del lenguaje (disartria, disfasia, afasia) en caso de afección del hemisferio dominante (80% es el derecho). 
  Alteraciones visuales (amaurosis fugax) 
 
DIAGNÓSTICO 
Soplo en triángulo carotídeo. 
Estudio inicial si clínica compatible o si >65a+>2FR CV ==> USG dúplex 
Estudios confirmatorios y prequirúrgicos son: angioTAC, angioRM, angiografía.a 
 
TRATAMIENTO 
Prevención secundaria: aas, estatinas, control de comorbilidades, cese de tabaquismo.   
  Tratamiento Qs: endarterectomía o angioplastia endovascular + colocación de stet en caso de estenosis críticas >0% en sintomáticos, 
>60% en asintomáticos.   
 
MASCULINO DE 75 AÑOS, QUIEN CUENTA CON ANTECEDENTE DE DIABETES MELLITUS TIPO 2, HAS, DISLIPIDEMIA MIXTA DEMÁS DE 20 
AÑOS DE DIAGNÓSTICO EN TRATAMIENTO IRREGULAR CON METFORMINA, INSULINA NPH Y AMLODIPINO.    INICIA SU PADECIMIENTO 
ACTUAL HACE 4 HRS DE FORMA SÚBITA CON DIARTRIA, POSTERIORMENTE SE AGREGA HEMIPARESIA FACIOCORPORAL DERECHA Y 
PARESTESIAS EN LA MISMA LOCALIZACIÓN, POR LO QUE ES LLEVADO A URGENCIAS.   
A LA EF DIRIGIDA ENCUENTRA SOPLO CAROTÍDEO IZQUIERDO, RUIDOS CARDÍACOS RÍTMICOS DE ADECUADA INTESIDAD SIN 
AGREGADOS, A LA EXPLORACIÓN NEUROLÓGICA DISARTRIA AL HABLA, RESTO DE LAS FUNCIONES MENTALES SIN ALTERACIONES, 
NERVIOS DEL CRÁNEO CON DESVIACIÓN DE LA COMISURA LABAL A LA IZQUIERDA, TONO Y TROFISMO CONSERVADO, FUERZA 
MUSCULAR 3/5 EN HEMICUERPO DERECHO, REMS 2/4 GENERALIZADOS, SIN DATOS DE LIBERACIÓN PIRAMIDAL, SENSIBILIDAD CON 
NIVEL HIPOESTÉSICO C5‐C6, L4‐S1, CEREBELO SIN ALTERACIONES Y SIN DATOS DE IRRITACIÓN MENÍNGEA.   
 
EL RIESGO DE EVENTO VASCULAR CEREBRAL POR ENFERMEDAD CAROTÍDEA ES MAYOR EN QUÉ TIPO DE PACIENTES: 
A) HOMBRES 
B) <50 AÑOS 
C) MUJERES 
D) PORTADORES DE FIBRILACIÓN AURICULAR.   
 
Efectivamente ser portador de fibrilación auricular aumenta el riesgo de embolismo pero NO por enfermedad carotídea.   
El de mayor peso es ser hombre. Predomina > 55 años, especialmente 60 años. 
¿Qué relación guarda la enfermedad carotídea con la fibrilación auricular? 
 
LOCALIZACIÓN MÁS FRECUENTE DE LAS PLACAS DE ATEROMA A NIVEL CAROTÍDEO: 
A) CARAÓTIDA EXTERNA 
B) CARÓTIDA COMÚN 
C) BIFURCACIÓN DE LA CARÓTIDA INTERNA Y EXTERNA 
D) CARÓTIDA INTERNA 
 
MECANISMO FISIOPATOLÓGICO MÁS FREUCNETE EN EL DESARROLLO DE EV POR ENFERMEDAD CAROTÍDEA: 
A) TROMBOSIS IN SITU 
B) MICROEMBOLISMO 
C) VASOCONSTRICCIÓN 
D) PLACA DE ATEROMA ESTABLE 
 
ESTUDIO INICIAL A REALIZAR EN PACINTES CON SOSPECHA DE ENFERMEDAD CAROTÍDEA: 
A) ANGIOGRAFÍA 
B) ANGIOTAC 
C) ANGIORM 
D) USG DÚPLEX 
 
RESPUESTA A, C, B, D 
 
INSUFICIENCIA VENOSA 
[Resumen la Salle] 
DEFINICIÓN 
Condición donde el sistema venoso es incapacidad de retornar la sangre, debido a anormalidades de la pared venosa y valvular que 
lleva a una obstrucción o reflujo sanguíneo en las venas.   
 
EPIDEMIOLOGÍA 
80% de la pob gral presenta algún grado de enfermedad venosa, de 25‐50% várices, 1‐2% úlceras flebo estáticas. Más frec en mujeres.   
 
FISIOLOGÍA 
Hay un sistema superficial que maneja 15‐20% ,uno profundo 70‐80%    y las venas perforantes que dirigen del superficial al profundo   
 
FISIOPATOLOGÍA 
Al dañarse las válvulas del sistema venoso o al presentar obstrucción al flujo (ej. trombosis venosa previa), el sistema de bomba 
muscular no resulta eficiente, por lo cual existe estasis y se genera hipertensión venosa, ocasionando cambios endolteliales y 
favoreciendo extravasación de líquido a nivel capilar (edema), extravasación de células y hematíes, inflamación, degradación de 
pigmentos derivados de hemoglobina (hemosiderina), acumulación de pigmentos en el TCSC    (dando color ocre a nivel del tobillo), 
estos factores producen lesion cutánea que deriva en lipodermatoesclerosis o atrofia, y por último ulceración.   
 
FACTORES DE RIESGO 
Edad, sexo femenino, gestación, antecedente familiar de IVC, ortostatismo prolongado, obesidad, sedentarismo, trabajos que 
impliquen actividad de pie o sedestación prolongada (más de 5hrs) 
 
CUADRO CLÍNICO 
Inicio incidioso, dolor, pesantez, prurito, cansancio, edema y calambres de predominio vespertino que se exacerba con el ortostatismo 
y con el calor, mejoran con decúbito y frío.    A la EF telangiectasia <1mm, venas reticulares 1‐3mm, várices >3mm, edema, 
pigmenación ocre, lipodermatoeslerosis, eccema o úlceras; es importante destacar que los cambios cutáneos inicialmente se presentan 
a nivel del maléolo medial.   
Se utiliza CEAP para su clasificación. 
 

 
 
Clase 1: telangiectasias y venas reticulares 
Clase 2: varices   
Clase 3: edema 
Clase 4: lipodermatoesclerosis + pigmentación ocre en maléolo medial 
Clase 5: úlcera cicatrizada 
Clase 6: úlcera activa 
 
DIAGNÓSTICO 
Exploración física 
Schwartz: percusión del sistema venoso superficial con transmisión distal de la onda 
Trendelemburg: evalúa reflujo superficial y competencia de perforantes. [si se llenan al levantarse fallan el superficial y las perforantes] 
Perthes: evalúa perforante y sistema venoso profundo. [la paciente debe caminar, si se vacían habla de que perforante y profundo 
funcionan bien] 
 
USG dúplex: el más usado, acorde a GPC se recomienda en pacientes candidatos a tratamiento quirúrgico. Reflujo > 1seg en venas 
superficiales de la ingle es patológico, a nivel de las perforantes >1seg. 
Pletismografía: para cambios cutáneos si várices si no se cuenta con doppler. Se usa en investigación.   
Medición invasiva de presión venosa del dorso del pie: es el estudio de mayor desempeño diagnóstico, es el ideal según GPC; pero casi 
no se usa por que los no invasivos arrojan datos suficientes generalmente.   
Flebografía: info anatómica precisa en pacientes con malformaciones anatómicas, candidatos a Qx con várices recidivantes. 
 
 
 
 
TRATAMIENTO 
Conservador: control del peso, evitar ortostatismo prolongado, realizar medidas físico‐posturales, uso de ropa cómica y calzadocómici, 
tacones <3cm, actividad física 30 min diarios, ejercicios de flexo extensión de tobillos, circunducción de pie y corregir estreñimiento.   
Soporte elástico de compresión: debe ser individualizado C2 18‐21mmHg, C3 22‐29mmHg, C4‐C6 30‐40mmHg. Se recomienda cmo 
preventivo en pacientes con actividad que condiciona ortostatismo o sedestación prolongada (5h) con compresión 22‐29mmHg.   
  Containdicaciones: enf arterial periférica con ITB <0.6 es absoluta, relativa 0.6‐0.89, dermatitis, AR en fase aguda, IC. 
      ¡Pálpale los pulsos por lo menos! 
Farmacológica: flebotónicos (evidencia baja, el de mayor evidencia es IIa y es diosmina/hesperidina 1000mg VO c 24h x 3‐4m)) y su 
indicación es únicamente alivio sintomático.   
Pentoxifilina: solo en pacientes con estadio C6 a dosis de 400mg VO c 8 hr hasta remisión de la úlcera. 
Quirúrgico: en pacientes con fracaso al tx conservador, várices complicadas o recidivantes. 
Escleroterapia (indicada si telangiectasias y venas reticulares), safeno excéresis o ablación con láser o radiofrecuencia (se indica en 
pacientes con insuficiencia venos superficial), ligadura de perforantes. 
Referir al 2do nivel: después de 6 meses con medidas de higiene vensa sin adecuada respuesta, IVC complicada (tromboflebitis, 
varicotrombosis, varicorragia, TVP), CEAP >3, várices recidivantes., 
 
¿Cuándo referir a paciente a hospital? 
 
FEMENINO DE 45 AÑOS, OCUPACIÓN AEROMOZA, QUIEN AHCE 6 MESES PRESENTA DOLOR EN AMBOS MIEMBROS PÉLVICOS, 
ACOMPAÑADO DE AUMENTO DE VOLUMEN DE AMBAS EXTREMIDADES EL CUAL ES DE PREDOMINIO VESPRTINO Y CEDE 
PARCIALMENTE AL ENCONTRASE EN DECÚBITO.   
ASÍ MISMO REFIERE QUE AL INICIO DE SU PADECIMIENTO CURSABA CON PRESENCIA DE TELANGIECTASIA A NIVEL DEL TOBILLO, SIN 
EMBARGO ESTAS HAN EVOLUCIONADO A TRAYECTOS VENOSOS TORTUOSOS Y PALPABLES QUE SE EXTIENDEN HASTA NIVEL DE LA 
RODILLA. TAMBIÉN PRESENTA CAMBIOS DE COLORACIÓN EN LA PIEL A NIVEL DEL MALÉOLO INTERNO. LA PACINETE ACUDE A 
CONSULTA EN SU PRIMER NIVEL DE ATENCIÓN.   
A LA EF SE ENCUENTRAN SIGNOS VITALES NORMALES, IMC 24, AMBIOS MIEMBROS PÉLVICOS CON EDEMA +++ EN TOBILLOS, 
TRAYTECTOS VENOSOS TORTUOSOS, PALPABLES HASTA TERCI SUPERIOR DE LA PIERNA, CON DOLOR A LA PALPACIÓN DE LOS MISMO, 
DERMATITIS OCRE EN MALÉOLO MEDIAL DE AMBAS EXTREMIDADES.   
 
¿CUÁL ES EL DIAGNÓSTICO MÁS PROBABLE EN ESTE CASO? 
A) TROMBISIS VENOSA PROFUNDA 
B) LINFEDEMA 
C) INSUFICIENCIA VENOSA CRÓNICA 
D) SECUELA POST TROMBÓTICA 
 
¿CUÁLES SON LOS FACTORES DE RIESGO EN ESTA PACIENTE PARA EL DESARROLLO DE INSUFICIENCIA VENOSA CRÓNICA? 
A) OBESIDAD 
B) SEXO 
C) SEDENTARISMO 
D) ANTECEDENTE FAMILIAR 
 
¿CUÁL ES EL MÉTODO DIAGNÓSTICO INDICADO EN ESTE CASO? 
A) PLETISMOGRAFÍA 
B) FLEBOGRAFÍA 
C) USG DÚPLEX 
D) MANIOBRA DE SCHWARTZ 
 
¿CUÁL ES EL SOPORTE ELÁSTICO DE COMPRESIÓN RECOMENDADO PARA PACIENTES EN RIESGO DE DESARROLLAR INSUFICIENCIA 
VENOSA? 
A) 30‐40MMHG 
B) 18‐21MMHG 
C) 41‐50MMHG 
D) 22‐29MMHG 
RESPUESTA C, B, C 
 
[r]MUJER DE 60 AÑOS DE EDAD QUE ACUDE A CONSULTA POR PRESENTAR EN LOS ÚLTIMOS MESES DOLOR Y PRURITO EN MIEMBROS 
INFERIORES.  LA  PACIENTE  REFIERE  QUE  EL  DOLOR  Y  EL  EDEMA  SON  DE  PREDOMINIO  VESPERTIN.  NIEGA  TRAUMA  Y  COMO  ÚNICO 
ANTECEDENTE PADECE HIPERTENSIÓN ARTERIAL SISTÉMICA DESDE HACE 10 AÑOS, EN TRATAMIENTO REGULAR. FC 89, FR 13, T 37.1, TA 
100/66. A LA EF SE OBSERVA HIPERPIGMENTACIÓN DE LA REGIÓN MALEOLAR MEDIAL SIN DATOS DE ÚLCERA. EL MÉDICO TRATANTE 
DECIDE SOLICITAR ECO DOPPLE, EL CUAL DEMIUSTRA AFECCIÓN DE VENAS PROFUNDAS, OBSTRUCCIÓN Y REFLUJO. 
¿CUÁL ES LA CLASIFICACIÓN DE NICOLAIDES DE ESTA PACIENTE? 
A) C1, EP, AP, PR‐0 
B) C0, EP, AP, PR‐0 
C) C4, EP, AP, PR‐0 
D) C6, EC, AP, PR‐0 
 
RESPUESTA C 
Los  FR  para  insuficiencia  venosa  crónica  son:  edad  avanzada,  historia  familiar  de  várices  y  profesiones  que  obliguen  a  un  estado  de 
bipedestación prolongado. La paciente cuenta con manifestaciones como pensantez, prurito y calambres musculares nocturnos que nos 
orientan al diagnóstico de esta entidad.   
La  clasificación  de  Nicolaides  es  la  refrida  en  las  GPC;  divide  hallazgos  clínicios,  etiología,  distribución  anatómica  y  condiciones 
fisiopatológicas. C4 corresponde a cambios cutáneos en este cas hiperpigmentación. Ep correspnde a etiología en este caso desconocida.   
Los hallazgos de l eco‐dppler demuestran afecci´n de vnas profundas lo cual corresponde a un AP. Respecto a su fisiopatología PR O 
correspnde a reflujo y obstrucción.   
 

 
 
individualizado C2 18‐21mmHg, C3 22‐29mmHg, C4‐C6 30‐40mmHg 
 
 
ENFERMEDAD TROMBOEMBÓLICA VENOSA 
[Resumen la Salle] 
 
DEFINICIÓN 
Es un complejo fisiopatológico que integra a la Trombosis Venosa Profunda y a la tromboembolia pulmonar. La tromboembolia 
pulmonar es la obstrucción aprcial o total de una o ambas arterias pulmoanres como consecuencia de la migración de un trombo.   
Es una de las causas de mayor morbimortalidad de pacientes con antecedente de truama, cirugía ortopédica o lesión espinal.   
El reisgo durante el embarazo es 5 veces amyor que en las pacientes no embarazadas y en el puerperio, y hasta 3er mes postparto 
puede llegar a ser 60 veces amyor, en México tEP es la 3era causa de muerte materna.   
 
FISIOPATOLOGÍA 
La cascada de la coagulación se activa cuando está presenta algún factor de la triada de Virchow, es ma´s frecuente en miembros 
pélvicos ya que la inmovildia y falla de la bomba muscular la condicionan   
En el 82% de los pacientes con TEP son consecuencia de una TVP. En 21% de los pacinetes con TvP sin sinatomatología de TEP se ha 
reportado gramagramma V/Q con alta probabilidad de TEP. 
Durante el embarazo la compresión de las venas ilíacas y la cava favorece la estasis, hay cambios protrómboticos en los factores de la 
coagulación (disminución de proteína S, aumento de fibrinógeno, V, VIII, IX; X) 
 
CUADRO CLÍNICO 
El de la TVP es (dolor, EDEMA, hipertermia, sensibilidad, fiebre, presencia de red venosa superficial así como dolor a dorsiflexión o 
compresión de la pierna). En TEP dependerá del área de las arteria pulmoanres afectada, puede ser desde asintomática: cuando es 
>50% se considera masiva y existen alteracioens hemodinámicas, hasta choque cardiogénico y la muerte. Se pueden presentar disnea, 
dolor torácico, palpitaciones, tos, hemoptisis, ortopnea, síncope, arritmias, taquicardia, hipotensión, hioxemia, sin embargo debido a 
los datos inespecíficos no se puede fundamenter dx con la EF. 
 
DIAGNÓSTICO 
Cuadro clínico con criterios de Wells para TEP no son diagnósticos pero establecen probabilidad. 
Rx de tórax: signo de Westermark (disminución de trama vascular), joroba de Hampton (radioopacidad en forma de domo con base en 
la pleura), mayor tulidad para descartar otras patologías pulmonares.   
 
 
 
 
EKG: es noramal hasta en 70%, SIQIIITIII es patognomónico pero raro, pueden encontrarse datos de sobrecarga de ventrículo derecho.   
Ondas S profundas en DI 
Ondas Q en DIII 
Onda T negativa en DIII 
GA: hipoxemia 
Dímero D, sirve para excluir. 
Ecocardiograma, gamagrama V/Q puden ser confirmatorios 
TAC helicocoidal contrastada: es estándar de oro 
Se clasifica en aguda, masiva, submasiva y no masiva y crónica.   
 
TRATAMIENTO 
Depende de la severidad: medidas de soporte (VM, aminas). 
Trombolisis o trombectomía en pacientes con TEP masiva y choque cardiogénico debido a su alta mortalidad es >50%.    Para 
submasiva y no masiva se recomienda anticoagulación iniciando con heparina convencional o HBPM y continuar con anticoagulación 
oral con INR 2‐3. En los pacientes que no pueden recibir anticoagulación que presentan TEP a pensar de la decuada anticoagulacion 
está inciado la colocación de un filtro de vena cava para prevenir nuevo evento.   
 
PACIENTE FEMENINO DE 32 AÑOS, SE ENCUENTRA EN PUIERPERIO FISIOLÓGICO MEDIATO. INICIA CON EDEMA DE MIEMBRO PÉLVICO 
DERECHO A NIVEL DE GASTROCNEMIOS, ERITEMA, AUMENTO DE TEMPERATURA EN DICHA LOCALIZACIÓN, RED VENOSA SUPERFICIAL Y 
DOLOR A LA DORSIFLEXIÓN Y A LA COMPRESIÓN DE LA PIERNA. SE DIAGNOSTICA TVP Y SE INCIA MANEJO CON HEPARINA DE BAJO 
PESO MOLECULAR A DOSIS PLENAS.   
ES EFRESADA DEL SERVICIO A DOMICILIO SIN RECIBIR TRATAMIENTO CON ANTICOAGULACIÓN ORAL. TRES DÍAS DESPUÉS LA PACIENTE 
INGRESA A URGENCIAS POR PRESENTAR DISENA DE PEQUEÑOS ESFUERZOS, DOLOR TORÁCICO DE PREDOMINIO A LA INSPIRACIÓN, 
CIANOSIS PERIBUCAL Y PAPITACIONESL. 
A LA EF SE ENCUENTRA CON SIGNOS VITALES TA 100/60, FC 109LPM, FR 24, TEMP 37, SAT O2 A AIRE AMBIENTE 75%, TORÁX ANTERIOR 
CON RUIDOS CARDÍACOS RÍTMICOS DE ADECUADA INTENSIDAD, CON FC 1110, SIN AGREGADOS, TORAX POSTERIOR CO MURMULLO 
VESICULA RPRESNETE, LIGERAMNETE DISMINUIDO BASAL DERECHO, SIN OTRAS ALTERACIONES DE IMPORTANCIA. MIEMBRO PÉLVICO   
DERECHO CON EDEMA +++, AUMENTO DE TEMPERATURA Y DOLOR A LA PALPACIÓN. SE INICIA MANEJO CON CON OXÍGENO 
SUPLEMENTARIO POR MASCARILLA,    Y SE SOLICITA DD CUYO RESULTADO REPORTA 2563. 
 
DE ACUERDO A LOS DATOS CLÍNICOS PROPORCIONADOS, ¿CUÁL SERÍA SU IMPRESIÓN DAIGNÓSTICA EN ESTE CASO? 
A) TVP 
B) TEP 
C) IAM 
D) NEUMONÍA NOSOCOMIAL 
¿CUÁL ES EL ESTUDIO DE GABINETE ESTÁNDAR ORO PARA EL DIAGNÓSTICO DE LA PATOLOGÍA PRESENTADA POR LA PACIENTE? 
A) USG DOPPLER 
B) EKG 
C) RX TÓRAX 
D) TOMOGRAFÍA HELICOIDAL CONTRASTADA. 
 
¿CUÁL ES EL SIGNO ELECTROCARDIOGRÁFICO PATOGNOMÓNICO DE LA PATOLOGÍA PRESENTADA EN ESTE CASO? 
A) ELEVACIÓN DEL SEGMENTO ST 
B) TAQUICARDIA SINUSAL 
C) SIQIIITIII 
D) INVERSIÓN DE ONDA T 
 
SE REALIZA TOMOGRAFÍA HELICOIDAL CONTRASTADA Y SE ENCUENTRA UNA TEP SUBMASIVA ¿CUÁL ES EL TRATAMIENTO DE ELECCIÓN 
PARA ESTA PACIENTE? 
A) ANTICOAGULACIÓN CON HEPARINA E INCIAR ANTICOAGULACIÓN ORAL 
B) ANTIAGREGANTE PLAQUETARIO 
C) COLOCACIÓN DE FILTRO DE VENA CADA 
D) ANTICOAGULACIÓN CON HEPARINA 
 
RESPUESTA B, D, C, B 
 
 
MUJER DE 45 AÑOS    DE EDAD CON DIAGNÓTICO DE SX ANTIFOSFOLÍPIDO, Y ANTECEDENTE DE CIRUGÍA DE MIEMBRO INFERIR DERECHO 
POR FRACTURA DE TIBIA HACE 2 SEMANAS. ACUDE A URGENCIAS POR UN CUADRO DE 2 HRS DE EVOLUCIÓN, CARACTERIZADO POR 
DISNEA SÚBITA Y DOLOR TORÁCICO. A LA EXPLORACIÓN TA 100/70 MMHG, FC 120LPM, FR 24, DISNEICA Y CON USO DE MÚSCULOS 
ACCESOREIOS DE LA RESPIRACIÓN. EN LOS EXAMENES: HIPOXEMIA E HIPOCAPNIA EN LA GASOMETRÍA, Y TAQUICARDIA SINUAL EN EKG, 
SIGNO DE WESTERMARCK CON LA RX DE TÓRAX Y ELEVACIÓN DE HEMIDIAFRAGMA DERECHO. 
¿CUÁL ES EL ESTUDIO MÁS APROPIADO EN ESTA PACIENTE? 
 
A) DÍMERO D 
B) TAC PULMONAR HELICOIDAL CON CONTRASTE 
C) ECO CARDIOGRAMA 
D) ANGIOGRAFÍA PULMONAR 
 
Clasificar al paciente con sistema Wells. 
  Probabilidad alta  TAC pulmonar helicoidal con contraste, si contraindicación grammagrafía ventilación perfusión. 
      Si ninguna de las dos son factibles: USG de extremidades 
 
No se debe usar gammagrafía si: derrame pleural o atelectasis 
Angiografía teóricamente es el gold stándar pero por ser invasiva no deberá utilizarse a menos que no se llegue al diagnóstico con los 
métodos anteriores. 
 
RESPUESTA COMENTADA B 
 
MUJER DE 50 AÑOS DE EDAD CON ANTECEDENTE DE TROMBOEMBOLIA PULMONAR HACE 2 AÑOS. ACUDE A URGENCIAS POR UN 
CUADRO QUE INICIÓ HAE 2 HORAS, CARACTERIZADO POR DISNEA SÚBITA, DOLOR TORÁCICO Y TOS. SIGNOS VITALES: TA 
100/70MMHG, FC 130LPM, FR 25 RPM, T 36.5 A LA EXPLORACIÓN FÍSICA DOLOR A LA PALPACIÓN EN PANTORRILLA IZQUIERDA. LA 
RADIOGRAFÍA DE TÓRAX MUESTRA CONDENSACIÓN PARENQUIMATOSA TRINAGULAR DE BASE PLEURAL DERECHA. 
 
¿CUÁL ES LA ALTERACIÓN ELECTROCARDIOGRÁFICA MÁS FREUCENTE EN PACIENTES CON ESTA ENFERMEDAD? 
A) TAQUICARDIA SINUAL 
B) SI, QIII, TIII 
C) DESVIACIÓN DEL EJE A LA DERECHA 
D) BLOQUEO DE RAMA DERECHA 
 
¿CUÁL ES EL ESTUDIO DE ELECCIÓN PARA REALIZAR EL DIAGNÓSTICO EN ESTA PACIENTE? 
A) ECOCARDIOGRAMA 
B) TAC HELICOIDAL CON CONTRASTE 
C) GAMMAGRAFÍA VENTILACIÓN‐PERFUSIÓN (V/Q) 
D) ANGIOGRAFÍA 
 
¿CUÁL ES EL TRATAMIENTO MÁS ADECUADO EN ESTE CASO? 
A) FIBRINÓLISIS 
B) ANTICOAGULACIÓN 
C) FILTRO DE VENA CAVA INFERIOR 
D) EMBOLECTOMÍA 
 
La pacinet tiene tromboembolia pulmonar. El diagnóstico se apoya en: antecedente de tromboembolia pulmonar, disnea súbita 
(síntomas más característicos), dolor torácico, dolor en pantorrilla (signo de Hommans), taquicardia sinusal e imagen radiográfica 
característica de joroba de Hampton. El diagnóstico diferencial se deberá realizar con: pericarditis, pleuritis, derrame pleural, 
hipertensión arterial pulmonar y neumonía; principalmente. En la pericarditis existe dolo precordial pero no es súbito y da imagen 
electrocarddiográfica característica con supradesnivel del segmento S‐T (en sonrisa); en la radiografía de tórax solo si va acompañda de 
derrame pericárido se ve aumento de la silueta cardiaca. En la pleuritis, el dolor no es subito y se reproduce a la inspiración profunda; 
en la radiografía de tórax si se acmpaña de derrame pericárdico, habrá borramiento de ángulos. En la hipertensión arterial pulmonar la 
disena es progresiva y no se acompaña de dolor precordial, sin embargo hay que considerar que la tromboembolia pulmonar es causa 
de hipetensión pulmonar aguda. El principal factor de riesgo para la tromboembolia pulmonar es el antecedente    enfermedad 
tromboembólica venosa previa; otros factores de riesgo son: cáncer, inmovilización, cirugía en los últimos 3 meses, evento 
cerebrovascular reciente,    obesidad, tabaquismo, anticonceptivos orales, síndrome antifosfolípido, resistencia a la proteína C activada 
(trombofilia hereditaria más frecuente). El 20% de los TEP son idiopáticos (sin factores de riesgo). La radiografía en una tromboembolia 
pulmonar es normal en la mayoría de los casos, aunque ocasionalmente se pueden observar infartos pulmonares, joroba de Hampton 
(condensación parenquimatosa trinagular de base peural ), signo de Westermarck (oligoemia focal que produce hiertransparencia 
pulmonar). La alteración electrocardiográfica más frecuente, es la taquicardia sinusal, aunque puede presentarse otros datos que 
sugieran hipertrofia ventricular derecha como desviación del eje hacia la derecha, bloqueo de rama derecha, SI, QIII, TIII. El estudio de 
elección para realizar el diagnóstico es la TAC helicoidal con contraste, a excepción de alergia al contraste o presencia de insuficiencia 
renal, ese caso está indicada la grammagrafía ventilación‐perfusión.   
El ecocardiograma está indicado para evaluar la gravedad de la hipetensión pulmonar, en búsqueda de disfunción      ventricula 
derecha, y en casos donde el diangóstico no quede claro o no se pueda confirmar el diagnóstico por otro estudio por contraindicacines. 
La angiografía    es el estándar de oro de la TEP, sin embargo no es el primer eestudio indicado en este caso. En en caso sde reisgo lee o 
intermedio (sin choque o hipotensión), el tratamiento más adecuado es la anticoagulación. En casos de riesgo alto se indica fibrinólsiis. 
La embolectomía se realizaen pacintes com compromiso hemodinámico grave, que no responden a fibrinólisis. El filtro de vena cada 
inferior se utiliza en pacientes con enfermed tromboemólica venosa o con alto riesgo de la misma, TEP recurent y que tengan 
contraindicación par anticoagulaicón. 
RESPUESTA A, B, B 
 
 
 
 
 
TROMBOSOSIS VENOSA PROFUNDA 
[Resumen la Salle] 
 
DEFINICIÓN 
Es parte del espectro de la enfemedad tromboembólica venosa que afecta al sistema venoso profundo de los miembros pélvicos. 
 
EPIDEMIOLOGÍA Y RELEVANCIA 
Se presenta en 1:10 000 adultos jóvenes y hasta en 1:100 adultos mayores al año. Junto con la tromboembolia pulmonar es la 3era 
causa de muerte hospitalaria y la 1era prevenible.   
 
FACTORES DE RIESGO 
Edad >40 años, hospitalizado, cirugía reciente o trauma son los de mayor riesgo. Neoplasias, inmovilización, embarazo, puerperio, AO e 
inyectados, procedimientos invasivos vaculares (catéteres), antecedentes de TVP. Trombofilia: La más frecuente en la polbación es la 
deficiencia del factor V de Leiden. 
 
FISIOPATOLOGÍA 
Tríada de Virchow (estasis, hipercoagulabilidad y lesión endotelial), no es necesario tener los 3.   
La leisón endotelial es parte angular del desarrollo de la trombosis. La estasis promueve plaquetas y los factores de la coagulación se 
acumulen y puedan ponerse en contaco con las peredes del vaso, la hipercoagulabilidad se presenta en pacientes con enfermedades 
agudas, traumatismos, cirugía reciente (como parte de la respuesta metabólica al trauma) o bien alteracioens de la coagulación.    La 
lesión o disfunción endotelial puede ser por traumatismo directo, la mayoría de las veces se presenta por hipoxia y citocinas 
proinflamatorias que hacen que se pierdan las características del endotelio. 
 
CUADRO CLÍNICO 
Son asintomáticas en 50% de los casos. Se puede presentar con dolor edema, eritema, hipertermia, sensibilidad, fiebre, presentcia de 
red venosa superficial, así como, dolor a la dorsiflexión o compresión de la pierna. Es de inicio súbito y progresivo.     
El edema es un dato PIVOTE. 
Signos clínicos: son poco frecuentes aprox en 10%   
Homans (dolor a la compresión en gastroctnemios, 
Ollow (dolor en gastrocnemios a la dorsiflexión de pie).   
Prats (presencia de red venosa colateral superficial),   
 
DIAGNÓSTICO 
Escala de Wells. Solicitar dímero D (en ámbito hospitalario) valor predictivo negativo 98%.   
Se confirma el Dx si Wells ≥2 + DD+. Realizar USG duplex en todos los paciente con sspecha de TVP con DD+ como confirmatorio 
 
TRATAMIENTO 
Anticoagulación con HBPM, HNF o fondaparinux por 5 días al mismo tiempo que anticoagulación oral. Objetivo INR de 2‐3. 
Duración mínimo 3 meses, puede extenderse hasta 1 año en 1er evento con múltiples factores de riesgo y de forma indefinida en 
recurrencia. Medias de higiene venosa y deambulación temprana.   
 
Debido al alto riesgo de TVP en pacientes quirúrgicos (los de mayor eisgo son la cirgía de cadera y rodilla) así como pacinetes 
hospitalziados o postrados se sugiere tromboprofilaxis de actuero a riesgo de TVP, desde calcetines de compresión y deambulación 
temprana hasta combinación de medidas físicas y psoturales y uso de anticoagulanes (HBPM, IVK, heparina convencional, 
fondaparinux, rivaroxaban, dabigatran) previa y posterior al tratamiento. Todas las pacientes a los que se les realizará cirugía mayor 
ginecológica, urológica o bariátrica, deben recibir profilaxis farmacológica.   
 
COMPLICACIONES 
Secuela post trombótica o secuela posflebítica: es un cuadro de insfuciencia venosa crónica secundaria, flegmasia cerúlea dolens. TEP 
en 10‐50% de los casos. 
 
PRONÓSTICO 
Recurrencia: 25% a 5 años y 30% a 10 años. Mortalidad hasta 19% sin tratamiento. <5% con adecuada anticoagulación.   
 
FEMEINO DE 45 AÑOS, CON ANTECEDENTE DE INGESTA DE ANTICONEPTIVOS ORALES DURANTE 10 AÑOS, SUSPENDIDOS HACE 7 AÑO. 
PRESENTA ACCIDENTE AUTOMOVILÍSTICO SUFRIENDO FRACTURA DE TERCIO MEDIO DE FÉMUR DERECHO QUE REQUIERE 
TRATAMIENTO QUIRÚRGICO E INMOVILIZACIÓN DE LA EXTREMIDAD. 
DOS DÍAS DESPUÉS DE LA CIRUGÍA LA PACINETE INICIA CON AUMENTO DE VOLUEM A NIVEL DE LA PIERNA DERECHA, DOLOR 
INTENSIDAD 9/10 DE LA MISMA LOCALIZACIÓN, ERITEMA Y AUMENTO DE TEMPERATURA; DATOS QUE SUTED CORROBORA A LA E, 
ADEMÁS ENCUENTRA SIGNOS DE OLLOW, HOMANS, Y SOLICITA DÍMERO D CUYO RESULTADO SE REPORTA EN 1200. 
 
TODOS SON FACTORES DE RIESGO PARA DESARROLLAR ENFERMEDAD TROMBOEMBÓLICA VENOSA, EXCEPTO: 
A) EDAD MAYOR A 30 AÑOS 
B) CIRUGÍA RECIENTE 
C) NEOPLASIA 
D) DEFICIENCIA DE FACTOR V DE LEIDEN 
 
EL SIGNO DE OLLOW SE CARACTERIZA POR: 
A) DOLOR A EN GASTROCTNEMIOS A LA DORSIFLEXIÓN DEL PIE 
B) DOLOR A LA COMPRESIÓN EN GASTROCTNEMIOS 
C) PRESNECIA DE RED VENOSA COLATERAL SUPERFICIAL 
D) INCREMENTO DEL ERITEMA EN LA EXTREMIDAD AFECTADA A LA PALPACIÓN 
 
ESTUDIO CONFIRMATORIO PARA TROMBOSIS VENOSA PROFUNDA: 
A) DÍMERO D 
B) ESCALA DE WELLS MAYOR A 2 
C) USG DÚPLEX 
D) USG DE TEJIDOS BLANDOS 
 
EL TRATAMIENTO EN ESTA PACIENTE SERÍA: 
A) ANTIAGREGACIÓN PLAQUETARIA CON ÁCIDO ACETILSALICÍLICO 
B) ANTICOAGULACIÓN CON HEPARINA NO FRACCIONADA O HBPM 
C) ANTICOAGULACIÓN ORAL CON CUMARÍNICOS 
D) ANTICOGULACIÓN CON HBPM O HNF E INICIAR AL MISMO TIEMPO ANTICOAGULACIÓN ORAL.   
 
RESPUESTA A, A, C 
 
 
PIE DIABÉTICO 
[Resumen la Salle] 
 
DEFINICIÓN 
Condición de etiopagoenia neuropática inducida por hiperglucemia sostenida. 
 
EPIDEMIOLOGÍA 
En México tiene una prevalencia estimada de 7‐10% de la polbación dependiente del grupo de edad. Siendo el pie diabético una de las 
complicaciones más frecuentes que se presenta en 15‐25% de los casos. El 80‐85% de las amputaciones de miembros pélvicos en 
diabéticos fueron precedidas por úlceras en los pies.    El pie diabético fue la 1era causa de amputación no traumátia en mayores de 50 
años, condicionando una oérdida    importante de AVISA y costos económicos levados. 
 
FISIOPATOLOGÍA 
Neuropatía: hiperglucemia actividad aumentada en vía del sorbitol, el cual se acumula teniendo efectos tóxicos resultando en 
desmilinización y disminución de la velocidad de conducción. Aproximadamente 45‐60% de las úlceras son exclusivamente 
neuropáticas. 
Isquemia: se genera por los mismo factores de riesgo que la enf arterial periférica, siendo de rápida progresión debido a la disfunción 
endotelial ocasionada por la hiperglucemia. Es de predominio distal (vasos tibiales). 
La media calcinosis de Mökenberg es un proceso independeinte a la ateroesclerosis, en la cual se presenta calcificación de la media con 
alteración de la distensibilidad subsecuente, por lo que afecta la vasodilatación mediada por endotelio, resultando en isquemia de los 
territorios distales, se considera un marcador de alto riesgo cardiovascular. Un 45% de las úlceras presnetan componente isquémico. 
Infección: debido a la pérdida de la sensibilidad protectora al dolor y la presión así como a la neuropatía autonómica (anhidrosis) se 
pueden genrar heridas por traumas menores que pasan inadvertidas. En heridas superficiales generalmente son monobacterians (S. 
aureus) en caso de abscesos y necrosis extensa la mayor parte de las veces son polimicrobianas.   
 
FACTORES DE RIESGO PARA DESARROLLO DE LAS ÚLCERAS 
Mal control glucémico (HbA1c >7%), tabaquismo, sedentarismo, >5 años de DM, enfermedad arterial periférica, neuropatía periférica, 
ceguera, pérdida visual, enf renal crónica, >40 años. 
Deformidad estructural del piel, trauma, calzado inadecuado, hiperqueratosis, historia previa de úlceras o amputación, ortostatismo 
prolongado, movilidad articular limitada.   
 
CLÍNICA 
Alteraciones sensitivas, afección inicial a parestesias. 
Alteraciones motoras: formación de puntos de hiperqueratosis por alteraciones de puntos de apoyo (mal perforante plantar‐ formación 
de úlceras en puntos de hiperqueratosis), deformidad estructural con pérdida de los arcos plantares, puntos de apoyo anómalos y 
dedos en martillo (pie de Charcot). 
Alteraciones autonómicas: anhidrosis, edema, hiperemia.   
Alteraciones vasculares: pérdida de anexos cutáneos, hiptermia, ausencia de pulsos distales, hiperemia reactiva. Úlceras: su localización 
en puntos de apoyo o sitios de traumatismo, pueden o no presentar signos de infección.   
 
DIAGNÓSTICO 
HC y EF completa, valoración de la sensibilidad con monofilamente 10g (10 puntos) o diapsón (palestesia). Enviar a valoración a 2 nivel 
de forma anual, si hay FR enviar cada 3‐6 meses.   
Se establece diagnóstico al encontrar alteraciones físicas (desde hiperqueratosis, deformidad hasta ulceras o necrosis) con datos de 
disminución de sensibilidad. 
Solicitar Rx de pie para determinar compromiso óseo. 
 
TRATAMIENTO 
Referir al 2nivel a cualquier pacientes con una nueva úlceras.   
En caso de presentar sigos de infección inciar AB empírica con cefalosporina VO, amoxiclav, macrólidos, peniclina y fluoroquinolonas.   
En una extremidad sin riesgo inminente cubrir para gram +, una extremidad en riesgo deberá cubririse con doble esquema y si pone en 
riesgo la vida valorar triple esquema con amplio espectro. 
En caso de determinarse isquemia, valorar la posibilidad de revascularización.   
En caso de lesiones extensas (W4 o W5, Texas III) se requerirá desbridación extensa, amputación menor y hasta amputación mayor 
para control del cuadro.   

 
 
PREVENCIÓN 
Control metabólico, lubricación, no cortar callos, actividad física >30 min diarios o por lo menos 3 dias de la semnaa. Inspección diaria 
asistida, valoración de especialidad periódica.   
 
PACIENTE MASCULINO DE 65 AÑOS, CUENTA CON ANTECEDENTE DE DM TIPO 2 DE 10 AÑOS DE DIAGNÓSTICO EN TRATAMIENTO 
IRREGULAR CON METFORMINA 850MG CADA 24 HRS, HAS DE 5 AÑOS DE DX EN TX CON CAPTOPRIL CADA 24 HRS, TABAQUISMO DE 
MAS DE 20 AÑOS DE EVOLUCIÓN A RAZÓN DE 10 CIGARRILLOS AL DÍA.   
ACUDE A CONSULTA MENSUAL DE UMF REFIRIENDO PRESENCIA DE LESIÓN EN PLANTA DE PIE DERECHO, DE APROXIMADAMENTE 
4X5CM DE DIÁMETRO, FÉTIDA Y CON EXUDADO BLANQUECINO MODERADO, REFIERE QUE AHCE 6 MESES INICIÓN CON DOLOR 
ARDOROSO EN LA MISMA EXTREMIDAD DISTAL. 
A LA EF SE OBSERVA MIEMBRO PÉLVICO DERECHO CON PRESENCIA DE ÚLCERA PLANTAR DE 4X5 CM DE DIÁMETRO A NIVEL DE LA 
CABEZA DE TERCER METATARSIANO, CON EXUDADO PURULENTO, FÉTIDO, QUE SE EXTIENDE HASTA TEJIDO CELULAR SUBCUTÁNEO SIN 
PENETRAR LA FASCIA, RESTO DE LA EXTREMIDAD HIPOTRÓFICA CON TONO CONSERVADO, AUSENCIA DE VELLO EN PATE INFERIOR DE 
LA PIERNA, PULSOS FEMORAL 2/2, POPLÍTEO 2/2, TIBIAL ANTEIOR, POSTERIOR Y PEDIO 0/2. FUERZA MUSCULAR 4/5. REMS 2/4, 
SENSIBILIDAD SUPERFICIAL CON HIPOESTESIA A NIVEL DE S1, SENSIBILIDAD PROFUNDA CON DISMINUCIÓN DE PALESTESIA, LLENADO 
CAPILAR 4S. 
 
¿CUÁL ES SU DIAGNÓSTICO PRESUNTIVO EN ESTE CASO? 
A) PIE DIABÉTICO CON ULCERACIÓN SIN INFECCIÓN NI ISQUEMIA 
B) PIE DIABÉTICO INFECCIOSO 
C) PIE DIEBÉTICO ISQUÉMICO 
D) PIE DIABÉTICO ISQUÉMICO E INFECCIOSO 
 
DE ACUERDO A LA CLASIFICACIÓN DE WAGNER ¿CUÁL ES EL GRADO DE LA LESIÓN DE ESTE PACIENTE? 
A) 0 
B) 1 
C) 2 
D) 3 
 
¿Es en serio? Parece más un Wagener 2 por que llega a TCSC y tiene datos de infección. 
 
DE ACUERDO A LOS DATOS CLÍNICOS PRESENTADOS USTED DECIDE INICIAR TRATAMIENTO ANTIBIÓTICO, ¿CUÁL SERÍA SU PRIMERA 
ELECCIÓN? 
A) TMP/SMX 
B) CLINDAMICINA 
C) AMOXI CLAV 
D) FOSFOMICINA 
 
TODAS SON MEDIDAS DE PREVENCIÓN PARA EL PACIENTE CON PIE DIABÉTICO, EXCEPTO: 
A) CORTE DE UÑAS CON CORTAUÑAS 
B) CONTROL GLUCÉMICO ADECUADO 
C) CALZADO CÓMOCO 
D) ACTIVIDAD FÍSICA 30 MINUTOS DIARIOS 
 
RESPUESTA D, B, C, A 
SECUELA POSTROMBÓTICA 
[Resumen la Salle] 
 
DEFINICIÓN 
Es el conjunto de signos y síntomas tardíos secundarios a la obstrucción y lesión valvular posteriores a una trombosis venosa profunda 
que dan como resultado a una insuficiencia venosa secundaria.   
 
FISIOPATOLOGÍA 
Durante la formación de un trombo se da inicialmente la adhesión plaquetaria y activación de la cascada de coagulación, dentro de este 
proceso se liberan citocinas proinflamatorias, se presenta acumulación de leucocitos (PMN), los cuales van a liberar enzimas 
proteolíticas como las metaloproteinasas (colagensas, elastasas) con la finalidad de en conjunto con las plasmina lograr la disolución del 
trombo. Sin embargo estas enzimas también lesionan las paredes del vaso ocasionado pérdida de la elasticidad y lesión de las válvulas 
venosas.     
“En una TVP se despliegan mecanismo antitrombóticos que se comen las válvulas”.   
En la historia natural de la trombisis existen 3 finales distinso: recanalización (85‐100% a las 6 semanas), la mayoría de los segmentos 
ocluidos inicialmente se recanalizan dejando como secuela la destrucción valvular. En los casos restante puede existir recanalizacion 
parcial o bien organización de trombo con estenosis venosa. Recanalizació parcial y organización del trombo con estenosis venosa.   
 
Se presenta en 29‐79% de los pacientes con TVP, aumentando la probabilidad de TVP proximal (ilio‐femoral) así como retrombosis, 
hasta 23% puede ser severa (C4), y en 4‐6% desarrolar úlceras flebo estáticas (C6). 
 
CLÍNICA 
Es el de insfuciencia venosa con: dolor, pesantez, prutito, cansancio, edema y calambre en los miembros pélvico, los cuales son de 
predominio vesperino, se exacerban con en orstostatismo y con el calor, y mejoran con el decúbito y el frío.    A la EF se verán cambios 
como telangiectasis, venas reticualres y várices, edema, pigemnetación ocre, lipodermatoesclerosis, eccema o úlceras; es importante 
destar que los cambios cutáneos inicialmente se presentan a nivel del maléolo medial.   
 
Importante destacar que el miembro afectado en la mayorías de las ocasiones presenta aumento de perimetría >1cm, respecto al 
contralateral.   
 
TRATAMIENTO 
Medidas de higiene venosa con elásticos de 30‐40mmHg. 
Destarcar que las várices que se encuentran en contexto de una secuela postrombótia deberán ser valoradas por angiólogo para 
determinar otra posibildiad de tratamiento.   
 
PACIENTE FEMENINO DE 36 AÑOS, QUIEN AHCE 6 MESES PRESENTA EVENTO DE TORMBOSIS VENOSA PROFUNDA DE SEGMENTO 
ILIOFEMORAL DE MIEMBRO PÉLVICO DERECHO DURANTE PUEPRERIO MEDIATO, FUE TRATADA CON ANTICOAGULACIÓN ORAL CON 
CUMARÍNICOS, ACTUALMENTE YA SUSPENDIDOS.   
ACUDE AL CENTRO DE SLUD YA QUE AHCE TRES MESES PRESENTA VÁRICES DE MIEMBRO PÉLVICO DERECHO, DOLOR PUNZANTE‐
URENTE, ACOMPAÑADO DE PRURITO, PENSANTEZ Y PRESENCIA DE CALAMBRES ESPECIALMENTE POR LA TARDE POSTERIOR A 
PERMANECER MUCHO TIEMPO SENTADA, ASÍ MISMO, REFIERE QUE SU PIERNA DERECHA SE ENCUENTRA “MÁS HINCHADA” QUE LA 
IZQUIERDA.   
A LA EXPLORACIÓN FÍSICA DIRIGIDA A DICHA EXTREMIDAD USTED ENCUENTRA EDEMA +++, PRESENCIA DE PAQUETES VARICOSOS EN 
CARA INTERNA DE PIERNA Y MUSLO, AUMENTO DE PERIMETRÍA MÁS DE UNA CM. RESPECTO AL MIEMBRO CONTRALATERAL, VENAS 
RETICUALRES Y DISCRETA PIGMENTACIÓN OCRE A NIVEL DEL MALÉOLO MEDIAL.   
DE ACUERDO AL CASO CLÍNICO PRESENTADO¿CUÁL ES SU IMPRESIÓN DIAGNÓSTICA? 
A) RETROMBOSIS VENOSA PROFUNDA 
B) INSUFICIENCIA VENOSA CRÓNICA PRIMARIA 
C) SECUELA POSTTROMBÓTICA 
D) CELULITIS DEL MIEMBRO PÉLVICO DERECHO 
 
¿QUÉ ESTRUCTURA SE ENCUENTRA AFECTADA A NIVEL VASCULAR PARA EL DESARROLLO DE UNA SECUELA POSTRÓMBÓTICA? 
A) VÉNULAS 
B) CAPILARES 
C) PARED ENDOTELIAL 
D) VÁLVULAS 
 
¿DÓNDE SE PRESENTAN INICIALMENTE LOS CAMBIOS CUTÁNEOS ASOCIADOS A ESTA PATOLGÍA? 
A) MALEOLO LATERAL 
B) MALEO MEDIAL 
C) SEGEMENTO INFERIOR DE LA PIERNA 
D) A NIVEL DE GASTROCNEMIOS 
 
¿CUÁNTA ES LA COMPRESIÓN QUE DEBE APLICAR EL SOPORTE ELÁSTICO AL MIEMBRO AFECTADA DE ESTA PACIENTE? 
A) 15‐20MMHG 
B) 8‐10 
C) 30‐40 
D) 50‐60 
 
RESPUESTA C, D, B 
 
 
 
VÁRICES 
[Resumen la Salle] 
 
DEFINICIÓN 
Dilataciones venosas del sistema superficial >3mm.   
Dentro de la insfuciencai venosa crónica se puede encontrar en C2 dentro de la clasificación de CEAP (C3 es edema, C4 cambios 
dermatológicos, C5 úlcera cicatrizada y C6 úlcera activa). 
Es importante destacar que las várices pueden presentarse en diferentes territorios: esófagno‐estógmado, recto (hemorroides), 
testículos (varicocele), las cuales van más allá del abordaje del angiólogo y son vistas por otros especialistas. 
 
FISIOPATOLOGÍA 
Misma que enfermedad venosa crónica.   
Hay factores genéticos como mutaciones en gen FOXC2 asociadas a falla valvular primaria. En la destrucción de las válvulas se ha 
determinado el papel de enzimas como metaloproinasas que generand egracion de fibras de colágeno. 
FR: edad, sexo femenino, gestación, antecedente familiar de IVC, ostostatismo prolonado, obesidad, sendentarismo.   
 
CLÍNICA 
Lo mismo que insuficiencia venosa crónica más dialtaciones benosas >3mm.   
 
TRATAMIENTO 
Medidas de higiene venosa con soporte elástico 18‐21 mmHg, en caso de no presenta rmejoría en 6 meses o presentar complicaciones 
(tromboflebitis, varicotrombosis, varicorragia) se considera cnadidato a tx quirúrgico con safeno excéresis. 
 
PACIENTE FEMENINO DE 48 AÑOS DE EAD, CON ANTECEDENTES DE IMPORTANCIA: OCUPACIÓN SECRETARIA, SEDENTARIA, GESTA 2, 
CESÁREA 2, SIN OTROS DE RELEVANCIA, QUIEN NOTA QUE DESDE HACE 6 MESES PRESENTA DILATACIONES VENOSAS EN CARA INTERNA 
DE LA PIERNA DERECHA, DOLOROSAS A LA PALPACÓN, ASOCIADAS A EDEMA DE PREDOMINIO VESPERTINO, PESANTES DE LA 
EXTREMIDAD Y PRURITO OCASIONAL QUE MEJORA CUANDO EN DECÚBITO; ADEMÁS REFIERE PRESENCIA DE TELANGIECTASIS EN 
MALEOLO INTERNO DERECHO DESDE HACE 10AÑOS.   
A LA EF Y SOMATOMETRÍA TA 120/80, FC 80, FR 18 T 3, GB, IMG 24, A LA EXPLORACIÓN DEL MIEMBRO PÉLVICO HALLA DICHA 
DIALTACIÓN DE PAQUETES VASCUALRES DESDE TERCIO INFERIOR DEL MUSLO, HASTA AL TERCIO MEDIO DE LA PIERNA, DE 4MM DE 
DIÁMETRO, DOLOR A LA PALPACIÓN Y EDEMA +++ A NIVEL DEL TOBILLO DERECHO. 
 
DE ACUERDO AL CASO CLÍNICO PRESENTADO ¿CUÁL ES SU IMPRESIÓN DIAGNÓSTICA? 
A) INSUFICIENCIA VENOSA CRÓNICA SECUNDARIA 
B) LINFEDEMA PRIMARIO 
C) VÁRICES PRIMARIAS 
D) VARICES SECUNDARIAS 
 
LA PACIENTE CUENTA CON TODOS LOS FACTORES DE RIESGO PARA DESARROLLO DE SU PATOLOGÍA, EXCEPTO: 
A) OBESIDAD 
B) SEXO 
C) SEDENTARIAMOS 
D) SEDESTACIÓN PROLONGADA. 
 
CARACTERÍSTICA DEL EDEMA QUE PRESENTA ASOCIADO A VÁRIDES: 
A) ASOCIADO AL DECÚBITO 
B) PREDOMINIO VESPERTINO 
C) LA CONSISTENCIA ES DURA 
D) MEJORA CON EL CALOR 
 
¿CUÁL ES EL TRATAMIENTO QUE USTED OFRECE INICIALMENTE A ESTA PACIENTE? 
A) SAFENO EXÉRESIS 
B) LIGADURA DE PERFORANTES 
C) MEDIDIAS DE HIGIENE VENOSA 
D) LIGADURA DE PAQUETES VASCULARES. 
 
Recordar que si solot eine várices las medias deben ser de 18‐21mmHg (equivale a un C2). 
Si fracasa luego de 3‐6 meses se refeire y se valora safeno exéresis. 
 
RESPUESTA C, A, B, C 
   
 
 
   
 
   
 
 
   
 
   
 
   
   
 
 
 
 
 
 
 
 
 
 
 
 
 
 
CASOS CLÍNICOS ANALIZADOS POR CLASIFICAR 
 
 
[r]HOMBRE DE 72 AÑOS DE EDAD, SE ENCUENTRA HOSPITALIZADO HACE 7 DÍAS POSTERIOR A UNA COLECTOMÍA SECUNDARIA A LA 
PERSENCIA  DE  DIVERTICULITIS  QUE  NO  CEDIÓ  A  TRATAMIENTO  ATNIMICROBIANO  PREVIAMENTE.  EL  PACIENTE  PRESENTA  DOLOR 
INTENSO EN ABDOMEN QUE NO CEDE ANTE LA INGESTA DE MEDICAMENTOS. A LA EXPLORACIÓN FÍSICA: FC 90 FR 12, T38 TA 100/60. SE 
REALIZAN ESTUDIOS DE IMAGEN MEDIANTE LOS QUE S ECORROBORA FUGA DE ANASTOMOSIS COLÓNICA, POR LO QUE EL PACIENT EES 
REINTE4RVENIDO QUIR´RUGICAMENTE.   
¿CUÁL ES TRATAMIENTO ANTIMICROBIANO MÁS ADECUADO PARA ESTE PACIENTE 
A) AMOXICILINA ‐ VANCOMICINA 
B) LINEZOLID‐ FLUCONAZOL 
C) PIPERACILINA‐TAZOBACTAM 
D)ANFOTERICINA‐ METONIDAZOL 
 
RESPUESTA C 
El  paciente  tiene  fuga  anastomótica.  La  fuga  anastomótica,  desencadena  sepsis  abdominal  o  en  absceso  intaperitoneales.  Las 
manifstaciones que presenta son dolor abdominal, fiebre que no tarda en vlucionar a peritonitis generalizada, y choque séptico pr lo que 
el tratamiento oportuno es lo ideal . El paciente debe tener cbertura poara gramnegativas, gram positivas y anerobios demás de cubrir 
pseudomonas spp. 
Piperacilina‐Tazobactam, debido a que el paciente fue tratad previamente con antibióticos y por su estancia hospitalaria.   
 
 
  
 
[QUISTE PILONIDAL] 
[r]HOMBRE  DE  38  AÑOS  DE  EDAD,  QUE  ACUDE  A  SU  CONSULTA  REFERIDO  DEL  CENTRO  DE  SALUD  CON  DIAGNÓSTICO  DE  QUISTE 
PILONIDAL. A LA EXPLORACIÓN LO ENCUENTRA EN FASE AGUDA, INFECTADO Y ABSCEDADO. 
EN ESTE PACIENTE EL TRATAMIENTO INDICADO ES: 
 
   
A) PUNCIÓN Y ASPIRACIÓN DE CONTENIDO. 
B) ADMINISTRACIÓN DE ANTIBIÓTICOS SISTÉMICOS. 
C) LAVADO CON ANTISÉPTICOS Y EMPAQUETAMIENTO CON GASAS CON ISODINE. 
D) DRENAJE ABIERTO Y MARSUPIALIZACIÓN. 
 
RESPUESTA 
El tratamiento del quiste pilonidal en fase aguda es el drenaje del absceso y marsupialización de los bores, se puede dar antibiótico de ser 
necesario.     
Una vez resuelta la fase aguda se puede vigilar la recidiva, o se puede operar directamente (excisión del seno pilonidal completo hasta 
fascia persacra y legrado del tej de granulación, herida puede quedar abierta y esperar cierre por segunda intención). 
 
[ÚLCERA PÉPTICA] 
HOMBRE DE APROXIMADAMENTE 30 AÑOS DE EDAD, ES ENCONTRADO EN LA CALLE CON PÉRDIDA DEL ESTADO DE ALERTA E INTENSO 
ALIENTO ALCOHÓLICO. ES LLEVADO AL SERVICIO DE URGENCIAS DEL HOSPITAL. A LA EXPLORACIÓN FÍSICA CON MALA HIGIENE, RESTOS 
DE  VÓMITO  EN  SU  ROPA,  GLASGOW  DE  13‐14,  TA  90/55,  FC  115X´,  FR  26X´,  TEMPERATURA  38.9°C,  EL  ABDOMEN  SE  ENCUENTRA 
DISTENDIDO,  REFLEJO  DOLOROSO  A  LA  PALPACIÓN  MEDIA  Y  PROFUNDA  PRINCIPALMENTE  EN  EPIGASTRIO,  REBOTE  GENERALIZADO, 
PÉRDIDA DE LA MATIDEZ HEPÁTICA Y TIMPANISMO GENERALIZADO. 
       
PARA APOYAR SU DIAGNÓSTICO DEBERÁ SOLICITAR DE MANERA INICIAL: 
A) ULTRASONIDO ABDOMINAL. 
B) ESTUDIOS CON MEDIO DE CONTRASTE. 
C) TOMOGRAFÍA COMPUTADA. 
D) RADIOGRAFÍA DE TÓRAX Y ABDOMEN. 
    
EL DIAGNÓSTICO CLÍNICO MÁS PROBABLE ES: 
A) APENDICITIS AGUDA COMPLICADA. 
B) VÓLVULO INTESTINAL. 
C) ULCERA PÉPTICA COMPLICADA. 
D) PANCREATITIS AGUDA. 
       
EL TRATAMIENTO INDICADO EN ESTE CASO ES:   
A) RESECCIÓN INTESTINAL Y COLOSTOMÍA EN BOLSA DE HARTMANN. 
B) CIERRE CON PARCHE DE GRAHAM. 
C) APENDICECTOMÍA. 
D) COLOCACIÓN DE SONDA, ANALGÉSICOS Y ANTIBIÓTICOS. 
 
RESPUESTAS D, C, B 
 
 
 
[HERNIA HIATAL POR DESLIZAMIENTO] 
  
CONTENIDO 
Principal objetivo de procedimiento antirreflijo: recuperar el ángulo de His, recuperar la presión del EEI. 
 
 
MUJER  DE  60  AÑOS  DE  EDAD  CON  OBESIDAD  MÓRBIDA.  PRESENTA  DESDE  HACE  6  MESES  CUADROS  INTERMITENTES  DE  PIROSIS, 
REGURGITACIÓN  Y  DISFAGIA  OCASIONAL,  QUE  CEDEN  CON  INHIBIDORES  DE  LA  BOMBA  DE  PROTONES.  SE  LE  INDICA  UNA  SERIE 
ESÓFAGOGASTRODUODENAL, PARA COMPLEMENTAR SU ESTUDIO, OBTENIÉNDOSE LA SIGUIENTE IMAGEN: 

 
EL DIAGNÓSTICO RADIOLÓGICO MÁS PROBABLE ES EL DE: 
A) HERNIA HIATAL POR DESLIZAMIENTO. 
B) REFLUJO GRASTROESOFÁGICO. 
C) ACALASIA. 
D) ANILLO DE SCHATZKI. 
 
RESPUESTA A 
Las hernias hiatales se dividen en 4 tipos principales: 
TipoI/deslizante: 95%, secundaria    debilidad y elongación de las fibras frenoesofágicas. 
Tipo II/paraesofágicas: esta es una hernia real donde el fundus se torna paralelo al esófago secundario a la debilidad de la membrana 
peluroperitoneal. 
Tipo III/mixtas: componentes de las antriores 
Tipo IV/complejas: migración intratorácica de cualquier otro intrabdominal 
 
Síntomas clásicos: regurgitación, pirosis y disfagia. La disfagia puede ser secundaria a edema de mucosa, anillo de Schatzki, estenosis    o 
incapacidad para organizar la actividad peristáltica en el cuerpo del esófago. Estudio de elección: SEGD 
 
ACALASIA 
[r]HOMBRE DE 30 AÑOS DE EDAD, ACUDE POR PRESENTAR DISFAGIA Y REGURGITACIÓN PROGRESIVA. USTED SOSPECHA LA POSIBILIDAD 
DE ACALASIA. 
EL ESTUDIO QUE DEBERÁ SOLICITAR PARA CONFIRMAR SU SOSPECHA DIAGNÓSTICA ES: 
A) UN ESOFAGOGRAMA CON BARIO. 
B) UNA ENDOSCOPÍA. 
C) UNA PH METRÍA DE 24 HRS. 
D) UNA MANOMETRÍA. 
 
EN CASO DE CONFIRMAR SU DIAGNÓSTICO EL PACIENTE DEBERÁ SER TRATADO CON 
A) ANTAGONISTAS DE CALCIO. 
B) TOXINA BOTULINICA. 
D) DILATACIONES NEUMÁTICAS. 
E) MIOTOMÍA. 
 
RESPUESTAS D, E 
Para el ENARM es muy importante comprender qué es lo que te están preguntando.   
El estudio inicial en paciente con acalasia es el esofagograma  con bario, el cual demustra la imagen clásica en "punta de lápiz". Este 
estudio orienta al diagnóstico pero no lo confirma. La manometría es el estudio ideal para confirmarlo y demostrar ausencia de relajación 
del esfinter esofágico inferior y aperistalsis del cuerpo esofágico. Hay que recordar que en estadios iniciales aun no es posible encontrar 
la clásica imagen en el esofagograma. 
La endoscopia y la TAC son estudios útiles para diferenciar de acalasia de pseudoacalasia. 
La pHmetría en 24 hrs mide el reflujo ácido y es gold standar de ERGE, sin utilidad diagnóstica en acalasia. 
La miotomía y las dialtaciones son los únicos tratamiento duraderos en acalasia, y son más efectivos en pacientes vírgenes a tratamientos 
médicos. La miotomía de Heller con funduplicatura parcial tiene menor riesgo y es más eficaz que las dilatacines, por lo que en este 
paciente es gold standar. 
 
 
 
UN HOMBRE DE 65 AÑOS DE EDAD SE PRESENTA A SU MÉDICO QUEJÁNDOSE DE DIFICULTAD PARA DEGLUTIR, DOLOR EN EL PECHO DE 
VEZ EN CUANDO, Y REGURGITACIÓN DE ALIMENTOS. EN LOS ÚLTIMOS 2 MESES HA PERDIDO ALREDEDOR DE 7 KG. LOS RESULTADOS DE 
UN ESTUDIO DE DEGLUCIÓN DE BARIO SE MUESTRAN EN LA IMAGEN. ¿QUÉ PRUEBA SE DEBE REALIZAR PARA BUSCAR POSIBLES CAUSAS 
DE SU CONDICIÓN? 
 
 
A.  PHMETRÍA DE 24 HORAS 
B. MANOMETRÍA ESOFÁGICA 
C. MEDICIÓN DEL NIVEL DE GASTRINA SÉRICA 
D. ENDOSCOPIA SUPERIOR 
E. PRUEBA DEL ALIENTO CON UREA 
RESPUESTA D 
(YO) 
 
 
 
COLECISTITIS 
MUJER DE 35 AÑOS CON ANTECEDENTE DE CIRUGÍA BARIÁTRICA POR OBESIDAD MÓRBIDA HACE 6 MESES. ACUDE A CONSULTA DEBIDO 
A CUADRO CARACTERIZADO POR DOLOR INTERMITENTE EN CUADRNATE SUPERIOR ERECHO SOBRE TODO RELACIOADO A COSUMO DE 
ALIMENTOS GRASOSOS, REFIERE QUE HACE UN MES TUVO UN EPISODIO    SIMILAR. SE REALIZA USG DE HÍGADO Y VÍAS BILIARES CON 
REPORTE DE DOS LITS VESICULARES DE 10‐14MM, RESTO SIN ALTERACIONES. 
¿CUÁL ES LA CONDUCTA MÁS APROPIADA PARA ESTA PACIENTE? 
 
A)TRATAMIENTO SINTOMÁTICO 
B)COLECISTECTOMÍA 
C)ÁCIDOS BILIARES 
D)LITOTRICIA 
 
RESPUESTA    A 
El USG de la paciente solo muestra lito, más no "agudización" o inflamación de la vesícula.   
Criterios  para  colecistectomía  laparoscópica  profiláctica.  Vesícula  biliar  calcificada,  cálculos  mayores  de  3cm  o  paciente  candidato  a 
cirugía bariátrica o trasplante cardiaco.   
Criterios para litotricia: litiasis única, no calcificada con diámetro de 20‐30mm 
Acidos bilaires: se reservan para pacientes con alto riesgo quirúrgico, pero debido a su costo y efectos secundarios son poco utilizados. 
 
 
PANCREATITIS AGUDA 
HOMBRE DE 56 AÑOS DE EDAD, ACUDE AL SERVICIO DE URGENCIAS POR PRESENCIA DE DOLOR ABDOMINAL INTENSO, 
INCAPACITANTE  Y  QUE  IMPIDE  LA  DEAMBULACIÓN.  A  LA  EXPLORACIÓN  PRESENTA  ABDOMEN  DISTENDIDO, 
EXTREMADAMENTE  DOLOROSO  A  LA  PALPACIÓN  Y  LA  PERISTALSIS  SE  ENCUENTRA  DISMINUIDA.  SE  SOLICITA  RX 
ABDOMINAL ENCONTRÁNDOSE LA SIGUIENTE IMAGEN 
 

 
1)LA IMAGEN RADIOLÓGICA OBTENIDA APOYA EL DIAGNÓSTICO DE:   
A) APENDICITIS AGUDA 
B) ÚLCERA PÉPTICA PERFORADA 
C) PANCREATITIS 
D) COLECISTITIS AGUDA 
 
2)UNA VEZ ESTABLECIDO EL DIAGNÓSTICO, DEBERÁ INDICARSE EL SIGUIENTE TRATAMIENTO:   
A) APENDICECTOMÍA ABIERTA 
B) REPOSO PANCREÁTICO 
C) HEMOSTASIA ENDOSCÓPICA 
D)COLECISTECTOMÍA LAPAROSCÓPICA 
 
RESPUESTAS C, B 
La Rx de tórax de un paciente con pancreatitis puede mostrar elevación de hemidiafragmas, derrame pleural, atelectasia 
o neumonía, (la presencia de derrame pleural asocia pobre desenlace). Rx de abdomen puede msotrar cálculos biliares, 
asa centinela (segmento de intestino lleno de aire generalmente en cuadrante superior izquierdo), signo del colon cortado 
(se interrumpe de forma súbita donde hay inflamación pancrática), o aumento del espacio gastrocólico.   
Se recomienda reposo pancreático, junto con alimentación enteral temprana con sonda nasoyeyunal, transpilórica y en 
casos de cirugía yeyunostomía. Se realiza cirugía inmediata si se complica con pancreatisis necrótica o colangitis aguda. 
 
 
 
FEMENINA  DE  39  AÑOS,  QUE  POSTERIOR  A  PERMANECER  HOSPITALIZADA  POR  3  DÍAS  CON  DIAGNÓSTICO  DE 
PANCREATITIS AGUDA DE ORIGEN BILIAR, SE ENCUENTRA CON DOLOR ABDOMINAL PERSISTENTE, CON INCREMENTO DEL 
PERÍMETRO ABDOMINAL, SE MANTIENE AFEBRIL Y SIN ICTERICIA. SE SOLICITA BIOMETRÍA QUE REPORTA LEUCOCITOS DE 
18,000/MM3. 
 
EL PROCEDIMIENTO QUE SE DEBERÁ SOLICITAR PARA ESTABLECER EL DIAGNÓSTICO EN ESTE MOMENTO ES: 
A)LAVADO PERITONEAL DIAGNÓSTICO. 
B)TOMOGRAFÍA ABDOMINAL. 
C)ULTRASONOGRAFÍA ABDOMINAL. 
D)RADIOGRAFÍA DE TÓRAX. 
 
EN ESTE MOMENTO SE DEBE INDICAR: 
A) DEBRIDACIÓN PANCREÁTICA INMEDIATA. 
B) COLANGIOPANCREATOGRAFIA RETROGRADA ENDOSCOPICA(CPRE). 
C) COLECISTECTOMÍA ABIERTA URGENTE. 
D) SONDA NASOGÁSTRICA, LIQUÍDOS Y ELECTROLITOS, ANTIBIOTICOS, ANALGÉSICOS Y EL APOYO NUTRICIONAL. 
 
 
RESPUESTAS   
  B, D 
¿NO DEBERÍA SER SONSA NASOYEYUNAL? 
 
REFLUJOGASTROESOFÁGICO 
[r]HOMBRE DE 51 AÑOS DE EDAD QUIEN PADECE DESDE HACE 10 AÑOS ENFERMEDAD POR REFLUJO GASTROESOFÁGICO 
TRATADA SÓLO DE MANERA PARCIAL, ES SOMETIDO A ENDOSCOPÍA, REPORTÁNDOSE EPITELIO ROSA SALMÓN ARRIBA 
DE LA UNIÓN GASTROESOFÁGICA. 
 
CON BASE EN EL HALLAZGO ENDOSCÓPICO LO MÁS PROBABLE ES QUE EL PACIENTE PRESENTA EN ESTE MOMENTO: 
A) ESOFAGITIS. 
B) CARCINOMA ESCAMOCELULAR. 
C) METAPLASIA. 
D) ACALASIA. 
 
CORRESPONDE AL TRATAMIENTO DE ELECCIÓN EN ESTE CASO: 
A) INHIBIDORES DE LA BOMBA DE PROTONES. 
B) ANTIHISTAMÍNICOS H2. 
C) ESOFAGECTOMÍA. 
D) FUNDUPLICATURA 
 
RESPUESTA C, A   
Se trata de paciente con ERGE es somentido a endoscopia y se muestras el hallazgo de epitelio color salmón.   
Hay que recordar que una metaplasia es el cambio de una células por otra similar, se dice que que hay displasia cuando 
hay pérdida de la arquitectura habitual de la célula. El esófago de Barret es una metaplasia que endoscópicamente luce 
como  una  sona  de  epitelio  color  salmón,  razón  por  la  cual  el  apciente  presenta  una  metaplasia.  Y  el  tratamiento 
conducente es con IBP.   
 
Por medio de endoscopia la esofagitis se puede clasificar en grados: 
  Grado I‐  Erosiones pequeñas, circulares, no confluyen. 
  Grado II‐  Erosiones lineales cubiertas por tejido de granulación que sangran facil si se toca.   
  Grado III‐ Erosiones lineales o circulares que confluyen para formar área circular sin epitelio 
  Grado IV‐ Estenosis (obligada la toma de biopsias) 
   
Respecto al tratamiento hay que recordar que de primera elección están los IBP. 
  Displasia leve: IBP x 1‐2meses y repetir endoscopia. 
  Displasia de alto grado: vigilancia estricta endoscópica c/ 3 meses. 
Los inhibidores H2 solo están indicados junto con funduplicatura en pacientes con displasia leve y después del tratamiento 
inicial con IBP. 
Esofagectomía inciada en jovenes que no están dispuestos a control endoscópico trimestral.   
La funduplicatura está inciada en quienes no teleran los IBP o no responden a ellos.   
 
 
OSTEOSARCOMA 
MASCULINO DE 19 AÑOS, REFIERE DOLOR EN MUSLO IZQUIERDO INCREMENTÁNDOSE POR LAS NOCHES. NO HAY RELACIÓN CON LA 
ACTIVIDAD FÍSICA Y SE NIEGA FIEBRE O PÉRDIDA DE PESO. A LA EXPLORACIÓN SE PALPA MASA DURA Y FIJA A PLANOS PROFUNDOS EN 
TERCIO DISTAL DE FÉMUR. LA RADIOGRAFÍA PRESENTA LESIÓN PENETRANTE EN FÉMUR CON DESTRUCCIÓN CORTICAL CON ASPECTO DE 
"RAYOS SOLARES". 
 
EL DIAGNÓSTICO PROBABLE DE ESTE PACIENTE ES EL DE: 
A) OSTEOSARCOMA. 
B) QUISTE ÓSEO. 
C) OSTEOMIELITIS. 
D) OSTEOCONDROMA 
 
LOS HALLAZGOS RADIOLÓGICOS PRESENTES EN EL PACIENTE SON SECUNDARIOS A: 
A) INFECCIÓN EN UN SITIO CERCANO AL HUESO. 
B) DISPLASIA ÓSEA Y PLEOMORFISMO CELULAR METAFISIARIO. 
C) REMANENTE DE LA PLACA DE CRECIMIENTO DEL CARTÍLAGO 
D) INFECCIÓN QUE VÍA HEMATOGENA LLEGA A LA ARTICULACIÓN. 
 
RESPUESTAS 
A, B, 
 
 
MUJER  DE  23  AÑOS  DE  EDAD  PRESENTA  RADIOGRAFÍA  CON  DATOS  DE  TUMORACIÓN  MUTILOCULADA  EN  LA  ZONA  EPIFISARIA  DEL 
EXTREMO DISTAL DE FÉMUR IZQUIERDO. ¿CUÁL ES EL DIAGNÓSTICO MÁS PROBABLE? 
A) SARCOMA OSTEOGÉNICO 
B) TUMOR DE CÉLULAS GIGANTES 
C)SARCOMA DE EWING 
D)MIELOMA MÚLTIPLE 
  
RESPUESTA B 
EL ASPECTO MULTILOCULADO O EN POMPAS DE JABÓN ES CARACTERÍSTICO DEL TUMOR DE CÉLULAS GIGANTES, MUY FRECUENTE EN 
MUJRES DE 20‐40 AÑOS] CTO TRAUMATLOGÍA PAG 56 
El tumor de células gigantes es un tumor raro, constituye el 5% de los tumores óseos primarios. Ocurre en pacientes entre los 20 y 40 
años, siendo raro en los menores de 10 y en mayores de 50 años. Tiene un ligero predominio del sexo femenino y más frecuente en países 
orientales que occidentales. Habitualmente se loalizan en huesos largos, con afectación de la epífisis y de localización excéntrica, pueden 
propagarse a metáfisis, provocar destrucción cortical y extensión eventual a tejidos blando y al especio articular.   
Los tres sitios más habituales de localización en orden de frecuencia son: 1 extremo distal del fémur, 2 extremo proximal de tibia, 3 
extremo distal del radio.   
Clínicamente, el dolor es el signo más freucente de presentación, tanto si se asocia o no a fractura patológica. Localmente la piel puede 
estar hiperémica, eritematosa y si el tumoer crece puede aparecer ciruclación colateral con presencia de masa palpable. La clínica es 
infrecuente y si el TCG tiene localización raquídea o sacra presenta signos y síntomas neurológicos.   
El  diagnóstico  de  TCG  suele  realizarse  por  laws  manifestaciones  clínicas  y  radiográficas,  teniendo  en  cuenta  lo  spisible  sdiagnóstico 
diferenciales  Cmo  pruebas  complementarias,  la  RMN  es  el  método  más  aprovechoso  para  determinar  la  extensión  y  l  estadiaje,  la 
gammagrafía es uilizada para detectar TCG multicéntricos que son raros (<1%) m y los parámetros de laboratorios suelen ser normales.   
 
 
[PATOLOGIA DE LAS VÍAS VISUALES] 
HOMBRE DE 32 AÑOS DE EDAD, CON DIAGNÓSTICO RECIENTE DE CRANEOFARINGEOMA. 
LA MANIFESTACIÓN CLÍNICA OFTALMOLÓGICA ESPERADA EN ÉSTE PACIENTE SERÍA: 
A) HEMIANOPSIA BITEMPORAL HETERÓNIMO. 
B) HEMIANOPSIA HETERONIMA BINASAL. 
C) HEMIANOPSIA ALTITUDINAL. 
D) HEMIANOPSIA HOMÓNIMA CONTRALATERAL 
   
LA ALTERACIÓN EN LA VISIÓN DE ÉSTE PACIENTE ES SECUNDARIA A DAÑO:   
A) DEL QUIASMA ÓPTICO. 
B) DE LOS NÚCLEOS DE LA VISIÓN. 
C) DEL NERVIO ÓPTICO DESPUÉS DEL QUIASMA. 
D) DEL NERVIO ÓPTICO ANTES DEL QUIASMA. 
 
LAS ÁREAS DE LA CORTEZA DONDE SE INTEGRA LA VÍA VISUAL DE ESTE PACIENTE SON LAS SIGUIENTES: 
A) ÁREA 11 ‐ 12 ‐ 13. 
B) ÁREA 17 ‐ 18 ‐ 19. 
C) ÁREA 21 ‐ 22 ‐ 23. 
D) ÁREA 25 ‐ 26 ‐ 27. 
 
RESPUESTAS 
A, A, B 
 
CÁLCULOS URINARIOS 
MUJER DE 48 AÑOS DE EDAD, DIABÉTICA DE 10 AÑOS DE EVOLUCIÓN, CON ANTECEDENTE DE INFECCIONES URINARIAS DE REPETICIÓN. 
SE REPORTA UN ÚLTIMO UROCULTIVO HACE 5 DÍAS CON AISLAMIENTO DE PROTEUS VULGARIS PARA LO CUAL SE INICIA TRATAMIENTO 
ESPECÍFICO. ACUDE HOY AL SERVICIO DE URGENCIAS POR NÁUSEAS, DOLOR INTENSO EN REGIÓN LUMBAR DERECHA QUE SE IRRADIA 
HACIA  INGLE  DEL  MISMO  LADO.  EL  ULTRASONIDO  REPORTA  LIGERA  DILATACIÓN  URETERAL  EN  TERCIO  SUPERIOR  LO  QUE  SUGIERE 
URETEROLITIASIS. 
      
LO MÁS PROBABLE ES QUE EL CÁLCULO PRESENTE EN ESTA PACIENTE ESTÉ FORMADO POR: 
A) CALCIO. 
B) FOSFATO DE AMONIO‐MAGNESIO. 
C) ÁCIDO ÚRICO. 
D) OXALATO DE CALCIO. 
       
EL TRATAMIENTO ESPECÍFICO EN ESTE CASO DEBERÁ SER CON: 
A) HIDROCLOROTIAZIDA + ORTOFOSFATOS 
B) FOSFATO + ÁCIDO ASCÓRBICO 
C)PENICILINA ORAL + ALOPURINOL 
D) ÁCIDO ACETOHIDROXÁMICO + DOXICICLINA. 
 
RESPUESTAS   
B,D 
 
ESGUINCE CERVICAL 
 
 
HOMBRE DE 53 AÑOS DE EDAD, DEDICADO ACTUALMENTE AL CUIDADO DE ANCIANOS Y NIÑOS ENFERMOS. DURANTE SUS VACACIONES 
Y  TRAS  PRACTICAR  BUCEO  PRESENTA  DOLOR  EN  CUELLO  DE  TIPO  INCAPACITANTE.  A  SU  LLEGADA  AL  SERVICIO  DE  URGENCIAS  SE 
SOLICITAN RADIOGRAFÍAS EN 2 PROYECCIONES DE CUELLO. 
 
EL FACTOR DE RIESGO MÁS IMPORTANTE PARA PRESENTAR LESIÓN CERVICAL EN ESTE PACIENTE ES: 
A) SER HOMBRE. 
B) BUCEAR. 
C) LA EDAD. 
D) SER CUIDADOR DE PERSONAS. 
 
RESPUESTAS    B 
 
 
 
[OBSTRUCCIÓN INTESTINAL] 
 
Obstrucción alta: náuseas, vómito, distensión 
Obstrucción baja; más distensión, vómito fecaloide. 
 
Para ENARM importa mucho el gasto por SNG, >500cc es alto gasto y nos indica que está obstruído de forma completa.   
 
En incompletas o parciales mecánicas, tx con medio de contraste 200‐250ml, es osmótico, resuelve obstrucción.   
 
 
HOMBRE DE 38 AÑOS DE EDAD, CON ANTECEDENTE DE APENDICECTOMÍA HACE 6 MESES Y EN ESTA OCASIÓN INGRESA CON DATOS DE 
OBSTRUCCIÓN INTESTINAL. 
LA CAUSA MÁS PROBABLE POR LA CUAL EL PACIENTE PRESENTA ESTE CUADRO CLÍNICO ES: 
   
A) ADHERENCIAS INTESTINALES. 
B) ABSCESO RESIDUAL. 
C) CÁNCER DE COLÓN. 
D) ADENITIS MESENTÉRICA. 
 
RESPUESTAS A 
El  bloqueo  de  la  luz  intestinal  se  produce  en  su  mayoría  por  factores  extrínsecos  (adherencias,  hernia  inguinal,  hernia  de  pared 
estrangulada) y en pocas ocasiones por factores intrínsecos.    Las adherencias intraabdominales por intervenciones quirúrgicas previas 
representan el 75% de los casos de obstrucción intestinal.   
 
  
[COLECISTITIS CRÓNICA LITIÁSICA] 
MUJER  DE  46  AÑOS  DE  EDAD,  SORDOMUDA,  CON  SOBREPESO  EVIDENTE.  LLEGA  SOLA  A  URGENCIAS  POR  DOLOR  ABDOMINAL.  SE 
DESCONOCEN ANTECEDENTES. DURANTE SU ESTANCIA EN URGENCIAS PRESENTA DOS VÓMITOS DE CONTENIDO GÁSTRICO. SE SOLICITA 
UNA RADIOGRAFÍA DE ABDOMEN DE PIE OBTENIENDO LA SIGUIENTE IMAGEN. 

 
 
EL DIAGNÓSTICO MÁS PROBABLE DE ESTA PACIENTE ES: 
   
A) LITIASIS URETERAL. 
B) LITIASIS RENAL. 
C) COLECISTITIS AGUDA LITIÁSICA. 
D) TUMOR RENAL. 
 
RESPUESTA C 
 
La ausencia de sintomatología urinaria y la localización de los litos nos hace descartar litiasis ureteral y renal. Recordar que los litos renales 
puden  ser  de  calcio,  ác  urico,  estuvita  y  cisteína.  Los  de  ác  úrico  se  forman  en  orina  ácida  y  son  radiotranspartentes,  el  resto  son 
radioopacos.  Pueden  originarse  por  hipocitraturia,  hiperoxaluria,  hipercalciuria  e  hiperuricosuria.  Un  pH  >7.6  nos  hace  pensar  en 
bacterias  productoras  de  ureasa.  Si  pH  6‐7  será  necesario  considerar  acidosis  tubular  como  causa  de  nefrocalcinosis.  Si  pH<5.5  es 
compatible con formación de ác. úrico y cisteína.   
Hipercalciuria + hipercalcemia + hipofosfatemia==> hiperparatiroidismo (medir parathormona) 
Los cálculos no radioopacos se pueden observar como un defecto de llenado cuando se usa medio de contraste.   
 
El 70‐80% de los tumores malignos del riñón corresponde a carcinoma de células claras, 50‐60% hace mets a pulmones, 25 a ganglios 
hiliaries renales, 12 suprarrenal ipsilateral, 30‐40% con lesiones líticas en huesos. Principal manifestación es hematuria macro o micro 
(60%). Dx de imagen de elección TAC abdominal. 
 
 
[ABSCESO PERIANAL] 
MUJER DE 33 AÑOS DE EDAD, DE OCUPACIÓN SECRETARIA, POR LO QUE LA MAYOR PARTE DE SU DÍA LABORAL LA PASA EN POSICIÓN 
SENTADA. USTED CONFIRMA EL DIAGNÓSTICO DE ABSCESO PERIANAL AGUDO 
EL MECANISMO FISIOPATOLÓGICO QUE MÁS PROBABLEMENTE ANTECEDÍO A LA FORMACIÓN DEL ABSCESO PERIANAL DE LA PACIENTE 
FUE: 
   
A) UNA FISURA ANAL. 
B) UNA FÍSTULA PERIANAL. 
C) INFLAMACIÓN CRIPTOGLANDULAR. 
D) COLITIS ISQUÉMICA.   
 
LA COMPLICACIÓN ESPERADA DEL ABSCESO PERIANAL DE LA PACIENTE EN CASO DE NO TENER UNA RESOLUCIÓN PRONTA SERÍA:   
A) FISURA ANAL 
B) FÍSTULA PERIANAL 
C) SEPSIS ABDOINAL 
D) PROLAPSO RECTA.   
 
RESPUESTAS C, B 
 
El Absceso anal    frecuente entre 20‐60 años, en ambos sexos. Causas específicas:    cuerpos extraños, traumatismos, enfermedades 
inflamatorias intestinales (Crohn) procesos infecciosos específicos, tumores, secuelas de tratamientos radioterápico y otras enf anales 
como la fisura anal. Causas inespecíficas: como la enf criptoglandular.    La teoría criptoglandular es la base fisiopatológica más aceptada 
del absceso perianal. 
 
La Fisura anal    es una úlcera longitudina por debajo de la línea dentada. Generalmente posterior, menos habitual anterior y en 3% 
pueden  coexistir  las  dos.    Incidencia  igual  en  ambos  sexos,  frec  en  edad  media  de  la  vida.  Generalmente  idiopáticas.  Etiología  más 
probable traumatismo agudo del conducto anal durante la defecación.    Si fístulas múltiples pensar en tuberculosis, sífilis, Crohn o sx de 
inmunodeficiencia.    Si es crónica se verá como úlcera con bordes indurados, acompañada de pliegue cutáneo indurado en el extremo 
distal  (hemorroide  centinela)  y  una  papila  hipertrófica  en  el  borde  proximal  (pólipo  de  Lane).  La  físura  puede  acompañar  al  absceso 
perianal agudo e incluso favorecerlo, pero no forma parte de la fisiopatología.   
 
La Fístula perianal es un conducto que comunica una cripta anal o el lumen del recto con la piel. Las fístulas simples son normalmente 
interesfinterianas  [B]  o  transesfinterianas  bajas  [C],  generalmente  de  trayecto  único.    La  fístula  compleja  tiene  un  trayecto  que 
comprende más del 30‐50% del esfínter externo (transesfintérica alta [C] , supraesfintérica [D] o extraesfintérica[E]), es de localización 
anterior en mujeres.    Se presentan en 50%    de los pacientes con absceso anorrectal desarrollan una fístula.    Los antecedentes de 
importancia  en  este  caso  son:    patología  intestinal  asociada,  eventos  que  comprometan  el  esfínter,  cirugía  anorrectal,  trauma 
obstétrico,  procesos  infecciosos  anorrectales.  Presentan  como  descarga  a  través  del  orificio  externo,  en  ocasiones  dolor.  EF  orificio 
externo con descarga y se palpa trayecto con fibrosis. La fístula perianal es más bien una complicación del absceso.   
 
La Colitis isquémica es la forma de presentación más frecuente de isquemia intestinal (70‐75%). Afecta princilamente a adultos mayores, 
y en jovenes asocia DM2, LES o crisis de células falciformes. La pancreatitis se ha asociado como factor al ocluir en ocasiones los vaso 
mesocólicos.    La  fisiopatlogía  de  la  colitis  isquémica  yace  en  una  insuficiencia  venosa  o  arterial  previa.  La  colitis  isquémica  derecha 
aislada de relaciona con    cardiopatía crónica, en especial estenosis aórtica. Se manifiesta como un dolor abdominal agudo, hemorragia 
gastro intesitnal y diarrea aguda. No se relaciona con abscesos perianales, patología distinta.   
 
  Causas de sepsis abdominal:   
  Colecistopancreatitis      27.5% 
  Perforación de colon    17.2 
  Heridas penetrantes abd.  13.7 
  Apendicitis aguda    13.7 
  Politraumtizado    10..3 
  Abscesos peripancráticos  6.9 
  Trombosis mesentéricas  3.4 
 
El prolapso rectal se asocia a: demensia senil, parasitosis (esquistosomiasis, tricocefalosis, amebiasis), enf neurológias (espina bífida), 
multiparidad, histectomía, estreñimiento de larga evolución, hipermotilidad sigmdea.   
 
 
 
         
 
  
 
 
 
[CÁNCER DE PRÓSTATA] 
HOMBRE DE 64 AÑOS DE EDAD QUE ACUDE A LA CONSULTA PORQUE DESDE HACE 3 MESES PRESENTA DISMINUCIÓN DEL CALIBRE DEL 
CHORRO URINARIO, GOTEO TERMINAL, POLAQUIURIA Y DISURIA. A LA EXPLORACIÓN SE REALIZA TACTO RECTAL ENCONTRANDOSE LA 
PRÓSTATA CON UN NÓDULO PÉTREO EN LA PARTE POSTERIOR DE ESTA GLÁNDULA. 
PARA CONFIRMAR EL DIAGNÓSTICO SE DEBERÁ INDICAR EL SIGUIENTE ESTUDIO: 
A) ULTRASONIDO PROSTÁTICO. 
B) FOSFATASA ALCALINA EN SUERO. 
C) NIVELES DE ANTÍGENO PROSTÁTICO ESPECÍFICO. 
D) BIOPSIAS TRANSRECTALES DE PRÓSTATA. 
 
RESPUESTA D 
No se recomienda tamizaje para próstata a población asintomática entre 50‐69 años    de edad empleando la prueba de APE y TR (ya que 
no asocia reducción de mortalidad). 
La detección basada en APE demostró reducción del 20% pero se relaciona con importante riesgo de sobrediagnóstico.   
La mayor parte de cánceres se presentan en la parte posterior de la glándula en forma de nódulos indoloros y pétreos.   
La biopsia de próstata guida por USG es la única prueba confirmatoria, idelmente tres de la derecha, tres de la izquieda y si procede 
clínicamente 1 de la zona transicional. No se recomienda si prostatitis, a menos que haya recibido un ciclo de antibioticoterapia.   
TR VPP 21%. 17% de pacientes con PSA 2.5‐4ng/ml y datos normales en TR tiene cáncer.   
 
 
[r] 
HOMBRE  DE  60  AÑOS  DE  EDAD  QUE  ACUDE  A  CONSULTA  CON  SINTOMATOLOGÍA  URINARIA  IMPORTANTE.  USTED  SOSPECHA  LA 
POSIBILIDAD DE CÁNCER DE PROSTATA. 
EN ESTE CASO DEBERÁ IDENTIFICAR EL SIGUIENTE FACTOR DE RIESGO: 
   
A) CONSUMO DE DIETA CON ELEVADO CONTENIDO DE GRASAS. 
B) CONSUMO OLIGOELEMENTOS. 
C) EL CONTACTO PERMANENTE CON SUBSTANCIAS DERIVADAS DE LA ACTIVIDAD INDUSTRIAL. 
D) LA RAZA O GRUPO ÉTNICO 
 
RESPUESTA D 
Factores ambientales: zona geográfica, dieta alta en grasa, exposición a humo de automóviles, polución del aire, cadmio, fertilizantes, 
sustnacias de la industria de la goma, imprenta y pintura.   
Factores genéticos: gen HPC 1. 
Factores infecciosos: ITS. 
Factores hormonales: andrógenos (en eunucos no aparecen). 
 
El consumo elevado de grasa si es FR pero no el más importante, los oligolementos no tienen nada que ver.   
La sustnacias de la industria de pueden generar riesgo pero la respuesta es vaga, no especifica que sea de pintura y la goma.   
La raza negra es el mayor factor de riesgo de neoplasia prostática intraepilial multifocal y con gran inestabilidad. Quizás por altos niveles 
de testosterona. 
En términos generales son los factores de riesgo más imprtantes para Ca prostático: Edad avanzada, antc familiares, raza negra.   
 
 
HOMBRE DE 35 AÑOS DE EDAD ACUDE A CONSULTA CON MÉDICO GENERAL. EL PACIENTE SE MUESTRA CONSTERNADO DEBIDO A QUE 
DOS DE SUS TÍOS LES DETECTARON CÁNCER DE PRÓSTATA A LOS 50 Y EL OTRO A LOS 65 AÑOS, NIEGA SINTOMATOLOGÍA ASOCIADA Y 
NO CUENTA CON ANTECEDENTS DE IMPORTANCIA. AL TACTO RETAL SIN PRSENCIA DE NÓDULOS O ALTERACINES. EL APE ES DE 2.3NG/DL.   
¿CUÁL ES LA RECOMENDACIÓN MÁS APROPIADA PARA EL PACIENTE? 
A) DETERMINACIONES BIENALES DE APE A PARTIR DE LOS 65‐85 AÑOS 
B) DETERMINAICONES ANUALES Y BIENALES DE APE DE LOS 35‐75 AÑOS 
C) DETERMINACIONES ANUALES DE APE DE LOS 40‐75 AÑOS 
D) DETERMINACIONES ANUALES DE APE DE LOS 50‐75 AÑOS 
 
RESPUESTA C 
Ca de próstata tiene fenómeno de anticipación de 7 años. El paciente en cuestión tiene factores de riesgo y por lo tanto el cribado se 
inicia con APE a los 40 años, de no tenerlos sería    a los 50.    [info del proedumed] 
 
El antígeno prostático específico es el examen más importante según las guías de la Secretaría de Salud, a pesar de que hay varios países 
que lo han retirado de sus guías de práctica clínica, porque aunque detecta    el cánr no modifica la motalidad. 
un gram de cáncer produce 3ng/ml de antígeno.   
los valores normales van de 0‐4ng/ml. La velocidad de crecimiento anual de más de 1ng/ml por año se considera anormal. El porcentaje 
de antígeno libre también se puede detectar, y si este es menor a un 10% en general es debido a áncer y es más de 25% probablemnte 
se trate de un adenoma. El estándar de oro para el diag nóstico es la biopsia. Se sugiere realizar determinaciones deanuales de APE, pero 
denbido al crecimiento lento del cáncer, se podría realizar también en intervalos de 2 años iniciando desde los 40‐45 años hasta los 75‐
65 añls si el pacient presentó determinaciones bajas persistentes entre 0.5‐1ng/dl. [reto enarm] 
 
 
HOMBRE DE 67 AÑOS DE EDAD QUE ACUDE A CONSULTA PORQUE, HACE ALGUNOS MESES, DEBE PUJAR CON ÁS FUERZA PARA PODER 
ORINAR Y AUN ASÍ, EL CHORRO URINARIO HA DISMINUIDO DE CALIBRE. AL INTERROGATORIO DIRIGIDO REFIERE QUE SE PARA A ORINAR 
MÁS DE DOS VECES POR LA NOCHE. NIEGA DISURIA, HEMTURIA O ALGUNA CONDICIÓN ASOCIADA. TIENE DIAGNÓSTICO DE DM2 HACE 
10 AÑOS, EN TRATAMIENTO REGULRA. FC 77, FR 29 T 36 TA 115/67. A LA EXPLORACIÓN FÍSICA SE PALPA PRÓSTATA SIN NÓDULOS O 
INDURACIONES PERO AUMENTADA DE TAMAÑO. LOS ESTUDIOS DE LABORATORIO REPORTAN APE 4‐5NG/DL. SE REALZÓ UN USG QUE 
DEMOSTRÓ UNA PRÓSTATA DE 45 CC. SIN NÓDULOS NI CALCIFICACIONES. 
¿CUÁL ES EL MANEJO INICIAL DE ESTE PACIENTE? 
A) CLONIDINA + FINASTERIDE 
B) DANAZOL + DOXAZOSINA 
C) PRAZOSINA + DANAZOL 
D) DOXAZOSINA + FINASTERIDE 
 
RESPUESTA D 
La hiperplasia prostática benigna se reifere a una detección microscópica de la hiperplasia (una proloferación de estroma y epitelio), 
crecimiento de próstata detectado por examen rectal digital o por medio de ultrasonido además de presentar síntomas asociados a esta 
hiperplasia. Se recuerda que el tamaño de la próstata no siempre correlaciona con los síntomas. La prevalencia aumenta en forma lineal 
con la edad, teniendo una prevalencia de 90% en pacientes de 85 años o más. La clásica sintomatología obstructiva es pujo miccional, 
dismiinución del calibre de la orina, nicturia, etre otros. Entre los métodos diagnósticos está la USG con medici´n de orina residual. Hay 
una relación directa entre lso niveles de APE y el volumen prostático pero no tienen mayor riesgo de desarrollar cáncer de próstata. El 
tratamiento farmacológico evita la morbilidad asociada a la cirugía. En pórstata de más de 40cc el tratamiento combiada (alfablqoueador 
e inhibidor de la 5 alfa redutasa) es la mejor opciójn. La clonidina es una gonista alfa 2.   
 
  
 
  [OSTEOMALACIA] 
FEMENINA DE 50 AÑOS DE EDAD, DIABÉTICA DESDE HACE 15 AÑOS, CON MAL CONTROL DE SU ENFERMEDAD. ACUDE URGENCIAS POR 
PRESENTAR DOLOR INTENSO EN EXTREMIDAD INFERIOR DERECHA ASÍ COMO LIMITACIÓN DE LA MOVILIDAD DE LA MISMA EXTREMIDAD. 
GLUCEMIA DE 340 MG/DL. EXAMEN GENERAL DE ORINA CON PROTEINURIA Y ERITROCITOS 1 A 2 POR CAMPO. LA RADIOGRAFÍA DE LA 
EXTREMIDAD MUESTRA FISURA EN EL DEDO MEDIO DEL PIE DERECHO Y DESMINERALIZACIÓN EN HUESOS CIRCUNDANTES. 
 
  EN ESTA PACIENTE LA DESMINERALIZACIÓN OSEA ES SECUNDARIA A LA DISMINUCIÓN EN: 
   
A) VITAMINA A. 
B) VITAMINA B12. 
C) VITAMINA C. 
D) VITAMINA D. 
 
RESPUESTA D 
  Se trata de paciente con complicaciones microvasculares de DM2, tiene descontrol metabólico y proteinuria. Por otro lado presenta una 
fractura (fisura) patológica puesto que no existe antecedente de traumatismo y tiene desmineralización.   
La desmineralización se puede explicar por dos mecanismos: a) disminución de matriz protéica (osteoporosis), déficit de sales minerales 
(osteomalacia). En ocasines puede haber osteoporomalacia. 
La D3 es la forma endógena que se produce en la piel, D2 es la forma exógena. Se metaboliza en el hígado por la 25‐OH conviertiendose 
en 25 hidroxivitamina D. Luego pasa al RIÑÓN donde se hidroxila una vez más, conviertiéndose en 1‐25 OH vitamina D que es la forma 
de  mayor  actividad  metabólica.  Cuando  existe  daño  renal  se  limita  la  formación  de  este  último  metabolito  altamente  activo  y  en 
consecuencia hay desmineralización. 
 
 
  [HERNIA INGUINAL] 
MUJER DE 48 AÑOS DE EDAD, CON ANTECEDENTE DE HERNIA INGUINAL BILATERAL. ES VALORADA EN EL SERVICIO DE URGENCIAS POR 
PRESENTAR DOLOR INTENSO EN LA REGIÓN INGUINAL DERECHA. 
LA POSIBILIDAD DE QUE SE TRATE DE UNA HERNIA ESTRANGULADA SE CONFIRMARA EN CASO DE: 
 
A) QUE HAYAN DATOS DE OCLUSIÓN INTESTINAL. 
B) QUE HAYAN DATOS DE PERFORACIÓN INTESTINAL. 
C) QUE NO SEA REDUCTIBLE. 
D) QUE EXISTA COMPROMISO VASCULAR. 
 
RESPUESTA D 
Tanto en hernias incarderadas como estanguladas hay cierto grado de oclusión intestinal.   
La perforación intestinal puede ser producto de una isquemia prolongada.   
En una hernia estrangulada hay compromiso vascular,    está tensa y muy snesible, la piel que lo cubre tiene color rojizo o azulado. N 
existen ruidos intetinales dentro de la hernia. El paciente suele tener leucocitosis con desviación a la izquierda, se encuentra tóxico, 
deshidratacio, febril, GASA con acidosis metabólica.    El Tx es Qx de urgencia por riesgo de perforación, el tx de esta última complicación 
es la resección del segmento afectado.   
 
 
  [HIPERTENSIÓN INTRACRANEAL] 
MUJER OBESA EN LA CUARTA DÉCADA DE LA VIDA CON ANTECEDENTE DE CUADROS REPETITIVOS DE CEFALEA INTENSA. COMO PARTE 
DEL PROTOCOLO DE ESTUDIO SE REALIZA EXPLORACIÓN DEL FONDO DE OJO ENCONTRANDO LA SIGUIENTE IMAGEN. 
  

 
 
EL DIAGNÓSTICO MÁS PROBABLE DE ESTE PACIENTE ES: 
   
A) DEGENERACIÓN VÍTREA. 
B) DRUSEN DEL DISCO ÓPTICO. 
C) PAPILEDEMA. 
D) UVEITIS. 
 
CORRESPONDE AL MECANISMO FISIOPATOLÓGICO QUE PRODUCE ESTA ENFERMEDAD: 
 
A) INFECCIÓN SIFILÍTICA. 
B) ENVEJECIMIENTO. 
C) DEPÓSITOS CÁLCICOS EN EL NERVIO ÓPTICO. 
D) HIPERTESIÓN ENDOCRANEAL 
 
RESPUESTAS C, D 
La degeneración vítrea se presenta a la par del envejecimiento y presenta depósitos lipídicos en el gel vítreo.   
El drusen del NO se caracteriza por depósitos refrigentes situados dentro del parénquima de la cabeza del nervio óptico. A la EF partículas 
brillantes en la superficie del NO. Clínicamente oscurecimiento de la visión. Puede coexistir papiledema y drusen óptico. 
El papiledema se manifiesta como resultado de hipertensión intracraneal, la papila luce con borde deshilachados, exudados algodonosos 
peripapilares, hermorragias intra y extrapapilares,    exudados color cereza. Puede asociar cefalea pero no es un requisito. Clíniamente 
oscurecimiento transitorio de la visión es un signo clásico,    afección a uno o ambos ojos al mismo tiempo.   
  La uveítis hace referencia a la inflamación aguda o crónica intraocular. Puede tener causa infecciosa (sífilis‐imagen sal y pimienta + 
papiledema) o inmunológica.   
Uveítis anterior: células inflamatorias en humor acuoso, células en endotelio corneal (imagen en grasa de carnero), nódulos en iris. Es 
causa de ojo rojo doloroso, asocia fotofobia, pérdida visual unilateral.   
Uveítis posterior: pérdida visual gradual. Células en vítreo con inflmación en retina y corides.   
 
 
[DMI] 
HOMBRE  DE  22  AÑOS  DE  EDAD,  CON  DIAGNÓSTICO  DE  DIABETES  TIPO  I  DESDE  HACE  TRES  MESES.  A  ÉSTE  PACIENTE,  SE  LE DEBERÁ 
RECOMENDAR REALIZAR UNA REVISIÓN OFTALMOLÓGICA EN EL SIGUIENTE LAPSO DE TIEMPO: 
   
A) AL MOMENTO DEL DIAGNÓSTICO. 
B) UN AÑO. 
C) TRES AÑOS. 
D) CINCO AÑOS. 
 
RESPUESTA C 
En DM2 la revisión oftalmológica se hace al diagnóstico y cada año, se justifica porque al diagnóstico 39% tiene retinopatía diabética y 4‐
8% tienen comprometida la visión.   
Si tiene DM1 o 2 y desea embarazo, deberá relizarse cada 3 meses de forma rutinaria o si tiene retinopatía diabética leve , cada mes si 
retipotía diabética moderada a grave. 
El tamizaje en DM1 se realiza   
  a) A partir de los 12 años de edad 
  b) A partir del 5to año de diagnóstico y cada año.   
  c) A partir del 3er año del diagnóstico una vez pasada la pubertad 
 
 
UVEITIS ANTERIOR 
[r]HOMBRE DE 30 AÑOS DE EDAD CON LUMBALGIA INFLAMATORIA. ACUDE A URGENCIAS POR UN CUADRO DE 24 HRS DE EVOLUCIÓN, 
CARACTERIZADO PR: DOLOR, FOTOFOBIA, BLEFAROSPAMO, VISIÓN BORROSA EN OJO IZQUIERDO. A LA EF SE OSBSRVAN OJ IZQUIERDO 
CON  HIPEREMIA  PERICORNEAL,  PUPILA  IZQUIRDA  MIÓTICA  CON  BRADICORIA,  FENÓMENO  DE  TYNDALL,  PRECIPITADOS  RETRO 
CORENALES PQUEÑOS, FONDO DE OJO APARENTEMENTE SIN ALTERACIONES Y AGUDEZA VISUAL DISMINUIDA. 
SEGÚN TU DIAGNÓSTICO PRESUNTIVO, ¿ QUÉ ENFERMEDAD REUMATOLÓGIA PADECE CON MAYOR PROBABILIDAD ESTE PACIENTE? 
A) ESPONDILITS ANQUILOSANTE 
B) SARCOIDSIS 
C) BEHCET 
D) ARTRITIS IDIOPÁTICA JUVENIL 
 
RESPUESTA A 
  El paciente tiene una uveitis anterior secundaria a espondilitis anquilosante. El Dx se apoya en: antecedente personal de espndilitis 
anquilosante,  ojo  doloroso,  tiempo  de  evolución,  ,  fotofobia,  miosis  y  efecto  de  tyndall.  Las  enfermedades  que  más  frecunetente  se 
relacionada con uveítis anterior son: espondiloartritis (y de la más común la espondilitis aquilsante);, artritis idiopática juvenil, Behcet, 
Sarcoidosis,  Tuberculosis.  Sin  embargo,  en  este  caso  el  diangóstico  más  probable  es  espondilitis  anquilosante  por  el  antecedente  de 
lumbalgia inflamatoria y la suencia de otro dqato sistémico. La uveítis es debida a la inflamación del iris y cuerpo ciliar, lo cual provoca 
exudados en cámara anterio,; lo que nos da el fenóemno de Tyndall (células y proteínas fltante en cámara anterior que s eobservan con 
la lámpara de hendidura). La inflamación de las estructuas produce un síndrome ciliar, caracterizado por: dolor, fotfobia, blefarospasmo 
e inyección pericorneal. La inflmación del iris produce una miosis con bradicoria (reacción lenta). El manejo es con esteroides, midriáticos 
y tratando la causa si que s se cnoce. RECUERDA Repasar diagnótico difernecia de ojo rojo. [reto enarm] 
 
 
HOMBRE DE 32 AÑOS, SIN ANTECEDENTES PATOLÓGICOS DE INTERÉS, QUE ACUDE A LA CONSULTA POR VISIÓN BORROSA EN SU OJO 
DERECHO DE 3 DÍAS DE EVOLUCIÓN. LA AGUDEZA VISUAL ES DE 0., EL EXAMEN DEL POLO ANTERIOR NO MUESTRA ALTERACIONES, Y EN 
EL FONDO DE OJO SE OBSERVA CICATRIZ CORIORRETINIANA Y CÉLULAS EN LA CAVIDAD VITREA. EL PACIENTE RELATA DOS EPIDOSIOS 
OCULARES  SIMILARES  NO  DIAGNOSTICADOS.  ¿QUIÉ  EXPLORACIONES  COMPLEMENTARIAS  SOLICITARÍA  PARA  ESTABLECER  EL 
DIAGNÓSTICO? 
A) TEST DE MANTOUX Y TINCIÓN DE ZIEHL‐NIELSEN EN ESPUTO 
B) SEROLOGÍA TOXOPLÁSMICA 
C) RADIOGRAFÍA DE TÓRAX 
D) SEROLOGÍA LUÉTICA 
 
RESPUESTA 
 
UNA MUJER DE 40 AÑOS QUE PADECE ARTRITIS REUMATOIDE, TRATADA CON SALES DE ORO, PRESENTA HIPEREMIA E INYECCIÓN CILIAR 
EN OJO DERECHO, CON VISIÓN BORROSA, MIOSIS DERECHA IRREGULAR Y DOLOR OCULAR, CON TENSIÓN OCULAR NORMAL. ¿CUÁL DE 
LOS DIAGNÓSTICOS QUE A CONTINUACIÓN SE ENUMERAN ES EL CORRECTO? 
A) DESPRENDIMIENTO DE RETINA 
B) GLAUCOMA CRÓNICO SIMPLE   
C) CONJUNTIVITIS AGUDA 
D) UVEÍTIS ANTERIOR 
 
RESPUESTA 
 
A 27 
 
 
 
 
 
[RETINOPATÍA DIABÉTICA][r] 
HOMBRE  DE  63  AÑOS  DE  EDAD  CON  DIAGNÓSTICO  DE  DIABETES  MELLITUS  DESDE  18  AÑOS.  ACUDE  A  CITA  DE  CONTROL 
OFTALMOLÓGICA. A LA EXPLORACIÓN DEL FONDO DE OJO SE OBSERVA LA SIGUIENTE IMAGEN. 

 
 
 
CON BASE EN LA IMAGEN DE FONDO DE OJO MOSTRADA SE PUEDE CONCLUIR QUE EL PACIENTE PRESENTA: 
   
A) RETINOPATÍA NO PROLIFERATIVA. 
B) UNA HEMORRAGIA VÍTREA. 
C) RETINOPATÍA PROLIFERATIVA. 
D) EDEMA DE PAPILA. 
 
EL TRATAMIENTO DE ELECCIÓN EN ESTE CASO INCLUYE: 
   
A) CONTROL METABÓLICO + APLICACIÓN DE LÁSER. 
B) UNICAMENTE CONTROL METABÓLICO. 
C) CONTROL METABÓLICO + DERIVACIÓN DE LÍQUIDO CEFALORRAQUÍDEO. 
D) CONTROL METABÓLICO + REPOSO EN SEMIFOWLER 
 
RESPUESTA A, B 
En la retinopatía diabética podemos encontrar varias lesiones: microaneurismas, hemorragias, exudados duros, manchas algodonosas, 
alteraciones microvasculares, arrosaraiamiento venoso, neovasos, tejido fibroso.   
Se clasfica en RDP y RDNP.   
La RDNP se caracteriza por aneurismas vasculares retinianos, manchas hemorrágicas, dilatación venosa y exudados algonosos.   
RDNP mínima: microanerusismas 
RDNP moderada: microaneurismas + exudados duros + engrosamiento de capilares 
RDNP avanzada: Más de 20 hermorragias retinianas x cuadrante o Arrosaramiento venoso en 2 o más cuadrantes o AMIR en un cuadrante. 
 
 
 
[GLAUCOMA] 
MUJER DE 75 AÑOS DE EDAD, QUE ACUDE A LA CONSULTA CON SINTOMATOLOGÍA COMPATIBLE CON GLAUCOMA. 
PARA ENTENDER LA FISIOPATOLOGÍA DEL GLAUCOMA, ES IMPORTANTE SABER QUE EL DAÑO GLACOMATOSO SE DA EN LA SIGUIENTE 
PORCIÓN DEL NERVIO ÓPTICO: 
   
A) INTRAOCULAR. 
B) ORBITARIA. 
C) INTRACANALICULAR. 
D) INTRACRANEAL. 
 
RESPUESTA A 
El glaucoma es una neuropatía óptica crónica y progresiva, hay pérdida de fibras nerviosas de la retina, excavación y palidez progresivas 
de la papila y defectos del campo visual. Va asociado a hipertensión intraocular pero NO en todos los casos. De hecho hay glaucoma 
normotenso, y hay hipertensión intraocular (>21mmHg) sin glaucoma.    La papila corresponde a la parte intraocular del nervio óptico. 
El  glaucoma  de  ángulo  abierto  (GPAA)  es  el  más  freucente  con  un  60%.  El  ángulo  esta  abierto  y  suele  ser  simétrico.    Etiología 
desconocida.    Pero el principal defecto fisiopatológico es la resistencia al paso del humor por la malla trabecular.   
Factores de reisgo: 1) PIC, 2) antecdenetes familiares (4‐16% de riesgo de padercerla si son de primer grado, 3)>40 años. 
 
Epitelio  de  cuerpo  ciliar===>  humor  acuoso  en  cámara  posterior==>  pupila==>  cámara  anterior  ==>ándulo  iridocorneal==> 
trabeculum===> canal de Schlemm==>vaso espiesclerales==> circulación sanguínea general (eliminación del 90% del humor acuoso) 
 
Solo 10% se eliminará vía uveoescleral atravesando directamente el cuerpo ciliar. 
 
En el glaucoma de ángulo cerrado el origen está en un bloqueo pupilar relativo a una mayor aposición del iris y el cristalino que dificutla 
el paso del humor acuoso de la cámara posterior a la anterior a través de la pupila (empujando a la periferia el iris).   
 
Independientemente de la causa, habrá un pérdida progresiva de la capa de fibras nerviosas ya sea de forma mecánica directa o por 
compromiso vasculaar.   
Tratatamiento farmacológico: 
1)betabloqueadores: disminuyen la producción del humor acuoso al actuar sobre los receptores beta de los procesos ciliares [timolol, 
carteolol, betaxolol (selectivo B1)] 
2)Mióticos parasimpaticomiméticos    y    anticolinesterásicos: disminuyen la resistencia de la malla trabecular. [pilocarpina y aceclidina] 
3) Simpaticomiméticos A y B adrenérgicos:    [adrenalina] aumentan el flujo de salida del humor acuoso y disminuyendo la producción en 
el cuerpo ciliar. [NO en ángulo cerrado] 
4) Agonistas alfa 2 adrenérigcos: disminución de la prodccuón de humor acuoso 
5)  Prostaglandinas:  [latanaprost  al  0.005%]  aumenta  la  salida  de  humor  acuoso  a  través  de  vía  uveoescleral.  [NO  en  glaucoma 
inflamatorio] 
6) Inhibidores de anhidrasa carbónica 
 
Tratamiento láser: trabeculoplastia con lárser de argón en GPAA y trabeculectomía, iridotomía periférica con láser argón o lsa Ng‐YAG en 
GPAC. 
Tratamiento quirúrgico: trabeculectomía simple o combinada cirugía de catarata (facoemulsificación) 
 
¿CUÁL ES LA CAUSA MÁS FRECUENTE DE UNA EXCAVACIÓN PAPILAR CON RECHAZO NASAL DE LOS VASOS EMERGENTES DE LA MISMA? 
A) HIPERTENSIÓN INTRACRANEAL 
B) UVEÍTIS POSTERIOR 
C) CONJUNTIVITIS CRÓNICA 
D) GLAUCOMA CRÓNICO SIMPLE 
 
RESPUESTA D 
No entendí la pregunta :( 
 
UNA  MUJER  DE  64  AÑOS,  HIPERMÉTROPE  Y  CON  CATARATAS  EN  AMBOS  OJOS,  ACUDE  A  CONSULTA  CON  DOLOR  INTENSO  EN  OJO 
IZQUIERDO,    DE UNAS HORAS DE EVOLUCIÓN. LA EXPLORACIÓN DE ES OJO PONE DE MANIFIESTO UNA TENSIÓN OCULAR DE 40MMHG, 
REACCIÓN HIPERÉMICA CILIO‐CONJUNTIVAL, MIDRIASIS Y EDEMA CORNEAL. ¿CUÁL SERÍA LA ACTITUD INMEDIATA MÁS CORRECTA DE 
LAS QUE SE ENUMERAN A CONTINUACIÓN? 
A) MANITOL INTRAVENOSO, ASOCIADO A TRATAMIENTO MIÓTICO Y CORTICOIDES SISTÉMICOS 
B) PRACTICAR TRABECULECTOMÍA 
C) CORTICOIDES TÓPICOS, ASOCIADOS A TRATAMIENTO MIDRIÁTICO 
D) OPERAR CON URGENCIA LA CATARATA DE ESE OJO, DESENCADENANTE EL CUADRO. 
 
RESPUESTA A 
 
ACUDE ACONSULTA UN PACIENTE PORQUE DICE QUE EN LOS ÚLTIMOS MESES LE HA CAMBIADO EL COLOR DEL OJO IZQUIERDO. EN LA 
ANAMENSIS DEBE INSISTIR EN POSIBLE OCNSUMO DE ALGIUNA DE LAS SIGUIENTES MEDICAMENONES POR VÍA TÓPICA: 
A) COLIRIOS PARA GLAUCOMA 
B) AINE 
C) CORTICOIDES 
D)  ACICLOVIR,  PUES  ESTÁ  DEMOSTRADO  QUE  PUEDE  PRODUCIR  HIPERPIGMENTACIÓN  EN  EL  IRIS  CUANDO  SE  MANTIENE  EL 
TRATAMIENTO DURANTE UN PERIODO MUY LARGO DE TIEMPO 
 
RESPUESTA A 
Hay que recordar que los análogos de prostaglandinas F2 pueden producir como efectos secundarios cambios en la coloración del iris 
(hiperpigmentación) e hipermia y aumento del tamaño de las pestañas.    A pesar de ello son los medicamentos mejor tolerados y eficacez 
en el manejo de glaucoma de ángulo abierto. 
 
A 93‐YEAR OLD WOMAN, COMES TO THE EMERGENCY ROOM COMPAINING ABOUT ACUTE PAIN AND REDNESS IN HER RIGHT EYE. TEN 
YEARS AGO SHE WAS SUBMITTED TO CATARACT SURGERY OF HER LEFT EYE. HOWEVER THEY "DIDN'T OPERATE MY RIGHT EYE, BECAUSE 
IT IS A LAZY EYE". VISUAL ACUITY IS COUNTING FINGERS IN HER RIGHT    EYE AND 8/20 IN HER LEFT EYE. IOP IS 48MMHG IN HER RIGHT 
EYE  AND  15MMHG  IN  HER  LEFT  EYE.  SLIT  LAMP  EXAMINATION  SHOWS  CORNEAL  EDEMA,  HYPERMATURE  CATARACT  AND  A  DEEP 
ANTERIOR  CHAMBER  WITH  INTENSE  FLARE,  AND  GROSS  TYNDALL.  AGGREGATES  OF  WHIT  MATERIAL  ARE  PRESENT  OVER  THE 
ENDOTHELIUM AND THE ANTERIOR FACE OF THE LENS. FUNDUS EXAMINATION REVEALS THE PRESENCE OF DRUSEN IN HER LEF EYE, BUT 
IT IUS NOT POSSIBLE IT IN HER RIGHT EYE, DUE TO THE DENSITY OF THE CATARACT. THE MOST PROBABLE DIAGNOSIS IS: 
A) ACUDE ANGLE‐CLOSURE GLAUCOMA 
B) PHACOLYTIC GLAUCOMA 
C) PHACOMORPHIC GLAUCOMA 
D) CHOROIDAL MELANOMA 
 
THE MOST APROPIATE TREATMENT FOR THIS PATTIEN IS:   
A) OCULAR HYPOTENSIVE DRUGS, TOPICAL STEROIDS AND THE CATARACT SURGERY ON HER RIGHT EYE. 
B) OCULAR HYPOTENSIVE DRUGS, AND TOPICAL STEROIDS ON HER RIGHT EYE 
C) ENUCLEATION OF HER RIGHT EYE 
D) EVISCERATION OF HER RIGHT EYE 
 
ANSWERS 
B, A 
 
A 45‐YEAR‐OLD HEALTHY MAN HAS STARTED TREATMENT WITH LATANAPROST AND TIMOLOL 0.5% FOR A RECENTLY DIAGNOSED OPEN 
ANGLE GLAUCOMA. HE TELLS YOU THAT HE CAN'T KEEP ON WITH HIS FRIENDS WHEN HE IS PLAYING PADDLE. WICH OF THE FOLLOWING 
WOULD BE THE WISER ATTITUDE? 
A) CHANGE LATANAPROST FOR A DIFFERENT DRUG 
B) CHANGE TIMOLOL FOR A DIFFERENT DRUG 
C) CHANGE BOTH DRUGS 
D) SEND HIM T THE CARDIOLOGIST 
 
ANSWER B 
No entendí la pregunta pero es poco factible que latanaprost tenga que suspenderse. Timolol debe usarse con cuidado en cardiopatas y 
broncópatas. Probablemente este paciente tenga alguna contraindicación.   
 
 
 
[ÚLCERA CORNEAL] 
HOMBRE DE 35 AÑOS DE EDAD. ACUDE A SERVICIO DE URGENCIAS POR CUADRO DE 12 HORAS DE EVOLUCIÓN, CARACTERIZADO POR 
DOLOR  OCULAR  DERECHO  INTENSO,  ACOMPAÑADO  DE  FOTOFOBIA,  LAGRIMEO,  PRESENCIA  DE  SECRECIÓN  AMARILLENTA  Y 
DISMINUCIÓN IMPORTANTE DE LA AGUDEZA VISUAL ÚNICAMENTE EN ES OJO. AL INTERROGATORIO EL PACIENTE REFIERE QUE HACE 24 
HRS TUVO ENTRADA DE ARENILLA EN OJO DERECHO. A LA EXPLORAICÓN FÍSICA: EL OJO DERECHO SE OBSERVA CON HIPEREMIA MIXTA 
(PERIFÉRICA Y PERICORNEAL) INTENSA, QUEMOSIS CONJUNTIVAL Y DE LOS PÁRPADOS, CÓRNEA OPACA, BLEFAROESPASMO Y PUPILAS 
REACTIVAS. OJO IZQUIERDO SIN ALTERACINES. 
¿CUÁL DE LOS SIGUIENTES DIAGNÓSTICO ES EL MÁS PROBABLE? 
A) ÚLCRA CORNEAL 
B) UVETITIS AGUDA 
C) CONJUNTIVITIS BACTERIANA 
D) GLAUCOMA 
 
RESPUESTA A 
El  diagnóstico  se  apoya  en  ojo  doloroso  (unilateral),  hiperemia  mixta,  córnea  opaca,  quemosis  y  pupilas  reactivas.  No  se  trata  de 
conjuntivitis bacteriana, ya que esta no da dolor ocular. El glaucoma es una causa de ojo rojo doloroso, pero los reflejos pupilares se 
encuentran alterados (midriasis fija), y en este paciente no hay alteración pupilar. La uveítis es una causa frecuente de ojo rojo doloroso, 
pero hay alteración del reflejo pupilar (miosis) y fenóemnos de Tyndall (humor acuoso turbio por celularidad). La úlcera corneal es una 
urgencia oftalmológica, ya que los pacientes corren riesgo de perforación, se debe de inciar antibiótico, quitar el dolor y referir lo antes 
posible al oftalmólogo para su manejo.   
 
 
[INSUFICIENCIA ARTERIAL PERIFÉRICA] 
HOMBRE  DE  64  AÑOS  DE  EDAD,  TABAQUISMO  CRÓNICO,  DIABÉTICO  E  HIPERTENSO.  ACUDE  A  CONSULTA  POR  DOLOR  EN  LAS 
EXTREMIDADES INFERIORES. A LA EXPLORACIÓN FÍSICA FC 80 FR 19 T 37 TA 130/78, PULSOS PEDIOS, TIBIAL POSTERIOR DISMINUIDOS Y 
DOLOR DEL PIE. REALIZA PRUEBA DE ÍNDICE TOBILLO‐BRAZO.   
¿CUÁL ES EL VALOR APROXIMADA QUE PROBABLMENTE TENDRÁ EL PACIENTE EN ESTA PRUEBA? 
A) 1.2 
B) 1.5 
C) 0.9 
D) 0.6 
 
RESPUESTA D 
El  paciente  tiene  enfermedad  arterial  periférica.  La  prevalencia  de  enfermeda  arterial  periférica  aumenta  en  pacientes  >70  años, 
pacientes entre 50‐69 años con historia de tabaquismo o diabetes,    como en    este paciente . Se concen otros factores de riesgo como 
historia familiar de ateroesclerosis, sexo masculino, hipertensión, hiperlipidemia, entre otros. Hay una pruba simple que se puede realizar 
para orientar el diagnóstico de esta entidad, y se le llama índice tobillo brazo, el cual es la presión sistólica del tobillo dividido entre la 
presión sistólica braquial.    El valor normal es >0.9 por lo que en este caso encontraremos valores menores a 0.9. Valores mayores a 1.3 
sugieren la presencia de vasos calcificados por lo que se necesitarán estudios adicionales.   
 
 
 
 
 
 
 
 
[  
 
 
 
 
 
[LUXACIÓN DE CADERA] [r] 
FEMENINA DE 64 AÑOS DE EDAD, SUFRE CAÍDA. A SU LLEGADA A URGENCIAS REFIERE DOLOR INTENSO EN CADERA DERECHA. A LA 
EXPLORACIÓN SE OBSERVA ACORTAMIENTO DE UNA DE LAS EXTREMIDADES CON SENSACIÓN DE INESTABILIDAD AL MOVER LA 
EXTREMIDAD. 
 
SE SOSPECHA UNA LUXACIÓN POSTERIOR DE CADERA. LA URGENCIA EN LA ATENCIÓN DE ESTE PACIENTE ES DEBIDO A QUE ESTE 
PROCESO PUEDE OCASIONAR: 
A) OSTEOMIELITIS. 
B) RIGIDEZ ARTICULAR. 
C) FRACTURA SECUNDARIA. 
D) OSTEONECROSIS. 
 
LA SIGUIENTE POSICIÓN DE LA EXTREMIDAD LESIONADA DIRIGIRÍA EL DIAGNÓSTICO DE LUXACIÓN POSTERIOR DE CADERA. 
A) FLEXIÓN, ROTACIÓN INTERNA Y ABDUCCIÓN. 
B) FLEXIÓN, ROTACIÓN, INTERNA Y ADUCCIÓN. 
C) FLEXIÓN, ROTACIÓN EXTERNA Y ABDUCCIÓN. 
D) FLEXIÓN, ROTACIÓN EXTERNA Y ADUCCIÓN 
 
CORRESPONDE AL TRATAMIENTO DE ELECCIÓN EN ESTE CASO: 
A) TRACCIÓN CON REDUCCIÓN QUIRÚRGICA. 
B) TRACCIÓN CON SEDACIÓN Y ROTACIÓN EXTERNA. 
C) TRACCIÓN E INMOVILIZACIÓN DE LA EXTREMIDAD. 
D) TRACCIÓN LONGITUDINAL CON PACIENTE EN SUPINO 
 
RESPUESTAS D, B, D 
La luxación posterior de cadera es la más frecuente, corresponde hasta el 95% de las luxaciones de cadera. A la exploración se 
encuentra acortamiento de la extremidad, cadera flexionada, rotación interna y aducción.   
La luxación anterior de la cadera es la menos frecuente y se traduce en flexión, rotación externa y abducción.    [Posterior AD, anterior 
ABD] 
 
En el proceso de luxación de cadera puede haber interrupción vascular asociada y se debe evitar de forma urgente Osteonecrosis. La 
osteonecrosis es una complicación tanto de luxación anterior como posterior por la cual ameritan reducción urgente.    Si en la 
pregunta hubieran pregunta por complicación específica habría que considerar lesión del nervio ciático puest que es una complicación 
del 20%    de las luxaciones posteriores.   
La rigidez articular sí puyede ser una complicación pero crónica y por luxaciones recurrentes, por lo tanto no es urgencia.   
La fractura deberá buscarse asociada a la luxación y de forma impsilateral, pero NO es una complicación de la luxación.   
 
No está inciada la rotación de la cadera en el manejo de la luxación y tampoco inmovilización ( a menos que asocia fracturas). 
Lo ideal es la reducción con tracción de la extremidad estando el paciente supino: luego de ello deberá flexionarse cadera a 90° en 
posición neutral y aplicar fuerza hacia atrás en caso de alteración o subluxación está indicada la reducción quirúrgica urgente.   
 
[FRACTURA DE MUÑECA] [r] 
 
FEMENINA DE 75 AÑOS DE EDAD, ES LLEVADA A URGENCIAS POR SUFRIR CAÍDA. A LA EXPLORACIÓN PRESENTA LA MUÑECA DERECHA 
FORMA DE TENEDOR. USTED INFIERE QUE EL MECANISMO DE LA LESION FUE: 
   
A) ANTEBRAZO EN SUPINACIÓN. 
B) MUÑECA EN ANTEFLEXIÓN. 
C) BRAZO EN ABDUCCIÓN. 
D) MANO EN HIPEREXTENSIÓN. 
 
RESPUESTA C 
Cuando hablamos de deformidad en dorso de tenedor, se hace referencia a una fractura del extremo distal del radio, o fractura de 
Colles. El mecanismo para que se produzca esta lesión es caída sobre la mano con muñeca en hiperextensión o dorsiflexión. 
La fractura de Colles se observan tres desviaciones: 
  Enclavamiento de la epífisis en la metáfisis 
  Desviación dorsal de la epífisis dando la silueta en "dorso de tenedor" 
  Desviación radial de la epífisis dando la forma de "desviación en bayoneta" 
 
[PARÁLISIS DE CUERDAS VOCALES] [r] 
FEMENINA DE 44 AÑOS, CURSA POSTQUIRÚRGICO SECUNDARIO A TIROIDECTOMÍA TOTAL. PRESENTA SUBITAMENTE ESTRIDOR 
INTENSO Y SENSACIÓN DE FALTA DE AIRE. SE SOLICITA LARINGOSCOPÍA QUE MUESTRA PARÁLISIS DE CUERDAS VOCALES. 
EN ESTE MOMENTO ESTARÁ INDICADO REALIZAR: 
   
A) INTUBACIÓN OROTRAQUEAL. 
B) CORDECTOMÍA. 
C) TRAQUEOSTOMÍA. 
D) ARITENOIDECTOMÍA. 
 
RESPUESTA C 
Hay que tomar en cuenta que la parálisis de cuerdas vocales impide el paso de la cánula de intubación orotraqueal, razón por la cual 
habrá que asegurar la vía aérea mediante traqueostomía.   
La cordectomía consiste en extirpar parte de las cuerdas voales, requiere especialistas altamente capacitados, poco factible usarla en 
urgencias.   
La aritenoidectomía también permite liberar la vía aérea, pero requiere especialistas altamente capacitados, poco factible usarl en 
urgencias.   
El 90% de las parálisis son de causa periférica, 10% central. 
Dentro de las periféricas, cirugía tiroidea y cervical (40‐60%), tumoraciones torácicos y cervicales (5‐25%), idiopáticas (5‐20%), 
sistémicas y otras causas (5‐15%), traumatismo cervical y anestésico 1‐5%) 
 
 
 
 
 
  [QUEMADURAS] 
EN EL CENTRO DE SALUD SE DAN PLATICAS ORIENTANDO A LOS USUARIOS SOBRE LA PREVENCION DE LAS QUEMADURAS EN EL 
HOGAR. 
EN LA PLÁTICA SE COMENTA QUE HAY ALGUNAS QUEMADURAS QUE SON MAS GRAVES DE LO QUE APARENTAN. ESTE TIPO DE 
QUEMADURAS SON LAS: 
   
A) ELÉCTRICAS. 
B) QUÍMICAS. 
C) POR RADIACIÓN. 
D) POR LÁSER. 
 
RESPUESTA D   
Tipos de traumatismo eléctrico. 
Directo: 80% de todos los traumatismos eléctricos. 
  De bajo voltaje <1000 v produce lesiones pequeñas y profundas, así como arritmias graves. 
  De alto voltaje >1000 v produce lesiones graves como quemaduras, afetación multiorgánica, destrucción tisular, etc.     
      Similar a sx por aplastamiento 
Indirecto o arco voltáico: causado por campo magnético al rededor de líneas de alta tensión    (>10 000v) 
Flash eléctrico: realmente es una quemadura por llama, es una lesión térmicas.   
Rayo: produce parada respiratoria y muerte inmediata. Lesiones cutáneas en arborización típica. 
 
La rabdomiólisis causa liberación de mioglobina e insuficiencia renal. Tx inmediado con manejo de vía aérea, línea IV, monitorea EKG, 
sonda vesical. Si orina oscura pensar==> hemocromógenos. De hecho no se debe esperar a la confirmación de mioglobulinuria para 
empezar a administrar líquidos, deberá mantenerse una diuresis cuando menos de 100ml/hr en adulto. Si no se aclara el pigmento 
habrá que aumentar líquidos, 25gr de manitol, 12.5 a cada litro subsecuente. 
 
[OTITIS EXTERNA] 
HOMBRE DE 27 AÑOS DE EDAD QUE ACUDE A CONSULTA POR PRESENTAR OTALGIA DERECHA SEVERA. EL APCIENTE REFIERE QUE EL 
DOLOR SE IRRADIA HACIA LA REGIÓN TPM Y TAMBIÉN TIEN PRURIT. NIEGA ANTECEDENTES DE IMPORTNAICA. REFIERE REALZIAR 
NATACIÓN 3 VECES POR SEMANA. FC 67, FR 13, T37.5 TA 114/76.. TIENE ERITEMA Y EDEMA DIFUSO EN CONDUTO AUTIVIO DERECHO.   
¿CUÁL ES EL TRATAMIENTO MÁS ADEUCADA PARA ESTE PACIENTE? 
A) AMOXICILINA/CLAGVULANATO ORAL + GOTAS ÓTICAS + AC. AC´TICO 
B) NEOMICINA ÓTICA + DEXAMETASONA + AC ACÉTICO 
C) CIPROFLOACINO ORAL    + AINES 
D) GOITAS ÓTICAS CON DEXAMETASONA Y ÁC. AC´TICO 
 
RESPUESTA A 
Ciertos factores se asocian con la presencia de otitis externa, que es el diangóstico de este caso: la alteración del ph del conducto 
(seuncario a exposición a agua cmo pr ejempo antación), tauma local (cotonetes), limpieza agresiva entre otros. La manifestación 
clínica de esta patología es la hiperestesia locorregional a la manipulación de pabellón auricular (o también cnocido com signo del 
trago). La exploración física por medio de otoscopia demuesta un entorno inflamatorio (ritema, edema). De acuero a las guinas del 
CENETEC, el tratamiento es a base de gotas óticas que combinan esteroide o antibiótico +    ac acético. El uso de ác acético sólo es 
menos efectivo que la combinaci´n previamente mencinada. recordar que los agentes etiológicos más frecuentes son: peusomonas 
aeruginosa, S. aureus y el tratmiento tópico es de elección. 
[OTITIS MEDIA] 
MUJER DE 17 AÑOS QUE INICIA SU PADECIMIENTO HACE 7 DÍAS CON LA PRESENCIA DE CEFALEA Y OTALGIA IZQUIERDA. EL DÍA DE AYER 
SE AGREGA FIEBRE DE 38.5°C, HIPOACUSIA. A LA EXPLORACIÓN SE ENCUENTRA MEMBRANA TIMPÁNICA IZQUIERDA INTEGRA, 
HIPERÉMICA, ABOMBADA E INMÓVIL. 
EL DIAGNÓSTICO CLÍNICO MÁS PROBABLE ES: 
   
A) OTITIS MEDIA AGUDA. 
B) OTITIS MEDIA CRÓNICA. 
C) OTITIS SEROSA. 
D) OTITIS NECROTIZANTE. 
 
EL FACTOR DESENCADENANTE MÁS COMÚN EN ESTE TIPO DE PACIENTES ES: 
A) NADAR SIN PROTECCIÓN. 
B) LA ACUMULACIÓN DE CERUMEN. 
C) LAS INFECCIONES DE VÍAS RESPIRATORIAS SUPERIORES. 
D) LA PERFORACIÓN DE LA MEMBRANA TIMPÁNICA. 
 
  ADEMÁS DE ADMINISTRAR ANALGÉSICOS Y ANTIPIRÉTICOS SE DEBERÁ AGREGAR: 
A) TRIMETOPRIMA MÁS SULFAMETOXAZOL. 
B) CIPROFLOXACINA. 
C) AMOXICILINA. 
D) PENICILINA PROCAÍNICA. 
 
RESPUESTA A,C,C 
Otitis media aguda: principal agente etiológico 1) S. pneumoniae, 2)H. influenzae no tipable. Es una enfermedad íntimente relacionada 
con resfraido común del que es complicación habitual. La infección vírica de vías respiratorias altas provoca una toxicidad sobre el 
epitelio respiratorios, causante d discinesia ciliar en la trompa de Eustaquio, con alteración subsecuente del aclaramiento de moco. 
La otitis serosa es muy frecuente en la infancia es una condición muy frecuente, se caracteriza por la pesencia de líquido detrás del 
tímpano. Generalmente se asocia a resfraido o a otitis aguda previa, aunque no significa que el oido esté infectado. Mejora 
espontáneamente en unos meses. 
La osteomilitis de la base del cráneo también conocida como otitis externa maligna o necrotizante. Comineza como infección del 
conducto auditivo externo, lugo del temporal, meningitis, septicemia,    muerte.    Se observa en inmunocomprometidos (diabéticos de 
edad avanzada). Etiología: pseudomonas aeruginosa. Clínica: a parte de los síntomas óticos puede haner hipoacusia, trismus, parálisis 
facial o datos de septicemia. (en sus inicios hay síntomas de otitis externa aguda). 
 
Respecto al tatamiento amoxi es de primera elección cefalosporina de 2da generación es segunda eleccion, y cloranfenicol es de 
tercera. En otits media S.pneumonae tiene 50% de resistencia. TMP/SMX no ha    mostrado ser mejor que amoxicilina, 
 
 
ENFERMEDADES ANORRECTALES 
[Resumen la Salle] 
DEFINICIÓN 
 
Recto==> hemorroides internas NO duelen==> se pueden tratar endoscópicamente   
Línea dentada ==> son válvulas para continencia, tiene columnas de Morgani c glándulas, si se tapan se abscedan ==> enf 
criptoglandular 
Ano==> hemorroides externas sí duelen 
 
Hemorroides: 
  Son venas o várices 
  Cuadrante lateral izquierdo 
  Cuadrante anterior derecho 
  Cuadrante posterior derecho 
 
Internas (más frecuentes)  Si ingurgitan da síntomas, prolapso al ano 
Externas (menos frecuentes)      Solo cuando hay trombosis deja colgajo si se resuelve.   
Mixta 
 
 
PATOGENIA 
Causas: estreñimiento, hipertensión portal.   
 
DIAGNÓSTICO Y TRATAMIENTO 
Hemorroides‐ absceso/fístula ‐ fisura 
Hemorragia, dolor, secresión, cambios de hábitos 
EF: co  lación permeabilidad, pedir que puje y valorar prolapso de hemorroides o recto, y tacto rectal.   
Anoscopia y/o proctosigmoidoscopia, si quiere saber si tiene fístula o saber si enf inflamatoria 
        Tratamiento 
Grado I  Indoloro      Mx 
Grado II  Reducción espontánea    Mx 
Grado III  Reducción manual    Qx 
Grado IV  No reductible    Qx 
 
Mx: dieta con fibra, ejercicio. 
Qx: Ligadura c banda elástica, escleroterapia, hemorroidectomía cerrada.   
 
Si están trombosadas ==> trombectomía (anestesia local y drenaje de coágulo.   
 
Dx dif Ca, enf inflamatoria, divertículos, pólipos, fisuras (duelen mucho) ==> sangre fresca   
Complicaciones: sepsis perianal (etiología principal Clostrium perfringens), sangrado, dolor, necrosis, . 
Si no cambian de hábitos recurren tras la Qx.   
 
Fisuras:    Ocurren en línea media, 90% posteriorres, 10% anteriores, 1% anteroposteriores.   
Si fuera de 
Síntoma pivote al estar debajo de línea dentada "dolor", no requiere defecr. 
El recto se le dilata==> se desgarran más. clínica:; dolor desgarrantes, gotas de sangre rojos urtilante cojn las heces. Paraclínicps: quizas 
manomería (esfínter hipertónico) Complicación: sangrado, no malignizan fuera de la línea media si hago biopsia.   
Tratamiento: sediluvios (agua tibia), 90% curan, nitroglicerina 2%, isosorbide SL x cada mg /kg /d? de 2/3 diosmina, paciente, se mueve, 
calider, lido (8disparos) 1/3 ketanserina (sufrexal) luego de cada sediluvio. Lidocaina, esteroide tópico x 10 días al igual que en 
hemorroides.    Toxina botulínica. Si    Tx    Qx si crónicidad o recurrencia.    fisurectomía + esfinterotomía lateral izquierda parcial.   
 
Absceso y fístula:     
  Perianal      Se ven 
  Interesfinteriano    Se ven   
  Isquioanal      Se ven 
  Supraelevador    Se palpa, es el más riesgoso, si se rompe hace peritonitis, hay que hacer diagnóstico rápido.   
   
Explorar bajo antetesia, tx drenaje.   
Clínica: pueden hacer spesis, en DM2 Fournier Paraclínica: USG solo si requiere detectar el trayecto. Si transesfinteriana ‐Setón 
 
 
 
HEMORRAGIA DE TUBO DIGESTIVO ALTO 
[Resumen la Salle] 
DEFINICIÓN 
1) Lipotimia, síncope, diaforesis, melena, hematemesis.   
2) Sospecha! ¿Cuál es su estado de TA, FC, ortostatismo, hiptrmia= 
Sangrado pero TA, FC, temperatura normales, no ortostatismo (<15 mmHg)==> pérdida <10 de VC 
Hipotermina, palidez, resto normal==> pérdida de 10‐20% 
Palidez, hipotermina, hipotensión    >15mmHg, ortostatimo. ==> pérdida 20‐35%VC==> joven, sano, canalizo y vena permeables, 
normal en sala de observación.   
Si FC >100, TA >15 de pérdida, diaforesis, hipotermina==> 40‐.50% de VC perdido==> cateter venosos ce ntral, terapia intensiva.     
 
Indicación de transfusión: 
  Hemorragia activa    ==> sangre total 
  Sangra con tratamiento  ==> sangre total 
  Caída del 10% del Htc (33%)    ==> concentrado eritrocitario 
  Dolor torácico    cambios en EKG (isquemia) ==> concentrado eritrocitario 
Luego de 10 paquetes de lo que sea debo pasar 1 concentrado plaquetarios (aportan 10,000 plaquetas) 
 
STD con pérdida de 20‐30% de VC 
Joven y adulto joven      Enf ác péptica (úlcera duodenal, enf inflamatria intestinal, pólipos 
Aultos <60       Ca, enf diverticular, hemorroides y fisuras, malf arterio venosa, 
Ancianos        Ca, angiodisplasia, diverticulos.   
 
STDA: hematemesis, melena, hematoquezia.   
 
      STDA      STDB 
Presentan        Melena o hematemesis  Hematoquezia 
... por SNG     Sangre ??      Limpio 
Relación BUN/Cr    Elevado      Normal 
Auscultación cardiaca   Hiper 
 
 
 
 
 
 
 

S-ar putea să vă placă și